Maths and Science Last 15 Years Cbse Solved Question Papers

You might also like

Download as pdf or txt
Download as pdf or txt
You are on page 1of 603

CBSE-XII-2017 EXAMINATION CBSE-X-2008 EXAMINATION

MATHEMATICS
Series: RLH/2 Paper & Solution Code: 30/2/1
Time: 3 Hrs. Max. Marks: 80
General Instuctions :
(i) All questions are compulsory.
(ii) The question paper consists of 30 questions divided into four sections — A, B, C and D. Section A
comprises of ten questions of .1 mark each, Section B comprises of live questions of 2 marks each, Section C
comprises of ten questions of 3 marks each and Section D comprises of five questions of 6 marks each
(iii) All questions in Section A are to be answered in one word, one sentence or as per the exact requirement
of the question.
(iv) There is no overall choice. However, an internal choice has been provided in one question of 2 marks
each, three questions of 3 marks each and two questions of 6 marks each You have to attempt only one of
the alternatives in all such questions.
(v) In question on construction, the drawings should be neat and exactly as per the given measurement.
(vi) Use of calculators is not permitted.

SECTION – A
1. Complete the missing entries in the following factor tree :

Solution:

Lets assume the missing entries be a, b


b  3  7  21
a  2  b  2  21  42

2. If (x + a) is a factor of 2x2 + 2ax + 5x + 10, find a.


Solution:
(x + a) is factor of the polynomial p(x) = 2x2 + 2ax + 5x + 10
 p(a)  0 {By factor theorem}

www.vedantu.com 1 / 23
CBSE-X-2008 EXAMINATION

2(a)2  2a(a)  5(a)  10  0


2a2  2a2  5a  10  0
a 2

3. Show that x = - 3 is a solution of x2 + 6x + 9 = 0.


Solution:
x2 + 6x + 9 =0
x 2  2.3x  (3)2  0
(x  3)2  0
 x  3 is the solution of x2 + 6x +9 =0

4. The first term of an A.P. is p and its common difference is q. Find its 10th term.
Solution:
First term of an A.P. = p
Common difference = q
T10  P  (10  1)q
T10  P  9q

5
5. If tan A = , find the value of (sin A + cos A) sec A.
12
Solution:
5
tan A 
12
sin A cos A
 sin A  cos A  sec A  
cos A cos A
 tan A  1
5
 1
12
17

12

6. The lengths of the diagonals of a rhombus are 30 cm and 40 cm. Find the side of the rhombus.
Solution:
Length of the diagonals of a rhombus are 30cm and 40cm

www.vedantu.com 2 / 23
CBSE-X-2008 EXAMINATION

i.e., BD = 30 cm
AC = 40 cm
OD  OB  15cm
OA  OC  20cm
AOD
OA2  OD 2  AD 2
(20)2  (15)2  AD2
AD  25cm
Side of rhombus = 25 cm

ar (APQ)
7. In Figure 1, PQ II BC and AP : PB = 1 : 2. Find .
ar (ABC)

Solution:

PQ || BC

www.vedantu.com 3 / 23
CBSE-X-2008 EXAMINATION

AP 1

PB 2
PB 2

AP 1
PB 2
1  1
AP 1
PB  AP 3

AP 1
AP 1

AB 3
2
ar (APQ)  AP  1
   
ar (ABC )  AB  9

8. The surface area of a sphere is 616 cm2. Find its radius.


Solution:
Surface area of sphere = 616 cm2
4 r 2  616
22
4   r 2  616
7
r  7 cm
9. A die is thrown once. Find the probability of getting a number less than 3.
Solution:
Total possible outcomes = 6
Outcomes which are less than 3 = 1, 2
2
Probability
6
1
=
3

10. Find the class marks of classes 10 —25 and 35 —55.


Solution:

Class Class marks


10-25 10  25
 17.5
2
35-55 35  55
 45
2

SECTION – B
Questions number 11 to 15 carry 2 marks each.

www.vedantu.com 4 / 23
CBSE-X-2008 EXAMINATION

11. Find all the zeros of the polynomial x4 + x3 - 34x2 - 4x + 120, if two of its zeros are 2 and -2.
Solution:
p(x)  x 4  x 3  34 x 2  4 x  120
Lets assume other two zeroes are  , 
Sum of all zeroes     2  2
  
    1
   1  .........(1)
Product of zeroes = 120
..2.(2)  120
   
Substituting (1) in (2)
(1  )  30
  2  30
2    30  0
  6, 5
  5,  6
Zeroes of the polynomial are 6,  2, 2, 5

12. A pair of dice is thrown once. Find the probability of getting the same number on each dice.
Solution:
Total outcomes  6  6  36
Outcomes with same number on each = {(1, 1)(2, 2)(3, 3)(4, 4)(5, 5)(6, 6)}
No. of favorable outcomes = 6
6 1
Probability  
36 6

13. If sec 4A = cosec (A - 20°), where 4A is an acute angle, find the value of A.
OR
1
In a  ABC, right-angled at C, if tan A = , find the value of sin A cos B + cos A sin B.
3
Solution:
sec4 A  cosec(A  20)
sec4 A  sec(90  (A  20))[sec(90  x)  cosec x]
sec A  sec(110  A)
4 A  110  A
5A  110
A  22
OR

www.vedantu.com 5 / 23
CBSE-X-2008 EXAMINATION

In ABC , c  90 tan


1
tan A 
3
sin A cos B  cos A sin B  sin(A  B)
 sin(180  c)
 sin c
 sin90
1

14. Find the value of k if the points (k, 3), (6, -2) and (-3, 4) are collinear.
Solution:
Given points (K, 3), (6, -2), (-3, 4) are collinear
 Area of the triangle formed by these points = 0
1
k(2  4)  6(4  3)  3(3  2)  0
2
6k  6  15  0
3
k
2

15. E is a point on the side AD produced of a ||gm ABCD and BE intersects CD at F. Show that  ABE ~ 
CFB.
Solution:

In ABE and CFB,


A  C (Opposite angles of a parallelogram)
AEB  CBF (Alternate interior angles as AE||BC)
ABE ~ CFB (By AA similarly criterion)

SECTION C
Questions number 16 to 25 carry 3 marks each.
16. Use Euclid's Division Lemma to show that the square of any positive integer is either of the form 3m or
(3m + 1) for some integer m.
Solution:
Let’s ‘a’ be any positive integer and b = 3.
We know a = bq + r, 0  r <b.
Now, a  3q  r , 0  r  3.

www.vedantu.com 6 / 23
CBSE-X-2008 EXAMINATION

The possible of remainder = 0, 1 or 2


Case I – a =3q
a2  9q 2
 3  (3q 2 )
 3m (where m  3q 2 )
Case II – a = 3q+1
a2  (3q  1)2
 9q2  6q  1
 3(3q2  2q)  1
= 3m + 1 (where m = 3q2  2q )
Case III – a = 3q + 2
a2  (3q  2)2
 9q2  12q  4
 3(3q2  4q  1)  1
= 3m + 1 (where m  3q2  4q  1 )
From all the above cases it is clear that square of any positive integer (as in this a2) is either of the form 3m
or 3m + 1.

17. Represent the following pair of equations graphically and write the coordinates of points where the
lines intersect y-axis :
x + 3y = 6
2x - 3y = 12
Solution:
x  3y  6
2x  3y  12
Graph of x + 3y = 6:
When x= 0, we have y= 2 and when y = 0, we have x= 6.
Therefore, two points on the line are (0, 2) and (6, 0).
The line x + 3y= 6 is represented in the given graph.
Graph of 2x - 3y = 12:
When x= 0, we have y = - 4 and when y = 0, we have x = 6.
Hence, the two points on the line are (0, - 4) and (6, 0).
The line 2x - 3y =12 is shown in the graph.

www.vedantu.com 7 / 23
CBSE-X-2008 EXAMINATION

The line x + 3y = 6 intersects y-axis at (0,2) and the line 2x - 3y = 12 intersects y-axis at (0, - 4).

18. For what value of n are the nth terms of two A.P.'s 63, 65, 67, ... and 3, 10, 17, ... equal ?
OR
If m times the m term of an A.P. is equal to n times its nth term, find the (m + n)th term of the AP.
th

Solution:
nth term of 63, 65, 67, …..
 63  (n  1)(2)
 63  2n  2
 61  2n ...........(1)
nth term of 3, 10, 17, ……….
 3  (n  1)7
 3  7n  7
 7n  4 .......(2)
Given that nth terms of two AP’s are equal.
(1) = (2)
61  2n  7n  4
65  5n
n  13
OR
Lets assume first term = a
Common difference = d
Tm  a  (m  1)d
Tn  a  (n  1)d
Given m.Tm  n.Tn

www.vedantu.com 8 / 23
CBSE-X-2008 EXAMINATION

m(a  (m  1)d )  n(a  (n  1)d )


ma  m(m  1)d  na  n(n  1)d
(m  n)a  d (m2  m  n2  n)  0
a(m  n)  d (m  n)(m  n  1)  0
(m  n)[a  (m  n  1)d ]  0
m  n a  (m  n  1)d  0
Tmn  0
19. In an A.P., the first term is 8, nth term is 33 and sum to first n terms is 123. Find n and d, the common
difference.
Solution:
. First term (a) = 5
Tn  33
Sum of first n terms = 123
n
 a  Tn   123
2
n
8  33  123
2
n6
Tn  a  (n  1)d
33  8  (5)d
d 5

20. Prove that : (1 + cot A + tan A) (sin A— cos A) = sin A tan A— cot A cos A.
OR
Without using trigonometric tables, evaluate the following :
 cos58   cos38cosec52 
2   3 
 sin 32   tan15 tan 60 tan 75 
Solution:
(1  cos A  tan A)(sin A  cos A)
 1  sin A 
 1   tan A   1  cos A
 tan A  cos A 


1  tan 2
A  tan A   tan A  1  cos A
tan A


 tan A  1 cos A
3

tan A
 tan A cos A  cot A cos A
2

sin A
 tan A. .cos A  cot A cos A
cos A
 sin A tan A  cot A cos A

www.vedantu.com 9 / 23
CBSE-X-2008 EXAMINATION

OR
 cos 58   cos38cosec72 
2  3 
 sin32   tan15 tan60 tan75 
tan75  cot(90  15)  cot15
tan15 tan75  1, tan60  3
sin32  cos 58, cos38  sin72
 sin32   cos38 sec38 
 2  3 
 sin32   3 
 2 1
1

AP 3
21. If P divides the join of A(-2, -2) and B(2, -4) such that  , find the coordinates of P.
AB 7
Solution:

AP 3

AB 7
As, AB = 7, AP = 3
 AB  AP  PB
 7  3  PB
 PB  7  3  4
Let the point P(x, y) divide the line segment joining the points A(2,  2) and B(2, 4) in the ratio AP : PB =
3:4
2(3)  (2)(4) (4)(3)  (4)(2)
x and y 
3 4 34
68 12  8
x and y 
7 7
2 20
x and y 
7 7
 2 20 
 The coordinate of P(x , y)   , 
 7 7 

22. The mid-points of the sides of a triangle are (3, 4), (4, 6) and (5, 7). Find the coordinates of the vertices
of the triangle.
Solution:

www.vedantu.com 10 / 23
CBSE-X-2008 EXAMINATION

Consider a ABC with A(x1, y1),B(x2, y2) and C(x3, y3). If P(3, 4), Q(4,6) and R(5,7) are the midpoints of AB,
BC and CA. Then,
x x
3  1 2  x1  x2  6 ...(i)
2
y y
4  1 2  y1  y2  8 ...(ii)
2
x x
4  2 3  x2  x3  8 ...(iii)
2
y y
5  2 3  y2  y3  12 ...(iv)
2
x x
6  3 1  x3  x1  10 ...(v)
2
y y
7  3 1  y2  y1  14 ...(vi)
2
Adding (i), (ii) and (v) we get
2(x1  x2  x3 )  6  8  10  24
 x1  x2  x3  12 ...(iv)
From (i) and (vii), we get x3 - 12 - 6 = 6
From (iii) and (vii) we get v1, = 12 - 8 = 4
From (v) and (vii), we get x2 = 12 - 10 = 2
Now, adding (ii), (iv) and (vi), we get
20(y1 + y2 + y3)= 8 + 12 + 14 = 34
 y1  y2  y3  17 (viii)
From (ii) and (viii), we get y3 = 17 - 8 = 9
From (iv) and (viii), we get y1 = 17 - 12 = 5
From (vi) and (viii), we get y2 = 17 – 14 = 3
Hence, the vertices of ABC are A (4, 5), B(2, 3), C(6, 9)

23. Draw a right triangle in which the sides containing the right angle are 5 cm and 4 cm. Construct a
5
similar triangle whose sides are times the sides of the above triangle.
3
Solution:

www.vedantu.com 11 / 23
CBSE-X-2008 EXAMINATION

Steps:
1) Draw a line segment AB = 5 cm, Draw a ray SA making 90 with it.
2) Draw an arc with radius 4 cm to cut ray SA at C. Join BC. ABC is required 
3) Draw a ray AX making an acute angle with AB, opposite to vertex C.
4) Locate 5 points (as 5 is greater in 5 and 3), A1, A2, A3, A4, A5, on line segment AX such that AA = A1A2 =
A2A3 = A3A4 = A4A5
5) Join A3B. Draw a line through A5 parallel to A3B intersecting line segment AB at B’.
6) Through B’, draw a line parallel to BC intersecting extended line segment AC at C’. AB ' C ' is the
required triangle.

Justification:
The construction can be justified by provided that
5 5 5
AB '  AB, B ' C '  BC , AC '  AC
3 3 3
In ABC and AB ' C ',
ABC  AB ' C ' (Corresponding angles)
BAC  B ' AC ' (Common)
ABC ~ AB'C' (AA similarly criterion)
AB BC AC
   ............(1)
AB ' B ' C ' AC '
In AA3B and AA5B ',
A3 AB  A5 AB ' (Common)
AA3B  AA5B ' (Corresponding angles)
AA3B ~ AA5B ' (AA similarly criterion)

www.vedantu.com 12 / 23
CBSE-X-2008 EXAMINATION

AB AA3
 
AB ' AA5
AB 3
  ....(2)
AB ' 5
On comparing equations (1) and (2), we obtain
AB BC AC 3
  
AB ' B ' C ' AC ' 5
5 5 5
 AB '  AB, B ' C '  BC , AC '  AC
3 3 3
This justifies the construction.

24. Prove that a parallelogram circumscribing a circle is a rhombus.


OR
In Figure 2, AD  BC. Prove that AB2 + CD2 = BD2 + AC2.

Solution:

Given ABCD is a ||9m such that its sides touch a circle with centre O.
 AB = CD and AB || CD,
AD = BC and AD II BC
Now, P Q, R and S are the touching point of both the circle and the ||9th
We know that, tangents to a circle from an exterior point are equal in length.
 AP = AS [Tangents from point A] ... (1)
BP = BQ [Tangents from point B] ...(2)
CR = CQ [Tangents from point C] …..(3)

www.vedantu.com 13 / 23
CBSE-X-2008 EXAMINATION

DR = DS [Tangents from point D] …. (4)


On adding (1), (2), (3) and (4), we get
AP+BP+CR+DR=AS+BQ+CQ+DS
 (AP + BP) + (CR +DR) = (AS + DS) + (BQ + CQ)
 AB+CD = AD + BC
 AB + AB = BC + BC [ ABCD is a ||9m  AB = CD and AD= BC]
 2AB = 2BC
 AB = BC
Therefore, AB = BC implies
AB = BC = CD = AD
Hence, ABCD is a rhombus.
OR

In ABD.
by Pythagoras theorem,
AB2  BD2  AD2 ..........(i)
and in ADC ,
by Pythagoras theorem,
AC 2  CD2  AD2
CD2  AC 2  AD2 ........(ii)
On adding (i) &(ii), we get,
 AB2  CD2  BD2  AD2  AC 2  AD2
 AB2  CD2  BD2  AC 2
Hence, proved.

25. In Figure 3, ABC is a quadrant of a circle of radius 14 cm and a semi-circle is drawn with BC as diameter.
Find the area of the shaded region.

www.vedantu.com 14 / 23
CBSE-X-2008 EXAMINATION

Solution:

Given AC = AB = 14 cm
BC  142  142  14 2 cm
Area of shaded region = Area of semi-circle – (Area of sector ABDC – Area of ABC )
1
 Area of ABC   14  14  98 cm2
2
1 22
Area of Quadrant ABC = . .(14)2  154 cm2
4 7
Area of segment BC = ar (Quadrant ABC) – ar( ABC )
 154  98
 56 cm2
2
1  BC 
Area of semicircle BC =   
2  1 
1 22 1
    14 2  14 2
2 7 4
 154 cm2
Area of shaded region = Area of semicircle BC – Area of segment BC
 154  56
 98 cm2

SECTION D
Questions number 26 to 30 carry 6 marks each.

www.vedantu.com 15 / 23
CBSE-X-2008 EXAMINATION

26. A peacock is sitting on the top of a pillar, which is 9 m high. From a point 27 in away from the bottom of
the pillar, a snake is coming to its hole at the base of the pillar. Seeing the snake the peacock pounces on it.
If their speeds are equal, at what distance from the hole is the snake caught ?
OR
4
The difference of two numbers is 4. If the difference of their reciprocals is , find the two numbers.
21
Solution:
Let AB be the pillar of height 9 meter. The peacock is sitting at point A on the pillar and B is the foot of the
pillar. (AB = 9)
Let C be the position of the snake which is at 27 meters from B. (BC = 27 and ABC = 90)
As the speed of the snake and of the peacock is same they will travel the same distance in the same time
Now take a point D on BC that is equidistant from A and C (Please note that snake is moving towards the
pillar)

Hence by condition 1 AD = DC = y (say)


Take BD = x
Now consider triangle ABD which is a right angled triangle
Using Pythagorus theorem (AB + BD = AD)
9+x=y
9 = y - x = (y-x)(y + x)
81/(y + x) = (y-x)
y + x = BC = 27
Hence 81/27 = (y - x) = 3
y-x=3
y + x = 27
Adding gives 2y = 30 or y = 15
x = 27  y = 12
Thus the snake is caught at a distance of x meters or 12 meters from the hole.
OR
Lets assume the two numbers to be x, y (y > x)
Given that y  x  4y  4  x ...(1)

www.vedantu.com 16 / 23
CBSE-X-2008 EXAMINATION

1 1 4
 
x y 21
yx 4
 
xy 21
4 4
 
xy 21
xy  21
x(4  x)  21
x 2  4 x  21  0
(x  7)(x  3)  0
x  7, 3
y  3, 7
 Numbers are 7,  3 or 3, 7

27. The angle of elevation of an aeroplane from a point A on the ground is 60°. After a flight of 30 seconds,
the angle of elevation changes to 30°. If the plane is flying at a constant height of 3600 3 m find the
speed, in km/hour, of the plane.
Solution:

Height of aeroplane (CD) = 3600 3 mt = BE


BAD  60 and CAD  30
In ABE
BE
tan60 
AE
BE
AE 
tan60
AE  3600 mt
In ACD

www.vedantu.com 17 / 23
CBSE-X-2008 EXAMINATION

CD
tan30 
AD
3600 3
AD 
1
3
AD  10800 mt
BC  AD  AE  10800  3600
BC  7200 mt
distance
Speed of aeroplane 
time
7200
  240 m/s
30
Speed (in km/hr) = 864 km/s

28. If a line is drawn parallel to one side of a triangle to intersect the other two sides in distinct points,
prove that the other two sides are divided in the same ratio.
Using the above, prove the following
In Figure 4, AB II DE and BC || EF. Prove that AC || DF.

OR
Prove that the lengths of tangents drawn from an external point to a circle are equal. Using the above,
prove the following :
ABC is an isosceles triangle in which AB = AC, circumscribed about a circle, as shown in Figure 5. Prove that
the base is bisected by the point of contact.

www.vedantu.com 18 / 23
CBSE-X-2008 EXAMINATION

Solution:

Construction: Join BE and CD and draw perpendicular DN & EM to AC and AD respectively.


1
 EM  AD
ar (ADE ) 2

ar (BDE ) 1  BD  EM
2
ar (ADE ) AD
  ........(1)
ar (BDE ) BD
Similarly,
1
 AB  DN
ar (ADE ) 2

ar (CDE ) 1  EC  DN
2
ar (ADE ) AE
  ........(2)
ar (CDE ) EC
But ar(BDE)  ar(CDE) (triangle on same base DE and between the same parallels DE and BC)
Thus, equation (2) becomes,
ar (ADE ) AE
 .............(3)
ar (EDE ) EC
From equations (2) and (3), we have,

www.vedantu.com 19 / 23
CBSE-X-2008 EXAMINATION

AD AE

BD EC
In the given AB||DE and BC||EF. Prove that AC||DF.

In OPE , AB||DE (Given)


 By basic proportionality Theorem,
OA OB
 .......(1)
AD BE
Similarly, in OEF , BC ||EF (Given)
OB OC
  .....(2)
BE CF
Comparing (1) and (2), we get
OA OC

AD CF
Hence, AC ||DF
[By the converse of BPT]
OR

Given: PT and TQ we two tangent drawn from an external point T to the circle C (0, r).
To prove: 1. PT = TQ
2.  OTP =  OTQ
Construction: Join OT.
Proof: We know that a tangent to circle is perpendicular to the radius through the point of contact.
  OPT =  OQT = 90
In  OPT and  OQT,
OT = OT (Common)
OP = OQ (Radius of the circle)
 OPT =  OQT (90 )
  OPT

 OQT (RHS congruence criterion)

www.vedantu.com 20 / 23
CBSE-X-2008 EXAMINATION

 PT = TQ and  OTP =  OTQ (CPCT)


PT = TQ,
 The lengths of the tangents drawn from an external point to a circle are equal.

We know that the tangents drawn from an exterior point to a circle are equal in length.
 AP = AQ (Tangents from A) (1)
BP = RR (Tangents from B) (2)
CQ - CR (Tangents from C) (3)
Now, the given triangle is isosceles. so given AB = AC
Subtract AP from both sides, we get
AB- AP = AC-AP
= AB – AP = AC - AQ (Using (1))
BP = CQ
 BR = CQ (Using (2))
 BR = CR (Using (3))
So BR = CR, shows that BC is bisected at the point of contact.

29. If the radii of the circular ends of a conical bucket, which is 16 cm high, are 20 cm and 8 cm. find the
22
capacity and total surface area of the bucket. (Use   ]
7
Solution:
Radius of the bigger end of the frustum (bucket) of cone = R = 20 cm
Radius of the smaller end of the frustum (bucket) of the cone = r = 8 cm
Height = 16 cm
Volume = 1/3h[R2  r 2  R * r ]
= 1/3*22/7*16[202 + 82+ 20*8]
= 352/21[400 + 64 + 160]
=(352*624)/21
= 219648/21
= 10459.43 cu cm
Now,
Slant height of the cone = I = (R  r )2  h2

www.vedantu.com 21 / 23
CBSE-X-2008 EXAMINATION

l  (20  8)2  162


l  122  162
l  144  256
l  400
l  20 cm

Slant height is 20 cm
Now,
Surface area =  [R2  r 2  (R  r )*]
= 22/7[202 + 82 + (20 + 8)*16]
= 22/7[400 + 64 + 448]
= 22/7*912
= 20064/7
= 2866.29 sq cm

30. Find mean, median and mode of the following data :


Classes Frequency
0-20 6
20 -40 8
40 – 60 10
60 -80 12
80 – 100 6
100 – 120 5
120 -140 3
Solution:

Class Frequency class mark (xi) xifi


0-20 6 10 60
20-40 8 30 240
40-60 10 50 500
60-80 12 70 840
80-100 6 90 540
100-120 5 110 550
120-140 3 130 390
fi  50 fi xi  3120

www.vedantu.com 22 / 23
CBSE-X-2008 EXAMINATION

xi fi
Mean 
fi
3120

50
 62.4
Class f (less than)
cumulative
frequency
0-20 6 6
20-40 8 14
40-60 10 24
60-80 12 36
80-100 6 42
100-120 5 47
120-140 3 50
n  fi  50
n
 25
2
 Median class = 60 – 80
n 
 2  c. f 
M l   h
 f 
 
 25  24 
M  60     20
 12 
M  61.66
Mode:
Maximum class frequency = 12
 Model class = 60 – 80
 f1  f0 
Mode = l   h
 2 f1  f0  f2 
 12  10 
 60     20
 2  12  10  6 
 65

www.vedantu.com 23 / 23
CBSE-XII-2017 EXAMINATION CBSE-X-2009 EXAMINATION

MATHEMATICS
Series: HRL Paper & Solution Code: 30/1
Time: 3 Hrs. Max. Marks: 80
General Instuctions :
(i) All questions are compulsory.
(ii) The question paper consists of 30 questions divided into four sections — A, B, C and D. Section A
comprises of ten questions of 1 mark each, Section B comprises of five questions of 2 marks each, Section C
comprises of ten questions of 3 marks each and Section D comprises of five questions of 6 marks each
(iii) All questions in Section A are to be answered in one word, one sentence or as per the exact requirement
of the question.
(iv) There is no overall choke. However, an internal choice has been provided in one question of 2 marks
each, three questions of 3 marks each and two questions of 6 marks each. You have to attempt only one of
the alternatives in all such questions.
(v) In question on construction, the drawings should be neat and exactly as per the given measurement.
(vi) Use of calculators is not permitted.

SECTION – A
Questions number 1 to 10 carry 1 mark each.
1. Find the ( HCF  LCM ] for the numbers 100 and 190.
Solution:
Given two numbers 100 and 190
Product of 100 and 190
HCF  LCM  100  190
 19000

2. If 1 is a zero of the polynomial p(x) = ax2 - 3 (a - 1)x - 1, then find the value of a.
Solution:
If x = 1 is the zero of the polynomial p(x)  ax 2  3(a  1)x  1
Then p(1)  0
a(1)2  3(a  1)  1  0
2a  2  0
a 1

3. In  LMN,  L = 50° and  N = 60°. If  LMN ~  PQR, then find  Q.


Solution:

www.vedantu.com 1 / 23
CBSE-X-2009 EXAMINATION

Given LMN ~ PQR


In similar triangles, corresponding angles are equal.
L  P
M  Q x
N  R
In LMN
L  M  N  180
M  180  50  60
M  70
Q  70

4. If sec2 sec2  sin sin  k, then find the value of k.


Solution:
sec2 (1  sin )(1  sin )  k
 sec2  sin2  κ
 sec2.cos2   κ
cos2 
 k
cos2 
 k 1

5. If the diameter of a semicircular protractor is 14 cm, then find its perimeter.


Solution:

Given diameter of semicircular protractor (AB) = 14cm


d 
Perimeter of a semicircle      d
2
 14 
 Perimeter of protractor      14
 2 
22 14
   14
7 2
 36cm

6. Find the number of solutions of the following pair of linear equations :

www.vedantu.com 2 / 23
CBSE-X-2009 EXAMINATION

x + 2y — 8 = 0
2x + 4y = 16
Solution:

x  2y  8  0
2x  4y  16  0
For any pair linear equations
a1 x  b1y  c1  0
a2 x  b2 y  c2  0
a1 b1 c1
If   , then
a2 b2 c2
There exists infinite solutions
a 1 b 2 c 8
Here 1  , 1  , 1 
a2 2 b2 4 c2 16
a b c 1
 1 1 1
a2 b2 c2 2
 Lines are coincident and will have infinite solutions.

7. Find the discriminant of the quadratic equation


3 3x 2  10 x  3  0.
Solution:
3 3x 2  10 x  3  0
Discriminant for ax2  bx  c  0 will be b2  4ac
 For the given quadratic equation
 (10)2  4(3 3)( 3)
 100  36
 64

4
8. If , a, 2 are three consecutive terms of an A.P., then find the value of a.
5
Solution:
4
Given , a , 2 are in A.P.
5
4
a   2  a
5
4
 2a   2
5
14
2a 
5
7
a 
5

www.vedantu.com 3 / 23
CBSE-X-2009 EXAMINATION

9. In Figure 1,  ABC is circumscribing a circle. Find the length of BC.

Solution:

Given BR = 3cm, AR = 4cm & AC = 11 cm


BP  BR
AR  AQ
CP  CQ
{Lengths of tangents to circle from external point will be equal}
 AQ  4 cm and BP  3cm
As AC = 11 cm
QC + AQ = 11cm
 QC  7cm
PC  7cm
We know BC  BP  PC
BC  3  7
BC  10cm

10. Two coins are tossed simultaneously. Find the probability of getting exactly one head.
Solution:
Two coins are tossed simultaneously
Total possible outcomes  {HH , HT , TH , TT }
No. of total outcomes = 4

www.vedantu.com 4 / 23
CBSE-X-2009 EXAMINATION

Favourable outcomes for getting exactly


One head = {Ht , TH}
2 1
Probability = 
4 2

SECTION B
Questions number 11 to 15 carry 2 marks each.

11. Find all the zeroes of the polynomial x3 + 3x2 — 2x — 6, if two of its zeroes are  2 and 2.
Solution:
x3  3x2  2x  6  0
Given two zeros are  2, 2
Sum of all zeros = -3
Let the third zero be x
 
 x  2   2  3
x  3
 All zeros will be 3,  2, 2

12. Which term of the A.P. 3, 15, 27, 39, ... will be 120 more than its 21st term ?
Solution:
Given an A.P. 3, 15, 27, 39, ………
Lets say nth term is 120 more than 12st term
Tn  120  T21
a  (n  1)d  120  (a  20d )
(n  1)12  120  20  12
n  1  30
n  31
 31st term is 120 more than 12st term.

13. In Figure 2,  ABD is a right triangle, right-angled at A and AC  BD. Prove that AB2 = BC . BD.

Solution:

www.vedantu.com 5 / 23
CBSE-X-2009 EXAMINATION

In ABC
AB2  AD2  BD2 .......(1)
In ABC
AC 2  BC 2  AB2 ......(2)
In ACD
AC 2  CD2  AD2 ........(3)
Subtracting (3) from (2)
AB2  AD2  BC 2  CD2 ..........(4)
Adding (1) and (4)
2 AB2  BD2  BC 2  CD2
2 AB2   BC  CD   BC 2  CD2
2

2 AB2  BC 2  CD2  2BC .CD  BC 2  CD2


AB2  BC (BC  CD)
AB2  BC .BD

15 (2  2sin )(1  sin )


14. If cot   , then evaluate .
8 (1  cos )(2  2cos )
OR
Find the value of tan 60°, geometrically.
Solution:
15
cot  
8
(2  2sin )(1  sin ) 2(1  sin2 )

(1  cos)(2  2cos ) 2(1  cos2 )
cos2 
 2
sin 
 cot2 
225

64
OR

www.vedantu.com 6 / 23
CBSE-X-2009 EXAMINATION

Consider a right triangle with A  60, AC  1 cm and AB = 2cm


BC
tan60 
AC
By Pythagoras theorem
BC 2  4  1  BC  3
tan60  3

15. If the points A (4, 3) and B (x, 5) are on the circle with the centre O (2, 3), find the value of x.
Solution:

OA = OB (radii)
OA  (2  4)2  (0)2  2
OB  (2  x)2  (3  5)2  (2  x)2  4
 2  (2  x)2  4
4  (2  x)2  4
 x 2
SECTION C
Questions number 16 to 25 carry 3 marks each.
16. Prove that 3  2 is an irrational number.
Solution:
Lets assume 3  2 is a rational number
p
3  2  {p, q are integers and q  0}
q

www.vedantu.com 7 / 23
CBSE-X-2009 EXAMINATION

p
 2 3
q
p  3q
 2
q
P  2q
is rational number but we know 2 is an irrational.
q
Irrational  rational
3  2 is not a rational number.

17. Solve for x and y


ax by
 ab
b a
ax  by  2ab
OR
8
The sum of two numbers is 8. Determine the numbers if the sum of their . reciprocals is .
15
Solution:
ax by
  a  b ....(1)
. b a
ax  by  2ab .......(2)
1
Multiply (2) with and subtract (1) from (2)
b
a
x  y  2a
b
a by
  ab
b a
 
 ba 
y  ab
 a 
y  a
Substituting y = -a in (1)
a b
x  (a)  a  b
b a
a
x a
b
xb
 x  b and y  a
OR
Lets assume 2 numbers are x, y
Given x  y  8  x  8  y .....(1)

www.vedantu.com 8 / 23
CBSE-X-2009 EXAMINATION

1 1 8
 
x y 15
xy 8 8 8
  
xy 15 xy 15
 xy  15
From (1) xy  y(8  y)  15
 y 2  8y  15  0
y  3, 5  x  5,3
 The numbers are 3 and 5

18. The sum of first six terms of an arithmetic progression is 42. The ratio of its 10th term to its 30th term is
1 : 3. Calculate the first and the thirteenth term of the A.P.
Solution:
Lets say first term of given A.P. = a
Common difference = d
Sum of first six terms = 42
6
  2a  5d   42
2
2a  5d  14 .....(1)
Also given T10 : T30  1: 3
a  9d 1
 
a  29d 3
3a  27d  a  29d
 2a  2d
 a  d ......(2)
Substituting (2) in (1)
 2a  5a  14
a  2 and d  2
T13  a  12d
 2  24
T13  26

19. Evaluate :
2 2 5
cosec2 58  cot58 tan32  tan13 tan37 tan45 tan53 tan77
3 3 3
Solution:
2 2 5
cos ec 2 58  cot58 tan32  tan13 tan37 tan45 tan58
3 3 3

www.vedantu.com 9 / 23
CBSE-X-2009 EXAMINATION

cot58  tan(90  58)  tan32


tan77  cot(90  77)  cot13
tan53  cot(90  53)  cot37
tan45  1
Substituting in the given expression
2 2 5
 cosec2 58  cot2 58 
3 3 3
2 5
 cosec2 58  cot2 58 
3 3
2 5
 1 
3 3
 1

20. Draw a right triangle in which sides (other than hypotenuse) are of lengths 8 cm and 6 cm. Then
3
construct another triangle whose sides are times the corresponding sides of the first triangle.
4
Solution:

Given ABC which is a right angled triangle B  90


Steps:
1. Draw line segment BC = 8cm, draw a ray BX making 90 with BC
2. Draw an arc with radius 6cm from B so that it cuts BX at A
3. Now join AC to form ABC

4. Draw a ray by making an acute angle with BC, opposite to vertex A


5. Locate 4 points B1, B2, B3, B4, on by such that BB1 = B1B2 = B2B3 = B3B4
6. Join B4C and now draw a line from B3 parallel to B4C so that it cuts BC at C’

www.vedantu.com 10 / 23
CBSE-X-2009 EXAMINATION

7. From C’ draw a line parallel to AC and cuts AB at A’


8. A ' BC ' is the required triangle

Justification:
ABC and A ' BC '
ABC  A ' BC ' (Common)
ACB  A ' C ' B (Corresponding angles)
By AA criterion ABC ~ A ' BC '
AB BC AC
  
A ' B BC ' A ' C '
In BB4C and BB3C '
B4C ||B3C ' [By construction]
BB4C ~ BB3C '
BB4 BC B4C
  
BB3 BC ' B3C '
BB 4
We know that 4 
BB3 3
A ' B BC ' A ' C ' 3
   
AB BC AC 4

1
21. In Figure 3, AD  BC and BD = CD. Prove that
3
2 CA2 = 2 AB2 + BC2

OR
In Figure 4, M is mid-point of side CD of a parallelogram ABCD. The line BM is drawn intersecting AC at L
and AD produced at E. Prove that EL = 2 BL.

www.vedantu.com 11 / 23
CBSE-X-2009 EXAMINATION

Solution:

1
BD  CD;
3
BD  CD  BC
3
CD  BC
4
1
BD  BC
4
In right ACD,
AC2 = AD2 + CD2 …………(1) (Pythagoras Theorem)
In right ABD,
AB2  AD2  BD2 ....(2) (Pythagoras Theorem)
From (1) and (2), we get
AC2 = AB2 – BD2 + CD2
2 2
 BC   3BC 
 AC  AB  
2
  
 4   4 
BC 2 9BC 2
 AC  AB 
2 2

16 16
9BC  BC 2
2
 AC 2  AB2 
16
2
8BC
 AC 2  AB2 
16
BC 2
 AC 2  AB2 
2
2 AB  BC 2
2
 AC 2 
2

www.vedantu.com 12 / 23
CBSE-X-2009 EXAMINATION

OR

In DME and CMD


EDM  MCB [Alternate angles]
DM = CM [mis midpoint of CD]
DME  BMC [VOA]
By ASA congruency DME  CMB
By CPCT
BM = ME
DE = BC
Now in
ALE and BLC
ALE  BLC [VOA]
LAE  LCB [Alternate angles]
By AA similarly
ALE ~ BLC
AE AL LE
  
BC BL LC
EL AE
 
BL BC
EL AD  DE
 
BL BC
EL BC  BC
 
BL BC
 EL  2BL

22. Find the ratio in which the point (2, y) divides the line segment joining the points A (-2, 2) and B (3, 7).
Also find the value of y.
Solution:

Lets say ratio = m : n

www.vedantu.com 13 / 23
CBSE-X-2009 EXAMINATION

 3m  2n 2n  7m 
(2, y)   , 
 mn mn 
3m  2n
2
mn
2m  2n  3m  2n
m: n  4:1
2  7 4
y
5
30
y
5
y 6
 p(2,6)

23. Find the area of the quadrilateral ABCD whose vertices are A (— 4, — 2), B (— 3, — 6), C (3, — 2) and D
(2, 3).
Solution:

Joining AC
Area of Quadrilateral ABCD
 ar(ABC)  ar(ADC)
1
Area of triangle ABC =  4(5  (2))  (3)(2  (2))  3(2  (5))
2
1
  4(5  2)  (3)(2  2)  3(2  5)
2
1
 [4(3)  3(0)  3(3)]
2
1
 [12  0  9]
2
1
 [21] square units
2
1
Area of triangle ADC =  4(3  (2))  2(2  (2))  3(2  3)
2

www.vedantu.com 14 / 23
CBSE-X-2009 EXAMINATION

1
  4(3  2)  3(2  2)  3(2  3)
2
1
  4(5)  3(0)  3(5)
2
1
  20  0  15
2
1 35
 35  sq. units
2 2
21 35
 Area of quadrilateral (ABCD) = 
2 2
 28 sq.units

24. The area of an equilateral triangle is 49 3 cm2. Taking each angular point as centre, circles are drawn
with radius equal to half the length of the side of the triangle. Find the area of triangle not included in the
circles. [Take 3 = 1.73]
OR
Figure 5 shows a decorative block which is made of two solids — a cube and a hemisphere. The base of the
block is a cube with edge 5 cm and the hemisphere, fixed on the top, has a diameter of 4.2 cm. Find the
22
total 2 surface area of the block. [Take   ]
7

Solution:
Let be the side of equilateral triangle.
3a2
 49 3;
4
a2  49* 4;
a= 7 * 2 = 14 cm;
Radius of circle = 14/2 = 7 cm

www.vedantu.com 15 / 23
CBSE-X-2009 EXAMINATION

Area of the first circle occupied by triangle = area of sector


1 1 60 77 2
 r 2  *72 *  *  cm
2 2 180 3
Area of all the 3 sectors = 77/3 * 3 = 77 cm2
Area of triangle not Included In the circle = area of triangle- area of all the 3 sectors
 49 3 — 77 — 7. 87cm2
OR

The total surface area of the cube = 6  (edge)2 = 6  5  5 cm2 = 150 cm2
Note that the part of the cube where the hemisphere is attached is not Included in the surface area.
So, the surface area of the block = TSA of cube — base area of hemisphere + CSA of hemisphere
 150  r 2  2r 2  (150  r 2 )cm2
 22 4.2 4.2  2
 150 cm2      cm
 7 2 2 
 150  13.86  cm2  163.86 cm2

25. Two dice are thrown simultaneously. What is the probability that
(i) 5 will not come up on either of them ?
(ii) 5 will come up on at least one ?
(iii) 5 will come up at both dice ?
Solution:
Total outcomes =    
(i). Total outcomes when 5 comes up on either dice are (5, 1) (5, 2) (5, 3) (5, 4) (5, 5) (5, 6) (6, 5) (4, 5) (3, 5)
(2, 5) (1, 5)
11
P (5 will come up on either time)
36
11
P (5 will not come up) = 1 
36
25

36
11
(ii) P (5 will come at least once) =
36
1
(iii) P (5 will come up on both dice = )
36

SECTION D
Questions number 26 to 30 carry 6 marks each.

www.vedantu.com 16 / 23
CBSE-X-2009 EXAMINATION

26. Solve the following equation for x :


9x2 — 9 (a + b) x + (2a2 + 5ab + 2b2) = 0
OR
If (-5) is a root of the quadratic equation 2x + px — 15 = 0 and the quadratic equation p (x2 + x) + k = 0 has
2

equal roots, then find the values of p and k.


Solution:
9 x 2  9(a  b)x  (2a2  5ab  2b2 )  0
Discriminant
D  81(a  b)2  36  2a2  5ab  2b2 
D  9 9a2  9b2  18ab  8a2  8b2  20ab 
D  9 a2  b2  2ab 

 D  9(a  b)2

9(a  b)  9(a  b)2


x 
2 9
9(a  b)  3(a  b)
x
18
3a  3b  a  b 3a  3b  a  b
x ,
6 6
2a  b a  2b
x  ;
3 3
OR
2
-5 is root of 2x + 9x – 15 =0
2(5)2  p(5)  15  0
10  p  3  0
p  7
p(x2 + x) + k = 0 has equal roots
7x2 + 7x + k = 0 [as we know p =7]
 Discriminant = 0
D  49  28k
28k  49
7
k
4

27. Prove that the lengths of the tangents drawn from an external point to a circle are equal.
Using the above theorem prove that:
If quadrilateral ABCD is circumscribing a circle, then
AB + CD = AD + BC.
Solution:
PT and TQ are two tangent drawn from an external pant T to the circle C (O, r)

www.vedantu.com 17 / 23
CBSE-X-2009 EXAMINATION

To prove: 1. PT = TQ
2.  OTP =  OTQ
Construction Join OT
Proof: We know that, a tangent to circle is perpendicular to the radius through the point of contact
  OPT =  OQT = 90
In  OPT and  OQT
OT = OT (Common)
OP = OQ (Radius of the circle)
 OPT =  OQT (90 )
  OPT

 OQT (RHS congruence criterion)
 PT = TQ and  OTP =  OTQ (CPCT)
PT = TQ
The lengths of the tangents drawn from an external point to a circle are equal.
 OTP =  OTQ
Centre lies on the bisector of the angle between the two tangents.

Let AB touches the circle at P.BC touches the circle at Q.DC touches the circle at R.AD. touches the circle at
S
THEN, PB = QB ( Length of the tangents drawn from the external point are always equal)
QC =RC'
AP : AS
DS = DP
NOW, AB + CD
= AP + PB+DR+RC
= AS+QB+DS+CQ
= AS + DS + OB + CQ

www.vedantu.com 18 / 23
CBSE-X-2009 EXAMINATION

= AD + BC
HENCE PROVED

28. An aeroplane when flying at a height of 3125 m from the ground passes vertically below another plane
at an instant when the angles of elevation of the two planes from the same point on the ground are 30°
and 60° respectively. Find the distance between the two planes at that instant.
Solution:

Let the distance between the two planes = h m


Given that: AD = 3125 m and
ABC  60
ABD  30
In
ABD
AD
tan30 
AB
1 3125

3 AB
 AB  3125 3 ..........(1)
ABC
AC
tan60 
AB
AD  DC
3
AB
3125  h
3
AB
3125  h
 AB  ...........(2)
3
Equating equation (1) and (2), we have

www.vedantu.com 19 / 23
CBSE-X-2009 EXAMINATION

3125  h
 3125 3
3
h  3125  3  3125
h  6250
Hence, distance between the two planes is 6250 m.

29. A juice seller serves his customers using a glass as shown in Figure 6. The inner diameter of the
cylindrical glass is 5 cm, but the bottom of the glass has a hemispherical portion raised which reduces the
capacity of the glass. If the height of the glass is 10 cm, find the apparent capacity of the glass and its actual
capacity. (Use  = 3.14)

OR
A cylindrical vessel with internal diameter 10 cm and height 10.5 cm is full of water. A solid cone of base
diameter 7 cm and height 6 cm is completely immersed in water. Find the volume of
(i) water displaced out of the cylindrical vessel.
(ii) water left in the cylindrical vessel.
22
[Take   ]
7
Solution:
Apparent capacity of the glass = Volume of cylinder

Actual capacity of the glass = Volume of cylinder – Volume of hemisphere


Volume of the cylindrical glass = r 2h
 3.14  (2.5)2  10
 3.14  2.5  2.5  10
 3.14  6.25  10
 196.25 cm3

www.vedantu.com 20 / 23
CBSE-X-2009 EXAMINATION

2
Volume of hemisphere  r 3
3
2
 (2.5)3  32.7 cm3
3
Apparent capacity of the glass = Volume of cylinder = 196.25 cm3
Actual capacity of the glass
= Total volume of cylinder — volume of hemisphere
= 196.25 — 32.7
= 163.54cm3
Hence, apparent capacity = 196.25cm3
Actual capacity of the glass = 163.54cm3
OR
29.

Given, internal diameter of the cylinder =10 cm


Internal radius of the cylinder = 5cm
and height of the cylinder = 10.5 cm
Similarly, diameter of the cone = 7 cm
radius of the cone = 3.5 cm and Height of the cone = 6cm
i) Volume of water displaced out of cyindrical vessel = volume of cone
1
 r 2h
3
1 22
   3.5  3.5  6  77 cm3
3 7
ii) Volume of water left In the cylindrical vessel = volume of cylinder - volume of cone
 R2H  Volume of cone
22
=  5  5  10.5  77
7
= 825 —77 =748cm3

30. During the medical check-up of 35 students of a class their weights were recorded as follows:
Weight (in Number of students
kg)
38 – 40 3
40 – 42 2
42 – 44 4
44 – 46 5

www.vedantu.com 21 / 23
CBSE-X-2009 EXAMINATION

46 – 48 14
48 – 50 4
50 – 52 3
Draw a less than type and a more than type ogive from the given data. Hence obtain the median weight
from the graph.
Solution:

Weight cumulative (more than


type)
More than 38 35
More than 40 32
More than 42 30
More than 44 26
More than 46 21
More than 48 7
More than 50 3
More than 52 0

Weight (in kg) Number of students


Upper class limits (cumulative frequency)
Less than 38 0
Less than 40 3
Less than 42 5
Less than 44 9
Less than 46 14
Less than 48 28
Less than 50 32
More than 52 35

Taking upper class limits on x-axis and their respective cumulative frequency on y-axis its give can be
drawn as follows:

Here, n = 35
So,

www.vedantu.com 22 / 23
CBSE-X-2009 EXAMINATION

n
= 17.5
2
Mark the point A whose ordinate is 17.5 and its x-coordinate is 46.5. Therefore, median of this data is 46.5.
It can be observed that the difference between two consecutive upper class limits is 2. The class marks
with their respective frequencies are obtained as below:
Weight (In kg) Frequency (f) Cumulative,
frequency
Less than 38 0 0
38 – 40 3-0=3 3
40 – 42 5-3=2 5
42-44 9-5=4 9
44 – 46 14 – 9 = 5 14
46- 48 28- 14 = 14 28
48-50 32 -28 = 4 32
50 – 52 35 – 32 = 3 35
Total (n) 35
The Cumulative frequency just greater than
n 35 
 i.e.,  17.5 
2 2 
is 28. Belonging to class interval 46 - 48
Medan class = 46 - 48
Lower class lima (l) of median class = 46
Frequency (f) of median class = 14
Cumulative frequency (cf) of class preceding median class = 14
Class size (h) = 2
n 
 2  cf 
Median  l    h
 f 
 
 17.5  14 
 46   2
 14 
3.5
 46 
7
 46.5
=46.5
Therefore, median of this data is 46.5
Hence, the value of median is verified.

www.vedantu.com 23 / 23
CBSE-XII-2017 EXAMINATION CBSE-X-2010 EXAMINATION

MATHEMATICS
Series: LRH/1 Paper & Solution Code: 30/1/1
Time: 3 Hrs. Max. Marks: 80
General Instuctions :
(i) All questions are compulsory.
(ii) The question paper consists of 30 questions divided into four sections - A, B, C and D. Section A
comprises often questions of 1 mark each, Section B comprises of five questions of 2 marks each, Section C
comprises often questions of 3 marks each and Section D comprises of five questions of 6 marks each.
(iii) All questions in Section A are to be answered in one word, one sentence. or as per the exact
requirement of the question.
(iv) There is no overall choice. However, an internal choice has been provided in one question of 2 marks
each. three questions of 3 marks each and two questions of 6 marks each. You have to attempt only one of
the alternative in all such questions.
(v) In question on construction, the drawings should be neat and exactly as per the given measurements.
(vi) Use of calculators is not permitted.

SECTION – A
Question Numbers 1 to 10 carry 1 mark each.
441
1. Has the rational number 2 7 2 a terminating or a non-terminating decimal representation?
2 .5 .7
Solution:
441
Given a rational number 2 7 2
2 .5 .7
Since the denominator is not in the form of 2m×5n. the rational number has non terminating repeating
decimal expansion

2. If  ,  are the zeroes of a polynomial, such that    6 and   4, then write the polynomial.
Solution:
Given ∝ and β are the zeroes of quadratic polynomial with ∝+β = 6
∝β = 4
Quadratic polynomial =x2-6x+4
=x2-6x+4

3. If the sum of first p terms of an A.P., is ap2 + bp, find its common difference.
Solution:
Given an AP which has sum of first P terms =ap2+bp
Lets say first term=k & common difference =d
p
∴ap2+bp=  2k   p  1 d 
2
2ap  2b  2k   p  1 d
2b  2ap   2k  d   pd

www.vedantu.com 1 / 23
CBSE-X-2010 EXAMINATION

Comparing terms (r) both sides


 2a  d
2k  d  2b
2k  2b  2a
k  ab
Common difference =2a
First term = a + b

4. In Fig. 1, S and T are points on the sides PQ and PR, respectively of  PQR, such that PT = 2 cm, TR = 4 cm
and ST is parallel to QR. Find the ratio of the areas of  PST and  PQR.

Solution:

Given : PT  2cm,TR  cm
ST 11QR
solution :AS itisgiven that ST 11 QR
PST PQR

 PQ
PS PT ST
 
PR QR
ar  PST 
2 2
 PS   PT   ST 
2

Also,      
ar  PQR   PQ   TR   QR 
ar  PST  2
 PT   2 
2

    
ar  PQR   TR   4 
ratio  1 : 4

www.vedantu.com 2 / 23
CBSE-X-2010 EXAMINATION

5. In fig. 2,  AHK is similar to  ABC. If AK = 10 cm, BC = 3.5 cm and HK = 7 cm, find AC.

Solution:

Given ∆AHK ~ ∆ABC


AH HK AK
  
AB BC AC
Also we know AK=10cm, BC=3.5cm and HK=7cm
 AK  HK
AC BC
 10  7
AC 3.5
AC  5cm

3  1
6. lf 3x = cosec 9 and  cot , find the value of 3 x2  2 .
x  x 
Solution:
Given 3x=cosec θ
3
 cot 
x
We know that cosec2 θ-cot2θ=1
9
 9x2  2  1
x
 1 
 9  x2  2   1
 x 
 1 1
 3  x2  2  
 x 3

www.vedantu.com 3 / 23
CBSE-X-2010 EXAMINATION

7. If P(2, p) is the mid-point of the line segment joining the points A(6, -5) and B(-2, 11). find the value of p.
Solution:

Given P is midpoint of AB
 6  2 5  11 
∴(2,P)=  , 
 2 2 
 2 , p    2 ,3 
 p3

8. If A(1, 2), B(4, 3) and C(6, 6) are the three vertices of a parallelogram ABCD, find the coordinates of the
fourth vertex D.
Solution:

Given ABCD is a parallelogram


In a parallelogram diagonals each other.
 0 is midpoint to AC and BD
1 6 2  6 
0   , 
 2 2 
1 
0   ,4 
2 
Lets say D=(x,y)
 x  4 3 4
0 , 
 2 2 
7 
But we know 0   , 4 
2 
7 x4 3 y
  and 4 
2 2 2
x  3, y  5
 D   3, 5 

www.vedantu.com 4 / 23
CBSE-X-2010 EXAMINATION

9. The slant height of a frustum of a cone is 4 cm and the perimeters (circumferences) of its circular ends
are 18 cm and 6 cm. Find the curved surface area of the frustum.
22
[Use   ]
7
Solution:

Given slant height (ℓ)=4cm


Perimeters of circular ends:
2πr=6cm
2πR=18cm
C.S.A=πℓ(r+R)=4×12
=48cm2

10. A card is drawn at random from a well shuffled pack of 52 playing cards. Find the probability of getting
a red face card.
Solution:
A card is drawn from 52 card total no of possible outcomes =52
No of face cards =12
No of Red face cards =6
6
Probability of drawing =
52
3
A Red face card =
26

Section B
Question Numbers 11 to 15 carry 2 marks each.
11. If two zeroes of the polynomial x3 -4x2 - 3x + 12 are 3 and  3 , then find its third zero.
Solution:
Given a polynomial
X2-4x2-3x+12=0
Sum of all the zeroes of polynomial =(-4)=4
Given two zeroes of 3 , 3
Say the third zero = ∝
 3 3  4
4  third zero is 4

www.vedantu.com 5 / 23
CBSE-X-2010 EXAMINATION

12. Find the value of k for which the following pair of linear equations have infinitely many solutions
2x + 3y = 7; (k-1)x + (k + 2)y = 3k
Solution:
2x+3y=7
(k-1)x+(k+2)y =3k
For this pair of linear equations to have infinitely many solution, they need to be concident
2 3 7
  
k  1 k  2 3k
Upon solving we get
k 7

13. In an A.P., the first term is 2, the last term is 29 and sum of the terms is 155. Find the common
difference of the A.P.
Solution:
Given an AP with first term (a)=2
Last term (ℓ)=29
Sum of the term =155
Common difference (d)=?
n
Sum of the n term =  a  
2
n
 155   2  29 
2
 n  10
Last term which is Tn
= a +(n-1)d
= a +(9)d
∴29=2+9d
d 3
Common difference=3

14. If all the sides of a parallelogram touch a circle, show that the parallelogram is a rhombus.
Solution:

www.vedantu.com 6 / 23
CBSE-X-2010 EXAMINATION

Given a parallelogram PQRS in which a circle is inscribed


We know PQ=RS
QR=PS
DP=PA {tangents to the circle from external point have equal length}
QA=BQ
BR=RC
DS=CS
Adding above four equations
DP+QA+BR+DS=PA+BQ+RC+CS
(DP+DS)+(QA+BR)=(PA+QA)+(SR)
2QR=2(PQ)
∴PQ=QR
PQ=QR=RS=QS
∴PQRS is a rhombus

15. Without using trigonometric tables, find the value of the following expression
sec(90  ).cosec  tan(90-)cot+ cos2 25  cos2 65
3tan27 tan63
Or
Find the value of cosec 30°, geometrically.
Solution:

www.vedantu.com 7 / 23
CBSE-X-2010 EXAMINATION

sec  90    cos ec  tan  90    cot   cos 2 25  cos 2 65


3 tan 27.tan 63
cos ec 2  cot 2   sin  90  25    cos 2 65
2


3 tan 27.tan 63
1  sin 65  cos 2 65
2

3 cot  90  27  tan 63
2

3 cot 63 tan 63
2

3
15)(or)

∠A=∠B=∠C=600
Draw AD⊥BC
In ∆ABD and ∆ACD,
AD=AD (common)
∠ADB=∠ADC (900)
AB=AC (∆ABC is equalateral∆)
∴∆ABD≥∆ACD (RHS congruence criterion)
BD=DC (C.P.C.t)
∠BAD=∠CAD (C.P.C.t)
2a 600
BD= =a and ∠BAD= =300
2 2
In right ∆ABD,
BD  Perpendicular 
Sin 300=  sin   
AB  Hypotenuse 
a
 sin 300 
2a
1 1
 sin 300   2
2 sin 300
 cos ec 300  2

Section C
Question Numbers 16 to 25 carry 3 marks each.
16. Prove that 2  3 5 is an irrational number.
Solution:
lets assume 2-3 5 is a rational number

www.vedantu.com 8 / 23
CBSE-X-2010 EXAMINATION

p
 23 5  , where p,q are int egers pq  0 
q
p
2 3 5
q
2q  p
  5
3q
2q  p
is a rational number but we also know 5 is an irrational
3q
Rational ± irrational
 our assumption is wrong
∴2-3 5 is an irrational number

17. The sum of numerator and denominator of a fraction is 3 less than twice the denominator. If each of
1
the numerator and denominator is decreased by 1, the fraction becomes . Find the fraction.
2
Or
Solve the following pair of equations
4 6
 3y  8,  4 y  5
x x
Solution:
lets say numerator=x
Denominator=y
Given x+y=2y-3
 x  y  3  0  1
From the next condition
x 1 1

y 1 2
2x  y  1  o   2
Solving (1) and (2)
X=4
Y=7
4
∴ fraction=
7
17)(or)
4
 3 y  8  1
x
6
 4 y  5   2 
x
Multiplying 4 to (1) and 3 to (2)

www.vedantu.com 9 / 23
CBSE-X-2010 EXAMINATION

16
 12 y  32
x
18
 12 y  15
x
34
 17
x
x2
Substitute
X in (1)
2+3y=8
3y=6
Y=2
∴x=2
Y=2

18. In an A.P., the sum of first ten terms is -150 and the sum of its next ten terms is -550. Find the A.P.
Solution:
sum of first ten terms =-150
Sum of next ten terms =550
Lets say first term of A.P=a
Common difference =d
10
Sum of first ten terms =  2a  9d 
2
150  5  2a  9d 
2a  9d  30  1
For sum of next ten terms the first term would be T11=a+10d
10
 550   2  a  10d   9d 
2
 110  2a  29d   2 
Solving (1) and (2)
D= - 4
A=3
∴A.P will be 3,-1,-5,-9,-13,………..

19. In Fig. 3, ABC is a right triangle, right angled at C and D is the mid-point of BC. Prove that AB2 = 4AD2 -
3AC2.

www.vedantu.com 10 / 23
CBSE-X-2010 EXAMINATION

Solution:

Given that BD = CD
AC ⊥ BC
In ∆ABC AB2 =BC2 +AC2
AB2=(BD+CD)2+AC2
AB2 =(2CD)2+AC2
AB2=4CD2+AC2-----(1)
In ∆ADC AD2=CD2+AC2
CD2=AD2-AC2
Substituting CD2 in-------(2)
 AB2 =4AD2 – 4AC2+AC2
 AB2 =4AD2 -3AC2
Hence proved

20. Prove the following:


tan A cot A
  1  tan A  cot A
1  cot A 1  tan A
OR
Prove the following:
1
 cosec A  sin A  sec A  cos A  
tan A  cot A

www.vedantu.com 11 / 23
CBSE-X-2010 EXAMINATION

Solution:
Proved that
tan A cot A
  1  tan A  cot A
1- cot A 1- tan A
tan A cot A
L.H.S 
1- cot A 1- tan A
tan A cot A
Þ 
 1  1- tan A
1- 
 tan A 
- tan 2 A cot A
Þ 
1- tan A 1- tan A
1
1  tan
  tan 2 A  cot A 
1  1 
   tan A 
2

1  tan A  tan A 
1  tan 3 A

tan A 1  tan A 
1  tan A  1  tan 2 A  tan A 

tan A 1  tan A 
 cot A  tan A  1
Hence proved
OR
Proved that :
1
(cosec A- sin A) (sec A- cos A)=
tan A  cot A
 1  1 
LHS   sin A   cos A 
 sin A  cos A 


1  sin A 1  cos A 
2 2

sin A cos A
=sin A cos A
Multiply & divide by 2

www.vedantu.com 12 / 23
CBSE-X-2010 EXAMINATION

2siA cos A

2
sin 2A

2
2 tan A

2 1  tan 2 A 
1

1
 tan A
tan A
1

tan A  cot A
Hence proved

21. Construct a triangle ABC in which BC = 8 cm,  B = 45° and  C = 30°.


3
Construct another triangle similar to  ABC such that its sides are of the corresponding sides of  ABC.
4
Solution:

steps :
1) Draw a ∆ABC with BC =8cm, ∠B=450 &∠C=300
2) Draw a ray BX making acute angle with BC on the opposite side of vertex A
3) mark four points B1,B2,B3B4 on BX such that BB1=B1B2=B2B3=B3B4
4) join B4toc and draw a line parallel to B4C from B3 such it luts BC at C1
5) form C1draw another line parallel to AC such that it cuts AC at A1.
6) ∆A1BC1 is the required triangle
Tustification :

www.vedantu.com 13 / 23
CBSE-X-2010 EXAMINATION

In ∆ABC and ∆A1BC1


∠ABC = ∠A1BC1
∠ACB = ∠A1C1B
∴by AA criterion ∆ABC ~ ∆A1BC1
AB BC AC
 1  1 1 1
A B BC AC
In ∆BB4C and ∆BB3C1
B4C || B3C1
∴∆BB4C ~ ∆BB3C1
BC BB4 4
  
BC1 BB3 3
AB BC AC 4
 1
 1
 1 1 
A B BC AC 3
3
A1 B  AB
4
3
BC1  BC
4
3
A1C1  BC.
4

AP 1
22. Point P divides the line segment joining the points A(2, 1) and B(5, -8) such that  . If P lies on the
AB 3
line 2x -y + k = 0, find the value of k.
Solution:

Given :

www.vedantu.com 14 / 23
CBSE-X-2010 EXAMINATION

AP 1 AP 1
   PB  2AP  AP : PB  1: 2
AB 3 AP  PB 3
by sec tion formula
 2 2  5 2 8 
P   , 
 3 3 
P   3, 2 
Also it is given that P lines on 2x-y+k=0
  3  2  k  0
k  4

23. If R(x, y) is a point on the line segment joining the points P(a, b) and Q(b, a), then prove that x + y = a +
b.
Solution:
Since R(x,y) is a point on the line segment joining the point, (a,b) and Q(b,a)
∴P(a,b) ,Q(b,a) and R(x,y) are the collinear.
 Area of PQR  0
Area of triangle whose vertices are  x1 , y1  ,  x 2 , y2  and  x3 , y3 
1
is  x1  y 2  y3   x 2  y3  y1   x 3  y1  y 2  
2
1
 a  a  y   b  y  b   x  b  a    0
2
 a 2  ay  by  b 2  x  b  a   0
 yb  a  x b  a   0
  x  y  b  a    b  a  b  a 
xyab

24. In Fig; 4, the boundary of shaded region consists of four semicircular arcs, two smallest being equal. If
diameter of the largest is 14 cm and that of the smallest is 3.5 cm, calculate the area of the shaded region.
 22 
Use   7 

Or

www.vedantu.com 15 / 23
CBSE-X-2010 EXAMINATION

Find the area of the shaded region in Fig. 5, if AC = 24 cm, BC = 10 cm and O is the centre of the circle. [Use
 = 3.14]

Solution:

Given AB=14cm and AC =BD = 3.5cm


 DC  7cm
Area of shaded region =Area of semicircle AB +Area of semicircle CD -2(Area of semicircle AC)
  14 
2
7
2
   3.5  2 
       2 
2 2  22  2  2  
 
 196 49 49 
Area of shaded region =      86.625cm2
4 2 2 4
24)(or)

AC=24cm, BC= 10cm


AB  242  102

www.vedantu.com 16 / 23
CBSE-X-2010 EXAMINATION

AB=26cm
Diameter of circle =26cm Area of shaded region
=Area of semicircle – Area of ∆ABC
 1
 13   24  10
2

2 2
 145.33cm 2

25. Cards bearing numbers 1, 3, 5, _ _ _ , 35 are kept in a bag. A card is drawn at random from the bag.
Find the probability of getting a card bearing
(i) a prime number less than 15.
(ii) a number divisible by 3 and 5.
Solution:
Total possible outcomes when a cord is drawn = 35
6
i) Total prime numbers from 1 to 15 = 6 probability that a prime numbered card is drawn =
35
ii) Total numbers between 1 to 35 divisible by 3 and 5 = 2
2
∴ probability that when a card is drawn it has a number divisible by 3 &5 =
35

Section D
Question Numbers 26 to 30 carry 6 marks each.
26. Three consecutive positive integers are such that the sum of the square of l the first and the product of
the other two is 46. find the integers.
Or
The difference of squares of two numbers is 88. If the larger number is 5 less than twice the smaller
number. then find the two numbers.
Solution:
Let the required three integers be  x  1 , x and  x  1.
Now,  x  1   x.  x  1   46
2

x  2x  1   x 2  x   46
2

2x 2  x  45  0
2x 2  10x  9x  45  0
2x  x  5   9  x  5   0
 x  5 2x  9   0
x  5 or x  .9 / 2
So, x  5  because it is given that x is a positive int eger 
Thus, the required int egers are  5  1 , i. e. 4,5 and 6.
26)(or)
Let the smaller no. be x and larger no. be y
Y2- x2=88………..(1)

www.vedantu.com 17 / 23
CBSE-X-2010 EXAMINATION

Y=2x-5…………(2)
In equation 1
(2x-5)2-x2=88
4x2-20x+25-x2=88
3x2-20x-63=0
By splitting the middle term,
3x2-27x+7x-63=0
3x(x-9)+7(x-9)
(x-9)(3x+7)
=>x=9 and x =-7/3
Since no. cannot be negative therefore the nos. are smaller no. =9
And larger no. =2x-5 =18-5=13

27. Prove that the ratio of the areas of two similar triangles is equal to the square of the ratio of their
corresponding sides.
Using the above, prove the following
If the areas of two similar triangles are equal, then prove that the triangles are congruent.
Solution:

proof : Given ∆ABC ~ ∆PQR


∠A=∠P, ∠B=∠Q,∠C=∠R
AB BC AC
  1
PQ QR PR
Ratio of areas of ∆ABC & ∆PQR will be
1
ar  ABC  2  BC  AD
  2
ar  PQR  1
 QR  PS
2
In ABD & PQS
B  Q
ADB  PSQ  900
by AA similarity ABD PQS
AB AD BD
    3
PQ PS QS
From (1) and (3) we get

www.vedantu.com 18 / 23
CBSE-X-2010 EXAMINATION

AB BC CA AD
  
PQ QR PR PS
BC AD
   4
QR PS
from  2  and  4 
ar  ABC  BC  BC

ar  PQR  QR.QR
ar  ABC   BC   AB   CA 
2 2 2

   
ar  PQR   QR 2  PQ 2  PR 
2

if ar  ABC   ar  PQR 
then BC  QR
AB  PQ
AC  PR
All corresponding sides equal in these similar triangles
ABC  PQR
Triangle are congruent

28. From the top of a 7 m high building, the angle of elevation of the top of a tower is 60° and the angle of
depression of the foot of the tower is 30°. Find the height of the tower.
Solution:

Let Ab be the building and CD be the tower such that <EAD=600 and <EAC=<ACB =450
Now, in triangle ABC, tan 45 =1 =AB/BC
So,Ab=AE=7m
Again in triangle AED,
Tan 60=root 3=DE/AE
So, DE=AE root 3=7 root 3 m

www.vedantu.com 19 / 23
CBSE-X-2010 EXAMINATION

 h  7 3m

 
Height of tower  h  7  7 1  3 mt

3
29. A milk container is made of metal sheet in the shape of frustum of a cone whose volume is 10459
7
3
cm . The radii of its lower and upper circular ends are 8 cm and 20 cm respectively. Find the cost of metal
sheet used in making the container at the rate of Rs. 1.40 per square centimeter.
22
[Use   ]
7
Or
A toy is in the form of a hemisphere surmounted by a right circular cone of the same base radius as that of
2
the hemisphere. If the radius of base of the cone is 21 cm and its volume is of the volume of the
3
hemisphere, calculate the height of the cone and the surface area of the toy.
22
[Use   ]
7
Solution:

Let the rad11 of lower end of the frustum be r=8 cm


Let the rad11 of upper end of the frustum be R=20 cm
Let the height of the frustum be h cm
Volume of the frustum =

h  R 2  r 2  Rr   10459 
3 73216
3 7 7
Therefore substituting the value of R and r.
 h  202  82  20  8  
22 1 73216
7 3 7
73216 7
h  400  64  160    3
7 22
h  624  9984
9984
h  16 cm
624
Total surface area of the container =

www.vedantu.com 20 / 23
CBSE-X-2010 EXAMINATION

R  r R  r  h 2  r 2
2

22 22
 20  8  20  8  162   82
2

7 7
22 22
  28 122  162   64
7 7
22 22
  28 144  256   64
7 7

22
7
 28  400  64 
22
7
 28  20  64 
22 22
  560  64    624
7 7
Cost of 1 cm square metal sheet is 1. 40 Rs
Cost of required sheet =
22
 624  1.40  2745. 60Rs
7
OR

Let the radius of base of the cone be r=21 cm


Let the height of the cone be h cm
Volume of the cone = 2/3 volume of the hemisphere
1 2 2 2
r h   r 3
3 3 3
4 4
 h  r   21  28 cm
3 3
Surface area of cone = lateral surface area of cone + curved surface area of sphere =
r r 2  h 2  2r 2
22 22
  21  212  282  2   21  21
7 7
 66  441  784  2772
 66  35  2772
 2310  2772  5082cm 2

30. Find the mean, mode and median of the following frequency distribution:
Class: 0-10 10-20 20-30 30-40 40-50 50-60 60-70
Frequency: 4 4 7 10 12 8 5
Solution:

www.vedantu.com 21 / 23
CBSE-X-2010 EXAMINATION

class fi Class Fi x i
mark(xi)
0-10 4 5 20
10-20 4 15 60
20-30 7 25 175
30-40 10 35 350
40-50 12 45 540
50-60 8 55 440
60-70 5 65 325
Efi=50 Efixi=1910

1910
mean   38.2
50
Class frequency cumulative frequency
0-10 4 4
10-20 4 8
20-30 7 15
30-40 10 20
40-50 12 32
50-60 8 40
60-70 5 45
n = 45
n
 22.5
2
Cumulative frequency greater than 22.5 is 32.
Medium class 40-50

n 
  c.f 
m   2 
 f 
 
her  40
n  45
c.f  20, f  32, f  10
 22.5  20 
m]40   10
 32 
mediam  40.781
mod e :
Maximum frequency = 12 so modal class 40-50

www.vedantu.com 22 / 23
CBSE-X-2010 EXAMINATION

 fi  f o 
mod e    
 2fi  f o  f 2 
here  40, h  10
f o  10 f i  12 f 2  8
 12  10 
mod e  40     10
 2  12  10  8 
Mode =40 + 3.33
=43.33

www.vedantu.com 23 / 23
CBSE-XII-2017 EXAMINATION CBSE-X-2012 EXAMINATION

MATHEMATICS
Paper & Solution
Time: 3 Hrs. Max. Marks: 90
General Instuctions :
1. All questions are compulsory.
2. The question paper consists of 34 questions divided into four sections A, B, C, and D.
3. Section A contains of 10 questions of 1 mark each, which are multiple choice type question, Section B
contains of 8 questions of 2 marks each, Section C contains of 10 questions of 3 marks each and Section D
contains of 6 questions of 4 marks each.
4. Question numbers 1 to 8 in Section A are multiple choice questions where you are to select one correct
option out of the given four.
5. There is no overall choice. However, internal choice has been provided in one question of 2 marks, three
questions of 3 marks each and two questions of 4 marks each. You have to attempt only one of the
alternatives in all such questions.
6. Use of calculator is not permitted.

SECTION – A
1. The length of shadow of a tower on the plane ground is 3 times the height of the tower. The angle of
elevation of sun is:
(A) 45
(B) 30
(C) 60
(D) 90
Solution:
Correct answer: B

Let AB be the tower and BC be its shadow. Let be the angle of elevation of the sun.
According to the given information,
BC = 3 AB … (1)
In ABC,
AB AB 1
tan     [Using (1)]
BC 3 AB 3

www.vedantu.com 1 / 24
CBSE-X-2012 EXAMINATION

1
We know that tan 30 =
3
= 30
Hence, the angle of elevation of the sun is 30.

2. If the area of a circle is equal to sum of the areas of two circles of diameters 10 cm and 24 cm, then the
diameter of the larger circle (in cm) is:
(A) 34
(B) 26
(C) 17
(D) 14

Solution:
Correct answer: B
Diameters of two circles are given as 10 cm and 24 cm.
Radius of one circle = r1 = 5 cm
Radius of one circle = r2 = 12 cm
According to the given information,
Area of the larger circle = (r1 ) 2  (r2 ) 2
 (5) 2  (12) 2
 (25  144)
 169
 (13) 2
Radius of larger circle = 13 cm
Hence, the diameter of larger circle = 26 cm

3. If the radius of the base of a right circular cylinder is halved, keeping the height the same, then the ratio
of the volume of the cylinder thus obtained to the volume of original cylinder is:
(A) 1 : 2
(B) 2 : 1
(C) 1 : 4
(D) 4 : 1
Solution:
Correct answer: C
Let the original radius and the height of the cylinder be r and h respectively.
Volume of the original cylinder = r 2 h
r
Radius of the new cylinder =
2
Height of the new cylinder = h

www.vedantu.com 2 / 24
CBSE-X-2012 EXAMINATION

r 2 h
2
r
Volume of the new cylinder =    h 
2 4
r 2h
Volume of the new cylinder 1
Required ratio  42   1: 4
Volume of the original cylinder r h 4

4. Two dice are thrown together. The probability of getting the same number on both dice is:
1
(A)
2
1
(B)
3
1
(C)
6
1
(D)
12
Solution:
Correct answer: C
When two dice are thrown together, the total number of outcomes is 36.
Favourable outcomes = {(1, 1), (2, 2), (3, 3), (4, 4), (5, 5), (6, 6)}

Number of favourable outcomes 6 1


Required probability =  
Total number of outcomes 36 6

5. The coordinates of the point P dividing the line segment joining the points
A(1,3) and B(4,6) in the ratio 2 : 1 are:
(A) (2,4)
(B) 3,5)
(C) (4,2)
(D) 5,3)
Solution:
Correct answer: B
It is given that the point P divides AB in the ratio 2: 1.
Using section formula, the coordinates of the point P are

 1  1  2  4 1  3  2  6   1  8 3  12 
 ,  ,   (3,5)
 2 1 2 1   3 3 

Hence the coordinates of the point P are (3, 5).

6. If the coordinates of the one end of a diameter of a circle are (2,3) and the coordinates of its centre are
(-2,5), then the coordinates of the other end of the diameter are:

www.vedantu.com 3 / 24
CBSE-X-2012 EXAMINATION

(A) (-6,7)
(B) ( 6,-7)
(C) (6,7)
(D) (-6,-7)

Solution:
Correct answer: A
Let the coordinates of the other end of the diameter be (x, y).
We know that the centre is the mid-point of the diameter. So, O(-2, 5) is the mid-point of the diameter AB.
The coordinates of the point A and B are (2, 3) and (x, y) respectively.
Using mid-point formula, we have,

2 x
2   4  2  x  x  6
2
3 y
5  10  3  y  y  7
2
Hence, the coordinates of the other end of the diameter are (-6, 7).

7. The sum of first 20 odd natural number is :


(A) 100
(B) 210
(C) 400
(D) 420
Solution:
Correct answer: C
The first 20 odd numbers are 1, 3, 5, … … 39
This is an AP with first term 1 and the common difference 2.

Sum of 20 terms = S20

20
S20  [2(1)  (20  1)(2)]  10[2  38]  400
2

Thus, the sum of first 20 odd natural numbers is 400.

8. If 1 is a root of the equations ay2 + ay + 3 = 0 and y2 + y + b = 0, then ab equals:


(A) 3
7
(B) 
2
(C) 6
(D) -3
Solution:

www.vedantu.com 4 / 24
CBSE-X-2012 EXAMINATION

Correct answer: A
It is given that 1 is a root of the equations ay2 + ay + 3 = 0 and y2 + y + b = 0.
Therefore, y = 1 will satisfy both the equations.

a(1)2 + a(1) + 3 = 0
a + a + 3 = 0
2a + 3 = 0
3
a
2
Also, (1)2 + (1) + b = 0
1 + 1 + b = 0
b =2

3
 ab   2  3
2

9. In Fig., the sides AB, BC and CA of a triangle ABC, touch a circle at P, Q and R respectively. If PA = 4 cm,
BP = 3 cm and AC = 11 cm, then the length of BC (in cm) is:

(A) 11
(B) 10
(C) 14
(D) 15

Solution:
Correct answer: B
It is known that the lengths of tangents drawn from a point outside a circle are equal in length.
Therefore, we have:
AP = AR … (1) (Tangents drawn from point A)
BP = BQ … (2) (Tangents drawn from point B)
CQ = CR … (3) (Tangents drawn from point C)
Using the above equations,
AR = 4 cm (AP = 4 cm, given)
BQ = 3 cm (BP = 3 cm, given)
AC = 11 cm RC = 11 cm – 4 cm = 7 cm
CQ = 7 cm
Hence, BC = BQ + CQ = 3 cm + 7 cm = 10 cm

www.vedantu.com 5 / 24
CBSE-X-2012 EXAMINATION

10. In Fig., a circle touches the side DF of  EDF at H and touches ED and EF produced at K and M
respectively. If EK = 9 cm, then the perimeter of  EDF (in cm) is:

(A) 18
(B) 13.5
(C) 12
(D) 9
Solution:
Correct answer: A
It is known that the tangents from an external point to the circle are equal.
EK = EM, DK = DH and FM = FH … (1)
Perimeter of EDF = ED + DF + FE
= (EK – DK) + (DH + HF) + (EM – FM)
= (EK – DH) + (DH + HF) + (EM – FH) [Using (1)]
= EK + EM
= 2 EK = 2 (9 cm) = 18 cm
Hence, the perimeter of EDF is 18 cm.

SECTION – B
11. If a point A(0,2) is equidistant from the points B(3,p) and C(p,5) then find the value of p.
Solution:
Solution:
It is given that the point A (0, 2) is equidistant from the points B(3, p) and C(p, 5).
So, AB = AC AB2 = AC2
Using distance formula, we have:
 (0  3)2  (2  p)2  (0  p)2  (2  5)2
 9  4  p2  4 p  p 2  9
 4  4p  0
 4p  4
 p 1

Hence, the value of p = 1.

www.vedantu.com 6 / 24
CBSE-X-2012 EXAMINATION

12. A number is selected at random from first 50 natural numbers. Find the probability that it is a multiple
of 3 and 4.
Solution:
Solution:
The total number of outcomes is 50.
Favourable outcomes = {12, 24, 36, 48}

Number of favourable outcomes 4 2


Required probability   
Total number of outcomes 50 25

1
13.The volume of a hemisphere is 2425 cm3. Find its curved surface area.
2
 22 
 Use  
7 
Solution:
Solution:

1 4851 3
Given volume of a hemisphere  2425 cm2  cm
2 2

Now, let r be the radius of the hemisphere


2
Volume of a hemisphere  r 3
3
2 4851
 r 3 
3 2
2 22 3 4851
  r 
3 7 2
3
4851 3 7  21 
r 3
   
2 2 22  2 
21
 r  cm
2
So, Curved surface area of the hemisphere  2r 2
22 21 21
 2     693 sq. cm
7 2 2

14.Tangents PA and PB are drawn from an external point P to two concentric circle with centre O and radii
8 cm and 5 cm respectively, as shown in Fig., If AP = 15 cm, then find the length of BP.

www.vedantu.com 7 / 24
CBSE-X-2012 EXAMINATION

Solution:
Given: Tangents PA and PB are drawn from an external point P to two
concentric circles with centre O and radii OA = 8 cm, OB = 5 cm
respectively. Also, AP = 15 cm
To find: Length of BP
Construction: We join the points O and P.

Solution: OA ⊥ AP ; OB ⊥ BP
[Using the property that radius is perpendicular to the tangent at the point
of contact of a circle]
In right angled triangle OAP,
OP2 = OA2 + AP2 [Using Pythagoras Theorem]
= (8)2 + (15)2 = 64 + 225 = 289
∴ OP = 17 cm
In right angled triangle OBP,
OP2 = OB2 + BP2
⇒ BP2 = OP2 – OB2
= (17)2 – (5)2 = 289 – 25 = 264
BP  264  2 66 cm.

15.In fig., an isosceles triangle ABC, with AB = AC, circumscribes a circle.


Prove that the point of contact P bisects the base BC.

www.vedantu.com 8 / 24
CBSE-X-2012 EXAMINATION

OR
In fig., the chord AB of the larger of the two concentric circles, with centre O, touches the smaller circle at
C. Prove that AC = CB.

Solution:
Given: ABC is an isosceles triangle, where AB = AC, circumscribing a circle.
To prove: The point of contact P bisects the base BC.
i.e. BP = PC
Proof: It can be observed that
BP and BR ; CP and CQ; AR and AQ are pairs of tangents drawn to the circle
from the external points B , C and A respectively.
So, applying the result that the tangents drawn from an external point to a circle, we get
BP = BR --- (i)
CP = CQ --- (ii)
AR = AQ --- (iii)
Given that AB = AC
⇒ AR + BR = AQ + CQ
⇒ BR = CQ [from (iii)]
⇒ BP = CP [from (i) and (ii)]
∴ P bisects BC.
Hence proved.
OR
Given: The chord AB of the larger of the two concentric circles, with centre O, touches the smaller circle at
C.
To prove: AC = CB
Construction: Let us join OC.

www.vedantu.com 9 / 24
CBSE-X-2012 EXAMINATION

Proof: In the smaller circle, AB is a tangent to the circle at the point of contact C.
∴ OC ⊥ AB ------ (i)
(Using the property that the radius of a circle is perpendicular to the tangent at the point of contact)
For the larger circle, AB is a chord and from (i) we have OC ⊥ AB
∴ OC bisects AB
(Using the property that the perpendicular drawn from the centre to a chord of a circle bisects the chord)
∴ AC = CB

16. In fig., OABC is a square of side 7 cm. If OAPC is a quadrant of a circle with centre O, then find the area
 22 
of the shaded region.  Use   
 7

Solution:
Given, OABC is a square of side 7 cm
i.e. OA = AB = BC = OC = 7cm
∴ Area of square OABC = (side)2 = 72 = 49 sq.cm
Given, OAPC is a quadrant of a circle with centre O.

∴ Radius of the sector = OA = OC = 7 cm.


Sector angle = 90

www.vedantu.com 10 / 24
CBSE-X-2012 EXAMINATION

90
∴ Area of quadrant OAPC  r 2
360
1 22 77
   (7)2  sq. cm  38.5 sq. cm
4 7 2
∴ Area of shaded portion = Area of Square - OABC Area of quadrant OAPC
 (49  38.5)sq. cm  10.5 sq.cm

17.Find the sum of all three digit natural numbers, which are multiples of 7.
Solution:
First three- digit number that is divisible by 7 = 105
Next number = 105 + 7 = 112
Therefore the series is 105, 112, 119,…
The maximum possible three digit number is 999.
When we divide by 7, the remainder will be 5.
Clearly, 999 – 5 = 994 is the maximum possible three – digit number divisible by 7.
The series is as follows:
105, 112, 119, …., 994
Here a = 105, d = 7
Let 994 be the nth term of this A.P.
an  a  (n  1)d
 994  105  (n  1)7
 (n  1)7  889
 (n  1)  127
 n  128
So, there are 128 terms in the A.P.
n
 Sum  {first term + last term}
2
128
 {a1  a128 }
2
 64{105  994}  (64)(1099)  70336

18.Find the values (s) of k so that the quadratic equation 3x2 – 2kx + 12 = 0 has equal roots.
Solution:
Given quadratic equation is 3x2 – 2kx + 12 = 0
Here a = 3, b = -2k and c = 12
The quadratic equation will have equal roots if Δ = 0
 b2  4ac  0
Putting the values of a,b and c we get

www.vedantu.com 11 / 24
CBSE-X-2012 EXAMINATION

(2k)2  4(3)(12)  0
 4k 2  144  0
 4k 2  144
144
 k2   36
4
Considering square root on both sides,
k  36  6
Therefore, the required values of k are 6 and -6.

SECTION – C

AP K
19.A point P divides the line segment joining the points A(3,-5) and B(-4,8) such that  . If P lies on
PB 1
the line x + y = 0, then find the value of K.
Solution:
Solution:

Let the co-ordinates of point P be (x, y)


Then using the section formula co-ordinates of P are.
4 K  3 8K  5
x y
K 1 K 1
Since P lies on x+y=0
4K  3 8K  5
  0
K 1 K 1
 4K  2  0
2
K 
4
1
K 
2
1
Hence the value of K  .
2

20.If the vertices of a triangle are (1,-3), (4,p) and (-9,7) and its area is 15 sq. units, find the value (s) of p.
Solution:

www.vedantu.com 12 / 24
CBSE-X-2012 EXAMINATION

The area of a  , whose vertices are (x1, y1), (x2, y2) and (x3, y3) is
1
  | x1 (y2  y3 )  x2 (y3  y1 )  x3 (y1  y2 )|
2
Substituting the given coordinates
1
Area of   |1(p  7)  4(7  3)  (9)(3  p)|
2
1
 |(p  7)  40  27  9 p| 15
2
 10 p  60  30
 10 p  30 or 10 p  90
 p  3. or p  9
Ans hence the value of p=-3 or -9

21.Prove that the parallelogram circumscribing a circle is a rhombus.


OR
Prove that opposite sides of a quadrilateral circumscribing a circle subtend supplementary angles at the
centre of the circle.
Solution:
Let ABCD be a parallelogram such that its sides touching a circle with centre O. We know that the tangents
to a circle from an exterior point are equal in length.

 AP = AS [From A] …(i)
BP = BQ [From B] …(ii)
CR = CQ [From C] …(iii)
and, DR = DS [From D] …(iv)
Adding (i), (ii), (iii) and (iv), we get
AP+BP+CR+DR=AS+BQ+CQ+DS
 (AP+BP)+(CR+DR)=(AS+DS)+(BQ+CQ)
 AB+CD = AD+BC

www.vedantu.com 13 / 24
CBSE-X-2012 EXAMINATION

 2 AB = 2 BC [ ABCD is a parallelogram  AB=CD and BC = AD]


 AB=BC
Thus, AB=BC=CD=AD
Hence, ABCD is a rhombus.
OR
A circle with centre O touches the sides AB, BC, CD, and DA of a quadrilateral ABCD at the points P, Q, R
and S respectively.
TO PROVE :  AOB +  COD = 180
and,  AOD +  BOC = 180

CONSTRUCTION
Join OP, OQ, OR and OS.
PROOF Since the two tangents drawn from a external point to a circle subtend equal angles at the centre.
  1 =  2,  3 =  4,  5 =  6 and  7 =  8
Now,  1 +  2 +  3 +  4 +  5+  6 +  7 +  8 = 360
Sum of all the angles 
subtended at a point is 360
 
 2(  2 +  3 +  6 +  7) = 360 and 2(  1 +  8 +  4 +  5) = 360
 (  2 +  3) + (  6 +  7) = 180 and (  1 +  8) + (  4 +  5) = 180
 2  3  AOB, 6  7  COD 
1  8  AOD and 4  5  BOC 
 
 AOB  COD  180
and AOD  BOC  180
Hence Proved

22. From a solid cylinder of height 7 cm and base diameter 12 cm, a conical cavity of same height and same
 22 
base diameter is hollowed out. Find the total surface area of the remaining solid.  Use   
 7
OR
A cylindrical bucket, 32 cm high and with radius of base 18 cm, is filled with sand. This bucket is emptied on
the ground and a conical heap of sand is formed. If the height of the conical heap is 24 cm, then find the
radius and slant height of the heap.
Solution:

www.vedantu.com 14 / 24
CBSE-X-2012 EXAMINATION

Given: radius of cyl=radius of cone=r=6cm


Height of the cylinder=height of the cone=h=7cm
Slant height of the cone= l
72  62
 85cm
Total surface area of the remaining solid
= curved surface area of the cylinder + area of the base of the cylinder + curved surface area of the cone
(2rh  r 2  rl)
22 22 22
 2  6  7   62   6  85
7 7 7
792 132
 264   85
7 7
132
 377.1  85cm2
7
OR
Volume of the conical heap=volume of the sand emptied from the bucket.
Volume of the conical heap=
1 2 1
r h  r 2  24cm2 (height of the coneis24)----------(1)
3 3
 r 2h
Volume of the sand in the bucket
 (18)2 32cm2        (2)
Equating 1 and 2
1 2
r  24  (18)2 32
3
(18)2  32  3
 r2 
24
 r  36cm

23. In fig., PQ and AB are respectively the arcs of two concentric circles of radii 7 cm and 3.5 cm and centre
 22 
O. If  POQ = 30, then the area of the shaded region.  Use   
 7

www.vedantu.com 15 / 24
CBSE-X-2012 EXAMINATION

Solution:

Area of the shaded region=


Area of sector POQ-Area of sector AOB
   
 R 2  r 2 
 360 360 
30 22
Area of Shaded region    (72  3.52 )
360 7
77
 cm2
8

24. Solve for x: 4x2 – 4ax + (a2 – b2) = 0


Or
2
Solve for x: 3x - 6x + 2 = 0
Solution:

www.vedantu.com 16 / 24
CBSE-X-2012 EXAMINATION

4 x 2  4ax  (a2  b2 )  0
 (4 x 2  4ax  a2 )  b2  0
 (2 x 2 )  2.2 x.a  a2   b2  0

   2 x  a    b2  0
2

 
 (2 x  a)2  b  (2 x  a)  b  0
 (2 x  a)  b  0 or (2 x  a)  b  0
ab ab
x ;x
2 2
OR
3x 2  2 6 x  2  0
 3x 2  6 x  6 x  2  0
 3   3 x  2   2  3 x  2   0

 3x  2  3x  2   0
 2  0
2
3x 

 3x  2  0
 3x  2
2 2 3 6
x  2

3 ( 3) 3

25. A kite is flying at a height of 45 m above the ground. The string attached to the kite is temporarily tied
to a point on the ground. The inclination of the string with the ground is 60 . Find the length of the string
assuming that there is slack in the string.
Solution:
Given: Position of kite is B.
Height of kite above ground= 45 m
Angle of inclination = 60
Required length of string = AB

www.vedantu.com 17 / 24
CBSE-X-2012 EXAMINATION

In right angled triangle AOB,


OB
sin A 
AB
45
 sin  
AB
3 45
 
2 AB
45  2 90
 AB    30 3m
3 3
Hence, the length of the string is 30 3m

26. Draw a triangle ABC with side BC = 6 cm, C = 30 and A = 105. Then construct another triangle
2
whose sides are times the corresponding sides of ABC.
3
Solution:
It is given that A = 105C = 30 .
Using angle sum property of triangle, we get, B = 45
The steps of construction are as follows:
1. Draw a line segment BC = 6 cm.
2. At B, draw a ray making an angle of 45with BC.
3. At C, draw a ray making an angle of 30with BC. Let the two rays meet at point A.
4. Below BC, make an acute angle CBX.
5. Along BX mark off three points B1, B2 , B3 such that BB1 = B1B2 = B2B3.
6. Join B3C.
7. From B2, draw B2C|| B3C.
8. From C, draw CA|| CA, meeting BA at the point A.
Then ABCis the required triangle.

27. The 16th term of an AP is 1 more than twice its 8th term. If the 12th term of the AP is 47, then find its nth
term.
Solution:
Let a and d respectively be the first term and the common difference of the AP.

www.vedantu.com 18 / 24
CBSE-X-2012 EXAMINATION

We know that the nth term of an AP is given by an = a + (n – 1)d


According to the given information,
A16 = 1 + 2 a8
 a + (16 – 1)d = 1 + 2[a + (8 – 1)d]
 a + 15d = 1 + 2a + 14d
 a + 15d = 1 + 2a + 14d
 –a + d = 1 … (1)
Also, it is given that, a12 = 47
 a + (12 – 1)d = 47
 a + 11d = 47 … (2)
Adding (1) and (2), we have:
12d = 48
d = 4
From (1),
–a + 4 = 1 a = 3
Hence, an = a + (n – 1)d = 3 + (n – 1)(4) = 3 + 4n – 4 = 4n – 1
Hence, the nth term of the AP is 4n – 1.

28.A card is drawn from a well shuffled deck of 52 cards. Find the probability of getting (i) a king of red
colour (ii) a face card (iii) the queen of diamonds.
Solution:
Total number of outcomes=52
(i) Probability of getting a red king
Here the number of favourable outcomes=2
no.of favourable outcomes
probability 
total number of outcome
12

52
3

13
(iii)Probability of queen of diamonds
number of queens of diamond=1,hence
no.of favourable outcomes
probability 
total number of outcome
1
52

SECTION – D
29.A bucket is in the form of a frustum of a cone and its can hold 28.49 litres of water. If the radii of its
 22 
circular ends are 28 cm and 21 cm, find the height of the bucket.  Use   
 7

www.vedantu.com 19 / 24
CBSE-X-2012 EXAMINATION

Solution:

Here, R = 28 cm and r = 21 cm, we need to find h.


Volume of frustum = 28.49 L = 28.49×1000 cm3 = 28490 cm3
h 2
Now, Volume of frustum =
3
 R  Rr  r 2 


22h
73
282  28  21  212   28490
22
 h  1813  28490
21
28490  21
h  15cm
22  1813
Hence the height of bucket is 15 cm.

30.The angle of elevation of the top of a hill at the foot of a tower is 60 and the angle of depression from
the top of the tower of the foot of the hill is 30. If the tower is 50 m high, find the height of the hill.
Solution:

Let the height of hill is h.


In right triangle ABC,
50 50 1
 tan30    AB  50 3
AB AB 3
In right triangle ABD,
h h
 tan60   3  h  3AB
AB AB
 
 h  3 50 3  150m

www.vedantu.com 20 / 24
CBSE-X-2012 EXAMINATION

Hence the height of hill is 150 m.

31. AB + CD = AD + BC
Prove that the tangent at any point of a circle is perpendicular to the radius through the point of contact.
OR
A quadrilateral ABCD is drawn to circumscribe a circle. Prove that AB + CD = AD + BC.
Solution:

Given: AB is a tangent to a circle with centre O.


To prove: OP is perpendicular to AB.
Construction: Take a point Q on AB and join OQ.
Proof: Since Q is a point on the tangent AB, other than the point of contact P, so Q will be outside the
circle.
Let OQ intersect the circle at R.
Now OQ = OR + RQ
 OQ > OR  OQ > OP [as OR = OP]
 OP < OQ
Thus OP is shorter than any other segment among all and the shortest length is the perpendicular from O
on AB.
 OP  AB. Hence proved.
OR

Let ABCD be a quadrilateral, circumscribing a circle.


Since the tangents drawn to the circle from an external point are equal,
we have
AP = AS ... (1)

www.vedantu.com 21 / 24
CBSE-X-2012 EXAMINATION

PB = BQ ... (2)
RC = QC ... (3)
DR = DS ... (4)
Adding, (1), (2), (3) and (4), we get
AP + PB + RC + DR = AS + BQ + QC + DS
(AP + PB) + (DR + RC) = (AS + SD) + (BQ + QC)
AB + CD = AD + BC.
Hence, Proved.

32.A shopkeeper buys some books for ₹80. If he had bought 4 more books for the same amount, each
book would have cost ₹1 less. Find the number of books he bought.
OR
1
The sum of two number is 9 and the sum of their reciprocals is . Find the numbers.
2
Solution:
Total cost of books = Rs 80
Let the number of books = x

80
So the cost of each book = Rs
x

80
Cost of each book if he buy 4 more book  Rs
x4

As per given in question:

80 80
 1
x x4
80 x  320  80 x
 1
x(x  4)
320
 2 1
x  4x
 x 2  4 x  320  0
 (x  20)(x  16)  0
 x  20, 16

Since number of books cannot be negative,


So the number of books he brought is 16.
OR
Let the first number be x then the second number be 9 – x as the sum
of both numbers is 9.

1
Now the sum of their reciprocal is , therefore
2

www.vedantu.com 22 / 24
CBSE-X-2012 EXAMINATION

1 1 1
 
x 9x 2
9x x 1
 
x(9  x) 2
9 1
 
9x  x 2
2

 18  9 x  x 2
 x 2  9 x  18  0
 (x  6)(x  3)  0
 x  6,3
If x = 6 then other number is 3.
and if x = 3 then other number is 6.
Hence numbers are 3 and 6.

33.Sum of the first 20 terms of an AP is -240, and its first term is 7. Find its 24th term.
Solution:
Given: S20 240 and a 7
Consider, S20 240
20  n 
 (2  7  19d )  240  Sn  [2a  (n  1)d ]
2  2 
 10(14  19d )  240
 14  19d  24
 19d  38
 d  2
Now, a24  a  23d  7  23  2  39
Hence, a24  39

34.A solid is in the shape of a cone standing on a hemisphere with both their radii being equal to 7 cm and
 22 
the height of the cone is equal to its diameter. Find the volume of the solid.  Use   
 7
Solution:

www.vedantu.com 23 / 24
CBSE-X-2012 EXAMINATION

Radius of hemi-sphere = 7 cm
Radius of cone = 7 cm
Height of cone = diameter = 14 cm
Volume of solid = Volume of cone + Volume of Hemi-sphere
1 2
 r 2h  r 3
3 3
1
 r 2 (h  2r )
3
1 22
   49(14  14)
3 7
1 22
   49  28
3 7
22  7  28 4312 3
  cm
3 3

www.vedantu.com 24 / 24
CBSE-XII-2017 EXAMINATION CBSE-X-2013 EXAMINATION

MATHEMATICS
Paper & Solution
Time: 3 Hrs. Max. Marks: 90
General Instructions :
1. All questions are compulsory.
2. The question paper consists of 34 questions divided into four sections A, B, C, and D.
3. Section A contains of 8 questions of 1 mark each, which are multiple choice type question, Section B
contains of 6 questions of 2 marks each, Section C contains of 10 questions of 3 marks each and Section D
contains of 10 questions of 4 marks each.
4. Use of calculator is not permitted.

SECTION – A
1. The angle of depression of a car, standing on the ground, from the top of a 75 m high tower, is 30 The
distance of the car from the base of the tower (in m.) is:
(A) 25 3

(B) 50 3

(C) 75 3

(D) 150
Solution:
Correct answer: C

Let AB be the tower of height 75 m and C be the position of the car


In  ABC,

AC
cot 30 
AB
 AC  AB cot30
 AC  75m  3
 AC  75 3m

Thus, the distance of the car from the base of the tower is 75 3 m.

www.vedantu.com 1 / 23
CBSE-X-2013 EXAMINATION

2. The probability of getting an even number, when a die is thrown once, is:
1
(A)
2
1
(B)
3
1
(C)
6
5
(D)
6
Solution:
Correct answer: A
S = {1,2,3,4,5,6}
Let event E be defined as 'getting an even number'.
n(E) = {1,4,6}
Number of favourable outcomes 3 1
P E   
Number of possible outcomes 6 2

3. A box contains 90 discs, numbered from 1 to 90. If one disc is drawn at random from the box, the
probability that it bears a prime-number less than 23, is:
7
(A)
90

10
(B)
90

4
(C)
45

9
(D)
89
Solution:
Correct answer: C

S = {1, 2, 3,..90}

n(S) = 90

www.vedantu.com 2 / 23
CBSE-X-2013 EXAMINATION

The prime number less than 23 are 2, 3, 5, 7, 11, 13, 17, and 19.
Let event E be defined as 'getting a prime number less than 23'.
n(E) = 8
Number of favourable outcomes 8 4
P E   
Number of possible outcomes 90 45

4. In fig., a circle with centre O is inscribed in a quadrilateral ABCD such that, it touches the sides BC, AB,
AD and CD at points P, Q, R and S respectively, If AB = 29 cm, AD = 23 cm,  B  90 and DS = 5 cm, then
the radius of the circle (in cm) is:

(A)11

(B)18

(C)6

(D)15
Solution:
Correct answer: A
Given: AB, BC, CD and AD are tangents to the circle with centre O at Q, P, S and R respectively. AB = 29 cm,
AD = 23, DS = 5 cm and  B  90 Construction: Join PQ.

We know that, the lengths of the tangents drawn from an external point to a circle are equal.

www.vedantu.com 3 / 23
CBSE-X-2013 EXAMINATION

DS  DR  5 cm
 AR  AD  DR  23 cm  5 cm  18 cm
AQ  AR  18 cm
QB  AB  AQ  29 cm  18 cm  11 cm
QB  BP  11 cm

In  PQB,
PQ2  QB2  BP2  11 cm   11 cm   2  11 cm 
2 2 2

PQ  11 2 cm ... 1

In  OPQ ,
PQ2  OQ2  OP 2  r 2  r 2  2r 2
(11 2 )2  2r 2
121  r 2
r  11
Thus, the radius of the circle is 11 cm.

5. In fig., PA and PB are two tangents drawn from an external point P to a circle with centre C and radius 4
cm. If PA  PB, then the length of each tangent is:

(A) 3cm

(B) 4cm

(C) 5cm

(D) 6cm
Solution:
Correct answer: B
AP  PB (Given)

CA  AP , CB  BP (Since radius is perpendicular to tangent)

AC = CB = radius of the circle


Therefore, APBC is a square having side equal to 4 cm.
Therefore, length of each tangent is 4 cm.

www.vedantu.com 4 / 23
CBSE-X-2013 EXAMINATION

6. In fig., the area of triangle ABC (in sq. units) is:

(A) 15

(B) 10

(C) 7.5

(D) 2.5

Solution:
Correct answer: C

From the figure, the coordinates of A, B, and C are (1, 3), (-1, 0) and (4, 0) respectively.

Area of  ABC
1
 1  0  0    1  0  3   4  3  0 
2
1
 0  3  12
2
1
 15
2
 7.5 sq units

www.vedantu.com 5 / 23
CBSE-X-2013 EXAMINATION

22
7. If the difference between the circumference and the radius of a circle is 37 cm, then using   , the
7
circumference (in cm) of the circle is:
(A) 154

(B) 44

(C) 14

(D) 7
Solution:
Correct answer: B
Let r be the radius of the circle.
From the given information, we have:
2r  r  37 cm
 r 2  1  37cm
 22 
 r  2   1   37cm
 7 
37
 r   37cm
7
 r  7cm

22
Circumference of the circle  2r  2   7cm  44cm
7

1 1  6q 1  12q
8. The common difference of AP , , , …is:
3q 3q 3q
(A) q
(B) –q
(C) -2
(D) 2
Solution:
Correct answer: C
Common difference =
1  6q 1 1  6q  1 6q
    2
3q 3q 3q 3q

SECTION B
9. Prove that the parallelogram circumscribing a circle is a rhombus.
Solution:

www.vedantu.com 6 / 23
CBSE-X-2013 EXAMINATION

Given: ABCD be a parallelogram circumscribing a circle with centre O.


To prove: ABCD is a rhombus.

We know that the tangents drawn to a circle from an exterior point are equal in length. Therefore, AP = AS,
BP = BQ, CR = CQ and DR = DS. Adding the above equations,

AP + BP + CR + DR = AS + BQ + CQ + DS
(AP + BP) + (CR + DR) = (AS + DS) + (BQ + CQ)
AB + CD = AD + BC
2AB = 2BC
(Since, ABCD is a parallelogram so AB = DC and AD = BC)
AB = BC
Therefore, AB = BC = DC = AD.
Hence, ABCD is a rhombus.

10. Two circular pieces of equal radii and maximum area, touching each other are cut out from a
rectangular card board of dimensions 14 cm 7 cm. Find the area of the remaining card board.
 22 
Use  7 
 
Solution:
Dimension of the rectangular card board = 14 cm  7 cm Since, two circular pieces of equal radii and
maximum area touching each other are cut from the rectangular card board, therefore, the diameter of
14
each of each circular piece is  7 cm.
2

www.vedantu.com 7 / 23
CBSE-X-2013 EXAMINATION

7
Radius of each circular piece =  cm.
2
2
7 22 49
Sum of area of two circular pieces  2      2    77cm2
2 7 4

Area of the remaining card board


= Area of the card board - Area of two circular pieces

 14 cm  7 cm  77 cm2
 98 cm2  77 cm2
 21 cm2

11.In fig., a circle is inscribed in triangle ABC touches its sides AB, BC and AC at points D, E and F
respectively. If AB = 12 cm, BC = 8 cm and AC = 10 cm, then find the length of AD, BE and CF.

Solution:
Given: AB = 12 cm, BC = 8 cm and AC = 10 cm.
Let, AD = AF = x cm, BD = BE = y cm and CE = CF = z cm
(Tangents drawn from an external point to the circle are equal in length)
 2(x + y + z) = AB + BC + AC = AD + DB + BE + EC + AF + FC = 30 cm
 x + y + z = 15 cm
AB = AD + DB = x + y = 12 cm
z = CF = 15 - 12 = 3 cm
AC = AF + FC = x + z = 10 cm

y = BE = 15 - 10 = 5 cm
x = AD = x + y + z - z - y = 15 - 3 - 5 = 7 cm

12. How many three-digit natural numbers are divisible by 7?


Solution:
Three digit numbers divisible by 7 are

www.vedantu.com 8 / 23
CBSE-X-2013 EXAMINATION

105, 112, 119, … 994


This is an AP with first term (a) = 105 and common difference (d) = 7
Let an be the last term.

an  a  (n  1)d
994  105  (n  1)(7)
7(n  1)  889
n  1  127
n  128

Thus, there are 128 three-digit natural numbers that are divisible by 7.

13.Solve the following quadratic equation for x: 4 3 2 5  2 3  0


Solution:

4 3x 2  5x  2 3  0

 4 3x 2  8 x  3x  2 3  0
 4 x 3x  2  3 3x  2  0
 4 x  3 3x  2  0
3 2
x  or x  
4 3

14.A card is drawn at random from a well shuffled pack of 52 playing cards. Find the probability that the
drawn card is neither a king nor a queen.
Solution:
Let E be the event that the drawn card is neither a king nor a queen.
Total number of possible outcomes = 52

Total number of kings and queens = 4 + 4 = 8

Therefore, there are 52 - 8 = 44 cards that are neither king nor queen.

Total number of favourable outcomes = 44


Favourable outcomes 44 11
 Required probability = P(E) =  
Total number of outcomes 52 13

SECTION C

www.vedantu.com 9 / 23
CBSE-X-2013 EXAMINATION

15.A vessel is in the form of hemispherical bowl surmounted by a hollow cylinder of same diameter. The
diameter of the hemispherical bowl is 14 cm and the total height of the vessel is 13 cm. Find the total
 22 
surface area of the vessel. use  
 7
Solution:

Let the radius and height of cylinder be r cm and h cm respectively.

Diameter of the hemispherical bowl = 14 cm

 Radius of the hemispherical bowl = Radius of the cylinder


14
r  cm  7cm
2
Total height of the vessel = 13 cm
Height of the cylinder, h = 13 cm - 7 cm = 6 cm
Total surface area of the vessel = 2 (curved surface area of the cylinder + curved surface area of the
hemisphere) (Since, the vessel is hollow)
22
 2 2rh  2r 2  4r h  r  4   7  6  7 cm2
7
 1144 cm 2

16. A wooden toy was made by scooping out a hemisphere of same radius from each end of a solid
cylinder. If the height of the cylinder is 10 cm, and its base is of radius 3.5 cm, find the volume of wood in
 22 
the toy. use  
 7
Solution:

www.vedantu.com 10 / 23
CBSE-X-2013 EXAMINATION

Height of the cylinder, h = 10 cm

Radius of the cylinder = Radius of each hemisphere = r = 3.5 cm


Volume of wood in the toy = Volume of the cylinder - 2  Volume of each

Hemisphere
2
 r 2h  2  r 3
3
 4 
 r 2  h  r 
 3 
22 2 4 
   3.5  10   3.5 
7  3 
 38.5  10  4.67
 38.5 
  cm3

Radius = 21 cm

17.In a circle of radius 21 cm, an arc subtends an angle of 60 at the centre. Find: (i) the length of the arc
 22 
(ii) area of the sector formed by the arc. use  
 7
Solution:
The arc subtends an angle of 60o at the centre.


(i) I   2r
360

60 22
  2   21cm
360 7
 22cm

www.vedantu.com 11 / 23
CBSE-X-2013 EXAMINATION


(ii) Area of the sec tor   r 2
360

60 22
    21cm2
360 7
 231 cm2

18. In Fig., AB and CD are two diameters of a circle with centre O, which are perpendicular to each other.
 22 
OB is the diameter of the smaller circle. If OA = 7 cm, find the area of the shaded region. use  
 7

Solution:
AB and CD are the diameters of a circle with centre O.

OA = OB = OC = OD = 7 cm (Radius of the circle)


Area of the shaded region
= Area of the circle with diameter OB + (Area of the semi-circle ACDA – Area of  ACD)
2
7 1 1 
        2   CD  OA 
2 2 2 
22 49 1 22 1
     49   14  7
7 4 2 7 2
77
  77  49
2
 66.5 cm2

19. Find the ratio in which the y-axis divides the line segment joining the points (-4, -6) and (10, 12). Also,
find the coordinates of the point of division.
Solution:
. Let the y-axis divide the line segment joining the points (-4,-6) and (10,12) in the ratio k: 1 and the point of
the intersection be (0,y). Using section formula, we have:

www.vedantu.com 12 / 23
CBSE-X-2013 EXAMINATION

 10k  4 12k  6 
 ,   0, y
 k 1 k 1 
10k  4
  0  10k  4  0
k 1
4 2
k 
10 5
Thus, the y-axis divides the line segment joining the given points in the ratio 2:5
2  24  30 
12   6  
12k  6

5 5  6
y   
k 1 2
1 25 7
 
5  5 

 6
Thus, the coordinates of the point of division are  0,   .
 7

20. The horizontal distance between two poles is 15 m. The angle of depression of the top of first pole as
seen from the top of second pole is 30. If the height of the second pole is 24 m, find the height of the first
pole. Use 3  1.732
Solution:

Let AB and CD be the two poles, where CD (the second pole) = 24 m.

BD = 15 m

Let the height of pole AB be h m.

AL = BD = 15 m and AB = LD = h

So, CL = CD - LD = 24 – h

www.vedantu.com 13 / 23
CBSE-X-2013 EXAMINATION

In  ACL,
CL
tan30 
AL
24  h
 tan30 
15
1 24  h
 
3 15
15
 24  h  5 3
3
 h  24  5 3
 h  24  5  1.732 Taking 3  1.732
 h  15.34
Thus, height of the first pole is 15.34 m.

21. For what values of k, the roots of the quadratic equation (k  4) 2 (k  1)  1  0 are equal?
Solution:

k  4) 2 (k  1)  1  0
a  k  4, b  k  1, c  1

For equal roots, discriminant, D = 0

 b2  4ac  0
 (k  1)2  4(k  4)  1  0
 k 2  2k  1  4k  16  0
 k 2  2k  15  0
 k 2  5k  3k  15  0
 k(k  5)  3(k  5)  0
 (k  5)(k  3)  0
 k  5 or k  3

Thus, for k = 5 or k = -3, the given quadratic equation has equal roots.

22. The sum of first n terms of an AP is 3n2 + 4n. Find the 25th term of this AP.
Solution:

Sn  3n2  4n

First term (a1 )  S1  3(1)2  4(1)  7

www.vedantu.com 14 / 23
CBSE-X-2013 EXAMINATION

S2  a1  a2  3(2)2  4(2)  20
a2  20  a1  20  7  13

So, common difference (d) = a2 – a1 = 13 - 7 = 6


Now, an = a + (n - 1)d

a25  7   25  1  6  7  24  6  7  144  151

23. Construct a tangent of a circle of radius 4 cm from a point on the concentric circle of radius 6 cm.
Solution:

Steps of construction:
1. Draw two concentric circle with centre O and radii 4 cm and 6 cm. Take a point P on the outer circle and
then join OP.
2. Draw the perpendicular bisector of OP. Let the bisector intersects OP at M.
3. With M as the centre and OM as the radius, draw a circle. Let it intersect the inner circle at A and B.
4. Join PA and PB. Therefore, PA and PB are the required tangents.

24. Show that the points (-2, 3), (8, 3) and (6, 7) are the vertices of a right triangle.
Solution:
The given points are A(-2,3) B(8,3) and C(6,7). Using distance formula, we have:

www.vedantu.com 15 / 23
CBSE-X-2013 EXAMINATION

AB2  8  2 2  3  3 2
 AB2  102  0
 AB2  100
BC 2  6  8 2  7  3 2
 BC 2  (2)2  42
 BC 2  4  16
 BC 2  20
CA2  2  62  3  72
 CA  (8)2  (4)2
 CA2  64  16
 CA2  80

It can be observed that:

BC 2  CA2  20  80  100  AB2


So, by the converse of Pythagoras Theorem,
 ABC is a right triangle right angled at C.

SECTION D

25. Water is flowing through a cylindrical pipe, of internal diameter 2 cm, into a cylindrical tank of base
radius 40 cm, at the rate of 0.4 m/s. Determine the rise in level of water in the tank in half an hour.
Solution:
Diameter of circular end of pipe = 2 cm
2
 Radius r1 of circular end of pipe  m  0.01 m
200

Area of cross-section   r12   0.012  0.0001m2

Speed of water  0.4 m / s  0.4  60  24 metre /min

Volume of water that flows in 1 minute from pipe = 24  0.0001m3  0.0024m3

Volume of water that flows in 30 minutes from pipe =  0.0024m3  0.072m3
Radius (r2) of base of cylindrical tank = 40 cm = 0.4 m
Let the cylindrical tank be filled up to h m in 30 minutes. Volume of water filled in tank in 30 minutes is
equal to the volume of water flowed out in 30 minutes from the pipe.

www.vedantu.com 16 / 23
CBSE-X-2013 EXAMINATION

 r22  h  0.072


 0.4 2 h  0.072
 0.16 h  0.072
0.072
h
0.16
 h  0.45 m  45 cm

Therefore, the rise in level of water in the tank in half an hour is 45 cm.

26. A Group consists of 12 persons, of which 3 are extremely patient, other 6 are extremely honest and
rest are extremely kind. A person from the group is selected at random. Assuming that each person is
equally likely to be selected, find the probability of selecting a person who is (i) extremely patient (ii)
extremely kind or honest. Which of the above values you prefer more?
Solution:
The group consists of 12 persons.

Total number of possible outcomes = 12


Let A denote event of selecting persons who are extremely patient

Number of outcomes favourable to A is 3.


Let B denote event of selecting persons who are extremely kind or honest. Number of persons who are
extremely honest is 6. Number of persons who are extremely kind is 12 - (6 + 3) = 3  Number of
outcomes favourable to B = 6 + 3 = 9.

(i)
Number of outcomes favrouableto A 3 1
P A  
Total number of possible outcomes 12 4

(ii)
Number of outcomes favrouableto B 9 3
PB  
Total number of possible outcomes 12 4

Each of the three values, patience, honesty and kindness is important in one's life.

27.A bucket open at the top, and made up of a metal sheet is in the form of a frustum of a cone. The depth
of the bucket is 24 cm and the diameters of its upper and lower circular ends are 30 cm and 10 cm
respectively. Find the cost of metal sheet used in it at the rate of Rs 10 per 100 cm2. Use   3.14

www.vedantu.com 17 / 23
CBSE-X-2013 EXAMINATION

Solution:
Diameter of upper end of bucket = 30 cm
Radius  r1  of upper end of bucket = 15 cm
Diameter of lower end of bucket = 10 cm
 Radius  r1  of lower end of bucket = 5 cm
Slant height (l) of frustum

 r1  r2 2  h2
 15  52  24 2  10 2  24 2  100  576
676  26cm

Area of metal sheet used to make the bucket

  r1  r2 I  r22
        2

 520  25  545cm2

Cost of 100 cm2 metal sheet = Rs 10

Cost of 545  cm 2 metal sheet

545  3.14  10
 Rs.  Rs.171.13
100
Therefore, cost of metal sheet used to make the bucket is Rs 171.13.

28. In fig., l and m are two parallel tangents to a circle with centre O, touching the circle at A and B
respectively. Another tangent at C intersects the line l at D and m at E. Prove that DOE  90

www.vedantu.com 18 / 23
CBSE-X-2013 EXAMINATION

Solution:
Given: l and m are two parallel tangents to the circle with centre O touching the circle at A and B
respectively. DE is a tangent at the point C, which intersects l at D and m at E.
To prove:  DOE = 90
Construction: Join OC.
Proof:

In  ODA and  ODC,


OA = OC (Radii of the same circle)
AD = DC (Length of tangents drawn from an external point to a circle are equal)
DO = OD (Common side)
 ODA   ODC (SSS congruence criterion)
DOA  COD .........(1)
Similarly,  OEB   OEC
EOB  COE .........(2)
Now, AOB is a diameter of the circle. Hence, it is a straight line.
 DOA +  COD +  COE +  EOB = 180
From (1) and (2), we have:
2  COD + 2  COE = 180
  COD +  COE = 90
  DOE = 90
Hence, proved.

29. Sum of the areas of two squares is 400 Cm2. If the difference of their perimeters is 16 cm, find the sides
of the two squares.
Solution:
Let the sides of the two squares be x cm and y cm where x > y.
Then, their areas are x2 and y2 and their perimeters are 4x and 4y.
By the given condition:

www.vedantu.com 19 / 23
CBSE-X-2013 EXAMINATION

x2 + y2 = 400 … (1)
and 4x - 4y = 16
 4(x - y) = 16 x - y = 4
 x = y + 4 ... (2)
Substituting the value of x from (2) in (1), we get:
(y + 4)2 + y2 = 400
 y2 + 16 + 8y + y2 = 400
 2y2 + 16 + 8y = 400
 y2 + 4y - 192 = 0
 y2 + 16y - 12y - 192 = 0
 y(y + 16) - 12 (y + 16) = 0
 (y + 16) (y - 12) = 0
 y = -16 or y = 12
Since, y cannot be negative, y = 12.
So, x = y + 4 = 12 + 4 = 16
Thus, the sides of the two squares are 16 cm and 12 cm.

1 1 1 1
30. Solve that following for x:   
2a  b  2  2a b 2 
Solution:

1 1 1 1
  
2a  b  2 x 2a b 2 x
1 1 1 1
   
2a  b  2 x 2 x 2a b
2 x  2a  b  2 x b  2a
 
2 x 2a  b  2 x 2ab
2a  b b  2a
 
2 x 2a  b  2 x 2ab
1 1
 
x 2a  b  2 x ab
 2 x 2  2ax  bx  ab  0
 2x x  a  b x  a  0
 x  a 2x  b  0
 x  a  0 or 2 x  b  0
b
 x  a , or x 
2

31. Prove that the tangent at any point of a circle is perpendicular to the radius through the point of
contact.
Solution:

www.vedantu.com 20 / 23
CBSE-X-2013 EXAMINATION

Given: A circle with centre O and a tangent XY to the circle at a point P


To Prove: OP is perpendicular to XY.
Construction: Take a point Q on XY other than P and join OQ.
Proof: Here the point Q must lie outside the circle as if it lies inside the tangent XY will become secant to
the circle.
Therefore, OQ is longer than the radius OP of the circle, That is, OQ > OP.
This happens for every point on the line XY except the point P.
So OP is the shortest of all the distances of the point O to the points on XY.
And hence OP is perpendicular to XY.
Hence, proved.

32. Find the number of terms of the AP -12, -9, -6,... 12. If 1 is added to each term of this AP, then find the
sum of all terms of the AP thus obtained.
Solution:
Given AP is -12, -9, -6, ..., 21
First term, a = -12
Common difference, d = 3
Let 21 be the nth term of the A.P.
21 = a + (n - 1)d
 21 = -12 + (n - 1)  3
 33 = (n - 1)  3
 n = 12
Sum of the terms of the AP = S12
n 12
 2a  n  1 d   24  11  3  54
2 2
If 1 is added to each term of the AP, the sum of all the terms of the new AP will increase by n, i.e., 12.
Sum of all the terms of the new AP = 54 + 12 = 66

33. Two poles of equal heights are standing opposite each other on either side of the roads, which is 80 m
wide. From a point between them on the road, the angles of elevation of the top of the poles are 60 and
30 respectively. Find the height of the poles and the distances of the point from the poles.
Solution:
Let AC and BD be the two poles of the same height h m.

www.vedantu.com 21 / 23
CBSE-X-2013 EXAMINATION

Given AB = 80 m
Let AP = x m, therefore, PB = (80 - x) m
In  APC,
AC
tan  
AP
1 h
 ......(1)
3 x
In  BPD,
BD
tan60 
AB
h
3 ....(2)
80  x
Dividing (1) by (2),
1 h
3  x
3 h
80  x
1 80  x
 
3 x
 x  240  3x
 4 x  240
 x  60
From (1),
1 h

3 x
60
h  20 3m
3
Thus, the height of both the poles is 20 3 m and the distances of the point from the poles are 60 m and 20
m.

34. If the area of triangle ABC formed by A(x,y), B(1,2) and C(2,1) is 6 square units, then prove that x + y =
15.
Solution:

www.vedantu.com 22 / 23
CBSE-X-2013 EXAMINATION

The given vertices are A(x,y), B(1,2) and C(2,1).


It is know that the area of a triangle whose vertices are (x1, y1), (x2,y2) and (x3, y3) is given by
1
| x1 y2  y3  x2 y3  y1  x3 y1  y2 |
2
Area of  ABC
1
 | x 2  1  1  1  y  2 y  2|
2
1
 | x  1  y  2y  4|
2
1
 | x  y  3|
2
The area of  ABC is given as 6 sq units.
1
  x  y  3  6  x  y  3  12
2
 x  y  15

www.vedantu.com 23 / 23
CBSE-XII-2017 EXAMINATION CBSE-X-2014 EXAMINATION

MATHEMATICS
Paper & Solution
Time: 3 Hrs. Max. Marks: 90
General Instuctions :
1. All questions are compulsory.
2. The question paper consists of 34 questions divided into four sections A, B, C, and D.
3. Section A contains of 8 questions of 1 mark each, which are multiple choice type question, Section B
contains of 6 questions of 2 marks each, Section C contains of 10 questions of 3 marks each and Section D
contains of 10 questions of 4 marks each.
4. Use of calculator is not permitted.

SECTION – A
Q-1 The first three terms of an AP respectively are 3y – 1, 3y +5 and 5y +1. Then y equals

(A) -3

(B) 4

(C) 5

(D) 2

Solution:
The first three terms of an AP are 3y-1, 3y+5 and 5y+1, respectively.
We need to find the value of y.
We know that if a, b and c are in AP, then:
b − a = c − b ⇒ 2b = a + c
∴ 2 (3y+5) = 3y – 1 +5y +1
⇒6y +10 = 8y
⇒10 = 8y -6y
⇒ 2y =10
⇒y=5
Hence the correct option is C.

Q-2 In Fig. 1, QR is a common tangent to the given circles, touching externally at the point T. The tangent at
T meets QR at P. If PT = 3.8 cm, then the length of QR (in cm) is:

www.vedantu.com 1 / 29
CBSE-X-2014 EXAMINATION

(A) 3.8

(B) 7.6

(C) 5.7

(D) 1.9

Solution:

It is known that the length of the tangents drawn from an external point to a circle is equal.
∴ QP = PT = 3.8 cm ... (1)
PR = PT = 3.8 cm ... (2)
From equations (1) and (2), we get:
QP = PR = 3.8 cm
Now, QR = QP + PR
= 3.8 cm + 3.8 cm
= 7.6 cm
Hence, the correct option is B.

3. In Fig. 2, PQ and PR are two tangents to a circle with centre O. If ∠QPR = 46° , then ∠QOR equals:

www.vedantu.com 2 / 29
CBSE-X-2014 EXAMINATION

(A) 67°

(B) 134°

(C) 44°

(D) 46°

Solution:
Given: ∠QPR = 46°
PQ and PR are tangents.
Therefore, the radius drawn to these tangents will be perpendicular to the tangents.
So, we have OQ ⊥ PQ and OR ⊥ RP.
⇒ ∠OQP = ∠ORP = 90°
So, in quadrilateral PQOR, we have
∠OQP +∠QPR + ∠PRO + ∠ROQ = 360°
⇒ 90° + 46° + 90° + ∠ROQ = 360°
⇒ ∠ROQ = 360° − 226° = 134°
Hence, the correct option is B.

4. A Ladder makes an angle of 60° with the ground when placed against a wall. If the foot of the ladder is 2
m away from the wall, then the length ( in meters) is:

4
(A)
3

(B) 4 3

(C) 2 2

(D) 4

Solution:

www.vedantu.com 3 / 29
CBSE-X-2014 EXAMINATION

In the figure, MN is the length of the ladder, which is placed against the wall AB and makes an angle of 60°
with the ground.
The foot of the ladder is at N, which is 2 m away from the wall.
∴ BN = 2 m
In right-angled triangle MNB:

BN 2m
cos60  
MN MN
1 2m
 
2 MN
 MN  4m
Therefore, the length of the ladder is 4 m.
Hence, the correct option is D

Q5. If two different dice are rolled together, the probability of getting an even number on both dice, is:

(A) 1/36

(B) 1/2

(C) 1/6

(D) 1/4

Solution:
Possible outcomes on rolling the two dice are given below:
{(1, 1), (1, 2), (1, 3), (1, 4), (1, 5), (1, 6),
(2, 1), (2, 2), (2, 3), (2, 4), (2, 5), (2, 6),

www.vedantu.com 4 / 29
CBSE-X-2014 EXAMINATION

(3, 1), (3, 2), (3, 3), (3, 4), (3, 5), (3, 6),
(4, 1), (4, 2), (4, 3), (4, 4), (4, 5), (4, 6),
(5, 1), (5, 2), (5, 3), (5, 4), (5, 5), (5, 6),
(6, 1), (6, 2), (6, 3), (6, 4), (6, 5), (6, 6)}
Total number of outcomes = 36
Favourable outcomes are given below:
{(2, 2), (2, 4), (2, 6), (4, 2), (4, 4), (4, 6), (6, 2), (6, 4), (6, 6)}
Total number of favourable outcomes = 9
∴ Probability of getting an even number on both dice =
Total number of favourable outcomes 9 1
 
Total number of outcomes 36 4

Hence, the correct option is D.

Q6. A number is selected at random from the numbers 1 to 30. The probability that it is a prime number

(A) 2/3

(B) 1/6

(C) 1/3

(D) 11/30

Solution:
Total number of possible outcomes = 30
Prime numbers between 1 to 30 are 2, 3, 5, 7, 11, 13, 17, 19, 23 and 29.
Total number of favourable outcomes = 10
∴ Probability of selecting a prime number from 1 to 30
Total number of favourable outcomes 10 1
=  
Total number of outcomes 30 3

Hence, the correct option is C.

Q7 If the points A(x, 2), B (-3,-4) and C (7, -5) are collinear, then the value of x is:

(A) -63

(B) 63

(C) 60

www.vedantu.com 5 / 29
CBSE-X-2014 EXAMINATION

(D) -60

Solution:
It is given that the three points A(x, 2), B(−3, −4) and C(7, −5) are collinear.
∴ Area of ΔABC = 0
1
⇒ [x (y - y ) + x2(y3 - y1) + x3(y1 - y2) ] = 0
2 1 2 3

Here, x1 =x, y1 =2, x2 = -3, y2 = -4, and x3 = 7, y3 =-5

⇒ x[-4 - (-5)] - 3(-5 - 2) + 7[2 -(-4) ] = 0


⇒ x(-4 + 5) - 3(-5 -2) + 7(2 + 4) = 0
⇒ x - 3 × (-7) + 7 × 6 = 0
⇒ x + 21 + 42 = 0 ⇒ x + 63 = 0
⇒ x = -63
Thus, the value of x is − 63.
Hence, the correct option is A.

Q8 The number of solid of solid spheres, each of diameter 6cm that can be made by melting a solid metal
cylinder of height 45 cm and diameter 4 cm is:
(A) 3

(B) 5

(C) 4

(D) 6

Solution:
Let r and h be the radius and the height of the cylinder, respectively.
Given: Diameter of the cylinder = 4 cm
∴ Radius of the cylinder, r = 2 cm
Height of the cylinder, h = 45 cm
2 2 3 3
Volume of the solid cylinder = πr h =π × (2) × 45 cm = 180π cm
Suppose the radius of each sphere be R cm.
Diameter of the sphere = 6 cm
∴ Radius of the sphere, R = 3 cm

www.vedantu.com 6 / 29
CBSE-X-2014 EXAMINATION

Let n be the number of solids formed by melting the solid metallic cylinder.
∴ n × Volume of the solid spheres = Volume of the solid cylinder

4
 n  R 3  180
3
4
 n  R 3  180 k
3
180  3
n 5
4  27
Thus, the number of solid spheres that can be formed is 5.
Hence, the correct option is B.

SECTION-B
Q9 Solve the quadratic equation 2x2 +ax – a2 = 0 for x.
Solution:
We have: 2x2 + ax –a2 = 0
Comparing the given equation with the standard quadratic equation (ax2 + bx + c = 0), we get a =2, b = a
and c =-a2

Using the quadratic formula, x  b  b  4ac , we get:


2

2a

a  a2  4  2  (a)2
x
22
a  9a2

4
a  3a

4
a  3a a a  3a
x  or x   a
4 2 4

So, the solutions of the given quadratic equation are x  a or x  a.


2

Q10. The first and the last terms of an AP are 5 and 45 respectively. If the sum of all its terms is 400, Find
its common difference.
Solution:
Let a be the first term and d be the common difference.

Given: a = 5

www.vedantu.com 7 / 29
CBSE-X-2014 EXAMINATION

Tn  45
Sn  400

We know:

Tn  a  (n  1)d
 45  5  (n  1)d
 40  (n  1)d ..........(1)
n
And Sn  a  Tn
2
n
 400  (5  45)
2
n 400
 
2 50
 n  2  8  16
On substituting n = 16 in (1), we get:
40 =(16-1)d
⇒ 40 = (15)d
40 8
⇒d = 15  3

Thus, the common difference is 83.

Q11. Prove that the line segment joining the point of contact of two parallel tangles of a circle passes
through its centre.
Solution:
Let XBY and PCQ be two parallel tangents to a circle with centre O.
Construction: Join OB and OC.

Now, XB ||AO

www.vedantu.com 8 / 29
CBSE-X-2014 EXAMINATION

⇒ ∠XBO +∠AOB = 180° (sum of adjacent interior angles is 180°)


Now, ∠XBO = 90° (A tangent to a circle is perpendicular to the radius through the point of contact)
⇒90° +∠AOB =180°
⇒∠AOB = 180° - 90° =180°
Similarly , ∠AOC = 90⁰
∠AOB + ∠AOC = 90⁰ + 90⁰= 180°
Hence, BOC is a straight line passing through O.
Thus, the line segment joining the points of contact of two parallel tangents of a circle passes through its
centre.

Q12 If from an external point P of a circle with centre O, two tangents PQ and PR are drawn such that
∠QPR = 120° , prove that 2PQ = PO.
Solution:
Let us draw the circle with extent point P and two tangents PQ and PR.

We know that the radius is perpendicular to the tangent at the point of contact.
∴∠OQP = 90°
We also know that the tangents drawn to a circle from an external point are equally inclined to the joining
the centre to that point.
∴∠QPO = 60°
Now, in ΔQPO:
PQ
Cos 60°=
PO

www.vedantu.com 9 / 29
CBSE-X-2014 EXAMINATION

1 PQ
⇒ 
2 PO

⇒2PQ = PO

Q13 Rahim tosses two different coins simultaneously. Find the probability of getting at least one tail.
Solution:
Rahim tosses two coins simultaneously. The sample space of the experiment is {HH, HT, TH, and TT}.
Total number of outcomes = 4
Outcomes in favour of getting at least one tail on tossing the two coins = {HT, TH, TT}
Number of outcomes in favour of getting at least one tail = 3
∴ Probability of getting at least one tail on tossing the two coins
Number of favourable outcomes 3
 
Total number of outcomes 4

Q14 In fig. 3, a square OABC is inscribed in a quadrant OPBQ of a circle. If OA = 20 cm, find the area of the
shaded region (Use π= 3.14)

Let us join OB.

www.vedantu.com 10 / 29
CBSE-X-2014 EXAMINATION

In ΔOAB: OB2 = OA2 +AB2 = (20)2 + (20)2 = 2 × (20) 2

 OB  20 2
Radius of the circle, r  20 2cm

Area of quadrant OPBQ    r 2


360

90
  3.14  (20 2)2 cm2
360
1
  3.14  800 cm2
4
 628 cm2

Area of square OABC = (Side)2 = (20)2 cm2 = 400 cm2


∴ Area of the shaded region = Area of quadrant OPBQ – Area of square OABC

= (628 - 400) cm

 228 cm2

SECTION-C
4 5
Q15 Solve the equation 3 ; x  0,  3/2, for x.
x 2 x  3'

Solution:
Given equation:

4 5 3
3  ; x   
x 2x  3 2
4 5
3 
x 2x  3
4  3x 5
 
x 2x  3
 (4  3x)(2 x  3)  5x

⇒-6x2 +8x-9x+12 =5x


⇒6x2 +6x -12 =0
⇒ x2 +x -2 =0
⇒ x2 +2x –x-2 =0
⇒(x+2) (x-1) =0
⇒(x+2) =0 (x–1)=0

www.vedantu.com 11 / 29
CBSE-X-2014 EXAMINATION

⇒ x = -2 or x = 1
Thus, the solution of the given equation is -2 or 1.

Q16. If the seventh term of an AP is 1/9 and its ninth term is 1/7, find its 63rd term.
Solution:
Let a be the first term and d be the common difference of the given A.P.
Given:
1
a7 
9
1
a9 
7
1
a7  a  (7  1)d 
9

1
 a  6d  ................(1)
9
1
a9  a  (9  1)d 
7
1
 a  8d  .................(2)
7
Subtracting equation (1) from (2), we get:
2
2d 
63
1
d 
63
1
Putting d  in equation (1), we get:
63

 1  1
a  6  
 63  9
1
a
63
1  1  63
a63  a  (63  1)d   62     1
63  63  63

Thus, the 63rd term of the given A.P. is 1.

Q17. Draw a right triangle ABC is which AB = 6 cm, BC = 8 cm and ∠B = 90°. Draw BD perpendicular from B
on AC and draw a circle passing through the points B, C and D. Construct tangents from A to this circle.

www.vedantu.com 12 / 29
CBSE-X-2014 EXAMINATION

Solution:
Follow the given steps to construct the figure.
Step 1
Draw a line BC of 8 cm length.
Step 2
Draw BX perpendicular to BC.
Step 3
Mark an arc at the distance of 6 cm on BX. Mark it as A.
Step 4
Join A and C. Thus, ΔABC is the required triangle.
Step 5
With B as the centre, draw an arc on AC.
Step 6
Draw the bisector of this arc and join it with B. Thus, BD is perpendicular to AC.
Step 7
Now, draw the perpendicular bisector of BD and CD. Take the point of intersection as O.
Step 8
With O as the centre and OB as the radius, draw a circle passing through points B, C and D.
Step 9
Join A and O and bisect it Let P be the midpoint of AO.
Step 10
Taking P as the centre and PO as its radius, draw a circle which will intersect the circle at point B and G.
Join A and G.
Here, AB and AG are the required tangents to the circle from A.

www.vedantu.com 13 / 29
CBSE-X-2014 EXAMINATION

Q18. If the point A (0,2) is equidistant from the points B(3, p) and C(p, 5), find P. Also find the the length of
AB.
Solution:
The given points are A (0, 2), B (3, p) and C (p, 5).
It is given that A is equidistant from B and C.
∴ AB = AC
⇒ AB2 = AC2
⇒ (3-0)2 + (p-2)2 = (p-0)2 + (5-2)2
⇒ 9 + p2+ 4- 4p = p2 +9

⇒ 4 - 4p =0

⇒ 4p =4

⇒ p =1

Thus, the value of p is 1

Length of AB  (3  0)2  (1  2)2  32  (1)2  9  1  10units.

Q19. Two ships are there in the sea on either side of a light house in such a way that the ships and the light
house are in the same straight line. The angles of depression of two ships as observed from the top of the
light house are 60° and 45°. If the height of the light house is 200 m, find the distance between the two
ships. [Use 3  1.73 ]

www.vedantu.com 14 / 29
CBSE-X-2014 EXAMINATION

Solution:
Let d be the distance between the two ships. Suppose the distance of one of the ships from the light house
is X meters, then the distance of the other ship from the light house is (d-x) meter.

In right –angled ΔADO, we have.

OD 200
tan45  
AD X
200
1
X
 X  200 ............(1)

In right-angled Δ BDO, we have

OD 200
tan60  
BD d  x
200
 3
dx
200
dx 
3
Putting x=200. We have:

200
d  200 
3
200
d  200
3
 31 
 d  200  
 3 
 d  200  1.58
 d  316 (approx.)

Thus, the distance between two ships is approximately 316 m.

www.vedantu.com 15 / 29
CBSE-X-2014 EXAMINATION

Q20 If the points A(-2. 1), B (a, b) and C (4, -1) are collinear and a-b = 1, find the values of a and b.
Solution:
The given points are A (-2, 1), B (a, b) and C (4-1).
Since the given points are collinear, the area of the triangle ABC is 0.
1

2
 x1 (y2  y3 )  x2 (y3  y1 )  x 3(y1  y2 )  0
Here, x1=-2,y1 =1,x2=a, y2 =b and x3=4, y3=-1

1
  2(b  1)  a(1  1)  4(1  b)  0
2
 2b  2  2a  4  4b  0
 2a  6b  2
 a  3b  1 .............(1)

Given :
a-b= 1 ………(2)
Subtracting equation (1) from (2) we get:
4b =0
⇒b=0
Subtracting b= 0 in (2), we get:
a= 1
Thus, the values of a and b are 1 and 0, respectively.

Q21 In Fig 4, a circle is inscribed in an equilateral triangle ABC of side 12 cm. Find the radius of inscribed
circle and the area of the shaded region. [Use π =3.14 and 3  1.73 ]

Solution:
It is given that ABC is an equilateral triangle of side 12 cm.
Construction:

www.vedantu.com 16 / 29
CBSE-X-2014 EXAMINATION

Join OA, OB and OC.


Draw.
OP ⊥ BC
OQ ⊥ AC
OR ⊥ AB

Let the radius of the circle be r cm.


Area of ΔAOB +Area of ΔBOC + Area of ΔAOC = Area of ΔABC

1 1 1 3
  AB  OR   BC  OP   AC  OQ   (Side)2
2 2 2 4
1 1 1 3
  12  r   12  r   12  r   (12)2
2 2 2 4
1 3
 3   12  r   12  12
2 4
 r  2 3  2  1.73  3.46

Therefore the radius of the inscribed circle is 3.46 cm.


Now, area of the shaded region = Area of ΔABC – Area of the inscribed circle

 3 
  (12)2  (2 3)2  cm2
 4 
 36 3  12  cm2

= [36 ×1.73 – 12 × 3.14] cm2


= [62.28 – 37.68] cm2

= 24.6 cm2
Therefore, the area of the shaded region is 24.6 cm2.

www.vedantu.com 17 / 29
CBSE-X-2014 EXAMINATION

Q22. In Fig.5. PSR, RTQ and PAQ are three semicircles of diameters 10cm, 3cm and 7 cm respectively. Find
the perimeter of the shaded region. [Use π = 3.14]

Solution:
Radius of Semicircle PSR = 1 × 10 cm =5 cm
2

Radius of Semicircle RTQ = 1 × 3 = 1.5 cm


2

Radius of semicircle PAQ = 1 ×7 cm = 3.5 cm


2

Perimeter of the shaded region = Circumference of semicircle PSR + Circumference of semicircle RTQ +
Circumference of semicircle PAQ

1 1 1 
   2(5)   2(1.5)   2(3.5) cm
2 2 2 
 (5  1.5  3.5)cm
 3.14  10cm
 31.4 cm

Q23 A farmer connects a pipe of internal diameter 20 cm from a canal into a cylindrical tank which is 10 m
in diameter and 2 m deep? If the water flows through the pipe at the rate of 4 km per hour, in how much
time will the tank be filled completely?

Solution:
For the given tank.

Diameter = 10 m

Radius, R = 5m

Depth, H =2m

Internal radius of the pipe = r = 20 cm =10 cm = 1 m


2 10

Rate of flow of water = v = 4 km/h = 4000 m/h

www.vedantu.com 18 / 29
CBSE-X-2014 EXAMINATION

Let t be the time taken to fill the tank.

So, the water flown through the pipe in t hours will equal to the volume of the tank

r 2  v  t  R 2H
2
 1 
    4000  t  (5)2  2
 10 
25  2  100 1
t  1
4000 4
1
Hence, the time taken is 1 hours.
4

Q24. A solid metallic right circular cone 20 cm high and whose vertical angle is 60°, is cut into two parts at
the middle of its height by a plane parallel to its base. If the frustum so obtained be drawn into a wire of
1
diameter cm, find the length of the wire.
12

Solution:

Let ACB be the cone whose vertical angle ∠ACB = 60°. Let R and x be the radii of the lower and upper end
of the frustum.
Here, height of the cone, OC = 20 cm = H
Height CP = h = 10 cm
Let us consider P as the mid-Point of OC.
After cutting the cone into two parts through P.
20
OP = = 10 cm
2

1
Also, ∠ACO and ∠OCB = × 60° = 30°
2

www.vedantu.com 19 / 29
CBSE-X-2014 EXAMINATION

After cutting cone CQS from cone CBA, the remaining solid obtained is a frustum.
Now, in triangle CPQ:

x
tan30 
10
1 x
 
3 10
10
x cm
3

In triangle COB:
R
Tan30 
CO

1 R
 
3 20
20
R cm
3
1
Volume of the frustum, V  ( R 2 H  x 2 h)
3

1   20  
2 2
 10 
 V       
3   3   
20 10
 3 

1  8000 1000 
   
3  3 3 
1  7000 
  
3  3 
1
   7000
9
7000
 
9
The volumes of the frustum and the wire formed are equal.
2
 1  7000
  l   [Volume of wire  r 2 h]
 24  9

7000
l   24  24
9
 l  448000cm  4480m
Hence, the length of the wire is 4480 m.

www.vedantu.com 20 / 29
CBSE-X-2014 EXAMINATION

SECTION –D

Q 25. The difference of two natural number is 5 and the difference of their reciprocals is 110. Find the
numbers.

Solution:
Let the two natural numbers be X and Y such that x > y.
Given:
Difference between the natural numbers = 5
∴ X - Y =5 …..(1)
1
Difference of their reciprocals  (given)
10
1 1 1
 
y x 10
xy 1
 
xy 10

5 1
 
xy 10
 xy  50 ..............(ii)

Putting the value of x from equation (i) in equation (ii), we get


(y+5) y =50
⇒ y2 +5y - 50 = 0
⇒ y2 + 10y -5y - 50 = 0
⇒ y (y+10) – 5 (y+10) = 0
⇒ (y -5) (y + 10) = 0
⇒ y = 5 or -10
As y is a natural number, therefore y = 5
Other natural number = y + 5 = 5 + 5 = 10
Thus, the two natural numbers are 5 and 10.

Q26. Prove that the length of the tangents drawn from an external point to a circle are equal.

Solution:
Let AP and BP be the two tangents to the circle with centre O.

www.vedantu.com 21 / 29
CBSE-X-2014 EXAMINATION

To Prove : AP = BP

Proof:

In ΔAOP and ΔBOP

OA =OB (radii of the same circle)

∠OAP = ∠OBP = 90° (since tangent at any point of a circle is perpendicular to the radius through the point
of contact)
OP = OP (common)
∴ ΔAOP ≅ Δ OBP (by R.H.S. congruence criterion)
∴ AP = BP (corresponding parts of congruent triangles)
Hence the length of the tangents drawn from an external point to a circle are equal.

Q27. The angles of elevation and depression of the top and the bottom of a tower from the top Of a
building, 60 m high, are 30° and 60° respectively. Find the difference between the heights of the building
and the tower and the distance between them.

Solution:
Let AB be the building and CD be the tower.

In right ΔABD.

www.vedantu.com 22 / 29
CBSE-X-2014 EXAMINATION

AB
 tan60
BD
60
  3
BD
60
 BD 
3
 BD  20 3

In right ΔACE:

CE
 tan30
AE
CE 1
  ( AE  BD)
BD 3
20 3
 CE   20
3

Height of the tower = CE + ED = CE + AB = 20 m + 60 m = 80 m


Difference between the heights of the tower and the building = 80 m – 60m = 20 m

Distance between the tower and the building = BD = 20 3 m.

Q28. A bag contains cards numbers from 1 to 49. A card is drawn from the bag at random, after mixing the
cards thoroughly. Find the probability that the number on the drawn card is:

(1) An odd number

(2) A multiple of 5

(3) A perfect Square

(4) An even prime number.

Solution:
Total number of cards = 49
(1)
Total number of outcomes = 49
The odd numbers form 1 to 49 are 1, 3,5,7,9,11,13,15,17, 19,21, 23, 2527,29,31,33,35, 37, 39, 41, 43, 45,
47 and 49.
Total number of favourable outcomes = 25
Total number of favourable outcomes 25
∴ Required probability  
Total number of outcomes 49

www.vedantu.com 23 / 29
CBSE-X-2014 EXAMINATION

(ii) Total number of outcomes = 49


The number 5,10,15,20,25,30,35,40 and 45 multiples of 5.
The number of favourable outcomes = 9
Total number of favourable outcomes 9
∴ Required probability  
Total number of outcomes 49

(iii) Total number of outcomes = 49


The number 1, 4, 9, 16, 25, 36 and 49 are perfect squares.
Total number of favourable outcomes = 7
Total number of favourable outcomes 7 1
∴ Required probability   
Total number of outcomes 49 7

(iv)
Total number of outcomes = 49
We know that there is only one even prime number which is 2
Total number of favourable outcomes = 1
Total number of favourable outcomes 1
∴ Required probability  
Total number of outcomes 49

Q29. Find the ratio in which the point P (X, 2) divides the line segment joining the points A (12, 5)and B (4, -
3). Also find the value of X
Solution:
Let the Point P (x, 2) divide the line segment joining the points A (12, 5) and B (4, -3) in the ratio k: 1

 4k  12 3k  5 
Then, the coordinates of P are  , 
 k 1 k 1 

Now, the coordinates of P are (x,2)

4k  12 3k  5
  x and 2
k 1 k 1
3k  5
2
k 1
 3k  5  2k  2
 5k  3
3
k
5
3 4k  12
Substituting k  in  x , we get
5 k 1

www.vedantu.com 24 / 29
CBSE-X-2014 EXAMINATION

3
4   12
x 5
3
1
5
12  60
x
35
72
x
8
 x 9
Thus, the value of x is 9

3
Also, the point P divides the line segment joining the points A (12, 5) and (4, -3 ) in the ratio : 1, i.e. 3:5.
5

Q30. Find the values of k for which the quadratic equation (k + 4) x2 + (k + 1)x + 1 = 0 has equal roots. Also
find these roots.
Solution:
Given quadratic equation:

(k + 4)x2 + (k + 1)x + 1=0

Since the given quadratic equation has equal roots, Its discriminant should be zero.

∴D=0

⇒(k+1)2- 4 × (k+4) × 1 = 0

⇒k2+2k+1- 4k - 16=0

⇒k2- 2k -15 =0
⇒k2 - 5k+ 3k -15 = 0
⇒(k-5) (k+3) =0
⇒k-5 =0 or k+3 =0
⇒k = 5 or -3
Thus, the values of k are 5 and -3
For k = 5 (k+4)x2 +(k+1)x +1= 0
⇒9x2 +6x +1 = 0
⇒(3x)2 +2(3x) +1 =0
⇒(3x +1)2 =0

www.vedantu.com 25 / 29
CBSE-X-2014 EXAMINATION

1 1
 x  ,
3 3

For k = -3 (k +4)x2 +(k+1)x + 1 = 0


⇒x2 – 2x +1 =0
⇒(x - 1)2 = 0
⇒ x = 1, 1
1
Thus, the equal root of the given quadratic equation is either 1 or 
3

Q31. In an AP of 50 terms, the sum of first 10 terms is 210 and the sum of its last 15 terms is 2565. Find the
A.P.
Solution:
Let a and d be the first term and the common difference of an A. P. respectively.
nth term of an A. P, an = a + ( n -1 )d
n
Sum of n terms of an A. P , Sn  [2a   n  1  d ]
2

We have:
10
Sum of the first 10 terms = [2a  9d ]
2

 210 = 5[2a+9d ]

⇒ 42 = 2a+9d ………. (1)


15th term from the last = ( 50−15 + 1)th= 36th term from the beginning
Now, a36 = a + 35d
15
∴ Sum of the last 15 terms = (2a36  (15  1)d )
2

15
 [2(a  35d)  14d]
2
 15[a  35d  7d ]

⇒2565 = 15[a+42d]
⇒ 171= a+42d ………….(2)
From (1) and (2), we get,
d=4

www.vedantu.com 26 / 29
CBSE-X-2014 EXAMINATION

a=3
So, the A. P. formed is 3, 7, 11, 15 …. and 199.

Q32 . Prove that a parallelogram circumscribing a circle is a rhombus.


Solution:
Given ABCD be a parallelogram circumscribing a circle with centre O.
To Prove: ABCD is a rhombus.

We know that the tangents drawn to a circle from an exterior point are equal is length.

∴ AP = AS, BP = BQ, CR =CQ AND DR = DS.

AP +BP +CR +DR = AS +BQ +CQ +DS

(AP +BP) + (CR+DR) = (AS+DS) +BQ +CQ)

∴AB +CD =AD +BC OR 2AB = 2BC (since AB =DC and AD = BC)

∴ AB = BC = DC =AD

Therefore, ABCD is a rhombus.

Q33. Sushant has a vessel, of the form of an inverted cone, open at the top, of height 11 cm and Radius of
top as 2.5 cm and is full of water. Metallic spherical balls each of diameter 0.5 cm are put In the vessel due
2
to which th of the water in the vessel flows out. Find how many balls were put in the vessel. Sushant
5
made the arrangement so that the water that flows out irrigates the Flower beds. What value has been
shown by Sushant?
Solution:

www.vedantu.com 27 / 29
CBSE-X-2014 EXAMINATION

Height (h) of the conical vessel = 11 cm


Radius (r1) of the conical Vessel = 2.5 cm

0.5
Radius (r2) of the metallic spherical balls =  0.25 cm
2

Let n be the number of spherical balls = that were dropped in the the vessel.
Volume of the water spilled = Volume of the spherical balls dropped
2
 Volume of cone = n × Volume of one spherical ball
5

2 1 2 4 3
  r h  n  r
5 3 1 3 2
2 3
 r h  n  10r
1 2
 (2.5)  11  n  10  (0.25)3
2

⇒ 68.75 = 0.15625n

⇒ n = 440

Hence, the number of spherical balls that were dropped in the vessel is 440.

Sushant made the arrangement so that the water that flows out, irrigates the flower beds.

This shows the judicious usage of water.

Q34. From a solid cylinder of height 2.8 cm and diameter 4.2 cm. a conical cavity of the same height and
same diameter is hollowed out. Find the total surface area of the remaining solid. [Take π=22/7]
Solution:
The following figure shows the required cylinder and the conical cavity.

www.vedantu.com 28 / 29
CBSE-X-2014 EXAMINATION

Given Height (b) of the conical Part = Height (h) of the cylindrical part = 2.8 cm
Diameter of the cylindrical part = Diameter of the conical part = 4.2 cm

∴ Radius  of the cylindrical part = Radius  of the conical part = 2.1 cm

Slant height (l) of the conical part  r 2  h2

 (2.1)2  (2.8)2 cm
 4.41  7.81 cm
 12.25cm
 3.5 cm

Total surface area of the remaining solid = Curved surface area of the cylindrical part +Curved surface area
of the conical part + Area of the cylindrical base

 2rh  rl  r 2
 22 22 22 
  2   2.1  2.8   2.1  3.5   2.1  2.1  cm2
 7 7 7 
= (36.96 + 23.1 + 13.86) cm2
=73.92 cm2

Thus, the total surface area of the remaining solid is 73.92 cm2

www.vedantu.com 29 / 29
CBSE-XII-2017 EXAMINATION CBSE-X-2015 EXAMINATION

MATHEMATICS
Paper & Solution
Time: 3 Hrs. Max. Marks: 90
General Instructions:
(i) All questions are compulsory.
(ii) The question paper consists of 31 questions divided into four sections – A, B, C and D.
(iii) Section A contains 4 questions of 1 mark each. Section B contains 6 questions of 2 marks each, Section
C contains 10 questions of 3 marks each and Section D contains 11 questions of 4 marks each.
(iv) Use of calculators is not permitted.

SECTION – A
Question numbers 1 to 4 carry 1 mark each.

1. If the quadratic equation px2 - 2 5px + 15 = 0 has two equal roots then find the value of p.
Solution:
Given quadratic equation is,
px 2  2 5px  15  0
Here, a = p, b = 2 5 p, c= 15
For real equal roots, discriminant = 0
 b2  4ac  0

 
2
 2 5p  4 p(15)  0
20 p2  60 p  0
20 p(p  3)  0
 p  3 or p  0
But, p = 0 is not possible.
∴ p=3

2. In the following figure, a tower AB is 20 m high and BC, its shadow on the ground, is 20 3 m long. Find
the Sun’s altitude.

www.vedantu.com 1 / 23
CBSE-X-2015 EXAMINATION

Solution:
Let AB be the tower and BC be its shadow.

AB = 20, BC = 20 3

In  ABC,
AB
tan  
BC
20
tan  
20 3
1
tan  
3
1
but, tan  
3
  30

The Sun is at an altitude of 30.

3. Two different dice are tossed together. Find the probability that the product of the two numbers on the
top of the dice is 6.
Solution:
Two dice are tossed
S = [(1,1),(1,2),(1,3),(1,4),(1,5),(1,6),
(2,1),(2,2),(2,3),(2,4),(2,5),(2,6),
(3,1),(3,2),(3,3),(3,4),(3,5),(3,6),
(4,1),(4,2),(4,3),(4,4),(4,5),(4,6),
(5,1),(5,2),(5,3),(5,4), (5,5),(5,6),
(6,1),(6,2),(6,3),(6,4),(6,5),(6,6)]
Total number of outcomes when two dice are tossed = 6  6= 36
Favourable events of getting product as 6 are:
(1  6= 6), (6  1 =6),(2  3= 6),(3  2 =6)
i.e.(1,6), (6,1), (2,3), (3,2)
Favourable events of getting product as 6 = 4

www.vedantu.com 2 / 23
CBSE-X-2015 EXAMINATION

4 1
 P(getting product as 6)  
36 9

4. In the following figure, PQ is a chord of a circle with centre O and PT is a tangent. If ∠QPT = 60˚, find
∠PRQ

Solution:

m  OPT 90 ( radius is perpendicular to the tangent)


So,  OPQ =  OPT -  QPT
= 90 - 60
= 30
m  POQ = 2  QPT 2  60 =120
reflex m  POQ = 360 - 120 =240
1
mPRQ  reflexPOQ
2
1
  240
2

PRQ = 120

SECTION B
Question numbers 5 to 10 carry 2 marks each.
5. In the following figure, two tangents RQ and RP are drawn from an external point R to the circle with
centre O, If ∠PRQ = 120˚, then prove that OR = PR + RQ.

www.vedantu.com 3 / 23
CBSE-X-2015 EXAMINATION

Solution:

Given that m  PRQ = 120


We know that the line joining the centre and the external point is the angle bisector between the tangents.
120
Thus, mPRO  mQRO   60
2
Also we know that lengths of tangents from an external point are equal.
Thus, PR = RQ.
Join OP and OQ.
Since OP and OQ are the radii from the centre O,
OP  PR and OQ  RQ.
Thus,  OPR and  OQR are right angled congruent triangles.
Hence, POR  90  PRO  90  60  30
QOR  90QRO  90 60  30
1
sin QRO  sin30 
2
PR 1

OR 2
Thus,  OR  2PR
 OR  PR  PR
 OR  PR  QR

www.vedantu.com 4 / 23
CBSE-X-2015 EXAMINATION

6. In Figure 4, a ΔABC is drawn to circumscribe a circle of radius 3 cm, such that the segments BD and DC
are respectively of lengths 6 cm and 9 cm. If the area of Δ ABC is 54 cm2, then find the lengths of sides AB
and AC.

Solution:

Let the given circle touch the sides AB and AC of the triangle at points F and E respectively and let the
length of line segment AF be x.
Now, it can be observed that:
BF = BD = 6 cm (tangents from point B)
CE = CD = 9 cm (tangents from point C)
AE = AF = x (tangents from point A)
AB = AF + FB = x + 6
BC = BD + DC = 6 + 9 = 15
CA = CE + EA = 9 + x
2s = AB + BC + CA = x + 6 + 15 + 9 + x = 30 + 2x
s = 15 + x
s – a = 15 + x – 15 = x
s – b = 15 + x – (x + 9) = 6
s – c = 15 + x – (6 + x) = 9
Area of  ABC s(s  a)(s  b)(s  c)
54  (15  x)(x)(6)(9)
54  3 6(15x  x 2 )
18  6(15x  x 2 )
324  6(15x  x 2 )
54  15x  x 2

www.vedantu.com 5 / 23
CBSE-X-2015 EXAMINATION

x 2  15x  54  0
x 2  18 x  3x  54  0
x(x  18)  3(x  18)
(x  18)(x  3)  0
x  18 and x  3
As distance cannot be negative, x = 3
AC = 3 + 9 = 12
AB = AF + FB = 6 + x = 6 + 3 = 9

7. Solve the following quadratic equation for x:


4x2 + 4bx – (a2 – b2) = 0
Solution:
4 x 2  4bx  (a2  b2 )  0
 a2  b2 
 x 2  bx   0
 4 
b a 2  b2
 x  2  x 
2

2 4
2 2
 b  b a b b
2 2
 x  2  x    
2
 
2 2 4 2
2
 b  a2
x  
 2 4
b a
x 
2 2
b a
x 
2 2
b  a b  a
x ,
2 2
ab  ab 
Hence, the roots are    and  .
 2   2 

8. In an AP, if S5 + S7 = 167 and S10 = 235, then find the A.P., where Sn denotes the sum of its first n terms
Solution:
S5  S7  167 and S10  235
n
Now, Sn  2a  (n  1)d
2
 S5  S7  167
5 7
 2a  4d  2a  6d  167
2 2
 5a  10d  7a  21d  167
 12a  31d  167 .................(1)
Also, S10 = 235

www.vedantu.com 6 / 23
CBSE-X-2015 EXAMINATION

10
 2a  9d  235
2
 10a  45d  235
 2a  9d  47 ...................(2)
Multiplying equation (2) by 6, we get
12a + 54d = 282 ………………....(3)
Subtracting (1) from (3), we get
12a  54d  282
()12a  31d  167
  
23d  115
d  5
Substituting value of d in (2), we have
2a + 9(5)= 47
 2a + 45= 47
 2a = 2
a = 1
Thus, the given A.P. is 1, 6, 11, 16,..........

9. The points A(4, 7), B(p, 3) and C(7, 3) are the vertices of a right triangle, right-angled at B, Find the values
of P.
Solution:
 ABC is right angled at B.
 AC 2  AB2  BC 2 .............(1)
Also, A     p, 3 and C  (7, 3)
Now, AC 2  (7  4)2  (3  7)2  (3)2  (4)2  9  16  25
AB2  (p  4)2  (3  7)2  p2  8p  16  (4)2
 p2  8p  16  16
 p2  8p  32
BC 2  (7  p)2  (3  3)2  49  14 p  p2  0
 p2  14 p  49
From (1), we have
25= (p2 - 8p + 32)+ (p2 -14p +49)
 25 = 2p2- 22p +81
 2p2 - 22p +56= 0
 p2 - 11p + 28 = 0
 p2 - 7p - 4p+ 28 = 0
 p(p -7) – 4(p – 7) = 0
 (p – 7) (p – 4) = 0
 p = 7 and p = 4

10. Find the relation between x and y if the points A(x, y), B(-5, 7) and C(-4, 5) are collinear.
Solution:

www.vedantu.com 7 / 23
CBSE-X-2015 EXAMINATION

Given, the points A(x,y), B(-5,7) and C(-4,5) are collinear.


So, the area formed by these vertices is 0.
1
  x(7  5)  (5)(5  y)  (4)(y  7)  0
2
1
 2 x  25  5y  4 y  28   0
2
1
  2 x  y  3  0
2
 2x  y  3  0
 y  2 x  3

SECTION C
Question numbers 11 to 20 carry 3 marks each.

11. The 14th term of an A.P. is twice its 8th term. If its 6th term is -8, then find the sum of its first 20 terms.
Solution:
Here it is given that,
T14 = 2(T8)
⇒ a + (14 – 1)d = 2[a + (8 – 1)d]
⇒ a + 13d = 2[a + 7d]
⇒ a + 13d = 2a + 14d
⇒ 13d – 14d = 2a – a
⇒ – d = a …. (1)
Now, it is given that its 6th term is –8.
T6 = –8
⇒ a + (6 – 1)d = – 8
⇒ a + 5d = –8
⇒ –d + 5d = –8 [∵ Using (1)]
⇒ 4d = –8
⇒ d = –2
Subs. this in eq. (1), we get a = 2
Now, the sum of 20 terms,
n
Sn  2a  (n  1)d 
2
20
S20  2a  (20  1)d 
2
= 10 [2(2) + 19(-2)]
= 10[4 - 38]
= -340

12. Solve for x:


3x 2  2 2 x  2 3  0
Solution:
For the given equation, 3x 2  2 2 x  2 3  0
Comparing this equation with ax2 +bx + c = 0, we obtain

www.vedantu.com 8 / 23
CBSE-X-2015 EXAMINATION

a  3, b  2 2, c  2 3
Now, D  b2  4ac

 (2 2)2  4 4 3 2 3  
 8  24  32  4 2
Using quadratic formula, we obtain
b  b2  4ac
x
2a
(2 2)  4 2
x
2 3
2 2 4 2 2 2 4 2
x or x 
2 3 2 3
2 2 2 2 2 2
x or x 
3 3
3 2  2
x or x 
3 3
 2
 x  3 2 or x 
3
 2
 x  6 or x 
3
13. The angle of elevation of an aeroplane from point A on the ground is 60°. After flight of 15 seconds, the
angle of elevation changes to 30°. If the aeroplane is flying at a constant height of 1500 3 m, find the
speed of the plane in km/hr.
Solution:
Let BC be the height at watch the aeroplane is observed from point A.
Then, BC = 1500 3
In 15 seconds, the aeroplane moves from point A to D.
A and D are the points where the angles of elevation 60 and 30 are formed respectively.
Let BA = x metres and AD = y metres
BC = x + y

In CBA,

www.vedantu.com 9 / 23
CBSE-X-2015 EXAMINATION

BC
tan60 
BA
1500 3
3
x
 x  1500 m ...............(1)
In CBD ,
BC
tan30 
BD
1 1500 3

3 xy
 x  y  1500(3)  4500
1500  y  4500
 y  3000 m ..................(2)
We know that, the aeroplane moves from point A to D in 15 seconds and the distance covered is 3000
metres. (by 2)
distance
Speed 
time
3000
Speed 
15
Speed 200m/s
18
Converting it to km/hr  200   720km / hr
5

14. If the coordinates of points A and B are (-2, -2) and (2, -4) respectively, find the coordinates of P such
3
that AP = AB, where P lies on the line segment AB.
7
Solution:
Here, P(x,y) divides line segment AB, such that
3
AP  AB
7
AP 3

AB 7
AB 7

AP 3
AB 7
1  1
AP 3
AB  AP 7  3

AP 3
BP 4

AP 3
AP 3

BP 4
 P divides AB in the ratio 3: 4

www.vedantu.com 10 / 23
CBSE-X-2015 EXAMINATION

3  2  4(2)   


x ;y 
34 34
68 12  8
x ;y 
7 7
2 20
x  ;y 
7 7
 2 20 
 The coordinates of P are  , 
 7 7 

15. The probability of selecting a red ball at random from a jar that contains only red, blue and orange balls
1 1
is . The probability of selecting a blue ball at random from the same jar . If the jar contains 10 orange
4 3
balls, find the total number of balls in the jar.
Solution:
Here the jar contains red, blue and orange balls.
Let the number of red balls be x.
Let the number of blue balls be y.
Number of orange balls = 10
Total number of balls = x + y + 10
Now, let P be the probability of drawing a ball from the jar
x
P(a red ball) 
x  y  10
1 x
 
4 x  y  10
 4 x  x  y  10
 3x  y  10 ............(i)
Next,
y
P (a blue ball) 
x  y  10
1 y
 
3 x  y  10
 3y  x  y  10
 2y  x  10 ..............(ii)
Multiplying eq. (i) by 2 and adding to eq. (ii), we get
6 x  2y  20
 x  2y  10
5x  30
 x 6
Subs. x = 6 in eq. (i), we get y = 8
Total number of balls = x + y + 10 = 6 + 8 + 10 = 24
Hence, total number of balls in the jar is 24.

www.vedantu.com 11 / 23
CBSE-X-2015 EXAMINATION

16. Find the area of the minor segment of a circle of radius 14 cm, when its central angle is 60˚. Also find
22
the area of the corresponding major segment. [Use   ]
7
Solution:
Radius of the circle =14 cm
Central Angle,  = 60 ,
Area of the minor segment
 1
  r 2  r 2 sin 
360 2
60 1
   142   142  sin60
360 2
1 22 1 3
   14  14   14  14 
6 7 2 2
22  14
  49 3
3
22  14 147 3
 
3 3
308  147 3 2
 cm
3
308  147 3 2
Area of the minor segment  cm
3

17. Due to sudden floods, some welfare associations jointly requested the government to get 100 tents
fixed immediately and offered to contribute 50% of the cost. If the lower part of each tent is of the form of
a cylinder of diameter 4.2 m and height 4 m with the conical upper part of same diameter but height 2.8 m,
and the canvas to be used costs Rs. 100 per sq. m, find the amount, the associations will have to pay. What
22
values are shown by these associations? [Use   ]
7
Solution:
Diameter of the tent = 4.2 m
Radius of the tent, r = 2.1 m
Height of the cylindrical part of tent, hcylinder = 4 m
Height of the conical part, hcone = 2.8 m
Slant height of the conical part, l
 hcone2  r 2
 2.82  2.12
 2.82  2.12
 12.25  3.5 m
Curved surface area of the cylinder = 2  r hcylinder
22
 2   2.1  4
7
 22  0.3  8  52.8 m2

www.vedantu.com 12 / 23
CBSE-X-2015 EXAMINATION

22
Curved surface area of the conical tent  rl   2.1  3.5  23.1 m2
7
Total area of cloth required for building one tent
= Curved surface area of the cylinder + Curved surface area of the conical tent
= 52.8 + 23.1
= 75.9 m2
Cost of building one tent = 75.9 × 100 = Rs 7590
Total cost of 100 tents = 7590 × 100 = Rs 7,59,000
759000
Cost to be borne by the associations  = Rs. 3,79,500
2
It shows the helping nature, unity and cooperativeness of the associations.

18. A hemispherical bowl of internal diameter 36 cm contains liquid. This liquid is filled into 72 cylindrical
bottles of diameter 6 cm. Find the height of each bottle, if 10% liquid is wasted in this transfer.
Solution:
Internal diameter of the bowl = 36 cm
Internal radius of the bowl, r = 18 cm
2 2
Volume of the liquid, V  r 3    183
3 3
Let the height of the small bottle be ‘h’.
Diameter of a small cylindrical bottle = 6 cm
Radius of a small bottle, R = 3 cm
Volume of a single bottle =  R2h =  × 32 × h
No. of small bottles, n = 72
10 2
Volume wasted in the transfer     183
100 3
Volume of liquid to be transferred in the bottles
2 10 2
   183     183
3 100 3
2  10 
   183  1  
3  100 
2 90
   183 
3 100
Volume of the liquid to be transferred
Number of small cylindrical bottles 
Volume of a single bottle
2 90
  183 
 72  3 100
 32  h
2 9
 183 
 72  3 2 10
3 h
2 9
 18  18  18 
h 3 10
32  72
 h  5.4 cm

www.vedantu.com 13 / 23
CBSE-X-2015 EXAMINATION

Height of the small cylindrical bottle = 10.8 cm

19. A cubical block of side 10 cm is surmounted by a hemisphere. What is the largest diameter that the
hemisphere can have? Find the cost of painting the total surface area of the solid so formed, at the rate of
Rs. 5 per sq. cm. [Use π = 3.14]
Solution:
Side of the cubical block, a = 10 cm
Longest diagonal of the cubical block = a 3 = 10 3 cm
Since the cube is surmounted by a hemisphere, therefore the side of the cube should be equal to the
diameter of the hemisphere.
Diameter of the sphere = 10 cm
Radius of the sphere, r = 5 cm
Total surface area of the solid = Total surface area of the cube – Inner cross‐section area of the hemisphere
+ Curved surface area of the hemisphere
= 6a2 –  r2 + 2 +  r2
= 6a2 +  r2
 6  (10)2  3.14  52
 600  78.5  678.5 cm2
Total surface area of the solid = 678.5 cm2

20. 504 cones, each of diameter 3.5 cm and height 3 cm, are melted and recast into a metallic sphere, Find
22
the diameter of the sphere and hence find its surface area. [Use   ]
7
Solution:
No. of cones = 504
Diameter of a cone = 3.5 cm
Radius of the cone, r = 1.75 cm
Height of the cone, h = 3 cm
Volume of a cone
1
 r 2h
3
2
1  3.5 
     3
3  2 
1 3.5 3.5
     3 cm3
3 2 2
Volume of 504 cones
1 3.5 3.5
 504      3 cm3
4 2 2
Let the radius of the new sphere be ‘R’.
4
Volume of the sphere  R 3
3
Volume of 504 cones = Volume of the sphere

www.vedantu.com 14 / 23
CBSE-X-2015 EXAMINATION

1 3.5 3.5 4
504      3  R 3
3 2 2 3
504  1   3.5  3.5  3  3
  R3
3 2 2 4  
504  3  49
 R3 
64
7  8  9  3  72
 R3 
64
8  27  73
R 3

64
2 3 7
R
4
21
 R   10.5 cm
2
Radius of the new sphere = 10.5 cm

SECTION D
Question numbers 21 to 31 carry 4 marks each.

21. The diagonal of a rectangular field is 16 metres more than the shorter side. If the longer side is 14
metres more than the shorter side, then find the lengths of the sides of the field.
Solution:
Let l be the length of the longer side and b be the length of the shorter side.
Given that the length of the diagonal of the rectangular field is 16 metres more than the shorter side.
Thus, diagonal = 16 + b
Since longer side is 14 metres more than shorter side, we have,
l = 14 + b
Diagonal is the hypotenuse of the triangle.
Consider the following figure of the rectangular field.

By applying Pythagoras Theorem in  ABD, we have,


Diagonal2 = Length2 = Breadth2
 (16  b)2  (14  b)2  b2
 256  b2  32b  196  b2  28b  b2
 256  32b  196  28b  b2
 60  32b  28b  b2

www.vedantu.com 15 / 23
CBSE-X-2015 EXAMINATION

 b2  4b  60  0
 b2  10b  6b  60  0
 b(b  10)  6(b  10)  0
 (b  6)(b  10)  0
 (b  6)  0 or (b  10)  0
 b  6 or b  10
As breadth cannot be negative, breadth = 10 m
Thus, length of the rectangular field = 14 + 10 = 24 m

22. Find the 60th term of the AP 8, 10, 12, ……., if it has a total of 60 terms and hence find the sum of its last
10 terms.
Solution:
Consider the given A.P. 8, 10, 12, …
Here the initial term is 8 and the common difference is 10 ‐ 8 = 2 and 12 ‐ 10 = 2
General term of an A.P. is tn and formula to find out tn is
tn  a  (n  1)d
 t60  8  (60  1)  2
 t60  8  59  2
 t60  8  118
 t60  126
We need to find the sum of the last 10 terms.
Thus,
Sum of last 10 terms = Sum of first 60 terms ‐ Sum of first 50 terms
n
Sn  2a  (n  1)d 
2
60
 S60  2  8  (60  1)  2
2
 S60  30 16  59  2
 S60  30 134 
 S60  4020
Similarly,
50
 S50  2  8  (50  1)  2
2
 S50  2516  49  2
 S50  25114 
 S50  2850
Thus the sum of last 10 terms =S60 – S50 = 4020 – 2850 = 1170
Therefore,
Sum of last 10 terms = Sum of first 60 terms ‐ Sum of first 50 terms

www.vedantu.com 16 / 23
CBSE-X-2015 EXAMINATION

23. A train travels at a certain average speed for a distance of 54 km and then travels a distance of 63 km at
an average speed of 6 km/h more than the first speed. If it takes 3 hours to complete the total journey,
what is its first speed?
Solution:
Let x be the first speed of the train.
Distance
We know that = time
Speed
Thus, we have,
54 63
  3 hours
x x 6
54(x  6)  63x
 3
x(x  6)
 54(x  6)  63x  3x(x  6)
 54 x  324  63x  3x 2  18 x
 117 x  324  3x 2  18 x
 3x 2  117 x  324  18 x  0
 3x 2  99 x  324  0
 x 2  33x  108  0
 x 2  36 x  3x  108  0
 x(x  36)  3(x  36)  0
 (x  3)( x  36)  0
 (x  3)  0 or ( x  36)  0
 x  3 or x  36
Speed cannot be negative and hence initial speed of the train is 36 km/hour.

24. Prove that the lengths of the tangents drawn from an external point to a circle are equal.
Solution:
Consider the following diagram.

Let P be an external point and PA and PB be tangents to the circle.


We need to prove that PA = PB
Now consider the triangles  OAP and  OBP
m  A = m  B = 90

www.vedantu.com 17 / 23
CBSE-X-2015 EXAMINATION

OP = OP [common]
OA = OB = radii of the circle
Thus, by Right Angle‐Hypotenuse‐Side criterion of congruence we have,
 OAP   OBP
The corresponding parts of the congruent triangles are congruent.
Thus,
PA = PB

25. Prove that the tangent drawn at the mid-point of an arc of a circle is parallel to the chord joining the
end points of the arc.
Solution:
In the figure, C is the midpoint of the minor arc PQ, O is the centre of the circle and
AB is tangent to the circle through point C.
We have to show the tangent drawn at the midpoint of the arc PQ of a circle is parallel to the chord joining
the end points of the arc PQ.
We will show PQ || AB.
It is given that C is the midpoint point of the arc PQ.
So, arc PC = arc CQ.
 PC = CQ

This shows that  PQC is an isosceles triangle.


Thus, the perpendicular bisector of the side PQ of  PQC passes through vertex C.
The perpendicular bisector of a chord passes through the centre of the circle.
So the perpendicular bisector of PQ passes through the centre O of the circle.
Thus perpendicular bisector of PQ passes through the points O and C.
 PQ  OC
AB is the tangent to the circle through the point C on the circle.
 AB  OC
The chord PQ and the tangent PQ of the circle are perpendicular to the same line OC.
PQ || AB.

26. Construct a Δ ABC in which AB = 6 cm, ∠A = 30° and ∠B = 60°, Construct another ΔAB’C’ similar to ΔABC
with base AB’ = 8 cm.
Solution:
Construct the  ABC as per given measurements.
In the half plane of AB which does not contain C, draw AX such that BAX is an acute angle.
3) With some appropriate radius and centre A, Draw an arc to intersect AX at B1. Similarly, with centre B1
and the same radius, draw an arc to intersect BX at B2 such that B1B2 = B3B4 = B4B5 = B5B6 = B6B7 = B7B8

www.vedantu.com 18 / 23
CBSE-X-2015 EXAMINATION

4) Draw B6B.
5) Through B8 draw a ray parallel to B6B. to intersect AY at B’.
6) Through B’ draw a ray parallel to BC to intersect AZ at C’.
Thus,  AB’C’ is the required triangle.

27. At a point A, 20 metres above the level of water in a lake, the angle of elevation of a cloud is 30°. The
angle of depression of the reflection of the cloud in the lake, at a A is 60°. Find the distance of the cloud
from A.
Solution:

Let AB be the surface of the lake and P be the point of observation such that
AP = 20 metres. Let C be the position of the cloud and C’ be its reflection in the lake.
Then CB = C’B. Let PM be perpendicular from P on CB.
Then m  CPM=30 and m  C'PM = 60
Let CM = h. Then CB = h + 20 and C’B = h + 20.
In CMP we have,

www.vedantu.com 19 / 23
CBSE-X-2015 EXAMINATION

CM
tan30 
PM
1 h
 
3 PM
 PM  3h...................(i)
In  PMC' we have,
C M
tan60 
PM
C B  BM
 3
PM
h  20  20
 3
PM
h  20  20
 PM  ...................(ii)
3
From equation (i) and (ii), we get
h  20  20
3h 
3
 3h  h  40
 2h  40
 h  20 m
Now, CB = CM + MB = h + 20 + 20 + 20 = 40.
Hence, the height of the cloud from the surface of the lake is 40 metres.

28. A card is drawn at random from a well-shuffled deck of playing cards. Find the probability that the card
drawn is
i. a card of spade or an ace.
ii. a black king.
iii. neither a jack nor a king
iv. either a king or a queen.
Solution:
Let S be the sample space of drawing a card from a well‐shuffled deck.
n(S)  52C1  52
(i)There are 13 spade cards and 4 ace's in a deck As ace of spade is included in 13 spade cards, so there are
13 spade cards and 3 ace's
a card of spade or an ace can be drawn in 13 C1  3 C1  13  3  16
16 4
Probability of drawing a card of spade or an ace  
52 13
(ii)There are 2 black King cards in a deck a card of black King can be drawn in 2 C1  2
2 1
Probability of drawing a black king  
52 26

(iii) There are 4 Jack and 4 King cards in a deck.

www.vedantu.com 20 / 23
CBSE-X-2015 EXAMINATION

So there are 52 - 8 = 44 cards which are neither Jacks nor Kings. a card which is neither a Jack nor a King
can be drawn in 44C1 = 44
44 11
Probability of drawing a card which is neither a Jack nor a King = 
52 13
(iv)There are 4 King and 4 Queen cards in a deck.
So there are 4 - 4 = 8 cards which are either King or Queen.
a card which is either a King or a Queen can be drawn in 8C1 = 8
8 2
Probability of drawing a card which is either a King or a Queen = 
52 13

29. Find the values of k so that the area of the triangle with vertices (1, -1), (-4, 2k) and (-k, -5) is 24 sq.
units.
Solution:
Take  x1 , y1   1,  1 ,  4, 2k  and  k,  5
It is given that the area of the triangle is 24 sq. unit
Area of the triangle having vertices  x1 , y1  ,  x2 , y2  and  x3 , y3  is given by
1
  x1 (y2  y3 )  x2 (y3  y1 )  x3 (y1  y2 )
2
1
24  1(2k  (5))  (4)((5)  (1))  (k)((1)  2k)
2
48  (2k  5)  16  (k  2k 2 )
2k 2  3k  27  0
(2k  9)(k  3)  0
9
 k   or k  3
2
9
The values of k are  and 3.
2

30. In the following figure, PQRS is square lawn with side PQ = 42 metres. Two circular flower beds are
there on the sides PS and QR with centre at O, the intersections of its diagonals. Find the total area of the
two flower beds (shaded parts).

Solution:
PQRS is a square.
So each side is equal and angle between the adjacent sides is a right angle.
Also the diagonals perpendicularly bisect each other.

www.vedantu.com 21 / 23
CBSE-X-2015 EXAMINATION

In  PQR using pythagoras theorem,


PR2 =PQ2 +QR2
PR2 = (42)2 + (42)2
PR  2(42)
1 42
OR  PR   OQ
2 2
From the figure we can see that the radius of flower bed ORQ is OR.
1
Area of sector ORQ = r 2
4
2
1  42 
 
4  2 
1
Area of the  ROQ   RO  OQ
2
1 42 42
  
2 2 2
2
 42 
 
 2 
Area of the flower bed ORQ
= Area of sector ORQ - Area of the ROQ
2 2
1  42   42 
   
4  2   2 
2
 42    
     1
 2  2 
= (441)[0.57]
= 251.37 cm2
Area of the flower bed ORQ = Area of the flower bed OPS
= 251.37 cm2
Total area of the two flower beds
= Area of the flower bed ORQ + Area of the flower bed OPS
= 251.37 + 251.37
= 502.74 cm2

31. From each end of a solid metal cylinder, metal was scooped out in hemispherical from of same
diameter. The height of the cylinder is 10 cm and its base is of radius 4.2 cm. The rest of the cylinder is
22
melted and converted into a cylindrical wire of 1.4 cm thickness. Find the length of the wire. [Use   ]
7
Solution:
Height of the cylinder (h) = 10 cm
Radius of the base of the cylinder = 4.2 cm
Volume of original cylinder  r 2h
22
  (4.2)2  10
7
 554.4 cm3

www.vedantu.com 22 / 23
CBSE-X-2015 EXAMINATION

2
Volume of hemisphere  r 3
3
2 22
   (4.2)3
3 7
 155.232 cm3
Volume of the remaining cylinder after scooping out hemisphere from each end
= Volume of original cylinder - 2  Volume of hemisphere
 554.4  2  155.232
 243.936 cm3
The remaining cylinder is melted and converted to
a new cylindrical wire of 1.4 cm thickness.
So they have same volume and radius of new cylindrical wire is 0.7 cm.
Volume of the remaining cylinder = Volume of the new cylindrical wire
243.936  r 2h
22
243.936  (0.7)2 h
7
h  158.4 cm
 The length of the new cylindrical wire of 1.4 cm thickness is 158.4 cm.

www.vedantu.com 23 / 23
CBSE-XII-2017 EXAMINATI CBSE – X – 2016 EXAMINATION

MATHEMATICS
SET-1 Paper & Solution
Time: 3 Hrs. Max. Marks: 90

General Instructions:
1. All questions are compulsory.
2. The question paper consists of 31 questions divided into four sections – A, B, C and D.
3. Section A contains 4 questions of 1 mark each. Section B contains 6 questions of 2 marks each, Section C
contains 10 questions of 3 marks each and Section D contains 11 questions of 4 marks each.
4. Use of calculators is not permitted.

SECTION A

Question 1. In the figure, PQ is a tangent at a point C to a circle with centre O. if AB is a diameter and
∠CAB = 30°, find ∠PCA.

Solution: In the given figure,

In ΔACO,
OA = OC …(Radii of the same circle)
 ΔACO is an isosceles triangle.
∠CAB = 30° …(Given)
 ∠CAO = ∠ACO = 30° …. (angles opposite to equal sides of an isosceles triangle are equal)
∠PCO = 90° …(radius drawn at the point of contact is perpendicular to the tangent)
Now ∠PCA = ∠PCO – ∠CAO
 ∠PCA = 90° – 30° = 60°
Marks: 1

www.vedantu.com 1 / 19
Question 2. For what value of k will k + 9, 2k – 1 and 2k + 7 are the consecutive terms of an A.P?
Solution: If k + 9, 2k – 1 and 2k + 7 are the consecutive terms of A.P., then the common difference will be
the same.
∴ (2k – 1) – (k + 9) = (2k + 7) – (2k – 1)
∴ k – 10 = 8
∴ k = 18
Marks: 1

Question 3. A ladder leaning against a wall makes an angle of 60° with the horizontal. If the foot of the
ladder is 2.5 m away from the wall, find the length of the ladder.
Solution:

Let AB be the ladder and CA be the wall.


The ladder makes an angle of 60° with the horizontal.
∴ ΔABC is a 30°-60°-90°, right triangle.
Given: BC = 2.5 m, ∠ABC = 60°
∴ AB = 5 m
Hence, length of the ladder is AB = 5 m.
Marks: 1

Question 4. A card is drawn at random from a well shuffled pack of 52 playing cards. Find the probability
of getting neither a red card nor a queen.
Solution: There are 26 red cards including 2 red queens.
Two more queens along with 26 red cards will be 26 + 2 = 28
28
∴ P(getting a red card or a queen) =
52
28 24 6
∴ P(getting neither a red card nor a queen) = 1 – = =
52 52 13
Marks: 1

www.vedantu.com 2 / 19
SECTION B
Question 5. If -5 is a root of the quadratic equation 2x2 + px – 15 = 0 and the quadratic equation p(x2 + x)k
= 0 has equal roots, find the value of k.
Solution: Given –5 is a root of the quadratic equation 2x2 + px – 15 = 0.
∴ –5 satisfies the given equation.
∴ 2(–5)2 + p(–5) – 15 = 0
∴ 50 – 5p – 15 = 0
∴ 35 – 5p = 0
∴ 5p = 35  p = 7
Substituting p = 7 in p(x2 + x) + k = 0, we get
7(x2 + x) + k = 0
∴ 7x2 + 7x + k = 0
The roots of the equation are equal.
∴ Discriminant = b2 – 4ac = 0
Here, a = 7, b = 7, c = k
b2 – 4ac = 0
∴ (7)2 – 4(7)(k) = 0
∴ 49 – 28k = 0
∴ 28k = 49
49 7
∴ k 
28 4
Marks: 2

Question 6. Let P and Q be the points of trisection of the line segment joining the points A(2, -2) and B(-7,
4) such that P is nearer to A. Find the coordinates of P and Q.
Solution: Since P and Q are the points of trisection of AB, AP = PQ = QB
Thus, P divides AB internally in the ratio 1 : 2
and Q divides AB internally in the ratio 2 : 1.

∴ By section formula,
 1(7)  2(2) 1(4)  2( 2)   7  4 4  4   3 
P ,  ,    , 0   (1, 0)
 1 2 1 2   3 3   3 
 2(7)  1(2) 2(4)  1( 2)   14  2 8  2   12 6 
Q ,  ,  ,   (4, 2)
 2 1 2 1   3 3   3 3
Marks: 2

Question 7. In figure, a quadrilateral ABCD is drawn to circumscribe a circle, with centre O, in such a way
that the sides AB, BC, CD and DA touch the circle at the points P, Q, R and S respectively. Prove that AB +
CD = BC + DA.

www.vedantu.com 3 / 19
Solution: Since tangents drawn from an exterior point to a circle are equal in length,
AP = AS ….(1)
BP = BQ ….(2)
CR = CQ ….(3)
DR = DS ….(4)
Adding equations (1), (2), (3) and (4), we get
AP + BP + CR + DS = AS + BQ + CQ + DS
∴ (AP + BP) + (CR + DR) = (AS + DS) + (BQ + CQ)
∴ AB + CD = AD + BC
∴ AB + CD = BC + DA …..(proved)
Marks: 2

Question 8. Prove that the points (3, 0), (6, 4) and (-1, 3) are the vertices of a right angled isosceles triangle.
Solution: Let A(3, 0), B(6, 4) and C(–1, 3) be the given points.
Now,
AB  (6  3) 2  (4  0) 2  (3) 2  (4) 2  9  16  25
BC  (1  6) 2  (3  4) 2  (7) 2  (1) 2  49  1  50
AC  (1  3) 2  (3  0) 2  (4) 2  (3) 2  16  9  25
∴ AB = AC
AB2 = ( 25) 2 = 25
BC2 = ( 50) 2 = 50
AC2 = ( 25) 2 = 25
∴ AB2 = AC2 = BC2
Thus, ∆ABC is a right-angled isosceles triangle.
Marks: 2

Question 9. The 4th term of an A.P. is zero. Prove that the 25th term of the A.P. is three times its 11th term.
Solution: 4th term of an A.P. = a4 = 0
∴ a + (4 – 1)d = 0
∴ a + 3d = 0
∴ a = –3d ….(1)
th
25 term of an A.P. = a25
= a + (25 – 1)d
= –3d + 24d ….[From (1)]
= 21d
3 times 11th term of an A.P. = 3a11
= 3[a + (11 – 1)d]

www.vedantu.com 4 / 19
= 3[a + 10d]
= 3[–3d + 10d]
= 3 × 7d
= 21d
∴ a25 = 3a11
i.e., the 25th term of the A.P. is three times its 11th term.
Marks: 2

Question 10. In figure, from an external point P, two tangents PT and PS are drawn to a circle with centre O
and radius r. If OP = 2r, show that ∠ OTS = ∠ OST = 30°.

Solution:

In the given figure,


OP = 2r … (Given)
∠OTP = 90° … (radius drawn at the point of contact is perpendicular to the tangent)
In ΔOTP,
OT 1
sin ∠OPT = = = sin30°
OP 2
∠OPT = 30°
∴ ∠TOP = 60°
∴ ΔOTP is a 30°-60°-90°, right triangle.
In ΔOTS,
OT = OS … (Radii of the same circle)
∴ ΔOTS is an isosceles triangle.
∴ ∠OTS = ∠OST … (Angles opposite to equal sides of an isosceles triangle are equal)
In ΔOTQ and ΔOSQ
OS = OT … (Radii of the same circle)
OQ = OQ ... (side common to both triangles)
∠OTQ = ∠OSQ … (angles opposite to equal sides of an isosceles triangle are equal)
∴ ΔOTQ ≅ ΔOSQ … (By S.A.S)
∴ ∠TOQ = ∠SOQ = 60° … (C.A.C.T)

www.vedantu.com 5 / 19
∴ ∠TOS = 120° … (∠TOS = ∠TOQ + ∠SOQ = 60° + 60° = 120°)
∴ ∠OTS + ∠OST = 180° – 120° = 60°
∴ ∠OTS = ∠OST = 60° ÷ 2 = 30°
Marks: 2

SECTION C
Question 11. In figure, O is the centre of a circle such that diameter AB = 13 cm and AC = 12 cm. BC is
joined. Find the area of the shaded region. (Take π = 3.14)

Solution: Diameter, AB = 13 cm
13
∴ Radius of the circle, r = = 6.5 cm
2
∠ACB is the angle in the semi-circle.
∴ ∠ACB = 90°
Now, in ∆ACB, using Pythagoras theorem, we have
AB2 = AC2 + BC2
(13)2 = (12)2 + (BC)2
(BC)2 = (13)2 – (12)2 = 169 – 144 = 25
BC = 25 = 5 cm
Now, area of shaded region = Area of semi-circle – Area of ACB
1 1
  r 2   BC  AC
2 2
1 1
  3.14  (6.5) 2   5 12
2 2
= 66.33 – 30
= 36.33 cm2
Thus, the area of the shaded region is 36.33 cm2.
Marks: 3

Question 12. In figure, a tent is in the shape of a cylinder surmounted by a conical top of same diameter. If
the height and diameter of cylindrical part are 2.1 m and 3 m respectively and the slant height of conical part
is 2.8 m, find the cost of canvas needed to make the tent if the canvas is available at the rate of Rs. 500/sq.
 22 
metre.  Use   
 7 

www.vedantu.com 6 / 19
Solution:

For conical portion, we have


r = 1.5 m and l = 2.8 m
∴ S1 = Curved surface area of conical portion
∴ S1 = πrl
= π × 1.5 × 2.8
= 4.2π m2
For cylindrical portion, we have
r = 1.5 m and h = 2.1 m
∴ S2 = Curved surface area of cylindrical portion
∴ S2 = 2πrh
= 2 × π × 1.5 × 2.1
= 6.3π m2
Area of canvas used for making the tent = S1 + S2
= 4.2π + 6.3 π
= 10.5 π
22
= 10.5 ×
7
2
= 33 m

www.vedantu.com 7 / 19
Total cost of the canvas at the rate of Rs. 500 per m2 = Rs. (500 × 33) = Rs. 16500.
Marks: 3

Question 13. If the point P(x, y) is equidistant from the points A(a + b, b – a) and B(a – b, a + b). Prove that
bx = ay.
Solution: P(x, y) is equidistant from the points A(a + b, b – a) and B(a – b, a + b).
∴ AP = BP
[ x  (a  b)]2  [ y  (b  a)]2  [ x  (a  b)]2  [ y  (a  b)]2
[ x  (a  b)]2  [ y  (b  a)]2  [ x  (a  b)]2  [ y  (a  b)]2
x 2  2 x(a  b)  (a  b) 2  y 2  2 y(b  a)  (b  a) 2  x 2  2 x(a  b)  (a  b) 2  y 2  2 y (a  b)  (a  b) 2
2 x(a  b)  2 y (b  a)  2 x(a  b)  2 y (a  b)
ax + bx + by – ay = ax – bx + ay + by
2bx = 2ay
∴ bx = ay ....(proved)
Marks: 3

Question 14. In figure, find the area of the shaded region, enclosed between two concentric circles of radii 7
 22 
cm and 14 cm where ∠AOC = 40°.  Use   
 7 

Solution: Area of the region ABDC = Area of sector AOC – Area of sector BOD
40 22 40 22
=   14  14   77
360 7 360 7
1 1
=  22 14  2   22  7 1
9 9
22
=  (28  7)
9
22
=  21
9
154
=
3
= 51.33 cm2
22 22
Area of circular ring =  14  14   7  7
7 7
= 22 × 14 × 2 – 22 × 7 × 1
= 22 × (28 – 7)
= 22 × 21

www.vedantu.com 8 / 19
= 462 cm2
∴ Required shaded region = Area of circular ring – Area of region ABDC
= 462 – 51.33
= 410.67 cm2
Thus, the area of shaded region is 410.67 cm2
Marks: 3

Question 15. If the ratio of the sum of first n terms of two A.P’s is (7n +1): (4n + 27), find the ratio of their
mth terms.
Solution: Let a1, a2 be the first terms and d1, d2 the common differences of the two given A.P’s.
n n
Thus, we have S n  [2a1  (n  1)d1 ] and S n '  [2a2  (n  1)d 2 ]
2 2
n
[2a1  (n  1)d1 ]
Sn 2 2a  (n  1)d1
   1
Sn ' n [2a  (n  1)d ] 2a2  (n  1)d 2
2 2
2
S 7n  1
It is given that n 
Sn ' 4n  27
2a1  (n  1)d1 7n  1
  ....(1)
2a2  (n  1)d 2 4n  27
To find the ratio of the mth terms of the two given A.P's, replace n by (2m – 1) in equation (1).
2a  (2m  1  1)d1 7(2m  1)  1
 1 
2a2  (2m  1  1)d 2 4(2m  1)  27
2a1  (2m  2)d1 14m  7  1
 
2a2  (2m  2)d 2 8m  4  27
a1  (m  1)d1 14m  6
 
a2  (m  1)d 2 8m  23
Hence, the ratio of the mth terms of the two A.P's is 14m – 6 : 8m 23.
Marks: 3

1 1 2
Question 16. Solve for x:   , x  1, 2,3
( x  1)( x  2) ( x  2)( x  3) 3
Solution:
1 1 2
 
( x  1)( x  2) ( x  2)( x  3) 3
( x  3)  ( x  1) 2

( x  1)( x  2)( x  3) 3
x  3  x 1 2

( x  3x  2)( x  3) 3
2

2x  4 2

x  3x  3x  9 x  2 x  6 3
3 2 2

2x  4 2

x  6 x  11x  6 3
3 2

www.vedantu.com 9 / 19
6 x  12  2 x 3  12 x 2  22 x  12
2 x 3  12 x 2  16 x  0
2 x( x 2  6 x  8)  0
x2  6x  8  0
x2  4x  2x  8  0
x( x  4)  2( x  4)  0
( x  4)( x  2)  0
x  4  0 or x  2  0
x  4 or x  2
Marks: 3

Question 17. A conical vessel, with base radius 5 cm and height 24 cm, is full of water. This water is
emptied into a cylindrical vessel of base radius 10 cm. Find the height to which the water will rise in the
 22 
cylindrical vessel.  Use   
 7 
Solution: Let the radius of the conical vessel = r1 = 5 cm
Height of the conical vessel = h1 = 24 cm
Radius of the cylindrical vessel = r2
Let the water rise upto the height of h2 cm in the cylindrical vessel.
Now, volume of water in conical vessel = volume of water in cylindrical vessel
1 2
πr1 h1 = r22h2
3
r12 h1 = 3r22h2
5 × 5 × 24 = 3 × 10 × 10 × h2
5  5  24
h2 = = 2 cm
3  10  10
Thus, the water will rise upto the height of 2 cm in the cylindrical vessel.
Marks: 3

Question 18. A sphere of diameter 12 cm, is dropped in a right circular cylindrical vessel, partly filled with
5
water. If the sphere is completely submerged in water, the water level in the cylindrical vessel rises by 3
9
cm. Find the diameter of the cylindrical vessel.
Solution: Radius of sphere = r = 6 cm
4 4
Volume of sphere = πr3 = π × (6)3 = 288π cm3
3 3
Let R be the radius of cylindrical vessel.
5 32
Rise in the water level of cylindrical vessel = h = 3 cm = cm
9 9
32 32 2
Increase in volume of cylindrical vessel = πR2h = πR2 × = πR
9 9
Now, volume of water displaced by the sphere is equal to volume of sphere.

www.vedantu.com 10 / 19
32 2
∴ πR = 288π
9
288  9
∴ R2 = = 81
32
∴ R = 9 cm
∴ Diameter of the cylindrical vessel = 2 × R = 2 × 9 = 18 cm
Marks: 3

Question 19. A man standing on the deck of a ship, which is 10 m above water level, observes the angle of
elevation of the top of a hill as 60° and the angle of depression of the base of a hill as 30°. Find the distance
of the hill from the ship and the height of the hill.
Solution:

Let CD be the hill and suppose the man is standing on the deck of a ship at point A.
The angle of depression of the base C of the hill CD observed from A is 30° and the angle of elevation of the
top D of the hill CD observed from A is 60°.
∴ ∠EAD = 60° and ∠BCA = 30°
In ΔAED,
DE
tan 60 
EA
h
3
x
h= 3x ....(1)
In ΔABC,
AB
tan 30 
BC
1 10

3 x
x = 10 3 ....(2)
Substituting x = 10 3 in equation (1), we get
h = 3 × 10 3 = 10 × 3 = 30
DE = 30 m

www.vedantu.com 11 / 19
CD = CE + ED = 10 + 30 = 40 m
Thus, the distance of the hill from the ship is 10 3 m and the height of the hill is 40 m.
Marks: 3

Question 20. Three different coins are tossed together. Find the probability of getting
(i) exactly two heads
(ii) at least two heads
(iii) at least two tails.
Solution: When three coins are tossed together, the possible outcomes are
HHH, HTH, HHT, THH, THT, TTH, HTT, TTT
∴ Total number of possible outcomes = 8
(i) Favourable outcomes of exactly two heads are HTH, HHT, THH
∴ Total number of favourable outcomes = 3
3
∴ P(exactly two heads) =
8
(ii) Favourable outcomes of at least two heads are HHH, HTH, HHT, THH
∴ Total number of favourable outcomes = 4
4 1
∴ P(at least two heads) = =
8 2
(iii)Favourable outcomes of at least two tails are THT, TTH, HTT, TTT
∴ Total number of favourable outcomes = 4
4 1
∴ P(at least two tails)= =
8 2
Marks: 3

SECTION D
Question 21. Due to heavy floods in a state, thousands were rendered homeless. 50 schools collectively
offered to the state government to provide place and the canvas for 1500 tents to be fixed by the
governments and decided to share the whole expenditure equally. The lower part of each tent is cylindrical
of base radius 2.8 cm and height 3.5 m, with conical upper part of same base radius but of height 2.1 m. If
the canvas used to make the tents costs Rs. 120 per sq. m, find the amount shared by each school to set up
 22 
the tents. What value is generated by the above problem?  Use   
 7 
Solution: Height of conical upper part = 3.5 m, and radius = 2.8 m
(Slant height of cone)2 = 2.12 + 2.82 = 4.41 + 7.84
Slant height of cone = 12.25 = 3.5 m
The canvas used for each tent
= curved surface area of cylindrical base + curved surface area of conical upper part
= 2πrh + πrl
= πr(2h + l)
22
= × 2.8(7 + 3.5)
7
22
= × 2.8 × 10.5
7

www.vedantu.com 12 / 19
= 92.4 m2
So, the canvas used for one tent is 92.4 m2.
Thus, the canvas used for 1500 tents = (92.4 × 1500) m2.
Canvas used to make the tents cost Rs. 120 per sq. m.
So, canvas used to make 1500 tents will cost Rs. 92.4 × 1500 × 120.
The amount shared by each school to set up the tents
92.4  1500  120
= = Rs. 332640
50
The amount shared by each school to set up the tents is Rs. 332640.
The value to help others in times of troubles is generated from the problem.
Marks: 4

Question 22. Prove that the lengths of the tangents drawn from an external point to a circle are equal.
Solution: Consider a circle centered at O.
Let PR and QR are tangents drawn from an external point R to the circle touching at points P and Q
respectively.
Join OR.

Proof:
In ∆OPR and ∆OQR,
OP = OQ ... (Radii of the same circle)
∠OPR = ∠OQR …. (Since PR and QR are tangents to the circle)
OR = OR ... (Common side)
∴ ∆OPR ≅ ∆OQ R ….(By R.H.S)
∴ PR = QR ….(c.p.c.t)
Thus, tangents drawn from an external point to a circle are equal.
Marks: 4

Question 23. Draw a circle of radius 4 cm. Draw two tangents to the circle inclined at an angle of 60° to
each other.
Solution: Steps of construction:
(i) Take a point O on the plane of the paper and draw a circle of radius OA = 4 cm.
(ii) Produce OA to B such that OA = AB = 4 cm.
(iii) Draw a circle with centre at A and radius AB.
(iv) Suppose it cuts the circle drawn in step (i) at P and Q.
(v) Join BP and BQ to get the desired tangents.

www.vedantu.com 13 / 19
Justification:
In ∆OAP, OA = OP = 4 cm ... (radii of the same circle)
Also, AP = 4 cm ….(Radius of the circle with centre A)
∴ ∆OAP is equilateral.
∴ ∠PAO = 60°
∴ ∠BAP = 120°
In ∆BAP, we have BA = AP and ∠ BAP = 120°
∴ ∠ABP = ∠APB = 30°
Similarly we can get ∠ABQ = 30°
∴ ∠PBQ = 60°
Marks: 4

Question 24. In figure, two equal circles, with centres O and O’, touch each other at X. OO’ produced meets
the circle with centre O’ at A. AC is tangent to the circle with centre O, at the point C. O’D is perpendicular
DO'
to AC. Find the value of .
CO

Solution: AO’ = O’X = XO = OC …..(Since the two circles are equal.)


So, OA = AO’ + O’X + XO …..(A-O’-X-O)
∴ OA = 3O’A
In ∆AO'D and ∆AOC,
∠DAO' = ∠CAO .... (Common angle)
∠ADO' = ∠ACO .... (both measure 90 )
∆ADO' ~ ∆ACO .... (By AA test of similarity)
DO ' O ' A O ' A 1
  
CO OA 3O'A 3
Marks: 4

www.vedantu.com 14 / 19
1 2 4
Question 25. Solve for x:   ; X  1, 2, 4
x 1 x  2 x  4
1 2 4
Solution:  
x 1 x  2 x  4
L.C.M. of all the denominators is (x + 1)(x + 2)(x + 4)
Multiply throughout by the L.C.M., we get
(x + 2)(x + 4) + 2(x + 1)(x + 4) = 4(x + 1)(x + 2)
(x + 4)(x + 2 + 2x +2) = 4(x2 + 3x + 2)
(x + 4)(3x + 4) = 4x2 + 12x + 8
3x2 + 16x + 16 = 4x2 + 12x + 8
∴ x2 – 4x – 8 = 0
Now, a = 1, b = –4, c = –8
b  b 2  4ac 4  16  32 4  48 4  4 3
x   
2a 2 2 2
∴ x = 2 2 3
Marks: 4

Question 26. The angle of elevation of the top Q of a vertical tower PQ from a point X on the ground is 60°.
From a point Y, 40 m vertically above X, the angle of elevation of the top Q of tower is 45°. Find the height
of the tower PQ and the distance PX. (Use 3 = 1.73)
Solution:

MP = YX = 40 m
∴ QM = h – 40
In right angled ∆QMY,
QM h  40
tan 45  1 ....(MY = PX)
MY PX
∴ PX = h – 40 ....(1)
In right angled ∆QPX,
QP QP
tan 60   3
PX PX
h
PX = ....(2)
3
h
From (1) and (2), h – 40 =
3
∴ 3 h – 40 3 = h
∴ 3 h – h = 40 3
∴ 1.73h – h = 40(1.73)  h = 94.79 m

www.vedantu.com 15 / 19
Thus, PQ is 94.79 m.
Marks: 4

Question 27. The houses in a row numbered consecutively from 1 to 49. Show that there exists a value of X
such that sum of numbers of houses preceding the house numbered X is equal to sum of the numbers of
houses following X.
Solution: Let there be a value of x such that the sum of the numbers of the houses preceding the house
numbered x is equal to the sum of the numbers of the houses following it.
That is, 1 + 2 + 3 + . . . . + (x – 1) = (x + 1) + (x + 2) + . . . . . + 49
∴ 1 + 2 + 3 + . . . . + (x – 1)
= [1 + 2 + .... + x + (x – 1) + .... + 49] – (1 + 2 + 3 +. . . . + x)
x 1 49 x
∴ [1  x  1]  [1  49]  [1  x]
2 2 2
∴ x(x – 1) = 49 × 50 – x(1 + x)
∴ x(x – 1) + x(1 + x) = 49 × 50
∴ x2 – x + x + x2 = 49 × 50
∴ x2 = 49 × 50
∴ x2 = 49 × 25
∴ x = 7 × 5 = 35
Since x is not a fraction, the value of x satisfying the given condition exists and is equal to 35.
Marks: 4

Question 28. In figure, the vertices of ∆ABC are A(4, 6), B(1, 5) and C(7, 2). A line-segment DE is drawn
AD AE 1
to intersect the sides AB and AC at D and E respectively such that   . Calculate the area of
AB AC 3
∆ADE and compare it with area of ∆ABC.

Solution:
AD AE 1
 
AB AC 3
AB AC
  3
AD AE
AD  DB AE  EC
  3
AD AE
DB EC
1  1 3
AD AE

www.vedantu.com 16 / 19
DB EC
  2
AD AE
AD AE 1
  
DB EC 2
 AD : DB  AE : EC  1: 2
So, D and E divide AB and AC respectively in the ratio 1:2.
So the coordinates of D and E are
 1  8 5  12   17   7  8 2  12   14 
 ,    3,  and  ,    5,  respectively.
 1 2 1 2   3   1 2 1 2   3 

Area of ∆ADE
1  17 14   17 14 
  4   3  5 6    3 6  5  4  
2  3 3   3 3
1  68   85 56 
   14  30   18   
2  3   3 3 
1  68  42  90   54  85  56 
   
2 3   3 
1  200   195 
   
2 3   3 
1 5
 
2 3
5
= sq. units
6

Area of ∆ABC
1
 | (4  5  1 2  7  6)  (1 6  7  5  4  2) |
2
1
 | (20  2  42)  (6  35  8) |
2
1
 | (64)  (49) |
2
15
= sq. units
2
Area of ADE 5 / 6 1
  
Area of ABC 15 / 2 9
Marks: 4

www.vedantu.com 17 / 19
Question 29. A number x is selected at random from the numbers 1, 2, 3, and 4. Another number y is
selected at random from the numbers 1, 4, 9 and 16. Find the probability that product of x and y is less than
16.
Solution: x is selected from 1, 2, 3 and 4
1, 2, 3, 4
y is selected from 1, 4, 9 and 16
Let A = {1, 4, 9, 16, 2, 8, 18, 32, 3, 12, 27, 48, 36, 64} which consists of elements that are product of x and
y
Number of outcomes less than 16
P(product of x and y is less than 16) =
Total number of outcomes
7

14
1

2
Marks: 4

Question 30. In figure, is shown a sector OAP of a circle with centre O, containing ∠θ. AB is perpendicular
to the radius OQ and meets OP produced at B. Prove that the perimeter of shaded region is
  
r  tan   sec   1 .
 180 

Solution: Perimeter of shaded region = AB + PB + arc length AP ...(1)


  r
Arc length AP =  2 r  ....(2)
360 180
In right angled ∆OAB,
AB
tan θ =  AB = r tan θ ....(3)
r
OB
sec θ =  OB = r sec θ
r
OB = OP + PB
∴ r sec θ = r + PB
∴ PB = r sec θ – r .....(4)
Substitute (2), (3) and (4) in (1), we get
Perimeter of shaded region = AB + PB + arc length AP
 r
 r tan   r sec   r 
180
  
 r  tan   sec   1
 180 

www.vedantu.com 18 / 19
Marks: 4

Question 31. A motor boat whose speed is 24 km/h in still water takes 1 hour more to go 32 km upstream
than to return downstream to the same spot. Find the speed of the stream.
Solution: Let the speed of the stream be s km/h.
Speed of the motor boat 24 km/h
Speed of the motor boat upstream 24 – s
Speed of the motor boat downstream 24 + s
According to the given condition,
32 32
 1
24  s 24  s
 1 1 
 32    1
 24  s 24  s 
 24  s  24  s 
 32   1
 576  s 
2

∴ 32 × 2s = 576 – s 2

∴ s2 + 64s – 576 = 0
∴ (s + 72)(s – 8) = 0
∴ s = –72 or s = 8
Since, speed of the stream cannot be negative, the speed of the stream is 8 km/h.
Marks: 4

www.vedantu.com 19 / 19
CBSE Class 10
Mathematics
Previous Year Question Paper 2011
Series: RHB/1 Code no. 30/1/1

● Please check that this question paper contains 16 printed pages.

● Code number given on the right hand side of the question paper should be
written on the title page of the answer-book by the candidate.

● Please check that this question paper contains 34 questions.

● Please write down the serial number of the question before attempting
it.

● 15 minutes time has been allotted to read this question paper. The question
paper will be distributed at 10.15 a.m. From 10.15 a.m. to 10.30 a.m., the
students will read the question paper only and will not write any answer on the
answer script during this period.

SUMMATIVE ASSESSMENT-II
MATHEMATICS

Time Allowed: 3 hours Maximum Marks: 80


General Instructions
1. All questions are compulsory.
2. The question paper consists of 34 questions divided into four sections A, B,
C and D.
3. Section A contains 30 questions of 1 mark each, which are multiple choice
type questions, Section B contains 8 questions of 2 marks each, Section C
contains 10 questions of 3 marks each, Section D contains 6 questions of 4
marks each.
4. There is no overall choice in the paper. However, internal choice is provided
in one question of 2 marks. 3 questions of 3 marks each and two questions
of 4 marks each.

Class X Mathematics www.vedantu.com 1


5. Use of calculators is not permitted.

SECTION-A
Question Numbers 1 to 30 carry 1 mark each. For each of the questions 1
to 30, four alternative choices have been provided, of which only one is
correct. Select the correct choice.
1. The roots of the equation x 2 +x-p ( p+1) =0 , where p is a constant, are:

(A) p,p+1

(B) -p,p+1

(C) p,-(p+1)

(D) -p,-(p+1) 1 Mark

Ans: The given equation is x 2 +x-p(p+1)=0 . Now, solving this equation using
-b± b 2 -4ac
Quadratic formula, i.e. x= Putting the values in the formula, we
2a
get :

1± 1-4 (1) ( -p ( p+1) )


⇒ x=
2
1± 1+4p 2 +p
⇒ x=
2

( 2p+1)
2
-1± 1 ( 2p+1)
⇒ x= =- ±
2 2 2
Thus, the two roots are:
1 1
α=- +p+ =p
2 2
1 1
β=- -p- =-p-1
2 2
Therefore, option is the correct answer.

2. In an AP, if, d=-2, n=5 and an=0, then the value of a is : 1 Mark

Class X Mathematics www.vedantu.com 2


a) 10
(B) 5
(C) -8
(D) 8
Ans: It is given that
d=-2
n=5
an=0
The formula for an is given as an=a+(n-1)d . Putting the values, we get
0=a+(5-1)-2
0=a+4(-2)
a=8
Hence, option (D) 8 is the correct answer.

3. In Fig. 1, O is the centre of a circle, AB is a chord and AT is the tangent


at A. If
< AOB =100o then < BAT is equal to: 1 Mark

Ans: It is given in this question that AO and BO are radius therefore, triangle
AOB is an isosceles triangle:
< AOB + < OBA + < OAB = 180
Putting values, we get
100 + < OAB + < OAB = 180
⇒ 2 < AOB=80o

Class X Mathematics www.vedantu.com 3


⇒< AOB=40o
We know that tangent and radius are perpendicular to each other,
∴< OAT=40o
Simplifying,
∴< OAB+<BAT=40o
<BAT=90o -40o =50o
Hence, option (C) 50 is the correct answer.

4. In Fig. 2, PA and PB are tangents to the circle with centre O. If <APB


=60o, then < OAB is : 1 Mark

a. 30o

b. 60o

c. 90o

d. 15o

Ans: It is given that PA and PB are two tangents. O is the centre of the circle
and OA and OB are joined. Given: < APB = 60 . We need to find < OAB PA
and PB are tangents to the circle, so, PA = PB , < PAB = < PBA. But < APB
= 60 . So, < PAB + < PBA = 180 - 60 = 120 .
⇒ 2 < PAB = 120
⇒ <PAB=60o
Now, OA is radius and PA is tangent, hence, OA ⊥ PA,
⇒ <OPA=90o

Class X Mathematics www.vedantu.com 4


⇒ <OAB+<PAB=90o
Putting the values, we get
⇒ <OAB+60o =90o
⇒ <OAB=30o
Hence, option (A) 30 is the correct answer.

5. The radii of two circles are 4 cm and 3 cm respectively. The diameter of


the circle having area equal to the sum of the areas of the two circles (in
cm) is: 1 Mark

a. 5
b. 7
c. 10
d. 14
Ans: The radius of one circle is 4 cm. Therefore, the area of the circle is :

A=π× ( 4 ) =16π . The radius of the second circle is 3 cm. Therefore, the area
2

of the circle is: A=π× ( 3) =9π . Thus, the sum of the areas of two circles is:
2

A'=9π+16π
⇒ πr'2 =25π
⇒ r'=5cm
Hence, the diameter of the circle is 10 cm. So, option (C) 10 is the correct
answer.

6. A sphere of diameter 18 cm is dropped into a cylindrical vessel of


diameter 36 cm, partly filled with water. If the sphere is completely
submerged, then the water level rises (in cm) by : 1 Mark

a) 3

(B) 4

Class X Mathematics www.vedantu.com 5


(C) 5

(D) 6

Ans: It is given that: The diameter of the sphere is 18 cm. So, the radius is 9
cm. The diameter of the cylindrical vessel is 36 cm. So, the radius is 18 cm. Let
us consider the height of water rise is H cm. Now, the sphere is completely
submerged, Volume of the vessel is equal to the volume of the sphere. Putting
the values, we get.
4 3
πR 2 H= πr
3
4 3
r 4× ( 9 )
3
3
⇒ H= 2 = =3cm
3× (18 )
2
R

Hence, option (A)3 is the correct answer.

7. The angle of elevation of the top of a tower from a point on the ground,
which is 30 m away from the foot of the tower is 45°. The height of the tower
(in metres) is:

a) 15

(B) 30

(C) 30 3

(D) 10 3 1 Mark

Ans: Let us draw a figure for the given tower and the point in the question :

Class X Mathematics www.vedantu.com 6


Let us consider the height of the tower to be h cm, now, in ΔOPQ
PQ
, tan45o =
OQ
Now the distance of the tower from the point is 30m, so,
h
tan45=
30
h
⇒ 1=
30
⇒ h=30m
Hence, option (B)30 m is the correct answer.

8. The point P which divides the line segment joining the points A(2, - 5)
and B(5, 2) in the ratio 2:3 lies in the quadrant: 1 Mark
a) I
(B) II
(C) III
(D) IV
Ans: Given the points A(2, - 5) and B(5, 2) are divided in the ratio 2:3. Thus,
the point P is given by:
 2×5+3×2 2×2-3×5   11 
P=  ,  =  3,- 
 2+3 2+3   5
Thus, the point clearly lies in the fourth quadrant. Hence option (D) IV is the
correct answer.

9. The midpoint of segment AB is the point P( 0, 4). If the coordinates of


Bare (-2, 3) then the coordinates of A are :
(A) (2,5)

(B) (-2,-5)

(C) (2,9)

Class X Mathematics www.vedantu.com 7


(D) (-2,11) 1 Mark

Ans: It is given that P( 0, 4) is the midpoint of segment AB .The point B is (-2,


3). Let the point A be (x, y), then,

x+ ( -2 )
. Solving this we get; x=2. For the point y.
2

y+3
=4
2

⇒ y=8-3=5 . Hence, the point A is given by (2,5). Therefore, option (a) (2, 5) is
the correct answer.

10. Which of the following cannot be the probability of an event? 1 Mark

(A) 1.5

3
(B)
5

(C) 25%

(D) 0.3
Ans: Out of these four options, 1.5 cannot be a probability of an event because
it is greater than 1 and probability of any event cannot be greater than 1.

SECTION-B
Question Numbers 11 to 18 carry 2 marks each.
11. Find the value of p so that the quadratic equation px(x - 3) + 9 = 0 has
two equal roots. 2 Marks

Ans: Given the quadratic equation is px ( x-3) +9=0 .Simplifying this equation
further, we get, px 2 -3px+9=0 .

-b± b 2 -4ac
Using the quadratic formula, we get x= .
2a

Class X Mathematics www.vedantu.com 8


3p± 9p 2 -4p ( 9 )
Putting the values, we get: x= . It is given that the equation has
2p
two equal roots, this implies, 9p 2 -4p ( 9 ) =0 . Solving this to find p, we get

9p 2 -4p ( 9 ) =0

9p ( p-4 ) =0

⇒ p=0,4
Therefore, the value of p is (0,4).

12. Find whether -150 is a term of the AP 17, 12, 7, 2,..........? 2 Marks

Ans: The given AP is 17,12,7,2………… Thus, we can see here that:


a=17
d=5
If -150 is a term, then, an =-150.Expanding the term to find the value of n,
an = a+(n-1)d
⇒ -150=17+(n-1)5
⇒ -150-17=(n-1)5
-167
⇒ =n-1
5
162
⇒ n=-
5
But this can’t be an actual nth term, thus, -150 does not belong to this AP.

13. Two concentric circles are of radii 7 cm and r cm respectively, where r


>7. A chord of the larger circle, of length 48cm touches the smaller circle,
find the value of r. 2 Marks

Ans: First of all, let us draw a figure of the two concentric circles,

Class X Mathematics www.vedantu.com 9


We need to find the value of r. In the figure, we can clearly see that point P is
the midpoint of AB. So, in ΔOPB , using Pythagoras theorem, we get:
OB2 =OP 2 +PB2

⇒ r 2 = ( 7 ) + ( 24 )
2 2

⇒ r 2 =49+576
⇒ r 2 =625
⇒ r 2 =25cm
Hence, the value of r is 25cm.

14. Draw a line segment of length 6 cm. Using compasses and ruler, find a
point P on it which divides it in the ratio 3:4. 2 Marks
Ans: In order to solve this question, we are first of all going to draw a line
segment AB which is equal to length 6cm.

Next, you need to put the compass on points A and B and a perpendicular
bisector of AB is drawn. The point at which the perpendicular bisector meets

Class X Mathematics www.vedantu.com 10


AB is named O. The compass is put on points O and B and a perpendicular
bisector of OB is drawn. Then, you should note that the Perpendicular bisector
meets OB at a point which is named P which as given in the question divides
the segment into 3:4.Thus,
3
AP= of AB
4

15. Fig. 3, APB and CQD are semi-circles of diameter 7 cm each, while
ARC and BSD are semi-circles of diameter 14 cm each. Find the
22
perimeter of the shaded region. [Use π= ]. 2 Marks
7

Ans: Let us consider the semicircle ARC and BSD. They have diameters 14cm
each, thus, the perimeter of ARC and BSD is equal to:
p1 =p 2 =2πr=2π ( 7 ) =14π

The semicircles APB and CQD have diameters 7cm each.


Perimeter of the shaded region = Length of APB + Length of ARC + Length of
CQD + Length of DSB . Now,
1  7  22 7
perimeter of APB ×2π  cm  = × =11cm .
2 2  7 2
1 22
Perimeter of ARC = ×2π ( 7cm ) = ×7cm=22cm
2 7
1  7  22 7
Perimeter of CQD = ×2π  cm  = × cm=11cm
2 2  7 2
1 22
Perimeter of DSB = ×2π ( 7cm ) = ×7cm=22cm
2 7

Class X Mathematics www.vedantu.com 11


Thus, the perimeter of the shaded region = 11 cm + 22 cm + 11 cm + 22 cm =
66 cm.
Therefore, the perimeter of the shaded region is 66cm.
Or
Find the area of a quadrant of a circle, where the circumference of the
22
circle is 44 cm. [Use π= ] 2 Marks
7
Ans: It is given that the circumference of the circle is equal to 44 cm. If we let
the radius of the circle to be r cm. 2πr=44 .
Solving this to find the radius.
44 44
r= = =7 cm
2π 2× 22
7
Thus, the area of the quadrant is
1 1 22 77
A= ×πr 2 = × ×7= . Thus, the area of the quadrant of the circle is equal to
4 4 7 2
77
cm 2 .
2

16. Two cubes, each 4 cm, are joined end to end. Find the surface area of
the resulting cuboid. 2 Marks
Ans: It is given that the cubes of side 4 cm each are joined end to end. The
joining of two cubes will result in a cuboid of length, breadth and height equal
to 8cm, 4cm, 4cm respectively. Hence, the surface area of the resulting cuboid
is:
S= 2(lb+bh+hl)
S=(8×4+4×4+4×8)
S=160 cm2

17. Find that value(s) of x for which the distance between the points P(x,
4) and Q(9, 10) is 10 units. 2 Marks

Class X Mathematics www.vedantu.com 12


Ans: It is given that the distance between the points P(x, 4) and Q(9, 10) is 10
units. Finding the distance using the distance formula for two points, we get

( 9-x ) + (10-4 )
2 2
PQ=

( 9-x )
2
⇒ PQ= +36

Putting the value of the distance, we get:

( 9-x )
2
⇒ 10= +36

⇒ 100= ( 9-x ) +36


2

⇒ 64= ( 9-x )
2

⇒ 9-x=±8
Taking the two different cases, we get :
⇒ x=9-8=1
And
⇒ 9-x=-8
⇒ x=9+9=17
Thus, x=1,17.

18. A coin is tossed two times. Find the probability of getting at least one
head. 2 Marks
Ans: When a coin is tossed twice, then the total number of outcomes is 4. The
probability of getting no heads is: The possible outcome is (T,T) The probability
1
is . Thus, the probability of getting at least one head is:
4
1 3
1− =.
4 4

SECTION-C
Question numbers 19 to 28 carry 3 marks each.
19. Find the roots of the following quadratic equation: 2 3x 2 -5x+ 3=0 .

Class X Mathematics www.vedantu.com 13


3 Marks
Ans: We are given the equation, 2 3x 2 -5x+ 3=0 .

-b± b 2 -4ac
x=
2a

( -5)
2
5± -4×2 3× 3
x=
2×2 2
5± 25 − 24
x=
4 2
5±1
x=
4 3
Further simplifying, we get
6 4
x= ;
4 3 4 3

3 1
x= ;
2 3

3 1
Thus, the roots of the equation are: x= ;
2 3

20. Find the value of the middle term of the following AP: - 6, -2, 2, ......, 58.
3 Marks

Ans: We are given the AP, - 6, -2, 2, ......, 58


For finding the middle term , we need to find the position of the middle term.
Thus, we will first find the total number of terms in this AP.
That is,
a=-6
d=4
l=58
Thus, finding the value of n: l=a+ ( n-1) d

Class X Mathematics www.vedantu.com 14


Putting the values in this, we get:
58 =−6 + ( n-1) 4

⇒ 58 + 6 =( n-1) 4
64
⇒ =n − 1
4
⇒ 16 + 1 =n
⇒ n=17
Hence, the middle term will be,
n+1
nm = .
2
17+1
⇒ nm = =9 So, we need to find the 9th term,
2
a 9 =a+ ( n-1) d=-6+ ( 9-1) 4

⇒ a 9 =-6+8×4=-6+32

⇒ a 9 =26

Thus, the middle term of the AP - 6, -2, 2, ......, 58 is 26.


Or
Determine the AP whose fourth term is 18 and the difference of the ninth
term from the fifteenth term is 30. 3 Marks
Ans: In this question, we are given the fourth term and the difference between
ninth and fifteenth term, i.e. a 4 =18 . Expanding the terms in terms of the formula
to find the AP, a15 -a 9 =30

a+ ( 4-1) d=18 . Finding the other equation as well,

⇒ a+3d=18
a+14d-a-8d=30
⇒ 6d=30
⇒ d=5
So, the common difference is 5. Putting the above equation:

Class X Mathematics www.vedantu.com 15


a+ ( 4-1) d=18

⇒ a+3 ( 5 ) =18

⇒ a=18-15=3
Thus, the AP which is a,a+d,a+2d,18,…3+14d,…… becomes:
3,3 + 5,3 + 10,18,.....3 + 70,....
.
3,8,13,18,.....,73,....

21. In Fig. 4, a triangle ABC is drawn to circumscribe a circle' of


radius 2 cm such that the segments BD and DC into which BC is divided by
the point of contact D are of lengths 4 cm and 3 cm respectively. If area of
ABC= 21 cm2, then find the lengths of sides AB and AC. 3 Marks

Ans: In this question, we are given that the radius of the circle is 2cm and the
area of triangle ABC is 21 cm2, we need to find the lengths of the triangle.
Construction: we need to join OA, OB, OC, OE ⊥ AB at E and OF ⊥ AC at F.
The resulting figure will be:

Now we have, the following data:


AE = AF

Class X Mathematics www.vedantu.com 16


BD = BE =4  cm
CD = CF = 3 cm
Let the lengths of AE and Af be x. This implies, area of the triangle equals:
A=area ( ∆BOC ) +area ( ∆AOB ) +area ( ∆AOC )

1 1 1
⇒ 21= ×BC×OD+ ×AB×OE+ ×AC×OF
2 2 2
⇒ 42=7×4+ ( 4+x )×2+ ( 3+x )×2

Solving this further, we get:


⇒ 21=7+ ( 4+x ) + ( 3+x )

⇒ 21=14+2x
⇒ x=3.5
Thus, the lengths of the sides AB and AC are:
AB=4+3.5=7.5
AC=3+3.5=6.5 .

22. Draw a triangle ABC in which AB = 5 cm, BC = 6 cm and ∠ABC=60o .


Then construct a triangle whose sides are times the corresponding sides of
ABC. 3 Marks

Ans: Construction of the triangle:

Class X Mathematics www.vedantu.com 17


1. A line segment AB of length equal to 5 cm is drawn.

2. An angle ABY equal to 60° is drawn from point B.

3. Now in order to measure angle at B:

3.1 With B as centre and with any radius, another arc are drawn cutting the line
which is present at D.

3.2 Taking the point D as centre and taking the same radius, an arc cutting the
first arc is drawn at point E.

3.3 A ray BY that passes through the point E is forming the angle equal to 60°
with the line AB.

4. Taking point B as centre and radius equal to 6 cm, an arc intersecting the line
BY is drawn at C.

5. A and C are joined, the required triangle is ∆ABC.

6. A ray from point A, named AX is drawn downwards which makes an acute


angle.

7. Some points are marked named on the segment AX such that the lengths:

AA1 =A1A 2 =A 2 A 3 =A 3A 4 =A 4 A 5 =A 5A 6 =A 6 A 7  

8. The points A7 and B are joined and from the point A5 , the segment A5 M is
drawn parallel to A7 B that intersects AB at M. 9. Taking the point M, a segment
MN is drawn parallel to BC intersecting AC at the point N. Then, ∆AMN is the
5
required triangle, the sides of this triangle are equal to of the corresponding
7
sides of ∆ABC.

23. Find the area of the major segment APB, in Fig 5, of a circle of radius
22
35 cm and ∠AOB=90o . [ Use π= ] 3 Marks
7

Class X Mathematics www.vedantu.com 18


Ans: For finding the area of the major segment, firstly we will write the equation
for area of minor segment. Area of minor segment = Area of sector AOBC- Area
of AOB
Putting the values in this equation, we get :
90o 2 1
A= ×π ( OA ) - ×OA×OB
360o 2
1 22 2 1
⇒ A= × × ( 35 ) - ×35×35
4 7 2
⇒ A=962.5-612.5
⇒ A=350cm 2
Thus, the area of major segment is equal to Area of major segment= Area of
circle – Area of minor segment. Therefore,

A'=π ( OA ) -350
2

22
× ( 35 ) -350
2
⇒ A'=
7
⇒ A'=3850-350=3500cm 2
Hence, the area of the major segment is 3500cm 2 .

24. The radii of the circular ends of a bucket of height 15 cm are 14 cm and
r cm (r < 14 cm). If the volume of the bucket is 5390 cm3, then find the value
22
of r. [Use π= ]. 3 Marks
7
Ans: In the question, we are given that: Height of bucket, h=15cm . Radius of
outer end,

Class X Mathematics www.vedantu.com 19


R=14cm . Radius of inner end, =r . Volume of the bucket is, V = 5390cm3 .
Putting the formula for the volume above, we get:
1
πh  R 2 +r 2 +Rr  =5390cm3
3
1 22
⇒ × ×15 (14 ) +r 2 +14r  =5390
2

3 7  

110
⇒ 196+r 2 +14r  =5390
7
⇒ r 2 +14r+196=343
⇒ r 2 +14r-147=0
Now solving this equation to find the value of r.
⇒ r 2 +21r-7r-147=0
⇒ r ( r+21) -7 ( r+21) =0

⇒ r=7,-21
The value of a radius cannot be negative, so, the value of r is 7cm.

25. Two dice are rolled once. Find the probability of getting such numbers
on two dice, whose product is a perfect square. 3 Marks

Ans: When two dice are rolled, the total number of possible outcomes is equal
to 36
Now, the number pairs on the two dices whose product form perfect square have
the possibilities, (1,1) , ( 2,2 ) , ( 3,3) , ( 4,4 ) , ( 5,5 ) , ( 6,6 ) , (1,4 ) , ( 4,1) . Thus, the
number of possible outcomes for this event is 8. Hence, the probability of getting
8 2
such numbers on two dice, whose product is a perfect square is = .
36 9
Or
A game consists of tossing a coin 3 times and noting its outcome each time.
Hanif wins if he gets three heads or three tails, and loses otherwise.
Calculate the probability that Hanif will lose the game. 3 Marks

Class X Mathematics www.vedantu.com 20


Ans: It is given that the coin is tossed thrice which means that the total number
of outcomes is 23 = 8 . The probability that Hanif loses is equal to one minus the
probability that he wins. So, we will find the probability of him winning the
game first Now, the outcomes for getting three heads or three tails will be:
2 1
( H,H,H ) , ( T,T,T ) . Thus, the probability of Hanif winning is = . Thus, the
8 4
1 3
probability that Hanif will lose the game is 1 − =.
4 4

26. From the top of a tower 100 m high, a man observes two cars on the
opposite sides of the tower with angles of depression 30° and 45°
respectively. Find the distance between the cars. [Use 3=1.73 ] 3 Marks

Ans: First of all, we are going to construct a diagram for the given situation.
Now it is given that Height of tower, H=100m . Angle of depression of car 1,
∠EAC=30o . Angle of depression of car 2, ∠FAD=45o . Now, in the right angled
AB 1
triangle ABC, tan30o = . As AB=100m and tan30 =
o
. Thus, putting these
BC 3
1 100 o AB
values in above equation, = . In right angled triangle ABD, tan45 =
3 BC BD

⇒ BC=100 3
Now, AB=100m and
tan45o =1. Substituting the values in above equation.
100
1= .
BD

Class X Mathematics www.vedantu.com 21


⇒ BD=100m

Now, ⇒ CD=CB+BD=100 3+100=100 ( 3+1) .

Putting 3=1.732 . Thus, CD=100 (1.73+1) =273.2m .

Therefore, the distance between the cars is 273.2m .

27. If (3, 3), (6, y), (x, 7) and (5, 6) are the vertices of a parallelogram taken
in order, find the values of x and y. 3 Marks

Ans: It is given that the vertices of the parallelogram are (3, 3), (6, y), (x, 7) and
(5, 6).
The figure is:

Now finding the midpoints of the diagonals AC and BD Coordinates of mid-


point of diagonal BD are:
 5+6 6+y 
 , .
 2 2 

 11 6+y 
 , 
2 2 
Coordinates of mid-point of diagonal AC are:
 3+x 7+3 
 , .
 2 2 

 3+x 10 
 , 
 2 2
Now comparing the x and y coordinates of the midpoint as the mid points of
both the diagonals coincide:

Class X Mathematics www.vedantu.com 22


3+x 11
= . And the y-coordinate :
2 2
⇒ x=11-3=8
6+y 10
=
2 2
⇒ y=10-6=4

Thus, the values of x and y are 8 and 4 respectively.

28. If two vertices of an equilateral triangle are (3, 0) and (6, 0), find the
third vertex. 3 Marks

Ans: Let the third vertex of the triangle be (x ,y). Thus, the vertices of the
triangle are:
A= ( 3,0 )
B= ( 6,0 )
C= ( x,y )

Finding the lengths of the sides of the triangle:

( 6-3)
2
AB= =3

( x-6 )
2
BC= +y 2

( x-3)
2
CA= +y 2

Equating CA and BC:

( x-6 ) +y 2 = ( x-3) +y 2
2 2

( x-6 ) = ( x-3)
2 2

⇒ x-6=- ( x-3)

⇒ 2x=9
9
⇒ x=
2

Class X Mathematics www.vedantu.com 23


Using equation AB=BC and putting value of x:

( x-6 )
2
+y 2 =9
2
9 
⇒  -6  +y 2 =9
2 
2
 3
⇒  -  -9=-y 2
 2
9
⇒ y 2 =- +9
4
27
⇒ y2 =
4
3 3
⇒ y=±
2
9 3 3
Thus, the values of x and y are and ± respectively.
2 2
Or
Find the value of k, if the points P(5, 4), Q(7, k) and R(9, -2) are collinear.
3 Marks
Ans: The given points are collinear, hence taking two of them in pairs each, and
finding the slope for them and equating the two slopes will give us the value of
k. Thus, slope of PQ
k-4 k-4
=
7-5 2
Slope of QR:
-2-k -k-2
=
9-7 2
Comparing both the slopes , we get:
k-4 -k-2
=
2 2
⇒ 2k=-2+4=2

Class X Mathematics www.vedantu.com 24


⇒ k=1

SECTION: D

Question Numbers 29 to 34 carry 4 marks each.

29. A motor boat whose speed is 20 km/h in still water, takes 1 hour more
to go 48 km upstream than to return downstream to the same spot. Find
the speed of the stream. 4 Marks

Ans: Before proceeding with the question, we must know the formulas that will
be required to solve this question.
We have a formula, time = distance/speed ......................(1). In the question, it
is given that the motorboat takes 1 hour more to go 48 km upstream than to
return downstream to the same spot. Also, it is given that the speed of the motor
boat in still water is equal to 20 km/hr. We are required to find the speed of the
stream.
Let us assume the speed of the stream = x km/hr. It is given that the speed of the
motor boat in still water is equal to 20 km/hr. While going upstream, the
direction of the stream will be against the direction of the motorboat. So, the
speed of the motorboat while going upstream will be equal to (20-x) km/hr. It is
given that the distance to be travelled is equal to 48 km. So, using formula (1),
the time taken by the motorboat to go upstream is equal to
48
..........(2) .
20-x
While going downstream, the direction of the stream will be along with the
direction of the motorboat. So, the speed of the motorboat while going
downstream will be equal to (20+x)km/hr . While going downstream, the
direction of the stream will be along with the direction of the motorboat. So, the
speed of the motorboat while going downstream will be equal to (20+x) km/hr.
It is given that the distance to be travelled is equal to 48 km. So, using formula
(1), the time taken by the motorboat to go downstream is equal to
48
..........(3) . In the question, it is given that the motor boat takes 1 hour more
20+x
to go 48 km upstream than to return downstream. So, we can say that,
(Time taken while going downstream) + 1 = Time taken while going upstream

Class X Mathematics www.vedantu.com 25


Substituting these times from equation (1) and equation (3), we get,
48 48
+1= .
20+x 20-x
48 48
⇒ - =1
20-x 20+x
 ( 20+x ) - ( 20-x ) 
⇒ (48)   =1
 ( 20-x ) (20+x) 
⇒ ( 48 )( 2x ) = ( 20-x )( 20+x )

⇒ 96x=400+20x-20x-x 2
⇒ x 2 +96x-400=0
⇒ x 2 -4x+100x-400=0
⇒ x ( x-4 ) +100 ( x-4 ) =0

⇒ ( x+100 )( x-4 ) =0

⇒ x=4,x= ( -100 )

Since speed cannot be negative, hence, the speed of the stream is 4 km/hr.
Or
1 1 11
Find the roots of the equation: - = ,x¹-4,7 . 4 Marks
x+4 x-7 30
1 1 11
Ans: By expressing the given equation - = ,x¹-4,7 in the form of a
x+4 x-7 30
general quadratic equation we get:
Taking a common denominator ( x+4 )( x-7 ) in the LHS,

( x-7 ) - ( x+4 ) = 11
( x+4 )( x-7 ) 30
-11 11
2
=
x -7x+4x-28 30
Cross multiplying the terms to express as a quadratic equation, cancelling out
11 and multiplying with (−1) on both sides, x 2 -3x-28=-30 .Expressing the

Class X Mathematics www.vedantu.com 26


equation in the simplest form possible, x 2 -3x+2=0 . Comparing the above
equation with the general quadratic equation we get the values of a, b, c as,
a=1,b=-3,c=2 . We know the formula to find the roots of a general quadratic
-b± D
equation is given as where D=b 2 -4ac is known as the discriminant.
2a
Applying the above formula by substituting the values of a, b, c to find the
discriminant, for the roots to be real and unequal the discriminant must be
greater than 0. ( -3) -4 (1)( 2 )  >0 . Squaring −3 and multiplying −4 with 2 we
2
 
get, ( 9 − 8 ) > 0 . Subtracting 8 from 9 the resultant is greater than 0. 1 > 0 .Hence
the roots are real and unequal. Applying the formula to find the roots of a
general quadratic equation by substituting the values of a, b, c and the value of
- ( -3) ± 1
the discriminant b 2 -4ac we get, .By taking square root of 1 and
2
-b+ D -b- D
simplifying the expression for: and we get, x=2 .
2a 2a
x=1
1 1 11
Hence the roots of the equation - = are 2 and 1.
x+4 x-7 30

30. If the sum of first 4 terms of an AP is 40 and that of first 14 terms is


280, find the sum of its first n terms.
Ans: Given that S4 = 40 and S14 =280’
n
Sn =  2a+ ( n-1) d 
2
n
Sn =  2a+ ( 4-1) d  = 40
2
⇒ 2a+3d=20.......( i )

14
S14 =  2a+ (14-1) d  =280
2
⇒ 2a+13d=40.....( ii )

( ii ) - ( i )
10d
= 20 ⇒ d=2

Class X Mathematics www.vedantu.com 27


Substituting the value of d in (i) we get
2a+6=10 ⇒ a=7
n
Sn =  2a+ ( n-1) d 
2
n
= 14+ ( n-1) 2 
2
= n(7+n-1)
= n(n+6)
=n2+6n
Therefore, Sn = n2 +6n
Or
Find the sum of the first 30 positive integers divisible by 6
According to the question series is
6,12,18,24,….
Here,
a=6, d=6
n
Sn =  2a+ ( n-1) d 
2
n = 30
30
S30 =  2×6+ ( 30-1)×6 
2 
S30 =15 (12+29×6 )

S30 =15×186
S30 =2790

31. A train travels 180 km at a uniform speed. If the speed had been 9
km/hour more, it would have take 1 hour less for the same journey. Find
the speed of the train.
Ans:
Let the speed of train be x km /h

Class X Mathematics www.vedantu.com 28


Distance = 180 km
So, time = 180 / x
When speed is 9 km/h more, time taken = 180 / x+9
According to the given information:
180 180
- =1
1 x+9
180 (x+9-x) / x(x+9) = 1
180×9 = x(x+9)
1620 = x2 + 9x
x2+ 9x - 1620 = 0
x2 + 45x - 36x -1620 = 0
x(x+45) - 36(x+45) = 0
(x-36)(x+45) = 0
x=36 or-45
But x being speed cannot be negative.
So, x = 36
Hence, the speed of the train is 36 km/h
Or
1 1 3
Find the roots of the equation + =1;x¹ ,5
2x-3 x-5 2
1 1 3
The given equation is + =1;x¹ ,5
2x-3 x-5 2
Taking L.C.M
x-5+ ( 2x-3)
=1
( x-5)( 2x-3)
3x-8
=1
2x 2 -13x+15
Cross-multiplying
2x2-13x + 15 = 3x-8

Class X Mathematics www.vedantu.com 29


2x2-16x + 23 = 0
Which is a quadratic equation.
Comparing with
ax2 + bx + c = 0, then a = 2, b = -16, c = 23
So, D = b2 - 4ac
= (-16)2 - 4.2. 23
= 256 - 184
= 72 > 0
Therefore, using quadratic formula

-b± D
x=
2a

6± 72
=
2.6

16+6 2 16-6 2
= ,
4 4
8+3 2 8-3 2
= ,
2 2
8+3 2 8-3 2
So the roots are = ,
2 2

32. In Figure 6, three circles each of radius 3.5 cm are drawn in such a
way that each of[ It them touches the other two. Find the area enclosed.
 22 
between these three circles (shaded region). use=π= 
 7

Class X Mathematics www.vedantu.com 30


Given that, three circles are in such a way that each of them touches the other
two.
Now, we join centre of all three circles to each other by a line segment. Since,
radius of each circle is 3.5 cm.
So; AB = 2 × Radius of circle
= 2 × 3.5 = 7 cm.
AC = BC = AB = 7cm
which shows that, ∆ ABC is an equilateral triangle with side 7 cm.
We know that, each angle between two adjacent sides of an equilateral triangle
is
60°
Area of sector with angle ∠ A = 60°.

∠A 60o
×π ( 3.5 )
2 2
= o
×πr = o
360 360
So, area of each sector = 3×Area of sector with angle A
60o
( )
2
=3× ×π× 3.5
360o
1 22
= × ×3.5×3.5
2 7

Class X Mathematics www.vedantu.com 31


5 35 11 7
=11× × = ×
10 10 2 2
77
= =19.25cm 2
4
3
×( 7 )
2
And Area of ΔABC=
4
3
[area of an equilateral triangle = (side)2]
4
3 2
=49 cm
4
Area of shaded region enclosed between these circles
=area of ∆ ABC - Area of each sector

3
= 49 -1925 -1225 × 3 -1925
4
= 21.2176 - 1925=1 9676 cm2?
Hence, the required area enclosed between these circles is 1.967 cm2? (approx).
33. From a solid cylinder whose height is 15 cm and diameter 16 cm, a
conical cavity of the same height and same diameter is hollowed out. Find
the total surface area of the remaining solid. [Taken=3.14]
4 Marks
Ans:

Class X Mathematics www.vedantu.com 32


In this question, we are given a solid cylinder with Height, h=15cm .The
diameter of the cylinder is, d=16cm . Thus, the radius, R=8cm . The slant height
of the cone can be found as: l= h 2 +R 2 . Putting the values above,

l= (15 ) + ( 8 ) = 225+64= 289 . Total surface of the solid is:


2 2

⇒l=17
T=Curved surface area of cone + Curved surface area of the cylinder
+Area of the bottom part

Putting the values, we get


T=πRl+2πRh+πR 2
⇒ T=3.14 [8×17+2×8×15+8×8]

⇒ T=3.14 [ 440]

⇒ T=1381.6cm 2
Thus, the total surface area of the remaining solid is1381.6cm 2 .

34. Two poles of equal heights are standing opposite to each other on either
side of the road, which is 100 m wide. From a point between them on the

Class X Mathematics www.vedantu.com 33


road, the angles of elevation of the top of the poles are 60° and 30°,
respectively. Find the height of the poles. 4 Marks

Ans: Let us firstly construct a figure reflecting the condition given in the
question,

Considering the right-angled triangles, ABP and BPD, let the distance
h
between the points C and P be x. Thus, in triangle ABP, tan60o = .
x
Putting here, the value of tan 60

h
3= . Now, in triangle BPD,
x

⇒ h= 3x--- (1)

BD h
tan30o = = . Using (1) in (2)
PD 100-x

1 h
⇒ = --- ( 2 )
3 100-x

1 3x
⇒ =
3 100-x

⇒ 3x=100-x

⇒ 4x=100

Class X Mathematics www.vedantu.com 34


⇒ x=25

Putting the value of x in (1); We get

h= 3x=25 3

The height of the poles is 25 3m .

Class X Mathematics www.vedantu.com 35


CBSE Class 10
Mathematics
Previous Year Question Paper 2011
Series: RHB Code no. 30/1

● Please check that this question paper contains 16 printed pages.


● Code number given on the right hand side of the question paper
should be written on the title page of the answer-book by the
candidate.
● Please check that this question paper contains 34 questions.
● Please write down the Serial Number of the question before
attempting it.
● 15 minutes of time has been allotted to read this question paper.
The question paper will be distributed at 10.15 a.m. From 10.15
a.m. to 10.30 a.m., the students will read the question paper only
and will not write any answer on the answer script during this
period.

MATHEMATICS

Time Allowed: 3 hours Maximum Marks: 80


General Instructions:
1. All questions are compulsory.
2. The question paper consists of 34 questions divided into four sections
A, B, C and D.
3. Section A contains 30 questions of 1 mark each, which are multiple choice
type questions, Section B contains 8 questions of 2 Marks each, Section C
contains 10 questions of 3 Marks each, Section D contains 6 questions of 4
Marks each.

Class X Mathematics www.vedantu.com 1


4. There is no overall choice in the paper. However, internal choice is provided
in one question of 2 Marks. 3 questions of 3 Marks each and two questions of
4 Marks each.
5. Use of calculators is not permitted.

SECTION-A
1. The roots of the equation x 2 -3x-m(m+3)=0 , where m is a constant, are
1 Mark
A. m, m+3
B. -m, m+3
C. m, -(m+3)
D. -m, -(m+3)
Ans: Given quadratic equation is,
x 2 -3x-m(m+3)=0

⇒ x 2 -3x-m 2 -3m=0
⇒ x 2 -m 2 -3x-3m=0
⇒ (x+m)(x-m)-3(x+m)=0
⇒ (x+m)(x-m-3)=0
Thus, x+m=0 And, x-m-3=0
x=-m
x= m + 3
The roots are -m , m+3 ..
The correct option is B. -m , m+3

2. If the common difference of an A.P. is 3, then a 20 -a15 is, 1 Mark


A. 5
B. 3

Class X Mathematics www.vedantu.com 2


C. 15
D. 20
Ans: Let the first term be a1 .
We get, a 20 =a1 +(20-1)3=a1 +57----- (1)

Again, a15 =a1 +(15-1)3=a1 +42----- ( 2 )

Subtracting equation ( 2 ) from (1) we get,


a 20-a15 =a1 +57-a1 -42=15
So, the correct option is C. 15

3. In figure 1, O is the centre of a circle, PQ is a chord and PT is the tangent


at P. If ∠POQ=70o , then ∠TPQ is equal to 1 Mark

A.
B.
C.
D.
Ans:

Class X Mathematics www.vedantu.com 3


Given, ∠POQ =
70°
∠OPQ + ∠OQP + ∠POQ =180°

2∠OPQ =
110°
Therefore, ∠OPQ =
55°

A tangent subtends an angle 90° to the radius. Thus,


∠TPQ = ∠OPT − ∠OPQ = 90° − 55° = 35°

The correct option is D. 35°

4. In Figure 2, AB and AC are tangents to the circle with centre O such that
∠BAC=40o Then ∠BOC is equal to 1 Mark
A.
B.
C.
D.

Class X Mathematics www.vedantu.com 4


Ans: ∠ABO =
∠ACO =
90°
The sum of all the angles of the quadrilateral ABOC is 360°
So,
∠ABO + ∠ACO + ∠BOC + ∠BAC =360°
⇒ 90° + 90° + 40° + ∠BOC =360°
⇒ ∠BOC =140°
The correct option is C. 140° .

5. The perimeter (in cm) of a square circumscribing a circle of radius a cm,


is 1 Mark
A. 8a
B. 4a
C. 2a
D. 16a
Ans: Since, the square is circumscribing a circle then, the diameter of the circle
is the side of a square as shown in the figure.

Diameter = Side of the square = 2a


So, the perimeter of square =4 × 2a =8a
The correct option is A. 8a

Class X Mathematics www.vedantu.com 5


6. The radius (in cm) of the largest right circular cone that can be cut out
from a cube of edge 4.2 cm is 1 Mark
A. 4.2
B. 2.1
C. 8.4
D. 1.05
Ans: The base of the cone is a circle. So, the diameter of the base acts as the
side of the cube.
Diameter 4.2
Radius
= = = 2.1
2 2
The correct option is B. 2.1

7. A tower stands vertically on the ground. From a point on the ground


which is 25m away from the foot of the tower., the angle of elevation of the
top of the tower is found to be 45o. Then the height (in meters) of the tower
is 1 Mark
A. 25 2
B. 25 3
C.
D.
Ans:

Class X Mathematics www.vedantu.com 6


From the figure we get,
AB
tan45° =
BC
⇒ AB=BC
⇒ AB =
25
So, the height of the tower is 25 m.
The correct option is C. 25

a 
8. If P  ,4  is the mid-point of the line-segment joining the points A(-6,5)
2 
and B(-2,3), then the value of a is, 1 Mark
A. -8
B.
C. -4
D.
Ans: Since, P is the midpoint of A and B then,
a -2+(-6)
=
2 2
⇒a=−8
The correct option is A. -8.

9. A and B are the points (-6,7) and (-1,-5) respectively, then the distance
2AB is equal to 1 Mark
A. 13
B. 26
C. 169
D. 238
Ans: From Distance formula we get,

Class X Mathematics www.vedantu.com 7


( -6- ( -1) ) + ( 7- ( -5) )
2 2
⇒ AB=

⇒ AB= 25+144=13
Therefore, 2AB=26
The correct option is B. 26

10. A card is drawn from a well-shuffled deck of 52 playing cards. The


probability that the card will not be an ace is 1 Mark
1
A.
13
1
B.
4
12
C.
13
3
D.
4
Ans: The total numbers of aces in 52 playing cards = 13
13 1
The probability of finding Ace = =
52 4
1 3
Hence, the probability of not finding Ace =1- =
4 4
3
The correct option is D.
4

SECTION-B
11. Find the value of m so that the quadratic equation mx(x-7)+49=0 has
two equal roots. 2 Marks
Ans: We get,
mx(x-7)+49=0

Class X Mathematics www.vedantu.com 8


⇒ mx 2 -7mx+49=0
If the quadratic equation has two equal roots then,
D=0
⇒ b 2 -4ac=0
∴ (7 m) 2 -4×m×49=0
⇒ 49m(m-4)=0
⇒ m(m-4)=0
⇒ m=0,m=4
m=0 is invalid so, m=4
So, the value of m is 4 when the quadratic equation mx(x-7)+49=0 has two
equal roots.

12. Find how many two-digit numbers are divisible by 6. 2 Marks


Ans: The two digit-numbers are from 10 to 99.
The numbers which are divisible by 6 are 12,18,24……
Hence they have a common difference of 6 and the first number divisible by 6
is 12 and last number is 96.
Let n be the total two-digit numbers divisible by 6.
From A.P. we get,
96=12+(n-1)6
⇒ 14=n-1
⇒ n=15
There are 15 numbers which are divisible by 6.

13. In Figure 3, a circle touches all the four sides of a quadrilateral ABCD
whose sides are AB=6 cm, BC=9cm and CD=8 cm Find the length of side
AD. 2 Marks

Class X Mathematics www.vedantu.com 9


Ans:

Let E,F,G,H be the point of contact of tangent and circle.


We know that, two tangents drawn from the same point to the circle are of equal
length. Hence,
= ED DF,CG
= FC,GB = BH and AE = AH .
Therefore, ED + CG + GB + AE = DF + FC + BH + AH
⇒ (ED + AE) + (CG + GB) = (DF + FC) + (AH + HB)

⇒ AD + BC = CD + AB
⇒ AD = 8 + 6 − 9 = 5  cm
Length of AD is 5  cm .

Class X Mathematics www.vedantu.com 10


14. Draw a line segment AB of length 7cm. Using ruler and compasses, find
AP 3
a point P on AB such that = 2 Marks
AB 5
Ans:
Steps of construction:
- Draw a line segment AB = 7 cm using a ruler.
- Draw a ray AX with ∠BAX as an acute angle.
- Cut the line-segment AC into 5 equal parts as, AD
= DE
= EF
= FG
= GC
- Join the point C with point B
- From point E draw a parallel line to BC with would intersect the line AB
at P .

AP 3
Hence, we find = .
AB 5

15. Find the perimeter of the shaded region in Figure 4, if A B C D is a square


 22 
of side 14 cm and APB and CPD are semicircles.  Use π=  2 Marks
 7

Class X Mathematics www.vedantu.com 11


Ans: The given two semi-circles are identical as they have equal diameter = side
of the square
The perimeter of the shaded region is the circumference of the semi-circles APB
and DPC and the sides of square AD and CB.
Radius of the semi-circle =7  cm
22
Circumference of each semi-circle =π× radius = ×7=22  cm
7
Circumference of two semi-circle =2 × 22 =44  cm Total circumference of
shaded region = 14 + 14 + 44 = 72  cm

16. Two cubes each of volume 27 cm3 are joined end to end to form a solid.
Find the surface area of the resulting cuboid. 2 Marks
Ans: Let the side of the cube be a .

Class X Mathematics www.vedantu.com 12


We get a 3 =27
⇒ a=3
As the two cubes are joined it forms a cuboid of length 1 = 6 cm , breadth
b = 3 cm and height h = 3 cm .
Surface area of resulting cuboid = 2(1 b + bh + h1) = 2(18 + 9 + 18) = 90 cm 2

The surface area of resulting cuboid is 90  cm 2 .


Or
A cone of height 20 cm and radius of base 5 cm is made up of modelling
clay. A child reshapes it in the form of a sphere. Find the diameter of the
sphere. 2 Marks
πr 2 h
Ans: Volume of the given cone = where r is the radius of the base and h
3
is the height of the cone.
π×52 ×20 500
So, the volume = = π
3 3
As, the child reshapes it in a sphere its volume remains the same.
Let the radius of the sphere be r .
4 3 500
We get, =
πr π
3 3
⇒ r 3 =125
⇒ r=5
Diameter of the sphere =2×5=10  cm
The diameter of the sphere is 10  cm .

17. Find the value of for which the distance between the points A(3, -1) and
B(11, y) is 10 units. 2 Marks
Ans: From the Distance Formula we get,

(11-3) 2 +{y-(-1)}2 =10

Class X Mathematics www.vedantu.com 13


⇒ 64+(y+1) 2 =100
⇒ y+1=±6
Considering y+1=-6
We get, y=-7
Again considering y+1=6
We get, y=5
So, the value of y is both −7 and 5 .

18. A ticket is drawn at random from a bag containing tickets numbered


from 1 to 40. Find the probability that the selected ticket has a number
which is a multiple of 5. 2 Marks
Ans: The numbers which are divisible by 5 from 1 to 40 are,
5,10,15,20,25,30,35,40
Total numbers ( n ) divisible by 5 from 1 to 40 = 8
Total number of ticket numbers (N)=40
Probability that the selected ticket has a number which is a multiple of
n 8 1
5= = =
N 40 5
The probability that the selected ticket has a number which is a multiple of 5 is
1
.
5

SECTION-C
19. Find the roots of the following equation:
x 3 -3 5x+10=0 3 Marks
Ans: From Sreedhar Acharya's Formula we get,

-b± b 2 -4ac
The roots of the equation, ax 2 +bx+c=0 is x= Here, for equation
2a

Class X Mathematics www.vedantu.com 14


x 2 -3 5x+10=0 we get

3 5± (3 5) 2 -4×1×10
x=
2
3 5± 5
⇒ x=
2
Considering the sign as (+ve) we get,
4 5
x= =2 5
2
Considering the sign as (-ve) we get,
2 5
x= = 5
2
So, the roots of the given equation are 2 5 and 5.

20. Find an A.P. whose fourth term is 9 and the sum of its sixth term and
thirteenth term is 40. 3 Marks
Ans: Let the first term be a1 and the common difference be d.
Given, a 4 =9 and a 6 +a13 =40
a 4 =a1 +3d=9----- (1)

a 6 =a1 +5d----- ( 2 )

a13 =a1 +12d----- ( 3)


a 6 +a13 =a1 +5d+a1 +12d=40
⇒ 2a1 +17d=40
40-2a1
⇒ d= ----- ( 4 )
17
9-a1
From (1) we get, d=
3
Comparing ( 4 ) and ( 5 ) we get,

Class X Mathematics www.vedantu.com 15


40-2a1 9-a1
=
17 3
⇒ 120-6a1 =153-17a1
⇒ 11a1 =33
⇒ a1 =3

Putting the value of a1 in equation ( 5 ) we get,

d=2
The A.P. is then a1 , a1 + d , a1 + 2d ,……
So, the required A.P. is 3,5,7,9…

21. In Figure 5, a triangle PQR is drawn to circumscribe a circle of radius


6 cm such that the segments QT and TR into which QR is divided by the
point of contact T, are of lengths 12 cm and 9 cm respectively. If the area
of ΔPQR=189cm 2 , then find the lengths of sides PQ and PR. 3 Marks

Ans:

Class X Mathematics www.vedantu.com 16


Given QT =12  cm and TR =9  cm and area of ∆ PQR =189  cm 2
The tangents PQ, QR and RP at A, T and B are at right angles.
We get, QT = AQ =12  cm and TR = BR =9  cm and AP = BP.
Radius OT = OB = OA =6 cm
Area of ∆ PQR = Area of ∆ AOP + Area of ∆ AOQ + Area of ∆ TOQ + Area
of ∆ TOR + Area of ∆ BOR + Area of ∆ BOP
1
Þ189= [(AP×OA)+(OA×AQ)+(OT×QT)+(OT×TR)+
2
(OB×RB)+(OB×OP)]
1
Þ189= [6AP+72+72+54+54+6BP]
2
⇒ 21=AP+BP
⇒ 2AP=21
⇒ AP=PB=10.5
Length of side PQ =(12+10.5)=22.5  cm and length of side PR
=(10.5+9)=19.5 cm .

22. Draw a pair of tangents to a circle of radius 3 cm, which are inclined to
each other at an angle of . 3 Marks

Class X Mathematics www.vedantu.com 17


Ans: Steps of Construction:
- Drawn a circle with radius = 3 cm centered at O.
- Diameter AOB is drawn.
- Drawn a radius OC in which ∠BOC =
60° .
- FromC, drawn CM ⊥ OC and from A dawn AN ⊥ OA
- Extended the perpendiculars which intersect each other at point P.
Thus, PA and PC are the required tangents.
Or
Draw a right triangle in which the sides (other than hypotenuse) are of
3
lengths 4 cm and 3 cm. Then construct another triangle whose sides are
5
times the corresponding sides of the given triangle. 3 Marks
Ans:

3
Here scale factor= <1
5
Steps of Construction:
- Drawn a line-segment BC of 4  cm .
- From B, ∠B =90° is drawn and a segment BA =3  cm is cut.

Class X Mathematics www.vedantu.com 18


- A and C are joined. Now, a right triangle ABC is formed.
- A ray BY is drawn from B making an acute angle ∠CBY on the opposite side
of vertex A.
- The ray BY is then cut at point B1 , B2 , B3 , B4 and B5 such that BB1 =

B=
1 B2 B=
2 B3 B=
3 B4 B4 B5

- B5 and C are joined.

- A parallel line B5C ‖B3C′ from B3 is drawn which cuts BC at C’.

- Now, from point C′ ,C′A′ ‖CA is drawn which intersects AB at A 2 . Hence,


3
ΔA'BC' is the required triangle whose sides are times the corresponding side
5
of a given triangle.

23. A chord of a circle of radius 14 cm subtends an angle of 120o at the centre.


22
Find the area of the corresponding minor segment of the circle. Use π=
7
and 3=1.73 3 Marks

Ans: We have to find the area of the section ACBD.

In ∆ AOC and ∆ BOC we get,


(i) AO = OB = Radius of the circle

Class X Mathematics www.vedantu.com 19


(ii) ∠OCB =
∠OCA =
90°
(iii) OC is common
Thus, we get, ∆ AOC ≅ ∆ BOC
So, ∠BOC =
∠AOC =
60°
In ∆ AOC we get,
AC
sin 60 =
OA

14 3
⇒ AC= =7 3
2
Again,
OC
cos60 =
OA
7
⇒ OC =

Hence, AC = 7 3 as ∆ AOC ≅ ∆ BOC.


= BC
1
Area of ∆OAB = × 14 3 × 7 = 49 3 cm 2 ∣
2
120° 196π
Area of the section OADB= °
×π×142 =
360 3
Now, Area of section ACBD = Area of the section OADB − Area of ∆ OAB
196π
= -49 3=205.33-84.77=120.56
3
Area of the minor segment of the circle ACBD is 120.56  cm 2 .

24. An open metal bucket is in the shape of a frustum of a cone of height


21cm with radii of its lower and upper ends as 10cm and 20cm respectively.
Find the cost of milk which can completely fill the bucket at Rs. 30 per litre.
 22 
 Use π= 7  3 Marks

Class X Mathematics www.vedantu.com 20


Ans:

Given, Radius of the upper part R = 20 cm and bottom part of the bucket
r = 10 cm and height h = 21 cm . Volume of the frustum of a cone
1
= πh ( R 2 +r 2 +Rr )
3
The volume of the given metal bucket,
1 22
= × ×21× ( 202 +102 +200 )
3 7
=15400 cm3
Therefore, Volume of milk that can be completely filled in the bucket
15400
= = 15.4 litre. [1000 cm 2 = 1 litre]
1000
Cost of milk to completely fill the bucket = Rs. 15.4 × 30 =
Rs. 462 .

25. Point P(x,4) lies on the line segment joining the points A(-5, 8) and B (4,
-10). Find the ratio in which point P divides the line segment AB. Also find
the value of x. 3 Marks
Ans: Let the point P divides the line segment AB in the ratio k:1

Class X Mathematics www.vedantu.com 21


By using section formula we get,
 4k+(-5) -10k+8 
(x,4)=  , 
 k+1 k+1 
Comparing the equation we get,
-10k+8
4=
k+1
⇒ 4k+4=-10k+8
⇒ 14k=4
2
⇒ k=
7
2
The radio in which P divides AB is, k:1= :1=2:7
7
P divides the line AB in the ratio 2 : 7
Again,
4k-5
x=
k+1
2
Substituting the value of k= we get,
7
2
4× -5
x= 7 =-3
2
+1
7
The value of x is -3.

26. Find the area of the quadrilateral ABCD, whose vertices are A(-3, -1),
B(-2, -4), C(4, -1) and D(3, 4) 3 Marks
Ans:

Class X Mathematics www.vedantu.com 22


By using geometry, we get,
F (−2, −1) and E (3, −1)

For ∆ ACD we get,

Height DE= (3 − 3) 2 + (−1 − 4)=


2
5

Base AC
= (4 + 3) 2 + (−1 − (−1))=
2
7

1 35
Area of ∆ ACD = ×5× 7 = sq unit
2 2
For ∆ ABC we get,
Base AC = 7

Height BF= (−2 + 2) 2 + (−1 + 4) 2 = 3

1 21
Area of ∆ ABC = × 3× 7 = sq unit
2 2
35 21
∴ Area of the quadrilateral ABCD = + = 28 sq units
2 2
Or

Class X Mathematics www.vedantu.com 23


Find the area of the triangle formed by joining the mid-points of the sides of
the triangle whose vertices are A (2, 1) , B (4, 3) and C (2, 5) . 3 Marks

Ans:

 2+4   1+3 
Let D be the mid-point of AB, D ( x1 ,y1 ) =  ,  =(3,2)
 2  2 

 2+4   5+3 
Let E be the mid-point of BC, E ( x 2 ,y 2 ) =  ,  =(3,4)
 2  2 

 2+2   1+5 
Let F be the mid-point of CA, F ( x 3 ,y3 ) =  ,  =(2,3)
 2  2 
1
Area of triangle DEF =  x1 ( y 2 -y3 ) +x 2 ( y3 -y1 ) +x 3 ( y1 -y 2 ) 
2
1
= [3(4-3)+3(3-2)+2(2-4)]
2
=1 unit 2

27. From the top of a vertical tower, the angles of depression of two cars, in
the same straight line with the base of the tower, at an instant are found to
be 45o and 60o. If the cars are 100 m apart and are on the same side of the
tower, find the height of the tower.  USe 3=1.73  3 Marks

Ans:

Class X Mathematics www.vedantu.com 24


The angle of depression ∠XDA =
∠BAD =
45° [ Alternate angle]
The angle of depression ∠XDB =
∠BDC =
60° [ Alternate angle ]
Let the distance BC be x m and the height of the tower be h m .
Given, the distance between car A and B is 100  m .
From ∆ BCD we get,
h
tan60° =
x
w h= 3x---- (1)

From ∆ ACD we get,


h
tan45° =
100+x
w h=100+x----- ( 2 )

Comparing equation (1) and ( 2 ) we get,

3x =100+x
100 100 100
⇒ x= = = =136.98 m
3-1 1.73-1 0.73

Putting the value of x in equation (1) we get, h= 3×136.98=236.98  m

The height of the tower is 236.98  m .

Class X Mathematics www.vedantu.com 25


28. Two dice are rolled once. Find the probability of getting such numbers
on the two dice, whose product is 12. 3 Marks
Ans: The sample space of the given case is:
S = {(1,1),(1,2),(1,3),(1,4),(1,5),(1,6)
(2,1),(2,2),(2,3),(2,4),(2,5),(2,6)
(3,1),(3,2),(3,3),(3,4),(3,5),(3,6)

(4,1),(4,2),(4,3),(4,4),(4,5),(4,6) ,
(5,1),(5,2),(5,3),(5,4),(5,5),(5,6)
(6,1),(6,2),(6,3),(6,4),(6,5),(6,6)}
n(S) = 36
Let E be the event where the product of two numbers will be 12 are,
E = {(1,6),(2,3),(3,2),(6,1)}
n(E) = 4

n(E) 4 1
The probability of getting the product of 12= = =
n(S) 36 9
The probability of getting such numbers on the two dice, whose product is 12 is
1
.
9
Or
A box contains 80 discs which are numbered from 1 to 80 . If one disc is
drawn at random from the box, find the probability that it bears a perfect
square number. 3 Marks
Ans: Total numbers (n) = 80

The perfect square numbers between 1 to 80 are, 1,4,9,16,25,36,49,64.


Number of perfect square number (N) = 8 The probability of finding a perfect
8 1
square number in the box
= =
80 10

Class X Mathematics www.vedantu.com 26


The probability that it bears a perfect square number from a box that contains 80
1
discs which are numbered from 1 to 80 is
10

SECTION-D
29. Prove that the tangent at any point of a circle is perpendicular to the
radius through the point of contact. 4 Marks
Ans: Let us consider that the center of a circle be O .
Let AB be the tangent to the circle which is touching the circle at point P . So,
the radius of the circle is OP .
Let us consider a point Q other than P on the tangent A B.

Now, we join the point Q with the center Q making a line segment OQ
intersecting the circle at R.
From the diagram we get, OP = Radius of the circle
= OR
Now, OQ = OR + RQ = OP + RQ
We get to know that, OQ > OP
Hence, we conclude that any line which is drawn from the center to a point on the
tangent is greater than OP.
We know that the shortest distance of a point from a given line is the
perpendicular distance from that line. So, here OP is the shortest distance Thus,
we conclude that OP ⊥ AB .
It is proved that the tangent at any point of a circle is perpendicular to the radius
through the point of contact.

Class X Mathematics www.vedantu.com 27


30. The first and the last terms of an A.P. are 8 and 350 respectively. If its
common difference is 9, how many terms are there and what is their sum?
4 Marks
Ans: Let the first term of the A.P. be a and last term be 1 and the common
difference be d.
a 8,1
Given, = = 350 and d = 9

Let 1th term be the last term.


We get,
350 =8 + (l − 1)9

342
1
⇒ l −= = 38
9
⇒l =39
The total number of terms (n) in A.P. is 39. The Sum of the A.P.
n 39
= [2a+(n-1)d]= [2×8+38×9]=6981
2 2
So, there are 39 terms in this A.P. and the sum of them is 6981.
Or
How many multiples of 4 lie between 10 and 250? Also find their sum.
4 Marks
Ans: The first number between 10 and 250 which is a multiple of 4 is 12.
As, it is a multiple of 4 then we can say that it will form an A.P. with common
difference d = 4
The last number between 10 to 250 which is a multiple of 4 is 248.
So, here first term (a) of the A.P. = 12 and last term (1) = 248

Therefore,
12 + (l − 1)4 =
248

⇒ l −1 =59
⇒l =60

Class X Mathematics www.vedantu.com 28


There are a total of 60 multiples of 4 between 10 and 250. Sum of the multiples
60
of 4 between 10 and 250= [2×12+(60-1)4]=7800
2
There are a total of 60 multiples of 4 between 10 and 250 whose total sum is 7800.

31. A train travels 180km at a uniform speed. If the speed had been
9km/hour more, it would have taken 1 hour less for the same journey. Find
the speed of the train. 4 Marks
Ans: Let the speed of the train be xkm / hour and the time taken to cover the
distance of 180 km be t hour.
180
We get, x= ----(1)
t
When the speed becomes (x + 9)km / hour then the time required to complete
180
180 km distance is (t-1) hour. \x+9= …
t-1
Putting the value of x from equation (1) to equation ( 2 ) we get,
180 180
+9=
t t-1
180+9t 180
⇒ =
t t-1
⇒ 180t-180+9t 2 -9t=180t
⇒ t 2 -t-20=0
⇒ t 2 -5t+4t-20=0
⇒ t(t-5)+4(t-5)=0
⇒ (t-5)(t+4)=0
Either, t = 5 Or, t = −4 [Time can never be negative]

So, we get, t = 5 hour.


Substituting its value in equation (1) we get,

Class X Mathematics www.vedantu.com 29


180
x= =36
5
The speed of the train is 36  km / hour .
Or
1 1 3
Find the roots of the equation + =1,x¹ ,5 . 4 Marks
2x-3 x-5 2
Ans:
1 1
+ =1
2x-3 x-5
x-5+2x-3
⇒ =1
(2x-3)(x-5)

⇒ 3x-8=2x 2 -10x-3x+15
⇒ 2x 2 -16x+23=0
Using Sreedhar Acharya's Formula we get,

16± (-16) 2 -4×2×23


x=
2×2
16±6 2
⇒ x=
4
3 2
⇒ x=4±
2
3 2 3 2
So, the roots of the given equation are 4 + and 4 − .
2 2
32. In Figure 6, three circles each of radius 3.5 cm are drawn in such a way
that each of them touches the other two. Find the area enclosed between
 22 
these Mark three circles (shaded region).  Use π=  4 Marks
 7

Class X Mathematics www.vedantu.com 30


Ans:

The given circles are identical with radius r = 3.5 cm .


The ∆ ABC is an equilateral triangle with side = = 7 cm . The area of ∆ ABC
a 2r
3 49 3
= ×7 2 =  cm 2
4 4
As, ∆ ABC is an equilateral triangle so its all angles are 60° . Area of section PBR
= Area of section PAQ = Area of section QCR
Area of the shaded portion = Area of ∆ ABC = [Area of section PBR + Area of
section PAQ + Area of section QCR]

49 3
∴ Area of the shaded portion
= − 3 × Area of the section PBR
4
49 3 60° 49 3 1 22
= -3× °
×π×3.52 = -3× × ×3.52 =21.21-19.25=1.96
4 360 4 6 7
The area of the shaded portion is 1.96 cm 2 .

Class X Mathematics www.vedantu.com 31


33. Water is flowing at the rate of 15 km/hour through a pipe of diameter 14
cm into a cuboidal pond which is 50m long and 44m wide. In what time will
the level of water in the pond rise by 21cm ? 4 Marks
Ans: Length of the cuboidal pond 1 = 50 m , breadth b = 44 m and height h is up
to 21 m .
The volume of the cuboidal pond =l×b×h=50×44×0.21=462 m3
5 25
The rate of flowing of water = 15 km / hour =15×  m/sec=  m/sec
18 6
Hence, the volume of water that the pipe drops into the pond in 1sec
25
= ×π×0.07 2
6
= 0.06 m3
462
Time in which the water rise to 21 =
cm = 7700sec
= 128.33 m
=in 2.13
0.06
hour
It takes 2.13 hour to rise the level of water by 21 cm .

34. The angle of elevation of the top of a vertical tower from a point on the
ground is 60o From another point 10m vertically above the first, its angle of
elevation is 30o. Find the height of the tower. 4 Marks
Ans:

Class X Mathematics www.vedantu.com 32


Let the height of the tower be hm .
From the diagram we get, BC = DE
In ∆ AED we get,
= (h − 10)m
AE

h-10
tan30° =
DE
ÞDE= 3(h-10)=BC-L(1) [As,BC=DE]

Again in ∆ ABC we get,


h
tan60° =
BC
h
⇒ BC=
3
From (1) and (2) we get,
h
3(h-10)=
3
⇒ 3h-30=h
⇒ h=15 m
The height of the tower is 15 m .

Class X Mathematics www.vedantu.com 33


CBSE-XII-2017 EXAMINATION CBSE-X-2017 EXAMINATION

MATHEMATICS
Paper & Solution
Time: 3 Hrs. Max. Marks: 90
General Instructions :
(i) All questions are compulsory.
(ii) The question paper consists of 31 questions divided into four sections - A, B, C and D.
(iii) Section A contains 4 questions of 1 mark each. Section B contains 6 questions of 2 marks each, Section C
contains 10 questions of 3 marks each and Section D contains 11 questions of 4 marks each.
(iv) Use of calculators is not permitted.

SECTION – A
Question numbers 1 to 4 carry 1 mark each.
1. What is the common difference of an A.P. in which a21 - a7 = 84 ?
Solution:
Given
a21 – a7 =84 ....................(1)
In an A.P a1, a2, a3, a4 …………
an = a1 + (n – 1)d d = common difference
a21 = a1 + 20 d ……………..(2)
a7 = a1 + 6d ……………….(3)
substituting (2) & (3) in (1)
a1 + 20d – a1 – 6d = 84
14d = 84
d=6

 common difference = 6

2. If the angle between two tangents drawn from an external point P to a circle of radius a and centre O, is
60 , then find the length of OP.
Solution:

www.vedantu.com 1 / 22
CBSE-X-2017 EXAMINATION

Given that BPA = 60


OB = OA = a [radii]
PA = PB [length of tangents Equal]
OP = OP

PBO and PAO are congruent. [By SSS test of congruency]

60
BPO = OPA = = 30
2
a
In a PBO sin30 =
OP
OP = 2a units

3. If a tower 30 m high, casts a shadow 10 3 m long on the ground, then what is the angle of elevation of
the sun ?
Solution:

Angle of equation of sun = GST = 


Height of lower TG = 30m

Length of shadow GS = 10 3 m

TGS is a right angled triangle


30
 tan  =
10 3
tan  = 3
 = 60

www.vedantu.com 2 / 22
CBSE-X-2017 EXAMINATION

4. The probability of selecting a rotten apple randomly from a heap of 900 apples is 0·18. What is the
number of rotten apples in the heap ?
Solution:
Number of rotten apples
Probability of selecting rotten apple =
Total number of apples

No. of rotten apples


 0.18 =
900
No. of rotten apples = 900  0.18 = 162

SECTION B
Question numbers 5 to 10 carry 2 marks each.
5. Find the value of p, for which one root of the quadratic equation px 2 - 14x + 8 = 0 is 6 times the other.
Solution:
Given Quadriatic Equation Px2 – 14x + 8 = 0
Also, one root is 6 times the other
Lets say one root = x
Second root = 6x
14
From the equation : Sum of the roots = +
P
8
Product of roots =
P
14
 x + 6x = .
P
2
x=
P
8
 6x2 =
P
2
2 8
 6  =
P P
64 8
=
P2 P
P =3

www.vedantu.com 3 / 22
CBSE-X-2017 EXAMINATION

1 1 3
6. Which term of the progression 20, 19 , 18 , 17 , .... is the first negative term ?
4 2 4
Solution:
1 1 3
Given progression 20, 19 , 18 , 17 , .......
4 2 4
This is an Arithmetic progression because

1 1 1
Common difference (d) = 19 − 20 = 18 − 19 = ..............
4 2 4
−3
d=
4

 −3  83 − 3n
Any nth term an = 20 + (n − 1)   =
 4  4

Any term an < 0 when 83 < 3n


This is valid for n = 28 and 28th term will be the first negative term.

7. Prove that the tangents drawn at the end points of a chord of a circle make equal angles with the chord.
Solution:
Need to prove that

BAP = ABP

AB is the chord
We know that OA = OB (radii)

OBP = OAP = 90


Join OP and OP = OP
By RHS congruency

www.vedantu.com 4 / 22
CBSE-X-2017 EXAMINATION

OBP  OAP
By CPCT BP = AP

In ABP BP = AP

Angles opposite to equal sides are equal


BAP = ABP

Hence proved.

8. A circle touches all the four sides of a quadrilateral ABCD. Prove that AB + CD = BC + DA
Solution:

ABCD is the Quadrilateral


Circle touches the sides at P, Q, R,S points
For the circle AS & AP are tangents
 AS = AP...............(1)

In the similar way


BP = BQ……..(2)
CQ = CR…………(3)
RD = DS …………..(4)
Now AB + CD = AP + PB + CR + RD
BC + AD = BQ + QC + DS + AS
Using (1), (2), (3), (4) in above equation BC + AD = BP + CR + RD + AP
 AB + CD = BC + AD

www.vedantu.com 5 / 22
CBSE-X-2017 EXAMINATION

Hence proved

9. A line intersects the y-axis and x-axis at the points P and Q respectively. If (2, -5) is the mid-point of PQ,
then find the coordinates of P and Q.
Solution:
x y
Equation of a line: + =1
a b
Where a =x-intercept
b = y-intercept
Given that line intersects y-axis at P
 P lies on y-axis and P = (0, b)
Line intersects x-axis at Q
 Q lies on x-axis and Q = (a, 0)
Midpoint of PQ = (2, -5)

a b
 ,  = (2, −5)
2 2
a b
= 2, = −5
2 2
a = 4 & b = −10
 P = (0, −10)
Q = (4,0)

10. If the distances of P(x, y) from A(5, 1) and B(-1, 5) are equal, then prove that 3x = 2y.
Solution:
Given that
PA = PB
P(x, y), A(5, 1), B(-1, 5)

PA = (x − 5)2 + (y − 1)2
PB = (x + 1)2 + (y − 5)2
(x − 5)2 + (y − 1)2 = (x + 1)2 + (y − 5)2

Squaring on both sides

www.vedantu.com 6 / 22
CBSE-X-2017 EXAMINATION

x 2 + 25 − 10 x + y 2 − 2y + 1 = x 2 + 2 x + 1 + y 2 − 10y + 25
−10 x − 2y = 2 x − 10y
8y = 12 x
 3 x = 2y

SECTION C
Question numbers 11 to 20 carry 3 marks each.
11. If ad  bc, then prove that the equation
(a2 + b2) x2 + 2 (ac + bd) x + (c2 + d2) = 0 has no real roots.
Solution:

Given ad  bc for the equation (a2 + b2 )x2 + 2(ac + bd)x + (c2 + d 2 ) = 0

For this equation not to have real roots its discriminant < 0
D = 4(ac + bd)2 – 4(a2 + b2)(c2 + d2)
D = 4a2c2 + 4b2d2 + 8acbd – 4a2c2 – 4b2d2 – 4b2c2 – 4a2d2

D = −4(a2d 2 + b2c 2 − 2acbd )


D = −4(ad − bc)2

Given ad  bc

D  0

Quadratic equation has no real roots

12. The first term of an A.P. is 5, the last term is 45 and the sum of all its terms is 400. Find the number of
terms and the common difference of the A.P.
Solution:
First term (a) = 5
Last term (l) = 45
Sum of all the terms = 400
n
400 = (a + l)
2
800
=n
50
n = 16
No. of terms = 16
l = 45 (16th term)

www.vedantu.com 7 / 22
CBSE-X-2017 EXAMINATION

a + (n − 1)d = 45
5 + 15d = 45
40
d=
15
8
d=
3
8
Common difference =
3

13. On a straight line passing through the foot of a tower, two points C and D are at distances of 4 m and
16 m from the foot respectively. If the angles of elevation from C and D of the top of the tower are
complementary, then find the height of the tower.
Solution:

Given CF = 4m
DF = 16m

TCF + TDF = 90


Lets say TCF = 

TDF =  − 
In a right angled triangle TCF

TF TF
tan  = =
CF 4
TF = 4 tan  ..........(1)

In TDF
TF
tan(90 − ) =
16
TF = 16cot ..............(2)

Multiply (1) & (2)

www.vedantu.com 8 / 22
CBSE-X-2017 EXAMINATION

(TF )2 = 64  TF = 8mt

 Height of tower = 8mt

14. A bag contains 15 white and some black balls. If the probability of drawing a black ball from the bag is
thrice that of drawing a white ball, find the number of black balls in the bag.
Solution:
Bag contains 15 white balls
Lets say there are x black balls
Probability of drawing a black ball

x
P(B) =
15 + x
Probability of drawing a white ball
15
P(W) =
15 + x
Given that P(B) = 3P(W)
x 3  15
 =
15 + x 15 + x
x = 45
No. of black balls = 45

 24 
15. In what ratio does the point  , y  divide the line segment joining the points P(2, -2) and Q(3, 7) ?
 11 
Also find the value of y.
Solution:

Lets say ratio is m + n


Then

www.vedantu.com 9 / 22
CBSE-X-2017 EXAMINATION

 24   3m + 2n 7m − 2n 
 ,y  =  , 
 11   m + n m+n 
24 3m + 2n 7m − 2n
= ,y =
11 m+n m+n
24(m + n) = 11(3m + 2n)
24m + 24n = 33m + 22n
2n = 9m
m 2
 =  ratio = 2: 9
n 9
m = 2, n = 9
72 − 29
y=
11
−4
y=
11

16. Three semicircles each of diameter 3 cm, a circle of diameter 4·5 cm and a semicircle of radius 4·5 cm
are drawn in the given figure. Find the area of the shaded region.

Solution:

Given that AB = BC = CD = 3 cm
Circle c has diameter = 4.5 cm
Semicircle s1 has diameter = 9cm
Area of shaded region

www.vedantu.com 10 / 22
CBSE-X-2017 EXAMINATION

= Area of s1 – Area of (s2+s4) – Area of c + Area of s3


Area of shaded region
2 2 2 2 2
 9  3  3  4.5    3 
=   −   −   −   +  
22 22 22  2  22
 81  
= −
16 8
= 12.36 cm2

17. In the given figure, two concentric circles with centre O have radii 21 cm and 42 cm. If  AOB = 60 ,
find the area of the shaded region.
 22 
Use  = 7 
 

Solution:

Given OC = OD = 21 cm
OA = OB = 42 cm
Area of ACDB region
= Area of sector OAB – Area sector OCD
60 60
=  (42)2 −   (21)2
360 360
1 22
Area of ACDB region =   21  63
6 7

www.vedantu.com 11 / 22
CBSE-X-2017 EXAMINATION

= 11  63 = 693 cm2

Area of shaded region= area of c1 – Area of c2 – Area of ACDB region

= (42)2 − () − 693


22
=  21  63 − 693
7
= 3,465 cm2

18. Water in a canal, 5·4 m wide and 1·8 m deep, is flowing with a speed of 25 km/hour. How much area
can it irrigate in 40 minutes, if 10 cm of standing water is required for irrigation ?
Solution:
Given canal width = 5.4 mt
Depth = 1.8 mt
Water flow speed = 25 km/hr
Distance covered by water in 40 minutes

25  40
=
60
50
= km
3
50
Volume of water flows through pipe =  5.4  1.8  1000
3

= 162  103 m3
Area irrigate with 10 cm of water standing

162  103
=
10  10−2
= 162  104 m2

19. The slant height of a frustum of a cone is 4 cm and the perimeters of its circular ends are 18 cm and 6
cm. Find the curved surface area of the frustum.
Solution:

www.vedantu.com 12 / 22
CBSE-X-2017 EXAMINATION

Given:
2r = 6cm
2R = 18cm
l = 4cm
Curved surface area of frustum of a cone
1
= (2R + 2r )  l
2
1
= (6 + 18)4
2
= 48cm2

20. The dimensions of a solid iron cuboid are 4·4 m  2·6 m  1·0 m. It is melted and recast into a hollow
cylindrical pipe of 30 cm inner radius and thickness 5 cm. Find the length of the pipe.
Solution:
Volume of cuboid = 4.4  2.6 1

11.44m3

length = l
Inner radius = 30 cm
outer radius = 35 cm
Volume of cuboid = volume of cylindrical pipe

www.vedantu.com 13 / 22
CBSE-X-2017 EXAMINATION

 l  (352 − 302 )
11.44 =
100  100  100
l = 10.205  104 cm
l = 102.05 km

SECTION D
Question numbers 21 to 31 carry 4 marks each.
21. Solve for x :
1 3 5 1
+ = , x  −1, − , − 4
x + 1 5x + 1 x + 4 5
Solution:
1 3 5 −1
+ = ; x  −1, , − 4
x + 1 5x + 1 x + 4 5
Take L.C.M. on the left hand side of equation

5x + 1 + 3(x + 1) 5
=
(x + 1)(5x + 1) x + 4
(x + 4)(8 x + 4) = 5(5x + 1)(x + 1)
8 x 2 + 4 x + 32 x + 16 = 25x 2 + 5 + 5x + 25x
17 x 2 − 6 x − 11 = 0

17 x 2 − 17 x + 11x − 11 = 0
17 x(x − 1) + 11(x − 1) = 0
(x − 1)(17 x + 11) = 0
−11
x = ,1
17

1
22. Two taps running together can fill a tank in 3 hours. If one tap takes 3 hours more than the other to
13
fill the tank, then how much time will each tap take to fill the tank ?
Solution:
1
Two taps when run together fill the tank in 3 hrs
13
Say taps are A, B and
A fills the tank by itself in x hrs
B fills tank in (x+3) hrs

1
Portion of tank filled by A (in 1hr) =
x

www.vedantu.com 14 / 22
CBSE-X-2017 EXAMINATION

1
Portion of tank filled by B (in 1hr) =
x +3
13
Portion of tank filled by A & B (both in 1hr) =
40
1 1 13
 + =
x x + 3 40
(x + 3 + x)40 = 13(x)(x + 3)
80 x + 120 = 13x 2 + 39 x
13x 2 − 41x − 120 = 0
x=5
 A fills tank in 5hrs

B fills tank in 8hrs

23. If the ratio of the sum of the first n terms of two A.Ps is (7n + 1) : (4n + 27), then find the ratio of their
9th terms.
Solution:
Given two A.P’s with n terms each
A.P I = first term = a1
A.PII = first term = a2
Common difference = d2
Sum of first n terms for A.PI = S1
n
S1 = 2a1 + (n − 1)d1 
2
n
Similarly S2 = 2a2 + (n − 1)d2 
2
S1 7n + 1
=
S2 4n + 27

(n − 1)
a1 + d1
2 7n + 1
 =
(n − 1)
a2 + d2 4n + 27
2
a1 + 8d1
Ratio of their 9th terms =
a2 + 8d2

Comparing

www.vedantu.com 15 / 22
CBSE-X-2017 EXAMINATION

(n − 1)
a1 + d1
2 a + 8d1
& 1
(n − 1)
a2 + d2 a2 + 8d2
2
Upon comparing
n −1
=8
2
 (n = 17) substituting n value

(n − 1)
a1 + d1
a1 + 8d1 2 7n + 1 7(17) + 1 120
 = = = =
a2 + 8d2 a + (n − 1) d 4n + 27 4(17) + 27 95
2 2
2
ratio = 24 : 19

24. Prove that the lengths of two tangents drawn from an external point to a circle are equal.
Solution:

PA & PB are the length of the tangents drawn from an external point P to circle C with radius r
OA = OB = r
OA ⊥ PA
OB ⊥ PB
Join O & P
In the triangles OAP & OBP
OA = OB (radii)
OP = OP (common side)
OAP = OBP = 90 (Right angle)

By RHS congruency

OAP  OBP
 By CPCT

PA = PB

www.vedantu.com 16 / 22
CBSE-X-2017 EXAMINATION

25. In the given figure, XY and X’Y’ are two parallel tangents to a circle with centre O and another tangent
AB with point of contact C, is intersecting XY at A and X’Y’ at B. Prove that  AOB = 90 .

Solution:

Prove that AOB = 90


In AOC and AOP
OA = OA (hypotenuse)
OP = OC (radii)
ACO = APO (right angle)
AOC  AOP By RHS congruency

By CPCT AOC = AOP .....(1)

Similarly In BOC & BOQ

OC = OQ
OB = OB
BCO = BQO = 90

By RHS congruency BOC  BOQ

By CPCT BOC = BOQ .....(2)

PQ is a straight line
AOP + AOC + BOC + BOQ = 180

www.vedantu.com 17 / 22
CBSE-X-2017 EXAMINATION

From equations (1) and (2)

2 ( AOC + BOC ) = 180

180
AOB =
2
AOB = 90

26. Construct a triangle ABC with side BC = 7 cm, B = 45 , A = 105 .


3
Then construct another triangle whose sides are times the corresponding sides of the ABC.
4
Solution:

In the ABC , A + B + C = 180

C = 30

To construct the similar triangle first we need to construct ABC


For ABC

1). Draw BC = 7cm with help of a ruler

2) Take a protractor measure angle 45 from point B and draw a ray BX

3) From point c, mark 30 with help of protractor & draw a ray CY

4) Now both BX and BY intersect at a point and this point is A


Now we have ABC
3
To construct similar triangle with corresponding sides of the sides of ABC
4
Step 1: Draw any raw making an acute angle with BC
Step 2: Along the ray BZ mark 4 points B1, B2, B3, B4 such that BB1 = B1B2 = B2B3 = B3B4

www.vedantu.com 18 / 22
CBSE-X-2017 EXAMINATION

Step 3: Now join B4 to C and draw a line parallel to B4C from B3 intersecting the line BC line BC at C’
Step 4: Draw a line through C’ parallel to CA which intersects BA at A’
A’BC’ is the required tri

Justification:
C ' A'||CA By construction

A' BC ' ~ ABC [using AA similarity]

A ' B BC ' A ' C '


 = = [corresponding sides ratio will be proportional
AB BC AC
B4C ||B3C ' [By construction]

BB4C ~ BB3C ' [By AA similarity]

BC ' BB3
= [By BPT]
BC BB4

BB3 3
But we know =
BB4 4

A ' B BC ' A '.C ' 3


 = = =
AB BC AC 4

www.vedantu.com 19 / 22
CBSE-X-2017 EXAMINATION

27. An aeroplane is flying at a height of 300 m above the ground. Flying at this height, the angles of
depression from the aeroplane of two points on both banks of a river in opposite directions are 45 and
30 respectively. Find the width of the river. [Use 3 = 1·732]
Solution:

Given aeroplane is at height of 300m


 AB = 300m and XY ||PQ
Angles of depression of the two points P & Q are 30 and 45
XAP = 30 & YAQ = 45
XAP = APB = 30
[Alternative Interior angles]
YAQ = AQB = 45
In PAB
AB
tan30 =
PB
PB = 300 3 mt
In BAQ
AB
tan45 =
BQ
BQ = 300m
 Width of the river = PB + BQ
= 300(1 + 3)mt

28. If the points A(k + 1, 2k), B(3k, 2k + 3) and C(5k - 1, 5k) are collinear, then find the value of k.
Solution:
Given A(k + 1,2k), B(3k ,2k + 3), C(5k − 1,5k) are collinear.
If three points are collinear then the area of the triangle will be zero. For any 3 points (x 1, y1), (x2, y2), (x3,
y3) Area will be
1
 A = x1 (y2 − y3 ) + x2 (y3 − y1 ) + x3 (y1 − y2 )
2
1
O = (k + 1)(2k + 3 − 5k) + 3k(5k − 2k) + (5k − 1)(2k − 2k − 3)
2
O = (k + 1)(3 − 3k) + 3k(3k) − 15k + 3

www.vedantu.com 20 / 22
CBSE-X-2017 EXAMINATION

−3k 2 + 3 + 9k 2 + 3 − 15k = 0
6k 2 − 15k + 6 = 0
1
k = 2,
2

29. Two different dice are thrown together. Find the probability that the numbers obtained have
(i) even sum, and
(ii) even product.
Solution:
Two dice are through together total possible outcomes = 6  6 = 36
(i) Sum of outcomes is even
This can be possible when
 Both outcomes are even
 Both outcomes are odd
For both outcomes to be Even number of cases = 3  3 = 9
Similarly
Both outcomes odd = 9 cases
Total favourable cases = 9 + 9 =18
18
Probability that =
36
1
Sum of the even outcomes is
2
(ii) Product of outcomes is even
This is possible when
 Both outcomes are even
 first outcome even & the other odd
 first outcome odd & the other even
Number of cases where both outcomes are even = 9
Number of cases for first outcome odd = 9
and the other Even
No. of cases for first outcome odd & the other even = 9
Total favourable cases = 9 + 9 + 9 = 27
27
Probability =
36
3
=
4

30. In the given figure, ABCD is a rectangle of dimensions 21 cm  14 cm. A semicircle is drawn with BC as
diameter. Find the area and the perimeter of the shaded region in the figure.

www.vedantu.com 21 / 22
CBSE-X-2017 EXAMINATION

Solution:

Area of shaded region = Area of rectangle – Area of semicircle


(7)2
= 21  14 −
2
= 217 cm 2

Perimeter of shaded region


= AB + AD + CD + length of arc BC
180 22
= 21 + 14 + 21 + 2  7
360 7
= 78 cm

31. In a rain-water harvesting system, the rain-water from a roof of 22 m  20 m drains into a cylindrical
tank having diameter of base 2 m and height 3·5 m. If the tank is full, find the rainfall in cm. Write your
views on water conservation.
Solution:
Water from the roof drains into cylindrical tank
Volume of water from roof flows into the tank of the rainfall is x cm and given the tank is full we can write
volume of water collected on roof = volume of the tank
2
22  20  x 2
=     3.5
100 2
x = 2.5 cm
 .rainfall is of 2.5 cm

www.vedantu.com 22 / 22
X - CBSE BOARD - 2018

CODE ( 30/3 )
Date: 28.03.2018 Mathematics - QXHVWLRQPDSHU 6ROXWLRQV

SECTION A
1. What is the value of (cos267° – sin223°) ?

Ans. cos2 670 sin2 230


as cos  90 0     sin 
Let   230
cos  90 0 - 230  = sin230

cos 67 0  sin 230


 cos2 670  sin2 230
∴ cos2 670 −sin2 230 =0

2. In an AP, if the common difference (d) = –4, and the seventh term (a7) is 4, then find the first term.

Ans. a7  4
a  6d  4  as an  a   n 1 d 
but d  4
a  64  4
a  24  4
a  4  24  28
Therefore first term a = 28

AB 1 ar ABC
3. Given ABC  PQR , if  , then find .
PQ 3 ar PQR

A  ABC AB2
Ans. 
A  PQR  PQ2 (Ratio of area of similar triangle is equal to square of their praportional sides)

www.vedantu.com 1
2
A  ABC   1  1
  
A  PQR   3  9

4. What is the HCF of smallest prime number and the smallest composite number ?
Ans. Smallest prime number is 2.
Smallest composite number is 4
Therefore HCF is 2.

5. Find the distance of a point P(x, y) from the origin.


Ans. Using distance formual
2 2
  OP    x  0   y  0
  OP   x 2  y 2

6. If x = 3 is one root of the quadratic equation x 2  2kx  6  0 , then find the value of k.

Ans.  x = 3 is one of the root of x 2  2kx  6  0


 3 2  2k  3   6  0
9  6k  6  0
3  6k  0
3  6k
3 1
k 
6 2

SECTION B
7. Two different dice are tossed together. Find the probability :
(i) of getting a doublet
(ii) of getting a sum 10, of the numbers on the two dice.

Ans. Sample space  S  1,11,2 ......., 6,6 

n  s   36
i) A = getting a doublet
A = {(1, 1), (2, 2) ......., (6, 6)}
n A   6

www.vedantu.com 2
n A  6 1
 P A    
n  S  36 6
ii) B = getting sum of numbers as 10.
B = {(6, 4), (4, 6), (5, 5)}
n B  3
nB 3 1
 P B   
n  S  36 12

8.Find the ratio in which P(4, m) divides the line segment joining the points A(2, 3) and B(6, –3). Hence find m.
Ans. Suppose the point P(4, m) divides the line segment joining the points A(2, 3) and B(6, -3) in the ratio
K : 1. A K P 1 B
(2, 3) (4, m) (6, -3)
 6K  2 3K  3 
Co-ordinates of point P   , 
 K 1 K 1 
But the co-ordinates of point P are given as (4, m)

6K  2
4 ......(1) and
K 1
3K  3
m ......(2)
K 1
6K  2  4K  4
2K = 2
K=1
Putting K = 1 in eq. (2)
3 1  3
m
11
 m0
Ratio is 1 : 1 and m = 0
i.e. P is the mid point of AB

9. An integer is chosen at random between 1 and 100. Find the probability that it is :
(i) divisible by 8
(ii) not divisible by 8
Ans. An integer is chosen at random from 1 to 100
Therefore n(S) = 100
(i) Let A be the event that number chosen is divisible by 8
 A  8,16, 24,32, 40, 48,56, 64, 72,80,88,96

www.vedantu.com 3
 n  A   12

n A
Now, P (that number is divisible by 8)  P  A   n S
 
12 6 3
  
100 50 25
3
P A 
25
(ii) Let ‘A’ be the event that number is not divisible by 8.
 P  A '  1  P  A 

3 22
 1 P  A ' 
25 25

10. In figure.1, ABCD is a rectangle. Find the values of x and y.

D x+y C

14cm x–y

A 30cm B
Figure 1
Ans. Since it is a rectangle

  AB     CD 

x  y  30 ...(i)

  AD     BC 

x  y  14 ...(ii)
Adding (1) and (2), we get
2 x  44
x = 22
Putting x = 22 in equation (i)
22  y  14  22  14  y

 y 8

∴ x =22 and y =8

www.vedantu.com 4
11. Find the sum of first 8 multiples of 3.
Ans. First 8 multiples of 3 are
3, 6, 9, 12, 15, 18, 21, 24
The above sequence is an A.P.
a = 3, d = 3 and last term l = 24

n 8
Sn   a  l   3  24  4  27 
2 2

Sn  108

12.  
Given that 2 is irrational, prove that 5  3 2 is an irrational number..

Ans.  
Let us assume that 5  3 2 is rational. Then there exist co-prime positive integers a and b such that

a
53 2 
b
a
3 2 5
b
a  5b
2
3b
a  5b
 2 is rational. [ a, b are integers ,  3b is rational].
This contradicts the fact that 2 is irrational.
So our assumption is incorrect.

 
Hence, 5  3 2 is an irrational number..

SECTION C
13. If A(–2, 1), B(a, 0), C(4, b) and D(1, 2) are the vertices of a parallelogram ABCD, find the values of a and b.
Hence find the lengths of its sides.
Ans. M is midpoint of AC and BD using midpoint formula,
(–2,1) (a,0)
 2  4 1  b   a  1 2  0  A B
 ,  , 
 2 2   2 2  M

 2 1 b   a 1 2  D C
 ,  , 
2 2   2 2 (1,2) (4,b)

www.vedantu.com 5
2 a 1
   a 1  2  a  1
2 2

1 b 2
and   1 b  2  b  1
2 2

OR
If A(–5, 7), B(–4, –5), C(–1, –6) and D(4, 5) are the vertices of quadrilateral, find the area of the quadrilateral
ABCD.

1
Ans. A  ABC  
2
 x1  y2  y3   x2  y3  y1   x3  y1  y2  
If A   x1 , y1  , B   x2 , y2  , C   x3 , y3  are vertices of ABC .
A(–5,7) B(–4,–5)

D(4,5) C(–1,–6)

A  ABCD   A  ABC   A  ADC  ....(i)

1
A  ABC    5  5  6   4  6  7   1 7  5  
2

1
  5  52  12
2

1
 35
2

35
 Sq.units
2

1
A  ADC    5  5  6   4  6  7   1 7  5  
2

1
  55  52  2
2

109

2
 Area cannot be negative.

109
 A  ADC   sq.units
2

www.vedantu.com 6
35 109 144
 A  ABCD      72 sq. units
2 2 2

14.    
Find all zeroes of the polynomial  2 x 4  9 x 3  5 x 2  3 x  1 if two of its zeroes are 2  3 and 2  3 .

Ans.    
It is given that 2  3 and 2  3 are two zeros of f  x   2x 4  9x 3  5x 2  3x  1

x   2  3 x   2  3    x  2  3  x  2  3 
2
2
  x  2   3
 x 2  4x  1
 x 2
 4x  1 is a factor of f  x 

2x 2  x  1
x 2  4x  1 2x 4  9x 3  5x 2  3x  1
2x 4  8x 3  2x 2
____________________
     

x 3  3x 2  3x  1
x 3  4x 2  x
     
__________________
x 2  4x  1
x 2  4x  1
_________________
     
0
_________________

Let us now divide f  x  by x 2  4x  1


We have,

 f  x    x 2  4x  1 2x 2  x  1
Hence, other two zeros of f(x) are the zeros of the polynomial 2x 2  x  1
We have,
2x 2  x  1  2x 2  2x  x  1
 2x  x  1  1 x  1

www.vedantu.com 7
  2x  1 x  1

  
f  x   x  2  3 x  2  3  2 x  1 x  1

1
Hence, the other two zeros are  and 1.
2

15. Find HCF and LCM of 404 and 96 and verify that HCF × LCM = Product of the two given numbers.
Ans. Using the factor tree for the prime factorization of
404 and 96, we have
404 = 22 × 101 and 96 = 25 × 3
To find the HCF, we list common prime factors and their smallest exponent in 404 and 96 as under :
Common prime factor = 2, Least exponent = 2
 HCF  22  4
To find the LCM, we list all prime factors of 404 and 96 and their greatest exponent as follows :
Prime factors of 404 and 96 Greatest Exponent
2 5
3 1
101 1

 LCM  25  31  1011
 2 5  3 1  1 0 11
 9696
Now,
HCF × LCM = 9696 × 4 = 38784
Product of two numbers = 404 × 96 = 38784
Therefore HCF × LCM = Product of two numbers.

16. Prove that the lengths of tangents drawn from an external point to a circle are equal.

Ans. Given AP and AQ are two tangents from a point A to a circle C (O, r )

To prove AP = AQ
Construction join OP, OQ and OA
Proof In order to prove that AP  AQ , we shall first prove that OPA  OQA
since a tangent at any point of a circle is perpendicular to the radius through the point of contact.
 OP  AP and OQ  AQ

 OPA  OQA  900 ........(i)


Now, in right triangles OPA and OQA, we have

www.vedantu.com 8
OP  OQ [Radii of a circle]
P
OPA  OQA [from (i)]
and OA = OA A O
so, by RHS – criterion of congruence, we get
OPA  OQA Q

⇒AP =AQ

17. Prove that the area of an equilateral traingle described on one side of the square is equal to half the area of
the equilateral triangle described on one of its diagonal.
Ans. Let a be the side of square.
C
3 3 2
A  ABC    side 2   a ..... 1
4 4
using pythagoras theorem
A a
2 2 2 2 2 2
B
AD  AB  BD  a  a  2a

AD  2a 2a a
2a
3 2 3
 A  ADE  
4
  2a  
4
 2a 2 .....  2  D
E 2a
A  ABC  3 / 4a 2

A  ADE  3 / 4  2a 2

1
A  ABC   A  ADE 
2
Area of equivalent triangle describes on
1
  area of equilateral  described an one of its diagonal 
2

OR

If the area of two similar triangles are equal, prove that they are congruent.

A P

Ans.

B C Q R

www.vedantu.com 9
Let ABC is PQR

A  ABC  AB 2 BC 2 AC 2
   
A  PQR  PQ 2 QR 2 PR 2

Given that  ABC   A  PQR 

A  ABC 
 1
A  PQR 

AB 2 BC 2 AC 2
1  
PQ 2 QR 2 PR 2

 AB  PQ

BC  QR

AC  PR
Hence corresponding sides are equal.
 ABC  PQR (SSS rule)
hence proved.

18. A plane left 30 minutes late than its scheduled time and in order to reach the destination 1500 km away in
time, it had to increase its speed by 100 km/h from the usual speed. Find its usual speed.
Ans. Let the usual speed of the plane be x km/hr

1500
Time taken to cover 1500 km with usual speed  hrs
x

1500
Time taken to cover 1500 km with speed of  x  100  km / hr  hrs.
x  100

1500 1500 1
  
x x  100 2

1500 1500 1
 
x x  100 2

 x  100  x  1
1500   
 x  x  100   2

150000  2  x  x  100 

www.vedantu.com 10
x 2  100 x  300000  0

x 2  100 x  300000  0
x  600 or x  500
But speed can’t be negative
Hence usual speed 500 km/hr.

19. The table below shown the salaries of 280 persons:

Salary (In thousand ) No.of Person


5  10 49
10  15 133
15  20 63
20  25 15
25  30 6
30  35 7
35  40 4
40  45 2
45  50 1

Calculate the median salary of the data.


Ans.
Class Frequency Cumulative Frequency
5 - 10 49 49
10 - 15 133 182
15 - 20 63 245
20 - 25 15 260
25 - 30 6 266
30 – 35 7 273
35 - 40 4 277
40 – 45 2 279
45 – 50 1 280

Let N  total frequency


 we have N  280

N 280
   140
2 2

N
The cumulative frequency just greater than is 182 and the corresponding class is 10  15
2

www.vedantu.com 11
Thus, 10  15 is the median class such that

l  10, f  133, F  49 and h  5

N
F
 140  49 
Median  l  2  h  10   5
f  133 

=13.42

20. A wooden article was made by scooping out a hemisphere from each end of a solid cylinder, as shown in
Fig. 2. If the height of the cylinder is 10 cm and its base is of radius 3.5 cm. Find the total surface area of the
article.

Ans. Let r be the radius of the base of the cylinder and h be its height. Then, total surface area of the article
= Curved surface area of the cylinder + 2 (surface area of a hemisphere)

 2 rh  2  2 r 2 

 2 r  h  2r 

22
 2  3.5 10  2  3.5  cm 2
7

=22×17 cm2 =374 cm2

OR
A heap of rice is in the form of a cone of base diameter 24 m and height 3.5 m. Find the volume of the rice.
How much canvas cloth is required to just cover the heap?
Ans. Given
Base diameter = 24 m
Base radius = 12 m
Height = 3.5 m

1
Volume   r 2 h
3

www.vedantu.com 12
1 22
   12  12  3.5
3 7

 22  4 12  0.5
3.5m
 264  2
12m
 528 cubic meter

  2  12 2  3.52  144  12.25

 2  156.25

  156.25  12.5 m
Curved surface area   r

22 150  22
 12  12.5   471.428sq.meter
7 7

21. Find the area of the shaded region in Fig. 3, where arcs drawn with centres A, B, C and D intersect in pairs
at mid-points P, Q, R and S of the sides AB, BC, CD and DA respectively of a square of side 12 cm, [Use
  3.14 ]

A P B

S Q

C R D
Fig.–3

Ans. Given that ABCD is a square & P,Q,R & S are the mid points of AB,BC,CD & DA respectively
& AB  12 cm

 AP  6cm P bisects AB


area of the shaded region = Area of square ABCD – (Area of sector APEC + Area of sector PFQB + .....
Area of sector RGQC + Area of sector RHSD) A P B

   62   62  6 2  62  E F
2
 12      
 4 4 4 4  S Q
 
H G
2
 12    36
C R D

www.vedantu.com 13
=144 −113.04

=30.96 cm2

 4sin   cos   1 
22. If 4 tan   3, evaluate  
 4sin   cos  1 
Ans. Given that,

3 9
tan    tan 2  
4 16
we know that,

sec 2   1  tan 2 

9 25
 sec 2   1  
16 16
16
 cos 2  
25
4
 cos  
5
we know that,

sin 2   1  cos 2 

16 9
 sin 2   1  
25 25

3
 sin  
5
Now,

 3 4 
 4    1 
 4sin   cos   1   5 5 
 
 4sin   cos   1   4   3    4   1 
 5 5 
     

12  4  5

12  4  5

13

11

www.vedantu.com 14
OR
If tan 2 A  cot ( A  18), where 2 A is an angle, find the value of A.
Ans. Given that,

tan 2 A  cot  A  180 

Now,
we know that,

tan   cot  90 0   

 cot  900  2 A   cot  A  180 

 900  2 A  A  180

 3 A  1080

1080
A  360
3

∴A =360

SECTION D
23. As observed from the top of a 100 m high light house from the sea-level, the angles of depression of
two ships are 30° and 45°. If one ship is exactly behing the other on the same side of the light house, find the
distance between the two ships. [Use 3  1.732 ]
Ans. Let ships are at distance x from each other P

In APO
100m
100
tan 45  1  y  100 m ...(i)
y
45° 30°
y A x B
In POB O

OP 100 1
tan 30   
OB x  y 3

x y
3
100

x  y  100 3 ...(ii)

x  100 3  y  100 3  100  100  3 1

www.vedantu.com 15
 x  100 1.732  1

 100  0.732
 73.2 m
∴ Ships are 73.2 meters apart.

24. The diameters of the lower and upper ends of a bucket in the form of a frustum f a cone are 10 cm and 30
cm respectively. If its height is 24 cm, find:
(i) The area of the metal sheet used to make the bucket.
(ii) Why we should avoid the bucket made by ordinary plastic? [Use   3.14 ]

Ans. Let r1  5 cm and r2  15 cm are radii of lower and upper circular faces.
Metal sheet required = Area of curved surface + Area of Base

   r1  r2     r12 ...(i)

From diagram
AB = CD = 5 cm
DE = 15 – 5 = 10 cm
and BD = 24 cm

 BE 2  BD 2  DE 2 C D E
15cm
 576  100
24cm 24cm
BE 2  676
BE  26 cm   A 5cm B
2
Metal required    5  15  26    5 

   20  26    25

 5  4  26  5 

 5 109 

22
 5  109
7

=1712.85cm2
There is a chance of breakdown due to stress an ordinary plastic.

www.vedantu.com 16
sin A  2sin 3 A
25. Prove that  tan A.
2 cos3 A  cos A

Ans. To prove

sin A  2sin 3 A
 tan A
2 cos3 A  cos A

sin A 1  2sin 2 A 
L.H .S 
cos A  2 cos 2 A  1

We know that, sin 2 A  cos 2 A  1

sin A   sin A  cos A  2sin A  


2 2 2

  
cos A   2 cos 2 A  sin 2 A  cos 2 A 
 

 cos2 A  sin 2 A 
 tan A  2 2 
 cos A  sin A 

=tan A
=R.H.S. hence proved.

26. The mean of the following distribution is 18. Find the frequency f of the class 19-21.

Class 11-13 13-15 15-17 17-19 19-21 21-23 23-25


Frequency 3 6 9 13 f 5 4

Ans.

Class Mid values xi Frequence f i d i  xi  18 xi  18 f iui


ui 
2
11 – 13 12 3 6 3 9
13 - 15 14 6 4 2  12
15 - 17 16 9 2 1 9
17 - 19 18 13 0 0 0
19 - 21 20 f 2 1 f
21 - 23 22 5 4 2 10
23 - 25 24 4 6 3 12
f i  40  f

 fui i  f 8

we have

www.vedantu.com 17
h  2; A  18, N  40  f ,  f iui  f  8 X  18

1 
 Mean  A  h 
N
 f u 
i i

 1 
18  18  2  f  8
 40  f 

2  f  8
0
40  f

f −8 =0

f =8

OR
The following distribution gives the daily income of 50 workers of a factory :

Daily Income(In ) 100-120 120-140 140-160 160-180 180-200


Number of workers 12 14 8 6 10

Convert the distribution above to a less than type cumulative frequency distribution and draw its ogive.
Daily income Frequency Income less than Cumulative frequency
100-120 12 120 12
120-140 14 140 26
140-160 8 160 34
Ans.
160-180 6 180 40
180-200 10 200 50
Other than the given class intervals, we assum a class interval 80-100 with zero frequency.
Cumulative Frequency

50 (200,50)

40 (180,40)
30 (160,34)

20 (140,26)

10 (120,12)

100 120 140 160 180 200


Number of workers

www.vedantu.com 18
27. A motor boat whose speed is 18 km/hr in still water 1 hr more to go 24 km upstream than to return
downstream to the same spot. Find the speed of the stream.
Ans. Let the speed of stream be x km / hr

Now, for upstream: speed  (18  x ) km / hr

 24 
 time taken    hr
 18  x 

Now, for downstream: speed  (18  x ) km / hr

 24 
 time taken    hr
 18  x 
Given that,

24 24
 1
18  x 18  x

24 24
1  
18  x 18  x

24  (18  x)  (18  x) 
1 
(18)2  x 2

24  2 x 
1
324  x 2

 324  x 2   48 x

x 2  48 x  324  0

x 2  54 x  6 x  324  0

( x  54) ( x  6)  0

x   54 or x  6

x   54 km / hr (not possible)
Therefore, speed of the stream = 6 km/hr.

OR
A train travels at a certain average speed for a distance of 63 km and then travels at a distance of 72 km
at an average speed of 6 km/hr more than its original speed. It it takes 3 hours to complete total journey,
what is the original average speed ?
Ans. Let x be the original average speed of the train for 63 km.
Then, (x + 6) will be the new average speed for remaining 72 km.

www.vedantu.com 19
Total time taken to complete the journey is 3 hrs.
63 72
  3
x  x  6

 Distance 
 Time = 
 Speed 

63x  378  72x


 3
x  x  6

 135x  378  3x 2  18x


 x 2  39x  126  0
  x  42  x  3  0

 x  42 OR x  3
Since speed can not be negative.
Therefore x = 42 km/hr.

28. The sum of four consecutive numbers in an AP is 32 and the ratio of the product of the first and the last term
to the product of two middle terms is 7 : 15. Find the numbers.
Ans. Let the numbers be (a, –3d), (a – d), (a + d) and (a + 3d)

  a  3d    a  d    a  d    a  3d   32
 4a  32
a 8
 a  3d  a  3d   7
Also,
 a  d  a  d  15
 15a 2  135d 2  7a 2  7d 2
 8a 2  128d 2

2 8a 2 8  8  8
d  
128 128
d2  4
d  2
If d =2 numbers are : 2, 6,10, 14
If d = −2 numbers are 14,10,16,2

www.vedantu.com 20
29. Draw a triangle ABC with BC = 6 cm, AB = 5 cm and ABC  60 . Then construct a triangle whose sides
3
are of the corresponding sides of the ABC .
4
Ans. STEPS OF CONSTRUCTION :

5cm

C' C
B 6cm
B1
B2
B3
B4

(i) Draw a line segment BC = 6 cm, draw a ray BX making 60° with BC.
(ii) Draw an arc with radius 5 cm from B so that it cuts BX at A.

(iii) Now join AC to form ABC .


(iv) Draw a ray by making an acute angle with NC opposite to vertex A.

(v) Locate 4 points B1 , B2 , B3 , B4 on by such that BB1  B1B2  B2 B3  B3 B4 .

(vi) Join B4C and now draw a line from B3 parallel to B4C so that it cuts BC at C ' .

(vii) From C ' draw a line parallel to AC and cuts AB at A ' .


(viii) A ' BC ' is the required triangle.

1 2 2
30. In an equilateral ABC , is a point on side BC such that BD  BC . Prove that 9  AD   7  AB  .
3
A
Ans. Let the each side of ABC be ‘a’ unit
a
 BD 
3
2 2
to prove : 9  AD   7  AB  B D C
construction : Draw AM  BC :
A
a a a
DM   
2 3 6
 In ABM
AB2  BM 2  AM 2 .......(1)
B D M C

www.vedantu.com 21
and in ADM
AD 2  AM 2  DM 2 .......(2)
AM
In ABM, sin 600 
AB
 AM  ABsin 600

3
a
2
2
Now, taking 9  AD 

9  AM 2  DM 2 

  a 3 2 2
a 
9    
  2   6  
 

 3a 2 a 2  28a 2
9    9
 4 36  36
2
7  AB   7a 2
or
∴9 (AD2 = )7 (AB2 ) Hence proved.

OR
Prove that, in a right triangle, the square on the hypotenuse is equal to the sum of the squares on the other
two sides.
Ans. Given : A right - angled triangle ABC in which B  900
To Prove : (Hypotenuse)2 = (Base)2 + (Perpendicular)2
i.e., AC2 = AB2 + BC2
Construction from B draw BD  AC.
B

A D C
Proof : In triangle ADB and ABC, we have
ADB  ABC [Each equal to 90o]
and, A  A [Common]
So, by AA - similarity criterian, we have
ADB  ABC
AD AB
  [ In similar triangles corresponding sides are proportional]
AB AC

www.vedantu.com 22
 AB2  AD  AC .......(1)
In triangles BDC and ABC, we have
CDB  ABC [Each equal to 90o]
and, C  C [Common]
So, by AA-similarity criterian, we have
BDC  ABC
DC BC
  [ In similar triangles corresponding sides are proportional]
BC AC
 BC 2  AC  DC ......(2)
Adding equation (1) and (2), we get
AB2  BC 2  AD  AC  AC  DC
2 2
 AB  BC  AC  AD  DC 
 AB2  BC 2  AC  AC
 AB2  BC 2  AC 2
Hence, AC2 = AB2 + BC2

www.vedantu.com 23
Class X
Mathematics
(CBSE 2019)
Time: 3Hr
Marks : 80

GENERAL INSTRUCTIONS :
(i) All questions are compulsory.
(ii) This question paper consists of 30 questions divided into four sections - A, B, C and D.
(iii) Section A contains 6 questions of 1 mark each.
Section B contains 6 questions of 2 marks each.
Section C contains 10 questions of 3 marks each.
Section D contains 8 questions of 4 marks each.
(iv) There is no overall choice. However, an internal choice has been provided in two questions of
1 mark, two questions of 2 marks, four questions of 3 marks each and three questions of 4
marks each. You have to attempt only one of the alternatives in all such questions.
(v) Use of calculator is not permitted.

Section A

Question numbers 1 to 6 carry 1 mark each.


1. Find the coordinates of a point A , where AB is diameter of a circle whose centre is  2, –3
and B is the point 1, 4  .
Solution: Let the centre be O and coordinates of point A be  x, y 
x 1
2 [By Mid-point formula]
2
Implies that
x3
y4
 –3
2
 Coordinates of A   3, –10 
2. For what values of k, the roots of the equation x2  4 x  k  0 are real?
OR
Find the value of k for which the roots of the equation 3x2 –10 x  k  0 are reciprocal of each
other.
Solution:

x2  4 x  k  0
 Roots of given equation are real,
D0
Implies that

1
 4 – 4 k  0
2

Implies that
–4k  –16
Implies that
k4
 k has all real values  4
OR
3x –10 x  k  0
2

∵ Roots of given equation are reciprocal of each other.


1
Let the roots be  and

c
Product of roots 
a
Implies that
1 k
, 
 3
 k  3.
3. Find A if tan 2 A  cot  A – 24
OR
Find the value of  sin 2 33  sin 2 57 
Solution:
Given:
tan 2 A  cot  A  24 
Implies that tan 2 A  tan 90   A  24  
Implies that tan 2 A  tan 90  A  24
Implies that tan 2 A  tan 114  A
Implies that 2 A  114  A
Implies that 3 A  114
114
Implies that A 
3
Implies that A  38
OR
Given:
sin 2 33  sin 2 57
 sin 2 33  cos  90  57  
2

 sin 2 33  cos 2 33


1
4. How many two digits numbers are divisible by 3?

2
Solution:
Two digits numbers divisible by 3 are
12,15,18, .....,99.
a  12, d  15 –12  3
Implies that
Tn  99
Implies that
a   n –1 d  99
Implies that
12   n –1 3  99
Implies that
n  30
Number of two digit numbers divisible by 3 are 30.

5.In Fig. 1, DE || BC, AD  1 cm and BD  2 cm . What is the ratio of the ar  ABC  to the ar
(ADE ) ?

Solution:
DE || BC
ADE ABC [By AA similarity]
ar  ABC  AB 2
  [By area similarity theorem]
ar  ADE  AD 2
ar  ABC  32
 
ar  ADE  12
ar  ABC  9
 
ar  ADE  1
6. Find a rational number between 2 and 3 .
15 3
Sol. Rational number lying between 2 and 3 is 1.5  
10 2
 2 1.414and 3 1.732

Section B

3
7. Find the HCF of 1260 and 7344 using Euclid's algorithm.
OR
Show that every positive odd integer is of the form  4q  1 or  4q+3 , where q is some
integer.
Since 7344 > 1260
7344  1260  5  1044
Since remainder  0
1260  1044 1  216
1044  216  4  180
216  180 1  36
180  36  5  0
The remainder has now become zero.
 HCF of 1260 and 7344 is 36.
OR
Let a be positive odd integer
Using division algorithm on a and b = 4
a  4q  r Since 0  r < 4, the possible remainders are 0, 1, 2 and 3
 a can be 4q or 4q + 1 or 4q + 2 or 4q + 3, where q is the quotient
Since a is odd, a cannot be 4q and 4q + 2
 Any odd integer is of the form 4q + 1 or 4q + 3, where q is some integer.
8. Which term of the AP 3, 15, 27, 39, .... will be 120 more than its 21st term?
OR
If S n , the sum of first n terms of an AP is given by Sn  3n2 – 4n , find the nth term.
Solution:
Given AP is
3, 15, 27, 39 ....
where a = 3, d = 15 – 3 = 12
Let the nth term be 120 more than its 21st term.
tn  t21  120
 3   n –112  3  20 12  120
  n –1 12  363 – 3
360
  n  1 
12
 n  31
Hence, the required term is
t31  3  30 12
 363
OR
Sn  3n2 – 4n
Let Sn –1 be sum of  n –1 terms

4
tn  Sn – Sn –1
  3n 2 – 4n  – 3  n –1 – 4  n –1 
2
 
  3n – 4n  – 3n – 6n  3 – 4n  4 
2 2

 3n 2 – 4n – 3n 2  10n – 7
 t n  6n – 7
Therefore, required nth term  6n – 7
9. Find the ratio in which the segment joining the points (1, –3) and (4, 5) is divided by x-axis?
Also find the coordinates of this point on x-axis.
Solution:
Let C  x, 0  divides the line segment joining the points A 1,–3 and B  4,5 in k :1 ratio.
By section formula,

mx2  nx1 my2  ny1 


 x, y    , 
 mn mn 
Implies that
 4k  11 5k  1  3 
 x, 0    , 
 k 1 k 1 
Implies that
4k  1 5k  3 
 x, 0    , 
 k 1 k 1 
Implies that
5k  3
0
k 1
Implies that
5k  3  0

5
Implies that
5k  3
3
k
5
3
4 1
4k  1 5
and x  
k 1 3
1
5
12  5
x 5
35
5
17
x
8
 17 
Therefore, coordinates of point P are  , 0  .
 8 
10. A game consists of tossing a coin 3 times and noting the outcome each time. If getting the
same result in all the tosses is a success, find the probability of losing the game.
Solution:
Total possible outcomes are (HHH), (HHT), (HTH), (THH), (TTH), (THT), (HTT), (TTT) i.e., 8. [½]
The favourable outcomes to the event E 'Same result in all the tosses' are TTT, HHH. Therefore,
the number of favourable outcomes = 2
2 1
P E  
8 4
Hence, probability of losing the game = 1  P  E 
1 3
 1 
4 4
11. A die is thrown once. Find the probability of getting a number which (i) is a prime number
(ii) lies between 2 and 6.
Solution:
Total outcomes = 1, 2, 3, 4, 5, 6 Prime numbers = 2, 3, 5 Numbers lie between 2 and 6 = 3, 4, 5
3 1
(i) P (Prime Numbers)  
6 2
3 1
(ii) P (Numbers lie between 2 and 6)  
6 2
12. Find c if the system of equations cx  3 y   3  c   0,12 x  cy  c  0 has infinitely many
solutions?
Solution:
cx  3 y   3  c   0 ……………………………….(i)
12 x  cy  c  0 ………………………………………(ii)

6
a1 b1 c1
For infinitely many solutions  
a2 b2 c2
c 3 3c
   3 3c
12 c c 
c c
c 3
  or  c  c  6  0
12 c
 c  0, 6
 c 2  36
 c  6
Hence the value of c  6 .

Section C

Question numbers 13 to 22 carry 3 marks each.


13. Prove that 2 is an irrational number.
Solution:
Let 2 be rational.
p
 2  where p and q are co-prime integers and , q 0
q
Implies that 2q  p
2q 2  p 2 .......  i 
 2 divides p 2
 2 divides p.......  A 
Let p  2c for some integer c
p 2  4c 2
 2q 2  4c 2
 q 2  2c 2
 2 divides q 2
 2 divides q........  B
From  A  and  B  , we get
 2 is common factor of both p and q. But this contradicts the fact that p and q have no
common factor other than 1
 Our supposition is wrong Hence, 2 is an irrational number.
14. Find the value of k such that the polynomial x2 –  k  6 x  2  2k –1 has sum of its zeros
equal to half of their product.
Solution:
For given polynomial x2 –  k  6 x  2  2k –1
Here
a  1, b    k  6  , c  2  2k  1

7
Given that:
1
∵ Sum of zeroes = (product of zeroes)
2
    k  6   1 2  2k  1
   
1 2 1
 k  6  2k  1
 6  1  2k  k
k 7
Therefore, the value of k  7
15. A father’s age is three times the sum of the ages of his two children. After 5 years his age
will be two times the sum of their ages. Find the present age of the father.
OR
1 1
A fraction becomes when 2 is subtracted from the numerator and it becomes when 1 is
3 2
subtracted from the denominator. Find the fraction.
Solution:
Let the sum of ages of two sons be x years
Age of man  3x years
After 5 years age of the man   3x  5 years
Sum of ages of two sons   x  10  years
Given,  3x  5  2  x  10 
  3x  5  2 x  20
 x  15
Hence 3x  3 15  45
Thus the age of the man(father) is 45 years.
OR
x
Let’s assume the fraction be
y
First condition:
x2 1

y 3
 3x  6  y
 3x  y  6............. 1
Second condition:
x 1

y 1 2
 2x  y 1
 2 x  y  1............  2 
Using eliminated method:

8
Multiplying (2) by -1 and then adding (1) and (2)
 3x  y  6
 2 x  y  1
x7
Now,from (1),
 3x  y  6
 37  y  6
 21  y  6
 y  15
7
Hence the required fraction is .
15
16. Find the point on y-axis which is equidistant from the points (5, – 2) and (– 3, 2).
OR
The line segment joining the points A(2, 1) and B(5, – 8) is trisected at the points P and Q such
that P is nearer to A. If P also lies on the line given by 2x – y + k = 0, find the value of k.
Solution:
Since the point is on y-axis so,X-coordinate is zero
Let the point be (0, y)
It’s distance from A(5, –2) and B(– 3, 2) are equal
  0  5   y  2   0  3   y  2 

2 2 2 2

 25  y 2  4 y  4  9  y 2  4 y  4 squaring both sides 


 4 y  29  4 y  13
 4 y  4 y  13  29
16
 8 y  16 y   2
8
Thus, the point is  0, 2  .
OR
As line segment AB is trisected by the points P and Q. Therefore,
Case I: When AP : PB = 1 : 2.
Then, coordinates of P are
1 5  2  2 1  8  1 2 
 , 
 1  2 1 2 
Implies that
P  3, 2 
Since the point P  3, 2  lies on the line
2x  y  k  0
 2  3   2   k  0
 k  8

9
17. Prove that :  sin   cos ec    cos   sec   7  tan 2   cot 2 
2 2

OR
Prove that: 1  cot A – cos ecA1  tan A  sec A  2.
Solution:
L.H.S :  sin   cos ec    cos   sec  
2 2

 1 1 
 sin2  cosec 2  2  cos 2  sec 2  2  sin   cos ec and cos   sec  
 sin 2   cos2   1  cot 2   1  tan 2   4  cos ec2  1  cot 2  and sec2   1  tan 2  
 sin 2   cos 2   1  cot 2   1  tan 2   4  cos ec 2  1  cot 2  and sec2   1  tan 2  
 1  1  1  4  tan 2   cot 2   cos 2   sin 2   1
 7  tan 2   cot 2 
L.H.S=R.H.S
OR
 cos A 1  sin A 1 
L.H.S : 1   1   
 sin A sin A  cos A cos A 
 sin A  cos A  1  cos A  sin A  1 
  
 sin A  cos A 
 sin A  cos A  1
2 2


sin A  cos A
sin A  cos 2 A  2sin A  cos A  1
2

sin A  cos A
1  2sin A  cos A  1

sin A  cos A
2
Hence, L.H.S=R.H.S.

18.In Fig. , PQ is a chord of length 8 cm of a circle of radius 5 cm and centre O. The tangents at
P and Q intersect at point T. Find the length of TP.

10
Solution:

Given radius, OP=OQ=5 cm


Length of chord, PQ=4 cm
OT  PQ,
∴ PM=MQ= 4 cm [Perpendicular draw from the centre of the circle to a chord bisect the chord]
In right OPM,
OP2 =PM2 +OM2
 52 =42 + OM2
 OM2  25 –16  9
Hence OM=3cm
In right PTM ,
PT2 =TM2 +PM2  1
OPT  90º [Radius is perpendicular to tangent at point of contact]
In right OPT ,
OT2 =PT2 +OP2   2 
From equations (1) and (2), we get
OT 2 =  TM 2 +PM 2  +OP 2

  TM + OM    TM 2 + PM 2   OP 2
2

11
 TM 2 +OM 2 +2×TM×OM=TM 2 + PM 2 + OP2
 OM 2 +2×TM×OM=PM 2 +OP 2
 32 2  TM  3  42 52
 9  6TM  16  25
 6TM  32
32 16
 TM  
6 3
Equation (1) becomes,
PT 2  TM 2  PM 2
2
 16 
   42
 3


 256   256  144 
  16 
 9  9
2
 400   20 
  
 9   3 
20
Hence PT 
3
20
Thus, the length of tangent PT is cm.
3

19.In Fig. 3, ACB = 90° and CD  AB, prove that CD2 = BD × AD. [3] A B C D Fig. 3

OR
If P and Q are the points on side CA and CB respectively of  ABC, right angled at C, prove that
(AQ2 + BP2) = (AB2 + PQ2)
Solution:
Given that : CD  AB
ACB  90
To Prove : CD2 =BD×AD
Using Pythagoras Theorem in ACD
AC2 =AD2 +CD2 ...... 1

12
Using Pythagoras Theorem in CDB
CB2 =BD2 +CD2 ......  2 
Similarly in ABC ,
AB2 =AC2 +BC2 .....  3
As AB = AD + DB
Since, AB=AD+BD…..(4)
Squaring both sides of equation (4), we get
 AB =  AD+BD
2 2

Since, AB2 =AD2 +BD2 +2×BD×AD


From equation (3) we get
AC2 +BC2 =AD2 +BD2 +2×BD×AD
Substituting the value of AC2 from equation (1) and the value of BC2 from equation (2), we
get
AD2 +CD2 +BD2 +CD2 =AD2 +BD2 +2×BD×AD
Since, 2 CD2 = 2×BD×AD
Hence, CD2 =BD×AD
OR

Using the Pythagoras theorem in ABC ,


ACQ, BPC, PCQ , we get
AB2 =AC2 +BC2 ..... 1
AQ 2 =AC2 +CQ 2 .....  2 
BP 2 =PC2 +BC2 .....  3
PQ 2 = PC2 +CO 2 .....  4 
Adding the equations (2) and (3) we get
AQ2 +BP2 =AC2 +CQ2 +PC2 +BC2
=  AC2 +BC2  +  CQ2 +PC2 
=AB2 +PQ2
As L.H.S = AQ2 +BP2
=AB2 +PQ2 = R.H.S

13
20. Find the area of the shaded region in Fig. 4, if ABCD is a rectangle with sides 8 cm and 6
cm and O is the centre of circle. (Take  = 3.14)

Solution:
Here, diagonal AC also represents the diameter of the circle.
Using Pythagoras theorem:
AC  AB 2  BC 2
AC  82  62
AC  64  36
AC  100
AC  10
AC
 Radius of the circle, OC= =5cm
2
Area of the shaded region = Area of the circle – Area of rectangle
 πr 2  AB  BC
 π  OC   AB  BC
2

 3.14  52  8  6
 78.5  48
 30.5
Therefore,the area of shaded region is 30.5cm2 .
21. Water in a canal, 6 m wide and 1.5 m deep, is flowing with a speed of 10 km/hour. How
much area will it irrigate in 30 minutes; if 8 cm standing water is needed?
Solution: Width of the canal = 6 m
Depth of the canal = 1.5 m
1
Length of the water column formed in hr
2
= 5 km or 5000 m
1
 Volume of water flowing in hr
2
= Volume of cuboid of length 5000 m, width 6 m and depth 1.5 m.
= 5000 × 6 × 1.5 = 45000 m3
On comparing the volumes,
Volume of water in field = Volume of water coming out from canal in 30 minutes.

14
Irrigated area × standing water = 45000.
45000
Irrigated Area
8
 1 m=100 cm
100
45000 100
 5,62,500m3
8
22. Find the mode of the following frequency distribution.
Class 0-10 10-20 20-30 30-40 40-50 50-60 60-70
Frequency 8 10 10 16 12 6 7

Solution:

Class Frequency
0-10 8
10-20 10
20-30 10  f 0
30-40 16  f1
40-50 12  f 2
50-60 6
60-70 7

Here, 30 - 40 is the modal class, and I = 30, h = 10


 f -f 
 Mode=I+  0 1  ×h
 2f1 -f 0 -f 2 
 16-10 
=30+   ×10
 2 16-10-12 
16
=30+ ×10=30+6=36
10

Section D
7
23. Two water taps together can fill a tank in 1 hours. The tap with longer diameter takes 2
8
hours less than the tap with smaller one to fill the tank separately. Find the time in which
each tap can fill the tank separately.
OR
A boat goes 30 km upstream and 44 km downstream in 10 hours. In 13 hours, it can go 40 km
upstream and 55 km downstream.
Determine the speed of the stream and that of the boat in still water.
Solution:
Let the time in which tap with longer and smaller diameter can fill the tank separately be x
hours and y hours respectively.

15
1 1 8
  ………(i)
x y 15
And x  y  2 ………(ii)
On substituting x  y  2 from (ii) in (i), we get
1 1 8
 
y  2 y 15
y y2 8
 2 
y  2 y 15
 15  2 y – 2   8  y 2 – 2 y 
 30 y – 30  8 y 2 –16 y
 8 y 2 – 46 y  30  0
 4 y 2 – 20 y – 3 y  15  0
  4 y – 3 y – 5   0
3
 y ,y5
4
Substituting values of y in (ii), we get
3
x  2
4
5
x
4
5
x 
4
(time cannot be negative)
Hence, the time taken by tap with longer diameter is 3 hours and the time taken by tap with
smaller diameter is 5 hours, in order to fill the tank separately
OR
Let the speed of boat is x km/h in still water
And stream y km/h
According to question,
30 44
  10
x y x y
And
40 55
  13
x y x y
1 1
Let  u and v
x y x y
30u  44v  10.....(i)
40u  55v  13....(ii)
On solving equation (i) and (ii) we get,

16
1
u  x y 5  (iii)
5
1
v   x  y  11  (iv)
11
On solving equation (iii) and (iv) we get,
x  8 km/h
y  3km/h
24. If the sum of first four terms of an AP is 40 and that of first 14 terms is 280. Find the sum
of its first n terms.
Solution:
Given that: S4  40 and S14  280
n
Sn   2a   n  1 d 
2
4
S4   2a   4  1 d   40
2
 2a  3d  20.........(i)
14
S14   2a  14  1 d   280
2
 2a  13d  40.........(ii)
 ii  -  i  ,
10d  20  d  2
Substituting the value of d in (i),we get
2a  6  20  a  7
Sum of first n terms,
n
Sn   2a   n  1 d 
2
n
 14   n  1 2 
2
 n  7  n  1
 n  n  6
 n 2  6n
Therefore, Sn  n2  6n
sin A  cos A  1
L.H.S 
sin A  cos A  1
tan A  1  sec A
 (Dividing numerator & denominator by cos A )
tan A  1  sec A

 tan A  sec A  1
 tan A  sec A  1

17

 tan A  sec A  1  tan A  sec A
 tan A  sec A  1 tan A  sec A

 tan 2 A  sec2 A   tan A  sec A
tan A  sec A  1 tan A  sec A
1  tan A  sec A

tan A  sec A  1 tan A  sec A
1

tan A  sec A
1

sec A  tan A
LHS=RHS
Hence proved.
26. A man in a boat rowing away from a light house 100 m high takes 2 minutes to change the
angle of elevation of the top of the light house from 60° to 30°. Find the speed of the boat in
metres per minute.  Use 3  1.732
OR
Two poles of equal heights are standing opposite each other on either side of the road, which
is 80 m wide. From a point between them on the road, the angles of elevation of the top of
the poles are 60° and 30° respectively. Find the height of the poles and the distances of the
point from the poles.
Solution:

AB is a lighthouse of height 100m. Let the speed of boat be x metres per minute. And CD is the
distance which man travelled to change the angle of elevation.
Therefore,
CD  2 x  Distance=Speed×Time

18
AB
tan  60  
BC
100
3
BC
100
 BC 
3
AB
tan  30  
BD
1 100
 
3 BD
BD  100 3
CD=BD-BC
100
2 x  100 3 
3
300  100
2x 
3
200
x
2 3
100
x
3
Using,
3  1.73
100
x  57.80
1.73
Hence,the speed of the boat is 57.80 meters per minute.
OR
Let the poles be AB, CD each of height h meter and E is the point between the poles on the
road.
Let AEB = 60°, CED = 30° and DE be x meter.

 BE  80 – x  m
In AEB,

19
AB
tan 60 
BE
h
 3
 20  x 
 h  3 80  x  m..........(i)
In CDE ,
CD
tan 30 
DE
1 h
 
3 x
x
h m........(ii)
3
From equation (i) and (ii), we get
x
 3  80  x 
3
 x  240 – 3 x
 4 x  240
 x  60 m
Put value of x in equation (ii), we get
h  20 3m, DE=60m and BE=20m
Hence, the heights of each pole is 20 3m and distance of the point
from the poles are 60 m and 20 m.

27. Construct a ABC in which CA = 6 cm, AB = 5 cm and BAC = 45º. Then construct a
triangle whose sides are 3 5 of the corresponding sides of ABC.
Solution:

20
Steps of construction:
1. Draw AB = 5 cm. With A as centre, draw BAC  45 . Join BC. ABC is thus formed.
2. Draw AX such that BAX is an acute angle.
3. Cut 5 equal arcs AA1 , A1A2 , A2 A3 , A3A4 and A 4 A5 .
4. Join A 5 to B and draw a line through A 3 parallel to A5 B which meets AB at B'.
3
Here, AB' = AB
5
5.Now draw a line through B' parallel to BC which joins AC at C'.
3 3
Here, B'C' = BC and AC’= AC
5 5
Thus, AB'C' is the required triangle.
28. A bucket open at the top is in the form of a frustum of a cone with a capacity of 12308.8
cm3. The radii of the top and bottom of circular ends of the bucket are 20 cm and 12 cm
respectively. Find the height of the bucket and also the area of the metal sheet used in
making it. (Use  = 3.14)
Solution:

21
Let the height of the bucket be h cm and slant height be l cm.
Here
r1  20 cm
r2  12 cm
And capacity of bucket = 12308.8 cm3
h 2 2
We know that capacity of bucket =
3
 r1  r2  r1r2 
h
=3.14   400  144  240 
3
h
=3.14   784
3
h
So we have =3.14   784  12308.8
3
12308.8  3
h
3.14  784
 15cm
Now, the slant height of the frustum,
l  h 2   r1  r2 
2

 152  82
= 289
=17 cm
Area of metal sheet used in making it
  r22    r1  r2 
2

 3.174  144   20  12  17 


 2160.32 cm 2
29. Prove that in a right angle triangle, the square of the hypotenuse is equal the sum of
squares of the other two sides.
Solution:
Given : A right triangle ABC in which B = 90°

22
To Prove: Hypotenuse  =  Base  +  Perpendicular 
2 2 2

i.e. AC2 =AB2 +BC2


Construction : From B, draw BD  AC

In ABC and ADB


BAC  DAB Common 
ABC  ADB  Each 90
 ABC ~ ADB  By AA similarity
AB AD
 =
AC AB
 AB2 =AD×AC … i 
Similarly, ABC ~ BDC
BC AC
 =
DC BC
 BC2 =AC×DC … ii 
On Adding (i) and (ii), we get
AB 2  BC 2  AD  AC  AC  DC
 AB 2  BC 2  AC  AD  DC 
 AB 2  BC 2  AC  AC
 AC 2  AB 2  BC 2
30. If the median of the following frequency distribution is 32.5. Find the values of f1 and f 2 .
Class 0-10 10-20 20-30 30-40 40-50 50-60 60-70 Total
Frequency f1 5 9 12 f2 3 2 40

OR
The marks obtained by 100 students of a class in an examination are given below.
Marks Number of students
0-5 2
5-10 5
10-15 6
15-20 8
20-25 10
25-30 25

23
30-35 20
35-40 18
40-45 4
45-50 2

Draw ‘a less than’ type cumulative frequency curves (ogive). Hence find median.
Solution:
Class Frequency Cumulative Frequency
0-10 f1 f1
10-20 5 5+f1
20-30 9 14+f1
30-40 12
40-50 26+f1
f2
50-60 26+f1  f 2
3
60-70
2 29+f1  f 2
31+f1  f 2
Total=40=n

f1  5  9  12  f 2  3  2  40
f1 + f 2 =40–31=9 ...  i 
Median=32.5 Given
Since,Median class is 30-40
l  30, h  10,cf  14  f1 ,f  12
n 
 2  cf 
Median  l    h
 f 
 
 40 
 2  14  f1  
32.5  30   10
 12 
 

10
2.5 
12  20  14  f1 
3  6  f1
f1  3
On putting in (i),
f1 + f 2 =9
f 2 =9–3  f1  3
6

24
OR
Marks Number of students Marks less than Cumulative frequency
0-5 2 Less than 5 2
5-10 5 Less than 10 7
10-15 6 Less than 15 13
15-20 2 Less than 20 21
20-25 10 Less than 25 31
25-30 25 Less than 30 56
30-35 20 Less than 35 76
35-40 18 Less than 40 94
40-45 4 Less than 45 98
45-50 2 Less than 50 100

Let us now plot the points corresponding to the ordered pairs (5, 2), (10, 7), (15, 13), (20, 21),
(25, 31), (30, 56), (35, 76), (40, 94), (45, 98), (50, 100). Join all the points by a smooth curve.

n 100
Locate =  50 on Y-axis
2 2
From this point draw a line parallel to X-axis cutting the curve at a point. From this point, draw
a perpendicular to X-axis. The point of intersection of perpendicular with the X-axis determines
the median of the data. Therefore median = 28.8

25
26
CBSE Class 10
Mathematics
Previous Year Question Paper 2020
Series: JBB/5 Code no. 30/5/3

• Please check that this question paper contains 23 printed pages.


• Code number given on the right hand side of the question paper should be
written on the title page of the answer-book by the candidate.
• Please check that this question paper contains 40 questions.
• Please write down the Serial Number of the question in the answer-
book before attempting it.
• 15 minutes of time has been allotted to read this question paper. The
question paper will be distributed at 10.15 a.m. From 10.15 a.m. to 10.30
a.m., the students will read the question paper only and will not write any
answer on the answer script during this period.

MATHEMATICS (Standard) - Theory

Time Allowed: 3 hours Maximum Marks: 80


General Instructions:
Read the following instructions very carefully and strictly follow them :
1. Please check that this question paper contains 23 printed pages.
2. The question paper comprises of four sections – A, B, C and D. This
question paper carries 40 questions. All questions are compulsory.
3. Section A: Question numbers 1 to 20 comprises of 20 questions of one
mark each.
4. Section B: Question numbers 21 to 26 comprises of 6 questions of two
mark each.

Class X Mathematics www.vedantu.com 1


5. Section C: Question numbers 27 to 34 comprises of 8 questions of three
mark each.
6. Section D: Question numbers 35 to 40 comprises of 6 questions of four
mark each.
7. There is no overall choice in the paper. However, internal choice is
provided in 2 questions of one mark, 2 questions of one mark, 2 questions
of two marks, 3 questions of three marks and three questions of four marks.
You have to attempt only one of the choices in such questions.
8. In addition to this, separate instructions are given with each section and
question, wherever necessary.
9. Use of calculators is not permitted.

SECTION-A
1. The value(s) of k for which the quadratic equation 2x 2 +kx+2=0 has equal
roots, is 1 Mark
(A) 4
(B) ±4
(C) -4
(D) 0
Ans: Given quadratic equation is 2x 2 +kx+2=0 .
For equal roots, D =0
⇒ b 2 -4ac=0
Here, a=2, b=k, and c=2
⇒ k 2 -4×2×2=0
⇒ k 2 -16 =
0
k 2 = 16
∴ k =±4
Therefore, the values of k is ±4 .
Hence, option (B) is correct.

Class X Mathematics www.vedantu.com 2


2. Which of the following is not an A.P.? 1 Mark
(A) -1.2,0.8, 2.8,…
(B) 3. 3+ 2 . 3+2 2 . 3+3 2
4 7 9 12
(C) , , , ,......
3 3 3 3
-1 -2 -3
(D) , , ,......
5 5 5
Ans: (A) We have,
⇒ a 2 -a1 =a 3 -a 2 =2
Hence, it is an A.P.
(B) We have,
⇒ a 2 -a1 =a 3 -a 2 = 2
Hence, it is an A.P.
2
(C) We have, a 2 -a1 =1 and a 3 -a 2 = .
3
So, a 2 -a1 ≠ a 3 -a 2 .
Hence, it is not an A.P.
(D) We have,
−1
a 2 -a1 =a 3 -a 2 =
5
Hence, it is an A.P.
Therefore, option (C) is correct.

3. The radius of a sphere (in cm) whose volume is 12πcm 3 , is 1 Mark


(A) 3
(B) 3 3
2

(C) 3 3

Class X Mathematics www.vedantu.com 3


1

(D) 3 3

4
Ans: Volume of a sphere = πr 3
3
Given, Volume of the sphere =12πr cm3
On equating we get,
4
⇒ πr 3 =12πr cm3
3
12π×3 3
⇒ r3 = cm

⇒ r 3 =9 cm3

⇒ r 3 = ( 3) cm3
2

2
⇒ r= ( 3) 3 cm
2
Therefore, radius of the sphere is ( 3) 3 cm .

Hence, option (C) is correct.

4. The distance between the points (m, -n) and (-m, n) is 1 Mark
(A) m 2 +n 2
(B) m+n

(C) 2 m 2 +n 2

(D) 2m 2 +2n 2
Ans: Let the points be A(m, -n) and B(-m, n).
From distance formula we get,

( -m-m ) + ( n- ( -n ) )
2 2
⇒ AB=

( −2m ) + ( 2n )
2 2
⇒ AB=

⇒ AB= 4m 2 +4n 2

Class X Mathematics www.vedantu.com 4


⇒ AB=2 m 2 +n 2

Therefore, the distance between the points (m, -n) and (-m, n) is 2 m 2 +n 2 .
Hence, option (C) is correct.

5. In Figuire-1, from an external point P, two tangents PQ and PR are


drawn to a circle of radius 4 cm with centre O. If ∠QPR=90o , then length
of PQ is 1 Mark
(A) 3 cm
(B) 4 cm
(C) 2 cm
(D) 2 2 cm

Ans: Given, ∠QPR=90o


Since, the line from the centre of the circle bisects the angle between the
tangents.
Therefore, ∠OPQ=45o .
In ΔPOQ , we have
OQ
⇒ tan45o =
PQ

Class X Mathematics www.vedantu.com 5


OQ
⇒ 1=
PQ
⇒ PQ=OQ
⇒ PQ=4cm
Therefore, PQ is 4cm.
Hence, option (B) is correct.

6. On dividing a polynomial p(x) by x 2 -4 , quotient and remainder are found


to be x and 3 respectively. The polynomial p(x) is 1 Mark
(A) 3x 2 +x-12
(B) x 3 -4x+3
(C) x 2 +3x-4
(D) x 3 -4x-3
Ans: As dividend = (divisor × quotient)+remainder
⇒ p ( x ) ( x 2 -4 )×x  +3

⇒ p ( x ) =x 2 -4x+3

Therefore, polynomial p(x) is x 2 -4x+3 .

AD 3
7. In Figure-2, DE || BC. If = and AE = 2.7 cm, then EC is equal to
DB 2
1 Mark
(A) 2.0 cm
(B) 1.8 cm
(C) 4.0 cm
(D) 2.7 cm

Class X Mathematics www.vedantu.com 6


Ans: Given, DE || BC.
AD AE
Therefore, =
DB EC
3 2.7 cm
⇒ =
2 EC
2×2.7cm
⇒ EC=
3
⇒ EC = 1.8 cm
Therefore, EC is equal to .
Hence, option (B) is correct.

8. The point on the x-axis which is equidistant from (-4, 0) and (10, 0) is
1 Mark
(A) (7, 0)
(B) (5, 0)
(C) (0, 0)
(D) (3, 0)
Ans: Let the point on the x-axis is P(x, 0) which is equidistant from A(-4, 0)
and B(10, 0).

We have, AP = PB
Using distance formula, we get

Class X Mathematics www.vedantu.com 7


( x- ( -4 ) ) + ( 0-0 ) = (10-x ) + ( 0-0 )
2 2 2 2

Squaring both the sides,


⇒ ( x+4 ) = (10-x )
2 2

⇒ x 2 +8x+16=100-20x+x 2
⇒ 8x+20x=100-16
⇒ 28x=84
⇒ x=3
Therefore, (3, 0) is equidistant from (-4, 0) and (10, 0).
Hence, option (D) is correct.
Or
The centre of a circle whose end points of a diameter are (-6, 3) and (6, 4)
is 1 Mark
(A) (8, -1)
(B) (4, 7)
 7
(C)  0, 
 2
 7
(D)  4, 
 2
Ans: Let centre be O(x, y) and end points of the diameter be A(-6, 3) and B(6,
4).

Since, centre is the midpoint of diameter. So,


-6+6 3+4
⇒ x= and y=
2 2

Class X Mathematics www.vedantu.com 8


7
⇒ x=0 and y=
2
 7
Therefore, centre of the circle is  0,  .
 2
Hence, option (C) is correct.

9. The pair of linear equations 1 Mark


3x 5y
+ =7 and 9x+10y=14
2 3
(A) consistent
(B) inconsistent
(C) consistent with one solution
(D) consistent with many solutions
3x 5y
Ans: Given + =7 and 9x+10y=14
2 3
3 5
Here, a1 = , b1 = , c1 =7, a1 =9, b1 =10 and c1 =14 .
2 3
3 5
a b c 7
⇒ 1 = 2 , 1 = 3 and 1 =
a 2 9 b 2 10 c 2 14
a1 1 b1 1 c 1
⇒ = , = and 1 =
a 2 9 b2 6 c2 2
a1 b1 c1
∴ = ¹
a 2 b1 c1
Therefore, these lines equation intersect each other at one point and only have
one possible solution.
Hence, the pair of linear equation is inconsistent.
Hence, option (B) is correct.

10. In Figure-3, PQ is tangent to the circle with centre at O, at the point B.

Class X Mathematics www.vedantu.com 9


If ∠AOB=100o , then ∠ABP is equal to 1 Mark
(A) 50o
(B) 40o
(C) 60o
(D) 80o

Ans: In ΔAOB, AO=OB


∴∠OAB=∠OBA=40o
Since PQ is tangent at the point B, ∠OBP=90o
⇒ ∠OBP = ∠OBA + ∠ABP
⇒ 90=
o
40o + ∠ABP
⇒ ∠ABP=90o -40o
⇒ ABP=50o
Therefore, ∠ABP is equal to 50o .
Hence, option (A) is correct.

Fill in the blanks in question numbers 11 to 15.


1+tan 2 A
11. Simplest form of is _________. 1 Mark
1+cot 2 A
1+tan 2 A
Ans: can be simplified as
1+cot 2 A
1+tan 2 A sec 2 A
=
1+cot 2 A cosec 2 A

Class X Mathematics www.vedantu.com 10


1 sin 2 A
⇒ ×
cos 2 A 1
⇒ tan 2 A

12. If the probability of an event E happening is 0.023, then P E = ( )


_________. 1 Mark
Ans: Given, P ( E ) =0.023 .

( )
As, P ( E ) +P E =1

⇒ 0.023+P ( E ) = 1

⇒ P ( E ) =1-0.023

⇒ P ( E ) =0.977

Therefore. P ( E ) =0.977

13. All concentric circles _________ to each other. 1 Mark


Ans: All concentric circles are similar to each other.
14. The probability of an event that is sure to happen, is _________.
1 Mark
Ans: The probability of an event that is sure to happen is .

15. AOBC is a rectangle whose three vertices are A(0, -3), O(0. 0) and B(4,
0). The length of its diagonal is _________. 1 Mark
Ans:

Class X Mathematics www.vedantu.com 11


( 4-0 ) + ( 0- ( -3) )
2 2
Length of diagonal AB =

( 25) + ( 3)
2 2
=

= 25
=5
Therefore, length of diagonal is 5 units.

Answer the following question numbers 16 to 20.


16. Write the value of sin 2 30o +cos 2 30o . 1 Mark
Ans: As, sin 2θ+cos 2θ=1
∴ sin 2 30o +cos 2 30o =
1

17. Form a quadratic polynomial, the sum and product of whose zeroes are
(-3) and 2 respectively. 1 Mark
Ans: Given, sum of zeroes =-3 and product of zeroes =2
The quadratic equation is given by
x2-(sum of zeroes)x+(product of zeroes) = 0
x2-(-3)x+(2)=0
x2+3x+2=0
Or
Can (x2-1) be a remainder while dividing x 4 -3x+5x-9 by ( x 3 +3 ) ?

Justify your answer with reasons. 1 Mark


Ans: On dividing x 4 -3x+5x-9 by ( x 3 +3) we get (5x+9) as a remainder.

x 2 -6
x 2 +3 x 4 -3x 2 +5x-9
x 4 +3x 2
-6x 2 +5x-9
-6x 2 -18
5x+9

Class X Mathematics www.vedantu.com 12


Therefore, ( x 2 -1) can’t be a remainder while dividing x 4 -3x 2 +5x-9 by ( x 2 +3)
.
18. Find the sum of the first 100 natural numbers. 1 Mark
Ans: Let, the sum be s100 =1+2+3+...+100
Here, a=1, d=1, n=100 and l =100
n
As, s n = ( a+l )
2
100
⇒ s100 = (1+l00 )
2
⇒ s100 =5050
Therefore, the sum of the first 100 natural numbers is 5050.

19. The LCM of two numbers is 182 and their HCF is 13. If one of the
numbers is 26, find the other. 1 Mark
Ans: Let the other number be x.
Product of number =LCM×HCF
⇒ 26×x=182×13
⇒ x=91
Therefore, other number is 91.

20. In Figure-4, the angle of elevation of the top of a tower from a point C
on the ground, which is 30 m away from the foot of the tower, is 30o. Find
the height of the tower. 1 Mark

Class X Mathematics www.vedantu.com 13


Ans: Let the tower be AB.
Since the tower is vertical, therefore ∠ABC=90o .
AB
In ΔABC, tab30o =
BC
1 AB
⇒ =
3 30

⇒ AB=10 3
Therefore, height of the tower is 10 3 .

SECTION-B
Question numbers 21 to 26 carry 2 marks each.
21. A cone and a cylinder have the same radii but the height of the cone is
3 times that of the cylinder. Find the ratio of their volumes. 2 Marks
Ans: Let radius of cone = radius of cylinder =r and height of cylinder be h,
then height of cone will become 3h.
1
Volume of a cone = πr 2 H
3
1
⇒ Volume of given cone = πr 2 ( 3h )
3
= πr 2 h
Volume of a cylinder = πr 2 H
Volume of given cylinder = πr 2 h
Volume of the cone πr 2 h
Therefore, =
Volume of the cylinder πr 2 h
Volume of the cone 1
⇒ =
Volume of the cylinder 1
Therefore, ratio of volume of cone to volume of cylinder is 1:1.

Class X Mathematics www.vedantu.com 14


22. In Figure-5, a quadrilateral ABCD is drawn to circumscribe a circle.
Prove that AB+CD=BC+AD . 2 Marks

Ans: As we know, length of tangents drawn from an external point are equal.
Therefore, we can write
⇒ AP=AS ---(1)
⇒ BP=BQ ---(2)
⇒ CR=CQ ---(3)
⇒ BR=DS ---(4)
On adding equation (1), (2), (3) and (4), we get
⇒ AP+BP+CR+DR=AS+BQ+CQ+DS
⇒ (AP+BP)+(CR+DR)=(AS+DS)+(BQ+CQ)
⇒ AB+CD=AD+BC
Hence, proved that AB+CD=AD+BC
Or
In Figure-6, find the perimeter of ΔABC , if AP=12 cm . 2 Marks

Class X Mathematics www.vedantu.com 15


Ans: As we know, tangents drawn from an external point are equal.
Therefore, BD=BP , CD=CQ and AP=Q.
Perimeter of Δ ABC = AB+BC+CA
=AB+BD+CD+AC
=AP+ AQ
=2Ap
=2×12
=24
Therefore, perimeter of Δ ABC is 24 cm.

23.Find the mode of the following distribution:


Marks: 0-12 10-20 20-30 30-40 40-50 50-60
Number of
4 6 7 12 5 6
students
2 Marks
Ans: Here, modal class is 30-40.
f1 -f 0
Mode=l+ ×h
2f1 -f 0 -f 2
12 − 7
⇒ Mode=30+ × 10
2 × (12 ) − 7 − 5

⇒ Mode=34.16
Therefore, mode is 34. 16.

QB DR
24. In Figure-7, if PQ || BC and PR || CD, prove that = . 2 Marks
AQ AR

Class X Mathematics www.vedantu.com 16


Ans: Since, PQ || BC in ΔABC
By basic proportionality theorem, we get
AQ AP
⇒ = --- (1)
AB AC
In ΔACD , PR || CD
By basic proportionality theorem, we get
AP AR
⇒ = --- ( 2 )
AC AD
From and , we get
AB AD
⇒ =
AQ AR
AQ+QB AR+RD
⇒ =
AQ AR
QB RD
⇒ 1+ =1+
AQ AR
QB RD
⇒ =
AQ AR
QB RD
Hence, proved that = .
AQ AR

25. Show that 5+2 7 is an irrational number, where 7 is given to be an


irrational number. 2 Marks
Ans: Let 5+2 7 is a rational number.
So, we can write
P
⇒ 5+2 7 =
q
P 5
⇒ 7= =
2q 2

Class X Mathematics www.vedantu.com 17


P 5
p, q, 5 and 2 are integers. So, - is a rational number.
2q 2

Therefore, 7 is also a rational number.


But 7 is given to be an irrational number.
This is a contradiction which raised due to our assumption that 5 + 2 7 is a
rational number.
Therefore, 5 + 2 7 is an irrational number.
Or
Check whether 12n can end with the digit 0 for any natural number n.
2 Marks
Ans: We can write, 12n = ( 2n ×3) .
n

If a number ends with then it is divisible by 5. But, prime factorisation of 12n


does not contains 5.
Therefore, 12n can’t end with the digit 0 for any natural number n.

26. If A, B and C are interior angles of a ΔABC , then show that


 B+C  A
cos   =sin   . 2 Marks
 2  2
 
Ans: In ∆ABC, ∠Α + ∠B + ∠C =180o or A+B+C=180o.
B+C=180o-A
 B+C 
L.H.S. =cos  
 2 
 180o − A 
=cos  
 2 
 A
=cos  90o − 
 2

A
=sin  
2

Class X Mathematics www.vedantu.com 18


 B+C  A
Therefore, cos   =sin   .
 2  2

SECTION-C
Question number 27 to 34 carry 3 marks each.
27. Prove that: ( sin 4θ-cos 4θ+1) cosec 2θ=2 3 Marks

Ans: On simplification we get,

( sin θ-cos θ+1) cosec θ= ( ( sin θ ) - ( cos θ ) )


2 2
4 4 2 2 2
+1 cosec 2θ

( sin θ-cos θ+1) cosec θ= ( sin θ+cos θ )( sin θ+cos θ ) +1 cosec θ
4 4 2 2 2 2 2 2

As sin 2θ+cos 2θ=1 , we get


⇒ ( sin 4θ-cos 4θ+1) cosec 2θ= ( sin 2θ-cos 2θ ) +1 cosec 2θ

⇒ ( sin 4θ-cos 4θ+1) cosec 2θ= 1-cos 2θ+sin 2θ  cosec 2θ

⇒ ( sin 4θ-cos 4θ+1) cosec 2 = sin 2θ+sin 2θ  cosec 2θ

⇒ ( sin 4θ-cos 4θ+1) cosec 2θ=2sin 2θcosec 2θ

1
⇒ ( sin 4θ-cos 4θ+1) cosec 2θ=2sin 2θ×
sin 2θ
⇒ ( sin 4θ-cos 4θ+1) cosec 2θ=2

Hence, proved that ( sin 4θ-cos 4θ+1) cosec 2θ=2 .

28. Find the sum: (-5)+(-8)+(-11)+…+(-230) 3 Marks


Ans: a1=-5, a2=-8, a3=-11
⇒ a 2 -a1 =a 3 -a 2 =-3
It is an A.P., in which first term is -5, common difference is -8 and last term is -
230.
∴ a1 =-5d, d=-3 and l=-230

Class X Mathematics www.vedantu.com 19


As l=a1 + ( n-1) d where n is number of terms in the A.P.

From this we get,


⇒ -230=-5+(n-1)(-3)
⇒ -230+5-3=3n+3
⇒ -230+3-3=-3n
⇒ 3n=228
⇒ n=76
n
As we know, sum of the series ( s n ) = ( a+l )
2
76
⇒ (sn ) = ( -5-230 )
2
76
⇒ (sn ) = × ( −235 )
2
⇒ ( s n ) = − 8930

Therefore, the sum of he given series is -8930.

29. Construct a ∆ ABC with sides BC=6 cm, AB=5 cm and ∠ABC=60o .
3
Then construct a triangle whose sides are of the corresponding sides of
4
∆ ABC . 3 Marks
Ans: Construction of ∆ ABC:
(1) Draw a line segment AB of length 5 cm.
(2) Draw a line segment BC by making an angle of 60o from point B.
(3) Join A and C to get the required ∆ ABC.
3
Now, construction of ΔA'BC' whose sides are of the corresponding sides of
4
∆ ABC are as follows:
(1) Draw a ∆ ABC with sides BC= 6cm , AB = 5 cm and ∠ABC=60o .
(2) On opposite side of vertex A, draw a ray BX making an acute angle with

Class X Mathematics www.vedantu.com 20


BC.
(3) On the line segment BX, locate four points B1,B2,B3 and B4.
(4) Join B4 and C. Draw a line through B3 parallel to B4C intersecting BC at
C
(5) Draw a line parallel to AC through C intersecting AB at A.
Hence, we obtained the required ΔA'BC' .

Or
Draw a circle of radius 3.5 cm . Take a point P outside the circle at a
distance of 7 cm from the centre of the circle and construct a pair of
tangents to the circle from that point. 3 Marks
Ans: Construction:
(1) Draw a line OP=6 cm.
(2) Draw a circle of radius 3.5 cm with O as centre.
(3) Draw a perpendicular bisector of OP that cuts OP at M.
(4) With M as a centre and MP (or OP) as radius, draw a circle which
intersects the first circle at A and B.
(5) Join PA and PB.
PA and PB are the required tangents.

Class X Mathematics www.vedantu.com 21


30. In Figure-8, ABCD is a parallelogram. A semicircle with a centre O
and the diameter AB has been drawn and it passes through D. If AB=12
cm and OD ⊥ AB , then find the area of the shaded region. (Use π =3.14)
3 Marks

Ans: Given, Diameter = AB = 12 cm


AO=OB=OD=6 cm
1
×π× ( 6 )
2
⇒ area of OBD =
4
=28.26cm 2
Area of parallelogram ABCD = base× height
= 12×6
=72 cm2
Area of shaded region = area of ABCD – area of OBD
=(72-28.26) cm2
= 43.74 cm2

Class X Mathematics www.vedantu.com 22


Therefore, area of shaded region is 43.74 cm2.

31. Read the following passage and answer the questions given at the end:
Diwali Fair A game in a booth at a Diwali Fair involves using a spinner
first. Then, if the spinner stops on an even number, the player is allowed
to pick a marble from a bag. The spinner and the marbles in the bag are
represented in Figure-9.Prizes are given, when a black marble is picked.
Shweta plays the game once.

(i) What is the probability that she will be allowed to pick a marble from
the bag?
(ii) Suppose she is allowed to pick a marble from the bag, what is the
probability of getting a prize, when it is given that the bag contains 20
balls out of which 6 are black? 3 Marks
Ans: The player is allowed to pick a marble from a bag if the spinner stops on
an even number.
So, the favourable outcomes are 2, 4, 6, 8 and 10.
Number of favourable outcomes =5
Total number of outcomes =6
(i) The probability that she will be allowed to pick a marble from the bag
Number of favourable outcomes
=
Total number of outcomes
5
=
6
(ii) The bag contains 20 balls out of which 6 are black and prizes are given,
when a black marble is picked.
Number of favourable outcomes =6

Class X Mathematics www.vedantu.com 23


Total number of outcomes =20
Number of favourable outcomes
The probability of getting a prize =
Total number of outcomes
6
=
20
3
=
10

1
32. A fraction becomes when 1 is subtracted from the numerator and it
3
1
becomes when 8 is added to its denominator. Find the fraction.
4
3 Marks
x
Ans: Let the numerator be x and denominator be y. So, fraction is .
y
1
Given, when is subtracted from the numerator the fraction becomes i.e.,
3
x-1 1
=
y 3
⇒ 3(x-1)y
⇒ 3x-3=y
⇒ 3x-y=3 ---(1)
1
Also given, the fraction becomes when 8 is added to its denominator i.e.,
4
x 1
=
y+8 4
⇒ 4x=y+8
⇒ y=4x-8--- (1)

Putting (2) in (1), we get


3x+4x+8=3
-x=-5

Class X Mathematics www.vedantu.com 24


x=5
Putting the value of x in (1), we get
y=4×5-8
y=4×5-8
y=12
x 5
Therefore, the fraction is i.e., .
y 12
Or
The present age of a father is three years more than three times the age of
his son. Three years hence the father’s age will be 10 years more than twice
the age of the son. Determine their present ages. 3 Marks
Ans: Let the son’s age be x.
Given, age of the father is three years more than three times the age of his son.
Therefore, father’s age is 3x+3.
After three years,
Age of the son =x+3
Age of the father =3x+3+3 ---(1)
But, according to question, after three years the father’s age will be 10 years
more than twice the age of the son.
Age of the father 10+2(x+3) ---(2)
From (1) and (2), we get
⇒ 3x+6=10+2(x+3)
⇒ 3x+6=10+2x+6
⇒ x=10
So, the present age of son is 10 years.
Present age of father =3×10+3
=33 years
Therefore, the present age of son is 10 years and father is 33 years.

Class X Mathematics www.vedantu.com 25


33. Find the ratio in which the y-axis divides the line segment joining the
points (6, -4) and (-2, 7). Also find the point of intersection. 3 Marks
Ans: Let the y-axis divides the line segment joining the points (6, -4) and (-2, -
7) in the ratio k:1 and the point be (0, y).
From section formula we know that if a point (x, y) divides the line joining the
 mx 2 +nx1 my 2 +ny1 
points (x1, y1) and (x2, y2) in the ratio m : n, then ( x,y ) =  , 
 m+n m+n 
Using this, we get
k× ( -2 ) +1×6
⇒ 0=
k+1
⇒ -2k+6=0
⇒ 2k=6
⇒ k=3
Therefore, the y-axis divides the line segment joining the points (6, -4) and (-2,
-7) in 3:1.
Now,
 3× ( -7 ) +1× ( -4 ) 
Coordinate of point of intersection =  0, 
 3+1 
 -25 
=  0, 
 4 
 -25 
Therefore, point of intersection is  0, .
 4 
Or
Show that the points (7, 10), (-2, 5) and (3, -4) are vertices of an isosceles
right triangle. 3 Marks
Ans: Let A(7, 10), B(-2, 5) and C(3, -4).
Using distance formula,

( -2-7 ) + ( 5-10 )
2 2
⇒ AB=

( -9 ) + ( -5)
2 2
=

Class X Mathematics www.vedantu.com 26


= 106

( 3+2 ) + ( -4-5)
2 2
⇒ BC=

( 5) + ( -9 )
2 2
=

= 106

( 3-7 ) + ( -4-10 )
2 2
⇒ AC=

( −4 ) + ( -14 )
2 2
=

= 212
We can see that AB = BC.
Also,
⇒ AB2 +BC2 =106+106
= 212
= AC2
Therefore, by Pythagoras theorem Δ ABC is a right-angled triangle.
Hence, the points (7, 10) (-2, 5) and (3, -4) are vertices of an isosceles right
triangle.

34. Use Euclid Division Lemma to show that the square of any positive
integer is either of the form 3q or 3q+1 for some integer q. 3 Marks
Ans: We know from Euclid Division Lemma that if a and b are two positive
integers then a=bm+r where 0 ≤ 0r<b.
Now, let the positive integer be a and b=3.
r is an integer greater than or equal to zero and less than 3. Therefore, r can be
either 0,1 or 2.
For r=0, the equation becomes
⇒ a=3m+0
⇒ a=3m
Squaring both the sides,

Class X Mathematics www.vedantu.com 27


⇒ a 2 = ( 3m )
2

⇒ a 2 =3 ( 3m )
2

Let 3m 2 =q .

⇒ a 2 =3q
For r=1, the equation becomes
⇒ a=3m+1
Squaring both the sides,
⇒ a 2 = ( 3m+1)
2

⇒ a 2 =9m 2 +6m+1
⇒ a 2 =3 ( 3m 2 +2m ) +1

Let q=3m 2 +2m .

⇒ a 2 =3q+1
For r=2, the equation becomes
⇒ a=3m+2
Squaring both the sides,
⇒ a 2 = ( 3m+2 )
2

⇒ a 2 =9m 2 +12m+4
⇒ a 2 =9m 2 +12m+3+1
⇒ a 2 =3 ( 3m 2 +4m+1) +1

Let q=3m 2 +4m+1 .

⇒ a 2 =3q+1
Hence proved that square of any positive integer is either of the form 3q or 3q+1
for some integer q.

SECTION-D
Question numbers 35 to 40 carry 4 marks each.

Class X Mathematics www.vedantu.com 28


35. Sum of the areas of two squares is 544m2. If the difference of their
perimeter is 32m, find the sides of the two squares. 4 Marks
Ans: Let the sides of two squares be x and y where x>y.
Perimeter of first square =4x
Perimeter of second square =4y
Given, the difference of their perimeter is 32m.
⇒ 4x-4y=32
⇒ x-y=8
⇒ x=5+y
Area of first square =x2
Area of second square =y2
Given, sum of the areas of two squares is 544 m2.
⇒ x 2 +y 2 =544
Putting x=8+y, we get
⇒ ( y+8 ) +y 2 =544
2

⇒ y 2 +16y+64+y 2 =544
⇒ 2y 2 +16y-480=0
⇒ y 2 +8y-240=0
⇒ y 2 +20y-12y-240-0
⇒ y ( y+20 ) -12 ( y+20 ) =0

⇒ ( y+20 )( y-12 ) =0

⇒ ( y=-20 ) or ( y=12 )

Length cannot be negative. So, y ≠ −20 .


∴ y=12m
As, x=8+y
x=8+12
x=20

Class X Mathematics www.vedantu.com 29


∴x=20
Hence, the sides of two squares are 20m and 12m.
Or
A motorboat whose speed is 18 km/h in still water takes 1 hour more to go
24 km upstream than to return downstream to the same spot. Find the
speed of the stream. 4 Marks
Ans: Let the speed of the stream be x km/h.
Given, speed of boat in still water is 18 km/h .
Speed of boat in downstream = speed of boat in still water + speed of the stream
=(18+x) km/h
Speed of boat in upstream = speed of boat in still water - speed of the stream
=(18-x) km/h
As given that motorboat takes 1 hour more to go upstream than to return
downstream to the same spot.
So, we can write
Time taken for upstream = time taken for downstream +1
Distance
As Time= , we can write
Speed
Distance covered in upstream Distance covered in downstream
⇒ = +1
Speed of upstream Speed of downstream
24 24
⇒ = +1
18-x 18+x
⇒ 24 (18+x ) =24 (18-x ) +18 (18-x )(18+x )

On simplifying we get,
⇒ ( 24×18 ) +24x= ( 24×18 ) -24x+ ( (18×18 ) -s 2 )

⇒ x 2 +48x-324=0
⇒ x 2 +54x-6x-324=0
⇒ x ( x+54 ) -6 ( x+54 ) =0

⇒ ( x+54 )( x-6 ) =0

Class X Mathematics www.vedantu.com 30


⇒ ( x=-54 ) or ( x=6 )

Speed of stream cannot be negative. So, x ≠ −54 .


Therefore, speed of stream is 6 km/h.

36. For the following data, draw a ‘less than’ ogive and hence find the
median of the distribution.
Age
(in 0-10 10-20 20-30 30-40 40-50 50-60 60-70
years)
Number
of 5 15 20 25 15 11 9
persons:
4 Marks
Ans: Plotting age (in years) on the x-axis and cumulative frequency on y-axis.

Age Numbers of persons


Less than 10 5
Less than 20 20
Less than 30 40
Less than 40 65
Less than 50 80
Less than 60 91
Less than 70 100

Here, we have N=100


N
⇒ =50
2

Class X Mathematics www.vedantu.com 31


From the curve we get x-ordinate as 33.5 when ordinate is 50.
Therefore, the median of the given distribution is 33.5.
Or
The distribution given below shows the number of wickets taken by
bowlers in one-day cricket matches. Find the mean and the median of the
number of wickets taken.
Number
of 20-60 60-100 100-140 140-180 180-220 220-260
wickets:
Number
of 7 5 16 12 2 3
bowlers:
4 Marks
Ans: Assuming a=120 and h=40.
Number
Number
of
of
xi ui =
( x -a )
i
fiu i cf
bowlers h
wickets:
(f)
20-60 7 40 -2 -14 7
60-100 5 80 -1 -5 12
100-140 16 120 0 0 28
140-180 12 160 1 12 40
180-220 2 200 2 4 42
220-260 3 240 3 9 45

We get, ∑ f =40
i
and ∑ f u =6 .
i i

Mean = a+ ∑
fi u i
×h
∑ fi
6 × 40
= 120 +
45
=125.33
We have, N=45
N
⇒ =22.5
2

Class X Mathematics www.vedantu.com 32


Therefore, Median class = 100-140, Cumulative frequency = 28, i = 100, cf=12,
f=16 and h=40.
N 
 2 -cf 
Median=l+  ×h
 f 
 
 22.5 − 12 
⇒ Median=l00+  ×40
 16 
= 126.25
Therefore, the mean is 125.33 and the median is 126.25 of the number of wickets
taken.

37. A statue 1.6 m tall, stand on the top of a pedestal. From a point on the
ground, the angle of elevation of the top of the statue is 60o and from the
same point the angle of elevation of the top of the pedestal is 45o. Find the
height of the pedestal. (Use 3=1.73 ) 4 Marks
Ans: Let the height of pedestal be h metres.

In ∆ ABD, we have
BD
⇒ tan45o =
AB
h
⇒1=
AB
⇒ AB=h--- (1)

Class X Mathematics www.vedantu.com 33


In ∆ ABC, we have
BC
⇒ tan60o =
AB
BD+DC
⇒ 3=
AB
h+1.6
⇒ 3=
AB
h+1.6
⇒ AB= --- ( 2 )
3
From (1) and (2), we get
h+1.6
⇒ h=
3

⇒ 3h=h+1.6

⇒ ( )
3-1 h=1.6

1.6
⇒ h=
( 3-1 )
1.6
⇒ h=
(1.73-1)
⇒ h=2.19
Therefore, the height of the pedestal is 2.19 m.

38. Obtain other zeroes of the polynomial


p ( x ) =2x 4 -x 3 -11x 2 +5x+5 if two zeroes are 5 and - 5 . 4 Marks

Ans: Two zeroes are 5 and - 5 .


So, we can write, x= 5 and x=- 5 .
We get, x- 5 and x+ 5 =0.
Multiplying both the factors we get,

Class X Mathematics www.vedantu.com 34


( )(
⇒ x- 5 x+ 5 =0 )
⇒ x 2 -5=0
x 2 -5 is a factor of p ( x ) =2x 4 -x 3 -11x 2 +5x+5 .

Dividing 2x 4 -x 3 -11x 2 +5x+5 by x 2 -5 , we get the quotient as 2x 2 -x-1 .


On factorising 2x 2 -x-1 , we get
⇒ 2x 2 -2x+x-1=0
⇒ 2x ( x-1) +1( x-1) =0

⇒ ( x-1)( 2x+1) =0

1
⇒ x=1, x=-
2
1
Therefore, other two zeroes of the polynomial are 1 and - .
2
Or
What minimum must be added to 2x 3 -3x 2 +6x+7 so that the resulting
polynomial will be divisible by x 2 -4x+8 ? 4 Marks
Ans:
2x+5
x -4x+8 2x -3x +6x+7
2 3 3

2x 3 -8x+16x
5x 2 -10x+7
5x 2 -20x+40
10x-33
For 2x 3 -3x 3 +6x+7 to be divisible by x 2 -4x+8 , remainder should be zero when
we divide 2x 3 -3x 3 +6x+7 by x 2 -4x+8 .
Dividing 2x 3 -3x 3 +6x+7 by x 2 -4x+8 , we get the remainder as 10x-33.
Therefore, we have to add –(10x-33) i.e., 33-10x so that the resulting polynomial
will be divisible by x2-4x+8.

39. In a cylindrical vessel of radius 10 cm, containing some water, 9000

Class X Mathematics www.vedantu.com 35


small spherical balls are dropped which are completely immersed in water
which raises the water level. If each spherical ball is of radius 0.5 cm, then
find the rise in the level of water in the vessel. 4 Marks
Ans: Given, Radius of spherical balls =0.5 cm
Radius of cylindrical vessel =10 cm
4
As we know, Volume of a sphere = ×π× ( radius )
3

3
4
⇒ Volume of 9000 spherical balls =9000 × ×π× ( radius of spherical ball )
3

3
4
⇒ Volume of 9000 spherical balls =9000 × ×π× ( 0.5 )
3

3
⇒ Volume of cylinder =π× ( radius of cylinder ) × ( rise in the level )
2

Now, Volume of cylinder Volume of 9000 spherical balls


4
⇒ π× (10 ) × ( rise in the level ) =9000× ×π× ( 0.5 )
2 3

3
⇒ Rise in the level = 15 cm
Therefore, the rise in the level of water in the vessel is 15 cm.

40. If a line is drawn parallel to one side of a triangle to intersect the other
two sides at distinct points, prove that the other two sides are divided in the
same ratio. 4 Marks
Ans: Consider a Δ ABC in which DE is drawn parallel to BC which intersects
the side AB and AC at D and E respectively.

Class X Mathematics www.vedantu.com 36


AD AE
We have to prove = .
DB EC
Construction: Join BE and CD and draw DQ ⊥ AC and EP ⊥ AC .
1
Area of ∆ADE= × base × height
2
1
⇒ Area of ΔADE= ×AD×EP
2
Also,
1
⇒ Area of ΔADE= ×AE×DQ
2
Similarly,
1
⇒ Area of ΔBDE= ×BD×EP
2
1
⇒ Area of ΔDEC= ×EC×DQ
2
On taking ratio we get,
1
Area of ΔADE 2 ×AD×EP
⇒ =
Area of ΔBDE 1 ×BD×EP
2
Area of ΔADE AD
⇒ = ---- (1)
Area of ΔBDE BD
Similarly,

Class X Mathematics www.vedantu.com 37


1
Area of ΔADE 2 ×AE×DQ
⇒ =
Area of ΔDEC 1 ×EC×DQ
2
Area of ΔADE AE
⇒ = − − − ( 2)
Area of ΔDEC EC
Also, in ∆ BDE and ∆ DEC has the same base DE and between the same
parallel lines BC and DE.
So, Area of ∆ BDE = Area of ∆ DEC ----(3)
Therefore, from equation (1), (2), and (3) we get
AD AE
⇒ =
DB EC
Hence proved that if a line is drawn parallel to one side of a triangle to
intersect the other two sides at distinct points, then the other two sides are
divided in the same ratio.

Class X Mathematics www.vedantu.com 38


CBSE-XII-2017 EXAMINATI CBSE – X – 2007 EXAMINATION

SCIENCE
SET-3 Paper & Solution
Time: 2½ Hrs. Max. Marks: 60

General Instructions:
1. The question paper comprises of two Sections, A and B. You are to attempt both the sections.
2. All questions are compulsory.
3. The candidates are advised to attempt all the questions of Section A and Section B separately.
4. There is no overall choice. However, internal choice has been provided in some questions. You are to attempt only
one option in such questions.
5. Question numbers 1–4 in Section A and 17, 18 in Section B are very short answer questions. These questions carry
one mark each.
6. Question numbers 5–8 in Section A and 19, 20 in Section B are short answer questions and carry two marks each.
7. Question numbers 9–14 in Section A and 21–23 in Section B are also short answer questions and carry three marks
each.
8. Question numbers 15, 16 in Section A and 24 in Section B are long answer questions and carry five marks each.

SECTION A

Question 1. Write the type of reactions in the following:


i) Reaction between an acid and a base
ii) Rusting of iron
Solution: i) Neutralization reaction
ii) Oxidation reaction
Marks: 1

Question 2. Give the names of the functional groups.


i. –CHO
ii. C = O
Solution: i) Aldehyde group
ii) Ketone group
Marks: 1

Question 3. Write the function of iris in the human eye?


Solution: Iris is a coloured muscular diaphragm that controls the amount of light entering the eye by
adjusting the size of the pupil.
Marks: 1

Question 4. What is the S.I. unit of electrical potential?


Solution: S.I. unit of electrical potential is Volt.
Marks: 1

Question 5. a) Give Arrhenius definition of an acid and a base.


b) Choose strong acid and strong base from the following:
CH3COOH, NH4OH, KOH, HCl
Solution: a) According to Arrhenius, a scientist, an acid is a substance which gives H+ ions in its aqueous
solution. Base is a substance which gives OH- ions in its aqueous solution.

www.vedantu.com 1/9
b) HCl – Strong Acid
KOH – Strong Base
Marks: 2

Question 6. What are esters? Write an equation to show the formation of an ester.
Solution: Esters are sweet smelling organic compounds. Their functional group is
R1COOR2, where R1 and R2 are alkyl radicals.
CH3COOH  C2H5OH 
H2SO4
 CH3COOC2H5  H2O
Ethanoic acid (Ethanol) Warm Ethyl ethanoate (An ester)

Marks: 2

Question 7. What is geothermal energy? What are its advantages?


Solution: The heat energy available from the hot rocks present inside the earth is known as geothermal
energy.
Advantages of geothermal energy:
i) It is available throughout the year.
ii) Cost of production of electricity from the source of energy is very less.
iii) It does not cause any pollution.
Marks: 2

Question 8. An electric iron has a rating of 750 W, 220 V. Calculate


i) Current passing through it, and
ii) Its resistance, when in use.
Solution: Power, P = 750 W
Potential difference, V = 220 V
Current, I = ?
Resistance, R = ?
We Know P =VI
P 750
I   3.41 A
V 220
V 220
and R    64.51 
I 3.41
Marks: 2

Question 9. Name the raw materials that are required for the manufacture of washing soda by Solvay
process. Describe the chemical reactions involved in the process.
Solution: The raw materials needed for the manufacture of washing soda are: NaCl (sodium chloride),
water, ammonia gas and limestone to give CO2 gas.
Chemical reactions involved:
NaCl + H2O + CO2 + NH3 
 NH4Cl + NaHCO3
2NaHCO3 
Heat
 Na2CO3 + CO2 + H2O
Na 2CO3  10H2O 
 Na 2CO3.10H2O
Sodium carbonate Crystals of Washing Soda

Marks: 3

www.vedantu.com 2/9
Question 10. Write about different chemical processes used for obtaining a metal from its oxides, for metals
low in the activity series, metals in the middle of activity series and metals towards the top of the activity
series.
Solution: Metals low in the activity series: Oxides of such metals can be reduced to metal by heating alone
in the air.
Example: HgS is cinnabar ore of mercury metal. This on heating in air changes to metal oxide and metal
oxide then on heating gets reduced to mercury metal.
2HgS  3O2 
Heat
 2HgO  2SO2 
(air)

2HgO 
Heat
 2Hg  O2
Mercury metal

Metals in the middle of activity series: Metals in the middle of the reactivity series are usually present as
sulphides or carbonates. These sulphides and carbonates are converted into metal oxides and then these
metal oxides are reduced to corresponding metals by reduction. Coke (carbon) acts as reducing agent.
Example:
2ZnS  3O2 
 2ZnO  2SO2
ZnO  C 
Heat
 2Zn  CO 
(coke)

Metals on the top of the activity series: As these metals are quite reactive metals, their oxides are reduced by
electrolysis method. On passing the current through their molten state, we get the metals at the cathode.
Example:
2Al2O3 
Current
4Al3+  6O2
Enriched
Bauxite ore

At cathode 4Al3+ + 12e– 


 4Al
At anode: 6O2– 
 3O2 + 12e–
Marks: 3

Question 11. Explain the mechanism of the cleaning action of soaps.


Solution: Most of the dirt is oily in nature. The oil does not dissolve in water. Moreover, soap molecules are
sodium or potassium salts of long chair carboxylic acids. Each soap molecule has two parts which are (i)
ionic part and (ii) hydrocarbon chain. Ionic part is water-loving or hydrophilic and soluble in water. The
hydrocarbon chain is hydrophobic and is not soluble in water. Inside water, the soap molecules have a
unique orientation in which its clusters of molecules form a structure called micelle. In the micelle, the ionic
parts of soap molecules are oriented towards water and hydrocarbon chain tails away from water (as shown
in the diagram).
The oily dirt is collected in centre of the micelle and gets entrapped into it. These micelles stay in water as
colloids. Thus, dirt suspended in the micelles is easily rinsed away.

www.vedantu.com 3/9
Marks: 3

Question 12. A concave lens has focal length of 20 cm. At what distance from the lens a 5 cm tall object be
placed so that it forms an image at 15 cm from the lens? Also calculate the size of the image formed.
Solution: Focal length, f = –20cm,
Image distance, v = –15cm
Object height, h = 5 cm
Object distance, u = ?
Image height, h' = ?
1 1 1
 
v u f
1 1 1
  
u v f
1 1 1
  
u 15 20
1 4  3
 
u 60
1 1
 
u 60
 u = –60 cm
h' v

h u
v
 h'  h
u
15  5 75
h   1.25 cm
60 60
Marks: 3

www.vedantu.com 4/9
Question 13. a) Why is the Solar Cooker box covered with a plane glass plate?
b) Why is energy of water flowing in a river considered to be an indirect form of solar energy?
c) Write one advantage of nuclear fission reaction.
Solution: a) The Solar Cooker box is covered with a plane glass plate to help in trapping the solar heat
inside the box through the greenhouse effect.
b) The water cycle in nature occurs due to energy of the sun. Solar energy causes the water to evaporate to
form clouds. These water vapours have potential energy. When clouds fall as rain, potential energy of water
vapours changes into kinetic energy of water. Some of this rain water flows into the rivers, thus kinetic
energy of river water can be considered as indirect form of solar energy.
c) Enormous amount of electrical energy can be produced from fission of 1 atomic mass unit.
Marks: 3

Question 14. a) What is meant by ‘Electric Resistance’ of a conductor?


b) A wire of length L and resistance R is stretched so that its length is doubled and the area of cross-section
is halved. How will its:
i) Resistance change? ii) Resistivity change?
Solution: a) 'Electric resistance' of a conductor is the amount of opposition it offers to the flow of electric
current through it. It is numerically equal to the ratio of potential difference across its ends to the current
flowing through it.
p.d. (V )
Resistance (R) =
Current ( I )
l
b) We know R  
A
where, R is the resistance,
 is the resistivity,
l is the length,
A is the area of cross-section.
When the length is doubled and the area of cross-section is halved, the resistance of the wire becomes
2l l
R'    4  4R
A/ 2 A
i) New Resistance will be four times of the original resistance.
ii) Resistivity will remain the same because it is a characteristic property of the material of the wire.
Marks: 3

Question 15. a) Name one main ore of zinc metal. Write its formula. How is this metal ore changed into its
oxide compound?
b) Explain in brief about electrolytic refining method.
OR
a) Why is sulphuric acid called ‘King of Chemicals’?
b) State two ways to prevent the rusting of iron.
c) Why should water be never added dropwise to concentrated sulphuric acid?
Solution: a) Ore of zinc. Zinc blend ore
Formula - ZnS
This ore is changed into metal oxide by roasting it in the presence of air.

www.vedantu.com 5/9
2ZnS  3O2 
Roasting
 2ZnO  2SO2
(air) (Heat)

b) Many metals like copper, zinc, tin, nickel, silver etc. are purified by electrolytic refining method. In this
method, the impure metal is made the anode and a thin strip of same pure metal is made the cathode. An
aqueous solution of that metal salt is used as an electrolyte. The apparatus is set up as shown in the diagram.
On passing current through the electrolyte, pure metal from the anode dissolves into the electrolyte. An
equivalent amount of the metal from the electrolyte gets deposited on the cathode. The impurities settle
down at the bottom of the anode.

OR
a) Sulphuric acid is used for the manufacture of many types of industrial compounds, so, it is called the
‘King of Chemicals’.
b) Two ways to prevent rusting of iron:
i) By changing the iron metal to its alloys like stainless-steel, cobalt steel etc.
ii) By painting the iron articles with paint etc.
c) Water should never be added to concentrated sulphuric acid, as this reaction is highly exothermic and the
heat generated may splash out solution from the container and cause some accident in the laboratory.
Marks: 5

Question 16. Define the term, ‘Critical Angle’. What is meant by ‘total internal reflection’?
State two essential conditions for total internal reflection to take place. With the help of a ray diagram,
illustrate an application of total internal reflection.
OR
a) What is meant by a ‘magnetic field’?
b) How is the direction of magnetic field at a point determined?
c) Describe an activity to demonstrate the direction of the magnetic field generated around a current carrying
conductor.
d) What is the direction of magnetic field at the centre of a current carrying circular loop?
Solution: 'Critical angle' is that angle of incidence in the denser medium for which the angle of refraction is
90o in the rarer medium. Total internal reflection: When a ray of light travels from a denser medium to a
rarer medium and its angle of incidence is greater than the critical angle, it does not come out in the rarer
medium but is totally reflected within the denser medium. This phenomenon is known as 'total internal
reflection'.
Two essential conditions for total internal reflection:
i) Ray of light must travel from denser to rarer medium.

www.vedantu.com 6/9
ii) The angle of incidence must be greater than the critical angle for the given pair of media.
An application of total internal reflection: Formation of rainbow in the sky after a rain shower is based on
the formation of natural spectrum and total internal reflection of light. Rainbow is caused by the dispersion
of sunlight by tiny water droplets present in the atmosphere. The water droplets act like small prisms. They
refract the incident sunlight and then reflect it internally and finally refract it again when it comes out of the
rain-drop. A rainbow is always formed in a direction opposite to that of the Sun.

OR
a) Magnetic field: The space surrounding a magnet in which magnetic force is exerted is called magnetic
field.
b) The direction of magnetic field at a point is determined by placing a small magnetic compass at that point.
The direction of north-pole of the compass needle gives the direction of magnetic field at a point.
c) Connect a thick copper wire in an electric circuit having a rectangular cardboard inserted in the wire as
shown in the diagram. Then sprinkle some iron filings uniformly on the card-board. Close the key so that the
current flows through the copper wire. Gently tap the card-board a few times. You will observe that the iron
filings align themselves showing a pattern of concentric circle around the copper wire.

The concentric circles represent the magnetic field lines around the current carrying conductor (copper
wire).
d) The direction of magnetic field at the centre of a current carrying circular loop is perpendicular to the
plane of the loop.
Marks: 5

SECTION-B
Question 17. Name the term for transport of food from leaves to other parts of the plant.
Solution: Translocation of food.
Marks: 1

Question 18. What is a neuron?


Solution: A neuron is the structural and functional unit of nervous system of animals.
Marks: 1

www.vedantu.com 7/9
Question 19. What is lymph? Write its important functions.
OR
State the two vital functions of the human kidney. Name the procedure used in the working of artificial
kidney.
Solution: Lymph is an extra cellular fluid of the body which is also involved in transportation. Lymph
transports digested and absorbed food fats from the intestine to the blood.
OR
Two vital functions of human kidney:
i) Excretion (Removal) of nitrogenous wastes from the body in the form of urine.
ii) Osmoregulation of ions and water content inside the body organs. The procedure used in the working of
artificial kidney is called dialysis.
Marks: 2

Question 20. Name the two hormones secreted by pancreas. Write one function of each hormone named.
Solution: Two hormones secreted by pancreas:
i) Insulin
ii) Glucagon
Insulin hormone controls the sugar level (glucose) in the blood to normal level.
Glucagon hormone increases the sugar level in the blood.
Marks: 2

Question 21. a) What is fertilization? Distinguish between external fertilization and internal fertilization.
b) What is the site of fertilization in human beings?
Solution: a) Fertilization is the process of fusion of male gamete with the female gamete (ovum) to produce
zygote cell.
External fertilization takes place outside the body of the female while internal fertilization takes place inside
the sexual tract of the female.
b) Fallopian tube (oviduct) is the site of fertilization in the human beings.
Marks: 3

Question 22. Define the terms:


i) Analogous
ii) Vestigial
iii) Sex chromosome
Solution: i) Analogous organs are those organs which have same function but different basic structures in
different animals.
Example: Wings of insects and wings of birds.
ii) Vestigial organs are the organs which are functionless and reduced in size in an organism but are
functional and of normal size in organisms of its race.
Example: Third eye membrane and tail bones in the human beings.
iii) Sex chromosomes are the chromosomes which determine the sex of the offspring (next generation).
Example: XX chromosomes in human female. XY chromosomes in human
male.
Marks: 3

www.vedantu.com 8/9
Question 23. Give any two ways in which biodegradable substances would affect the environment.
OR
Suggest three ways to maintain a balance between environment and development to survive.
Solution: Biodegradable substances affect the environment in the following ways:
i) These substances are decomposed by the action of micro-organisms. This causes foul smell.
ii) During the process of decaying of biodegradable substances, various types of gases are released which
cause air pollution.
OR
Three ways to maintain balance between environment and development:
i) Judicious use of natural resources and replenishing of such resources which can be replenished like
growing of trees in the forests.
ii) Protecting the wild animals from hunting and preserving their natural habitats.
iii) Managing the waste and pollutant materials by decomposing and recycling of such materials.
Marks: 3

Question 24. a) Draw the diagram of cross-section of a leaf and label the following in it:
i) Chloroplast
ii) Guard cell
iii) Lower epidermis
iv) Upper epidermis
b) Name the two stages in photosynthesis.
Solution: (a)

b) Two stages in photosynthesis:


i) Light reactions
ii) Dark reactions
Marks: 5

www.vedantu.com 9/9
CBSE-XII-2017 EXAMINATI CBSE – X – 2008 EXAMINATION

SCIENCE
SET-3 Paper & Solution
Time: 2½ Hrs. Max. Marks: 60

General Instructions:
1. The question paper comprises of two Sections, A and B. You are to attempt both the sections.
2. All questions are compulsory.
3. All questions of Section A and all questions of Section B are to be attempted separately.
4. There is no overall choice. However, internal choice has been provided in some questions. Only one option in such
questions is to be attempted.
5. Question numbers 1–6 in Section A and 17 to 19 in Section B are short answer questions. These questions carry one
mark each.
6. Question numbers 7–10 in Section A and 20 to 24 in Section B are short answer questions and carry two marks
each.
7. Question numbers 11–14 in Section A and 25 and 26 in Section B are also short answer questions and carry three
marks each.
8. Question numbers 15 and 16 in Section A and 27 in Section B are long answer questions and carry five marks each.

SECTION A

Question 1. Balance the following chemical equation:


Fe(s) + H2O(g)  Fe3O4(s) + H2(g)
Solution: 3Fe(s) + 4H2O(l)  Fe3O4(s) + 4H2(g)
Marks: 1

Question 2. Why is respiration considered an exothermic process?


Solution: Respiration is called an exothermic reaction because during respiration, breaking down of
glucose/food in the presence of oxygen occurs with release of energy.
Marks: 1

Question 3. How does the flow of acid rain water into a river make the survival of aquatic life in the river
difficult?
Solution: When acid rain water flows into the rivers, it lowers the pH of river water making the survival of
aquatic life difficult.
Marks: 1

Question 4. Draw the following diagram in your answer-book and show the formation of image of the
object, AB with the help of suitable rays.

www.vedantu.com 1 / 10
Solution:

Marks: 1

Question 5. Why is a series arrangement not used for connecting domestic electrical appliances in a circuit?
Solution: A series arrangement is not used for connecting domestic electrical appliances in a circuit
because:
i) Same current flows through each device, but different devices need current of different values to operate.
ii) If one device in a series circuit is defective, current is cut off.
iii) Total resistance of the circuit increases, so current flowing is reduced.
iv) Selective operation of devices is not possible.
Marks: 1

Question 6. Out of 60 W and 40 W lamps, which one has a higher electrical resistance when in use?
Solution: 40 W lamp
Marks: 1

Question 7. Write the chemical formula for washing soda. How may it be obtained from baking soda?
Name an industrial use of washing soda other than washing clothes.
Solution: Washing soda: Na2CO3.10H2O
It is obtained by heating baking soda.

2NaHCO3   Na2CO3 + H2O + CO2
Na2CO3 + 10H2O   Na2CO3.10H2O
Uses: It is used in the manufacture of glass, soap and paper.
Marks: 2

Question 8. Give an example of a decomposition reaction. Describe an activity to illustrate such a reaction
by heating.

Solution: CaCO3   CaO + CO2
(or any other example)
Activity:
Take 2 g of ferrous sulphate crystals in a dry test tube. Heat this test tube over the flame for some time. On
heating, green color of the crystals changes into dark brown and a gas with characteristic smell of burning
sulphur is obtained.
Marks: 2

www.vedantu.com 2 / 10
Question 9. Draw ray diagrams to represent the nature, position and relative size of the image formed by a
convex lens for the object placed:
a) at 2F1.
b) between F1 and the optical centre O of lens.
Solution: (a)

(b)

Marks: 2

Question 10. What is meant by the term, 'magnetic field'? Why does a compass needle show deflection
when brought near a bar magnet?
Solution: Magnetic field - The region around a magnet in which force of the magnet can be experienced. A
compass needle is a small bar magnet so it experiences the force of the other bar magnet when brought near
it and deflects.
Marks: 2

Question 11. a) Why are covalent compounds generally poor conductors of electricity?
b) Name the following compound:

c) Name the gas evolved when ethanoic acid is added to sodium carbonate. How would you prove the
presence of this gas?
Solution: a) Covalent compounds do not provide ions in aqueous solutions hence they do not conduct
electricity.
b) Propanone/acetone

www.vedantu.com 3 / 10
c) CO2 gas is obtained when ethanoic acid is reacted with sodium carbonate. Presence of the gas can be
tested by passing the gas through lime water. Carbon dioxide gas turns lime water milky.
Marks: 3

Question 12. a) What are amphoteric oxides? Choose the amphoteric oxides from amongst the following
oxides:
Na2O, ZnO, Al2O3, CO2, H2O
b) Why is it that non-metals do not displace hydrogen from dilute acids?
Solution: a) Amphoteric oxides are metal oxides which show both basic as well as acidic behavior.
ZnO, Al2O3
b) Non-metals cannot lose electrons to H+ to form H2 gas because nonmetals are electron-acceptors hence
they do not react with dilute acids.
Marks: 3

Question 13. Two lamps, one rated 60 W at 220 V and the other 40 W at 220 V, are connected in parallel to
the electric supply at 220 V.
(a) Draw a circuit diagram to show the connections.
(b) Calculate the current drawn from the electric supply.
(c) Calculate the total energy consumed by the two lamps together when they operate for one hour.
Solution: (a)

P
(b) I 
V
60 W 3
I1   A
220 W 11
40 W 2
I2   A
220 W 11
3 2 5
I  I1  I 2  +  A  0.45 A
11 11 11
(c) E = P × t = (40 W + 60 W) × 1 h = 100 Wh or 0.1 kWh.
Marks: 3

Question 14. a) Distinguish between the terms 'overloading' and 'short-circuiting' as used in domestic
circuits.
b) Why are the coils of electric toasters made of an alloy rather than a pure metal?

www.vedantu.com 4 / 10
Solution: a) Short circuiting - When neutral and live wire come in direct contact.
Overloading - When too many appliances are connected to a single socket drawing much more current or
power than permissible.
b) Resistivity of an alloy is higher than its constituent metal and alloys do not oxidize as easily as constituent
metal at high temperature. That is why the coils of electric toasters are made of an alloy rather than a pure
metal.
Marks: 3

Question 15. On the basis of Mendeleev's Periodic Table given below, answer the questions that follow the
table:

(a) Name the element which is in


i) 1st group and 3rd period.
ii) 7th group and 2nd period.
(b) Suggest the formula for the following:
i) Oxide of nitrogen
ii) Hydride of oxygen
(c) In group VIII of the Periodic Table, why does cobalt with atomic mass 58.93 appear before nickel having
atomic mass 58.71?

www.vedantu.com 5 / 10
(d) Beside gallium, which two other elements have since been discovered for which Mendeleev had left gaps
in his Periodic Table?
(e) Using atomic masses of Li, Na and K, find the average atomic mass of Li and K and compare it with the
atomic mass of Na. State the conclusion drawn from this activity.
OR
a) Why do we classify elements?
b) What were the two criteria used by Mendeleev in creating his Periodic Table?
c) Why did Mendeleev leave some gaps in his Periodic Table?
d) In Mendeleev's Periodic Table, why was there no mention of Noble gases like Helium, Neon and Argon?
e) Would you place the two isotopes of chlorine, Cl-35 and Cl-37 in different slots because of their different
atomic masses or in the same slot because their chemical properties are the same? Justify your answer.
Solution: (a) (i) Sodium
(ii) Fluorine
(b) (i) N2O5
(ii) H2O/OH2
(c) In group VIII of the Periodic Table, cobalt appears before nickel so that elements with similar chemical
properties may fall in the same group.
(d) Scandium (Sc) and Germanium (Ge)
(e) Atomic mass of lithium = 7
Atomic mass of potassium = 39
So, average of atomic mass = (7 + 39)/2 = 23
Atomic mass of sodium = 23 i.e., both are same hence we can conclude that atomic mass of the middle
element is the average of the other two elements.
OR
(a) We classify elements to systematize the study of elements and make the understanding of properties of
elements and compounds simpler.
(b) Criteria used by Mendeleev:
i) Atomic mass
ii) Properties of hydrides and oxides of elements.
(c) Mendeleev left some gaps in his Periodic Table to leave scope of search for the yet undiscovered
elements.
(d) In Mendeleev's Periodic Table, there was no mention of noble gases since they had not been discovered
by that time.
(e) The two isotopes of chlorine, Cl-35 and Cl-37 will be placed in the same slot because their chemical
properties are same.
Marks: 5

Question 16. a) What is meant by dispersion of white light? Describe the formation of rainbow in the sky
with the help of a diagram.
b) What is hypermetropia? Draw ray diagrams to show the image formation of an object by:
i) Hypermetropic eye
ii) Correction made with a suitable lens for hypermetropic eye.
OR
(a) Give reasons for the following:
i) Colour of the clear sky is blue.

www.vedantu.com 6 / 10
ii) The sun can be seen about two minutes before actual sunrise.
iii) We cannot see an object clearly if it is placed very close to the eyes.
(b) What is Presbyopia? Write two causes of this defect.
Solution: (a) Dispersion - The splitting of white light into its constituent colours.
Rainbow formation (figure)

Water droplets in air refract and disperse the incident sunlight. Then, reflect it internally and finally refract it
again when it comes out of the droplet. Due to the dispersion of light and internal reflection, different
colours of sunlight reach the observer's eye and are visible in the form of a rainbow.
(b) Hypermetropia - The defect of vision due to which a person clearly sees distant objects but cannot
clearly see nearby objects.
(i)

(ii)

OR
(a) (i) Due to scattering of light
(ii) Due to atmospheric refraction
(iii) At the near point of eye, curvature of eye lens is maximum and focal length is minimum. If object is
placed nearer than it, eye lens cannot adjust its curvature.
(b) Presbyopia - The defect of vision in which the eye is unable to see nearby as well as far off objects
clearly.

www.vedantu.com 7 / 10
Causes:
--- weakening of ciliary muscles
--- diminishing flexibility of the eye lens.
Marks: 5

SECTION-B
Question 17. Which one of the following is a renewable resource?
Natural gas, Petroleum, Ground water, Coal
Solution: Ground water.
Marks: 1

Question 18. What is the effect of DNA copying which is not perfectly accurate on the reproduction
process?
Solution: Imperfect DNA copying in the reproduction process leads to variations or evolution.
Marks: 1

Question 19. How do autotrophs obtain CO2 and N2 to make their food?
Solution: CO2 is obtained from the environment and N2 is obtained from the soil and environment.
Marks: 1

Question 20. List any four characteristics of biogas on account of which it is considered an ideal fuel.
Solution: Biogas is considered as an ideal fuel because of the following:
(i) High Calorific Value
(ii) Produces no smoke on burning
(iii) Burns smoothly (without explosion)
(iv) No residue on combustion.
Marks: 2

Question 21. Discus one limitation each for the extracting of energy from:
a) Winds
b) Tides
Solution: (a) From wind:
(i) Wind energy cannot be harnessed at places where wind does not blow at a minimum speed of 15 km/h.
(ii) Wind is not a dependable source as sometimes air is still and at other times there are storms.
(b) From tides:
(i) There are only few sites suitable for building tidal dams.
(ii) The rise and fall of sea water during high and low tides are not enough to generate electricity on a large
scale.
Marks: 2

Question 22. Write one function each of the following components of the transport system in human beings:
a) Blood vessels
b) Blood platelets
c) Lymph
d) Heart

www.vedantu.com 8 / 10
Solution: (a) Blood Vessels: Transport of blood.
(b) Blood Platelets: Clotting of blood.
(c) Lymph: Carries digested fats.
(d) Heart: Helps to circulate blood in the whole body by acting as a pump.
Marks: 2

Question 23. Name one sexually transmitted disease each caused due to bacterial infection and viral
infection. How can these be prevented?
Solution: (i) Bacterial: Gonorrhea or syphilis.
Viral: Warts or AIDS.
(ii) These can be prevented by the use of condoms.
Marks: 2

Question 24. What are fossils? What do they tell about the process of evolution?
Solution: Fossils are the remains or traces of animals and plants of the past on rocks.
Fossils give information about evolutionary relationships between different species.
Marks: 2

Question 25. How is ozone formed in the upper atmosphere? Why is damage to ozone layer a cause of
concern to us? What causes this damage?
Solution: (i) UV rays in the atmosphere split some molecular oxygen (O 2) into free oxygen (O) atoms.
(ii) These atoms combine with molecular oxygen to form O3.
OR
O2  
UV rays
 O+O
O + O2   O3
Damage to ozone layer will allow UV rays to reach on the surface of earth causing skin cancer, cataract and
damage to crops.
Release of chlorofluoro carbons in the atmosphere which are used as refrigerants or in fire extinguishers
damages the ozone layer.
Marks: 3

Question 26. How are oxygen and carbon dioxide transported in human beings? How are lungs designed to
maximize the area for exchange of gases?
Solution: (i) Respiratory pigment haemoglobin takes up O2 from the air in the lungs and carries it to tissues.
(ii) CO2 is being transported from various tissues into the alveoli by blood and is released during exhalation.
Within the lungs, the trachea divides into smaller and smaller tubes which finally terminate in balloon like
structures called alveoli. These alveoli increase the surface area for the exchange of gases.
Marks: 3

Question 27. (a) Draw the structure of a neuron and label the following on it:
Nucleus, Dendrite, Cell body and Axon
(b) Name the part of neuron:
i) Where information is acquired.
ii) Through which information travels as an electrical impulse.
OR

www.vedantu.com 9 / 10
(a) What is (i) photoropism and (ii) geotropism? With labelled diagrams describe an activity to show that
light and gravity change the direction that plant parts grow in.
(b) Mention the role of each of the following plant hormones:
i) Auxin
ii) Abscisic acid
Solution: (a)

(i) Information is acquired through dendrite.


(ii) From the dendrite to the cell body and then along the axon to it’s end.
OR
(a) (i) Phototropism: The movement of a plant or it's part in response to light is called phototropism.
(ii) Geotropism: The movement of a plant or it’s part in response to gravity is called geotropism.
Activity to show that light and gravity change the direction that plant part grows in:
i. Fill a conical flask with water.
ii. Cover the neck of the flask with a wire mesh.
iii. Keep two or three freshly germinated bean seeds on the wire mesh.
iv. Take a cardboard box which is open from one side.
v. Keep the flask in the box in such a manner that the open side of the box faces light coming from a
window.
vi. After two or three days, you will notice that the shoots bend towards light and roots away from light.

(b) (i) Auxin: - It promotes growth and cell elongation.


(ii) Abscisic acid: It inhibits growth and causes wilting of leaves.
Marks: 5

www.vedantu.com 10 / 10
CBSE-XII-2017 EXAMINATI CBSE – X – 2009 EXAMINATION

SCIENCE
SET-3 Paper & Solution
Time: 2½ Hrs. Max. Marks: 60

General Instructions:
1. The question paper comprises of two Sections, A and B. You are to attempt both the sections.
2. All questions are compulsory.
3. All questions of Section A and all questions of Section B are to be attempted separately.
4. There is no overall choice. However, internal choice has been provided in some questions. Only one option in such
questions is to be attempted.
5. Question numbers 1–6 in Section A and 17 to 19 in Section B are short answer questions. These questions carry one
mark each.
6. Question numbers 7–10 in Section A and 20 to 24 in Section B are short answer questions and carry two marks
each.
7. Question numbers 11–14 in Section A and 25 and 26 in Section B are also short answer questions and carry three
marks each.
8. Question numbers 15 and 16 in Section A and 27 in Section B are long answer questions and carry five marks each.

SECTION A

Question 1. Balance the following chemical equation:


Pb(NO3)2(s)  
heat
 PbO(s) + NO2(g) + O2(g)
Solution: 2Pb(NO3)2(s)  
heat
 2PbO(s) + 4NO2(g) + O2(g)
Marks: 1

Question 2. Fresh milk has a pH of 6. When it changes into curd (yogurt) will its pH value increase or
decrease? Why?
Solution: pH value decreases when fresh milk changes to curd.
Marks: 1

Question 3. Name a reducing agent that may be used to obtain manganese from manganese dioxide.
Solution: Hydrochloric acid may be used as the reducing agent to obtain manganese from manganese
dioxide.
Marks: 1

Question 4. Why does a ray of light bend when it travels from one medium into another?
Solution: Light has different speeds in different media and it takes such a path of propagation for which
time taken is minimum.
Marks: 1

Question 5. Draw the given diagram in your answer book and complete it for the path of ray of light beyond
the lens.

www.vedantu.com 1 / 11
Solution:

Marks: 1

Question 6. Why does sky look blue on a clear day?


Solution: Sky looks blue on a clear day because blue colour of light is scattered most by the particles
present in the atmosphere.
Marks: 1

Question 7. A compound which is prepared from gypsum has the property of hardening when mixed with a
proper quantity of water. Identify the compound. Write the chemical equation for its preparation. For what
purpose is it used in hospitals?
Solution: Plaster of Paris (Calcium sulphate hemihydrate) CaSO 4.1/2H2O
1 1
CaSO4 .2H2O  Heat 373 K
 CaSO4 . H2O  1 H2O
Gypsum 2 2
Plaster of Paris
It is used in hospitals as a plaster to support fractured bones in the right position.
Marks: 2

Question 8. (a) What is the colour of ferrous sulphate crystals? How does this colour change after heating?
(b) Name: the products formed on strongly healing ferrous sulphate crystals. What type of chemical reaction
occurs in this change?
Solution: (a) Ferrous sulphate crystals are light green in color. On heating, the green color of the crystals
changes to white since its water of crystallisation is removed on heating.
(b) On strongly heating ferrous sulphate crystals, ferric oxide, sulphur dioxide and sulphur trioxide is
formed.
2FeSO4 (s) 
Heat
 Fe2O3 (s)  SO2 (g)  SO3 (g)
This is a decomposition reaction.
Marks: 2

Question 9. What is the minimum number of rays required for locating the image formed by a concave
mirror for an object? Draw a ray diagram to show the formation of a virtual image by a concave mirror.
Solution: At least two rays are required for locating the image formed by a concave mirror for an object.
Formation of virtual image by concave mirror:

Marks: 2

www.vedantu.com 2 / 11
Question 10. A piece of wire of resistance 20  is drawn out so that its length is increased to twice its
original length. Calculate the resistance of the wire in the new situation.

Solution: R 
A
If the length is increased to twice the original length, keeping the area of cross section same, then resistance
will become double of its original value.
So new resistance = 2 × 20 = 40 ohm.
Marks: 2

Question 11. What is meant by 'rusting'? With labeled diagrams, describe an activity to find out the
conditions under which iron rusts.
Solution: Rusting is corrosion of iron to a brown flaky substance in the presence of moist air.
Activity to find out the conditions under which iron rusts:
1. Take three test tubes and place clean iron nails in each of them.
2. Label these test tubes as A, B and C.
3. Pour some water in test tube A and cork it.
4. Pour boiled distilled water in test tube B, add about 1 mL of oil and cork it. The oil will float on water and
prevent air from dissolving in water.
5. Put some anhydrous calcium chloride in test tube C and cork it.
Anhydrous calcium chloride will absorb the moisture, if any, from the air.
6. Leave the three test tubes for a few days and then observe.
Observation: The iron nails in test tube A rusts. In test tubes B and C, no rusting occurs.

Conclusion: Both air and moisture are necessary for rusting of iron.
Marks: 3

Question 12. Give reasons for the following observations:


(a) The element carbon forms a very large number of compounds.

www.vedantu.com 3 / 11
(b) Air holes of a gas burner have to be adjusted when the heated vessels get blackened by the flame.
(c) Use of synthetic detergents causes pollution of water.
Solution: (a) The element Carbon forms a large number of compounds due to its unique properties:
catenation and tetravalency.
(b) If fuel in the gas burner does not burn completely, then incomplete combustion occurs resulting in
production of a sooty flame and hence the vessels get blackened from the bottom. So, for sufficient supply
of air for complete combustion, the air holes of a gas burner have to be adjusted.
(c) Synthetic detergents cause pollution because they are non biodegradable in nature.
Marks: 3

Question 13. What is hypermetropia? State the two causes of hypermetropia. With the help of ray diagrams,
show:
i. the eye-defect hypermetropia
ii. correction of hypermetropia by using a lens
Solution: Hypermetropia is an eye defect in which distant vision is clear while near vision is blurred.
Causes of Hypermetropia:
--- Shortening of the eyeball, that is, the eyeball becomes smaller
--- Increase in focal length of the eye lens
(i)

(ii)

Marks: 3

Question 14. Two resistors, with resistances 5  and 10  respectively are to be connected to a battery of
emf 6 V so as to obtain:
(a) How will you connect the resistances in each case?
(i) minimum current flowing (ii) maximum current flowing
(b) Calculate the strength of the total current in the circuit in the two cases.
Solution: (a) (i) To obtain the minimum current, the resistances should be connected in series.

www.vedantu.com 4 / 11
(ii) To obtain the maximum current, the resistances should be connected in parallel.
(b) (i) Resistances in series:

Total resistance in the circuit R = 5 + 10 = 15 ohm


Current in the circuit I = 6/15 = 0.4 A
(ii) Resistances in parallel:

Total resistance in the circuit R = (5 x 10)/(5 + 10) = 50/15 = 10/3 ohm


Current in the circuit I = 6 x 3/10 = 1.8 A
Marks: 3

Question 15. (a) Which two criteria did Mendeleev use to classify the elements in his periodic table?
(b) State Mendeleev's periodic law.
(c) Why could no fixed position be given to hydrogen in Mendeleev's periodic table?
(d) I tow and why does the atomic size vary as you go:
i) From left to right along a period?
ii) Down a group?
OR
(a) Why did Mendeleev have gaps in his periodic table?
(b) Stale any three limitations of Mendeleev's classification.
(c) How do electronic configurations of atoms change in a period with increase in atomic number?
Solution: (a) Increasing atomic mass and similarity in chemical properties of elements were the two criteria
used by Mendeleev to classify the elements. He took the formulae of the oxides and hydrides formed by the
elements as the basis for classification of elements.
(b) Mendeleev’s periodic law states that the properties of elements are periodic function of their atomic
masses.
(c) Hydrogen resembles alkali metals in its electronic configuration and halogens as it also exists as a
diatomic molecule and combines with metals and non- metals to form covalent compounds hence it could
not be assigned a fixed position in Mendeleev's periodic table.

www.vedantu.com 5 / 11
(d) (i) Atomic size decreases from left to right in the periodic table due to increase in nuclear charge.
(ii) Atomic size increases down the group because new shells are being added as we go down the group.
Marks: 5

Question 16. (a) What is a magnetic field? How can the direction of magnetic field lines at a placed be
determined?
(b) State the rule for the direction of the magnetic field produced around a current carrying conductor. Draw
sketch of the pattern of field lines due to a current flowing through a straight conductor.
OR
(a) What is a solenoid? Draw a sketch of the pattern of field lines of the magnetic field through and around a
current carrying solenoid.
(b) Consider a circular loop of wire lying in the plane of the table. Let the current pass through the loop
clockwise. Apply the right hand rule to find out the direction of the magnetic field inside and outside the
loop.
Solution: (a) Magnetic field is a region near a magnetised body where magnetic forces can be detected. The
direction of the magnetic field line at a place is determined by the direction in which a north pole of the
compass needle moves inside it.
(b) Direction of the magnetic field produced around a current carrying conductor is determined by the right
hand thumb rule.
According to this rule, if we hold a current-carrying straight conductor in right hand such that the thumb
points towards the direction of current, then fingers will wrap around the conductor in the direction of the
field lines of the magnetic field. This is also shown in the figure given below:

Pattern of field lines due to a current flowing through a straight conductor:

www.vedantu.com 6 / 11
OR
(a) A solenoid is a long coil (shaped like a cylinder) containing a large number of close turns of insulated
copper wire.

(b) Direction of magnetic field inside and outside the loop is given as follows:

Marks: 5

SECTION-B

Question 17. What are the two main components of our environment?
Solution: The two main components of our environment are:
i. Biotic components
ii. Abiotic components.
Marks: 1

www.vedantu.com 7 / 11
Question 18. What will happen to a plant if its xylem is removed?
Solution: Xylem tissue conducts water and minerals from the soil to different parts of the plant. If the xylem
tissue is removed, then the transport of water and mineral will not take place and the plant will die.
Marks: 1

Question 19. Name two tissues that provide control and coordination in multi cellular animals.
Solution: Nervous and muscular tissues provide control and coordination in multi cellular animals.
Marks: 1

Question 20. What natural resources? State two factors that work against an equitable distribution of these
resources.
Solution: Natural resources are naturally occurring substances that are considered valuable in their
relatively unmodified or natural form. The examples of such resources are coal, wildlife, oil etc.
Two factors that work against the equitable distribution of these resources are:
(i) Corruption
(ii) Lack of proper management
Marks: 2

Question 21. What is water harvesting? Mention any water harvesting structures.
Solution: Water harvesting is the activity of collecting rainwater for future use. Harvesting of rainwater is
essential for ensuring the availability of usable water. The rainwater collected can be stored in surface
containers for direct use or can be recharged into the groundwater.
Khadins and Nadis are two traditional water harvesting structures that are found in Rajasthan.
Marks: 2

Question 22. What are 'nastic' and 'curvature' movements? Give one example of each.
Solution: Nastic movements are non-directional and growth independent movements that occur in response
to stimuli such as light, temperature, humidity, etc.
For example: Touch-me-not plant leaves bend and droop on touching.
Curvature movements are the bending or curving movements of a plant in response to any stimuli.
For example: The bending of the shoot tip towards light.
Marks: 2

Question 23. What is biogas? Why is biogas considered an ideal fuel for domestic use?
Solution: Biogas is the gas made from the anaerobic decomposition of organic matter such as agricultural
wastes and animal wastes like animal dung. It is prepared in bio-gas plants. It consists of a methane, carbon
dioxide, hydrogen and hydrogen sulphide.
Biogas is considered as an ideal fuel because it:
(i) It has a high calorific value.
(ii) It burns without producing smoke.
(iii)It is a safe and efficient method of waste-disposal.
(iv) It leaves no residue after burning.
Marks: 2

www.vedantu.com 8 / 11
Question 24. (a) Distinguish between renewable and non-renewable sources of energy.
(b) Choose the renewable sources of energy from the following list: Coal, biogas, sun, natural gas.
Solution: (a)
Renewable sources of energy Non-renewable sources of
i. These are inexhaustible that energy is can be i. They are exhaustible and therefore cannot be
used over a long time. used over a long time.

ii. They are replenished at a rate faster than the ii. The rate at which they are replenished is
rate at which they are consumed. much slower than the rate of consumption.

(b) Biogas and sun


Marks: 2

Question 25. Explain analogous organs and homologous organs. Identify the analogous and homologous
organs amongst the following:
Wings of an Insect, wings of a bat, forelimbs of frog, forelimbs of a human
Solution: Analogous organs: These are organs that have different structural design and origin, but perform
similar functions.
Homologous organs: These organs have the similar basic structural design and origin, but are evolved to
perform different functions.
Analogous organs: Wings of an insect and wings of a bat. Homologous organs: Forelimbs of frog and
forelimbs of a human.
Marks: 3

Question 26. (a) Explain the terms:


i) Implantation
ii) Placenta
(b) What is the average duration of human pregnancy?
Solution: (a)
(i) Implantation is the event during pregnancy in which a fertilized egg or zygote adheres to the walls of the
uterus, inside the female body.
(ii) Placenta is a disc like vascular structure embedded in the uterine wall. The placenta supplies nutrients
like glucose and oxygen to the developing embryo. It also removes waste substances generated by the
embryo.
(b) The average duration of human pregnancy is nine months.
Marks: 3

Question 27. (a) Draw a diagram of human alimentary canal and label on it:
Oesophagus, Gall bladder, Liver and Pancreas
(b) Explain the statement, 'Bile does not contain any enzyme but it is essential for digestion'.
OR
(a) Draw a diagram of excretory system in human beings and label on it:
Aorta, vena cava, urinary, bladder, urethra
(b) List two vital functions of the kidney.

www.vedantu.com 9 / 11
Solution: (a)

(b) Bile does not contain any enzyme, but it plays an important role in digestion because:
(i) The bile salts emulsify fat by acting on large fat globules to break them into smaller globules. This
increases the efficiency of pancreatic enzymes.
(ii) The food entering the small intestine is acidic. It is made alkaline by the action of bile juice so as to
facilitate the action of pancreatic enzymes.
OR
(a)

www.vedantu.com 10 / 11
(b) The two vital functions of kidney are:
(i) It filters out the nitrogenous wastes from the blood and forms urine.
(ii) It also regulates the water balance and levels of mineral ions in the body.
Marks: 5

www.vedantu.com 11 / 11
CBSE-XII-2017 EXAMINATI CBSE – X – 2010 EXAMINATION

SCIENCE
SET-3 Paper & Solution
Time: 2½ Hrs. Max. Marks: 60

General Instructions:
1. The question paper comprises of two Sections, A and B. You are to attempt both the sections.
2. All questions are compulsory.
3. The candidates are advised to attempt all the questions of Section A and Section B separately.
4. There is no overall choice. However, internal choice has been provided in some questions. You are to
attempt only one option in such questions.
5. Question numbers 1–6 in Section A and 19 to 21 in Section B are very short answer questions. These
questions carry one mark each.
6. Question numbers 7–12 in Section A and 22 to 24 in Section B are short answer questions and carry two
marks each.
7. Question numbers 13–16 in Section A and 25 and 26 in Section B are also short answer questions and
carry three marks each.
8. Question numbers 17 and 18 in Section A and 27 in Section B are long answer questions and carry five
marks each.

SECTION A

Question 1. What change in the colour of iron nails and copper sulphate solution you observe after keeping
the iron nails dipped in copper sulphate solution for about 30 minutes.
Solution: When iron nails are dipped in copper sulphate solution for about 30 minutes, iron nails become
brownish in colour and the blue colour of copper sulphate solution fades and changes to light green.
Fe(s)  CuSO4 (aq)  FeSO4 (aq)  Cu(s)
Iron Copper sulphate Iron sulphate Copper
(Grey) ( Blue) (Light green ) (Brown)

Marks: 1

Question 2. State two characteristic features of carbon which when put together give rise to large number of
carbon compound.
Solution: Two characteristic features of carbon which give rise to a large number of carbon compounds are:
(a) Catenation: Carbon has the unique ability to form bonds with other atoms of carbon giving rise to large
molecules i.e. carbon has a tendency to catenate.
(b) Tetravalency: Since carbon has a valency of four, it is capable of bonding with four other atoms of
carbon or atoms of some other mono-valent element.
Marks: 1

Question 3. Explain why a ray of light passing through the centre of curvature of a concave mirror gets
reflected along the same path.
Solution: A ray of light passing through the centre of curvature of a concave mirror falls on the mirror along
the normal to the reflecting surface. Hence, it gets reflected along the same path following the laws of
reflection.
Marks: 1

www.vedantu.com 1 / 12
Question 4. What is the nature of the image formed by a concave mirror if the magnification produced by
the mirror is +3?
Solution: The nature of the image formed by a concave mirror if the magnification produced by the mirror
is +3 is virtual, erect and magnified.
Marks: 1

Question 5. A charged particle enters at right angles into a uniform magnetic field as shown. What should
be the nature of charge on the particle if it begins to move in a direction pointing vertically out of the page
due to its interaction with the magnetic field?

Solution: Using Fleming's left hand rule we can easily find out that the nature of the charge on the particle
is positive.
Marks: 1

Question 6. Name the part of our eyes that helps us to focus near and distant objects in quick succession.
Solution: Eye lens help us to focus near and distant objects in quick succession.
Marks: 1

Question 7. What happen when an aqueous solution of sodium sulphate reacts with an aqueous solution of
barium chloride? State the physical conditions of reactants in which the reaction between them will not take
place. Write the balanced chemical equation for the reaction and name the type of reaction.
Solution: When an aqueous solution of sodium sulphate reacts with an aqueous solution of barium chloride,
insoluble barium sulphate along with solution of sodium chloride is formed.
If the reactants are in solid state, then reaction will not take place between sodium sulphate and barium
chloride.
Na 2SO4 (aq)  BaCl2 (aq)  BaSO4 (s)  2NaCl(aq)
Sodium Sulphate Barium chloride Barium sulphate Sodium chloride

Reaction between aqueous solution of sodium sulphate and aqueous solution of barium chloride is a double
displacement reaction.
Marks: 2

Question 8.What is the main constituent of biogas? How is biogas obtained from biomass? Write any two
advantages of using this gas.
Solution: The main constituent of biogas is methane.

www.vedantu.com 2 / 12
Biogas is obtained by the anaerobic degradation of biomass in the presence of water.
Two advantages of using biogas are:
1. It burns without smoke and hence does not cause air pollution.
2. It has a high calorific value.
Marks: 2

Question 9. In the figure given below a narrow beam of white light is shown to pass through a triangular
glass prism. After passing through the prism it produces a spectrum XY on a screen.

(a) State the colour seen at X and Y.


(b) Why do different colours of white light bend through different angles with respect to the incident beam
of light?
Solution: (a) Red color will be seen at Y and violet colour will be seen at X.
(b) Different colors of white light travel at different speeds through the glass prism.
Hence, they bend through different angles with respect to the incident beam of light.
Marks: 2

Question 10. What is a solenoid? Draw the pattern of magnetic field lines of a solenoid through which a
steady current flows. What does the pattern of field lines inside the solenoid indicate?
Solution: A coil of many circular turns of insulated copper wire wrapped closely in the shape of a cylinder
is called a solenoid.
Magnetic field lines of a solenoid through which a steady current is flowing:

The field lines inside the solenoid are in the form of parallel straight lines. This indicates that the magnetic
field is the same at all points inside the solenoid. That is, the field is uniform inside the solenoid.
Marks: 2

Question 11. A coil of insulated wire is connected to a galvanometer. What would be seen if a bar magnet
with its north pole towards one face of the coil is
(i) moved quickly towards it,

www.vedantu.com 3 / 12
(ii) moved quickly away from the coil and
(iii) placed near its one face?
Name the phenomena involved.
Solution: (i) A momentary deflection in the galvanometer will be seen, indicating a flow of current in the
circuit.
(ii) A momentary deflection in the galvanometer (but in opposite direction) will be seen, indicating a flow of
current in the opposite direction in the circuit.
(iii) No deflection in the galvanometer will seen, indicating that not current flows in the circuit.
The phenomenon involved is electromagnetic induction.
Marks: 2

Question 12. Mention any four limitations in harnessing wind energy on a large scale.
Solution: Four limitations in harnessing wind energy on a large scale are:
(a) Wind energy farms can be established only at those places where wind blows at least with a speed of 15
km/h for the most part of the year.
(b) There should be some back-up facilities (like storage cells) to take care of the energy needs during a
period when there is no wind.
(c) Establishment of wind energy farms requires large area of land.
(d) Since the tower and blades are exposed to the vagaries of nature like rain, sun, storm and cyclone, they
need a high level of maintenance.
Marks: 2

Question 13. At what distance should an object be placed from a convex lens of focal length 18 cm to
obtain an image at 24 cm from it on the other side. What Will be magnification produced in this case?
Solution: Focal length, f = +18 cm
Image distance, v = +24 cm
Object distance, u = ?
Magnification, m = ?
According to lens formula:
1 1 1
 
v u f
1 1 1
 
u v f
1 1 1
 
u 24 18
1 3 4

u 72
u  72cm
v 24
m   0.33
u 72
Marks: 3

Question 14. No chemical reaction takes place when granules of a solid, A, are mixed with the powder of
another solid, B. However when the mixture is heated, a reaction takes place between its components. One

www.vedantu.com 4 / 12
of the products, C, is a metal and settles down in the molten state while the other product, D floats over it. It
was observed that the reaction is highly exothermic.
(i) Based on the given information make an assumption about A and B and write a chemical equation for the
chemical reaction indicating the conditions of reaction, physical state of reactants and products and thermal
status of reaction.
(ii) Mention any two types of reaction under which above chemical reaction can be classified.
Solution: (i) Based on the given information, solid A can be assumed to be manganese dioxide (MnO2) and
solid B can be assumed to be aluminium powder (Al). When manganese dioxide is heated with aluminium
powder, the following reaction takes place:
3MnO2 (S)  4Al(S) 
Heat
 2Al2O3 (s) 3Mn  l   Heat
A B D C
The reaction is highly exothermic reaction and a lot of heat is evolved.
(ii) (a) Displacement reaction
(b) Exothermic reaction
(c) Redox reaction
Marks: 3

Question 15. Name the functional group of organic compounds that can be hydrogenated. With the help of
suitable example, explain the process of hydrogenation mentioning the conditions of the reaction and any
one change in physical property with the formation of the product. Name any one natural source of organic
compounds that are hydrogenated.
Solution: The functional groups of organic compounds that can be hydrogenated are alkenes and alkynes.
H2C  CH2 
H2
 CH3  CH3
Ethene Nickel Catalyst Ethane
Unsaturated hydrocarbons undergo addition reactions with hydrogen in the presence of catalysts such as
palladium or nickel to give saturated hydrocarbons. During this reaction, unsaturated compounds like
vegetable oils which are in liquid state are converted to animal fats in solid state. Vegetable oil is an
example of natural source of organic compound that are hydrogenated.
Marks: 3

Question 16. Atoms of eight elements A, B, D, E, F, G and H have the same number of electronic shells but
different number of electrons in their outermost shell. It was found that elements A and G combine to form
an ionic compound. This compound is added in a small amount to almost all vegetable dishes during
cooking. Oxides of elements A and B are basic in nature while those of E and F are acidic. The oxide of D is
almost neutral.
Based on the above information answer the following questions:
(i) To which group or period of the periodic table, do the listed elements belong?
(ii) What would be the nature of compound formed by a combination of elements B and F?
(iii) Which two of these elements could definitely be metals?
(iv) Which one of the eight elements is most likely to be found in gaseous state at room temperature?
(v) If the number of electrons in the outermost shell of element C and G be 3 and 7 respectively, write the
formula of the compound formed by the combination of C and G.

www.vedantu.com 5 / 12
Solution: (i)

(ii) Nature of the compound formed by combination of element B and F is ionic.


(iii) Elements A and B are definitely metals.
(iv) Element H belonging to group 18 is most likely to be found in gaseous state at room temperature.
(v) Formula of the compound formed by combination of C and G is CG 3.

Marks: 3

Question 17. Write the name and symbols of two most reactive metals belonging to group I of the periodic
table. Explain by drawing electronic structure how either one of the two metals reacts with a halogen. With
which name is the bond formed between these elements known and what is the class of the compound so
formed known? State any four physical properties of such compounds.
OR
What is meant by refining of metals? Name the most widely used method of refining impure metals
produced by various reduction processes. Describe with the help of a labeled diagram how this method may
be used for refining of copper.
Solution: Names and symbols of the two most reactive metals belonging to group I of the periodic table:

Formation of sodium chloride:


Na  Na   e
2,8,1 2,8
(Sodium cation)

www.vedantu.com 6 / 12
Cl  e  Cl
2,8,7 2,8,8
(Chloride anion)

Sodium and chloride ions, being oppositely charged are held by strong electrostatic forces of attraction.
Bond formed between sodium and chloride ion is ionic bond.
The class of compounds formed by the transfer of electrons from a metal to a non-metal is known as ionic
compounds or electrovalent compounds.
Physical properties of ionic or electrovalent compounds:
1. Ionic compounds are solids and are somewhat hard.
2. Ionic compounds have high melting and boiling points.
3. Ionic compounds are generally soluble in water and insoluble in solvents such as kerosene, petrol, etc.
4. Ionic compounds conduct electricity in aqueous solution and in molten state. They do not conduct
electricity in solid state.
OR
Process of obtaining pure metal from its impure form is called refining of metals. The most widely used
method for refining impure metals is electrolytic refining.
Electrolytic refining of copper:

In electrolytic refining of copper, electrolyte is a solution of acidified copper sulphate. Anode is made up of
impure copper. Cathode is made up of a strip of pure copper metal.
On passing current through the electrolyte, pure copper metal from the anode dissolves into the electrolyte
i.e., acidified copper sulphate.
Cu  Cu 2  2e
(Impure
copper)

At cathode: An equivalent amount of pure metal from the electrolyte is deposited on the cathode.
Cu 2  2e  Cu
(Pure
copper)

The soluble impurities go into the solution, whereas, the insoluble impurities settle down at the bottom of
the anode and are known as anode mud.
Marks: 5

Question 18. Derive the expression for the heat produced due to a current ‘I’ flowing for a time interval ‘t’
through a resistor ‘R’ having a potential difference ‘V’ across its ends. With which name is the relation

www.vedantu.com 7 / 12
known? How much heat will an instrument of 12 W produce in one minute if it is connected to a battery of
12 V?
OR
Explain with the help of a labeled circuit diagram how you will find the resistance of a combination of three
resistors, of resistance R1, R2 and R3 joined in parallel. Also mention how you will connect the ammeter and
the voltmeter in the circuit when measuring the current in the circuit and the potential difference across one
of the three resistors of the combination.
Solution: Let us take a resistor of resistance R. Let the current flowing through this resistor is equal to I and
the potential difference across it is equal to V. Suppose in time t, Q amount of charge flows through the
resistor
Work done in moving this charge, W = VQ … (1)
According to the definition of electric current,
Q
I
t
Q  I t
Putting this in equation (1),
W=V×I×t
This work done is dissipated as heat.
Hence, heat produced, H = W = VIt
H = VIt … (2)
According to Ohm’s law, V = IR.
Putting this in equation (2),
H = IR × It
H = I2Rt
This relation is known as Joule’s law of heating
Numerical:
Power, P = 12 W
Potential difference, V = 12 volt
Time duration, t = 1 min = 60 s
H
P
t
H  Pt
= 12 W × 60 s
= 720 J
The heat generated by the instrument is 720 J.
OR

www.vedantu.com 8 / 12
The given figure shows a circuit consisting of three resistors R 1, R2 and R3 connected in parallel. The total
current in the circuit (I) gets divided among the three resistors as I 1, I2 and I3.
Thus, I = I1 + I2 + I3 ----- (1)
Applying Ohm’s law for each resistor,
V 
I1  
R1 
V 
I2   ----- (2)
R2 
V 
I3  
R3 
Let the equivalent resistance of the circuit be Req.
Applying Ohm’s law for the equivalent circuit,
V
I ----- (3)
Req
Using eqns. (1), (2) and (3),
V V V V
  
Req R1 R2 R3
1 1 1 1
  
Req R1 R2 R3
This is the expression for the equivalent resistance of a parallel combination of three resistances. An
ammeter has to be connected in series with the combination of all these resistors so that the current passing
through the ammeter is equal to the total current through the circuit.
The voltmeter has to be connected in parallel to that resistor across which the potential difference has to be
measured.
Marks: 5

SECTION-B

Question 19. Name the green dot like structures in some cells observed by a student when a leaf peel was
viewed under a microscope. What is this green colour due to?
Solution: The green dot-like structures are chloroplasts. This green colour is due to the presence of
chlorophyll.
Marks: 1

Question 20. How is the spinal cord protected in the human body?
Solution: The spinal cord is protected by the vertebral column or backbone.
Marks: 1

Question 21. How is the increasing demand for energy adversely affecting our environment?
Solution: Our increasing demand for energy is depleting our natural resources and polluting the
environment in one or the other way.
Marks: 1

www.vedantu.com 9 / 12
Question 22. What are hormones? Name the hormone secreted by thyroid and state its function?
Solution: A hormone is a chemical compound synthesized by a group of cells or endocrine glands that
affect cells in other parts of the body and is also used for control and coordination in the organisms.
Thyroid gland secretes the hormone thyroxin.
Thyroxin regulates carbohydrate, protein and fat metabolism in the body so as to provide the correct balance
for growth.
Marks: 2

Question 23. With the help of diagrams show the different stages of binary fission in Amoeba.
Solution:

Marks: 2

Question 24. Given one example each of characters that are inherited and the ones that are acquired in
humans. Mention the difference between the inherited and the acquired characters.
Solution: Example of inherited trait - Shape of the eye or hair colour.
Example of acquired trait – Building of muscles while exercising.
Inherited characters affect the DNA of germ cells and hence can be passed on to the future generations.
Acquired characters do not cause changes in DNA of the germ cells and hence cannot be passed on to future
generations.
Marks: 2

Question 25. Write the full form of DNA. Name the part of the cell where it is located. Explain its role in
the process of reproduction of the cell.
Solution: DNA – Deoxyribonucleic acid.
DNA is present in the nucleus of the cell.
DNA in the cell nucleus is the information source for making proteins and is thereby, responsible for
inheritance of features. A basic event in reproduction process is DNA copying, accompanied by the creation
of an additional cellular apparatus after which the DNA copies separate, each with its own cellular
apparatus.
The consistency of DNA copying during reproduction is important for the maintenance of body design
features. Variations occur in the DNA copying reactions during reproduction, due to which the surviving
cells are similar to, but subtly different from each other. This inbuilt tendency for variation during
reproduction is the basis for evolution.
Marks: 3

www.vedantu.com 10 / 12
Question 26. Explain the phenomenon of “biological magnification.” How does it affect organisms
belonging to different tropic levels particularly the tertiary consumers?
Solution: When non-biodegradable substances such as pesticides, enter the food chain, they get
accumulated progressively at each trophic level. This results in a cumulative increase in the concentration of
the substance in successively higher trophic levels of the food chain. This phenomenon is known as
biological magnification.
For example - Pesticides entering our food chain through soil or water are not degradable and hence gets
progressively accumulated at each trophic level, with maximum accumulation in human bodies.
Biomagnification of a toxic substance has the potential to cause harm to organisms, particularly to the
tertiary consumers. This is because tertiary consumers occupy the top level in a food chain and hence
maximum concentration of such chemicals gets accumulated in their bodies.
Marks: 3

Question 27. Explain the process of digestion of food in mouth, stomach and small intestine in human body.
OR
(a) List the three events that occur during the process of photosynthesis. Explain the role of stomata in this
process.
(b) Describe an experiment to show that “sunlight is essential for photosynthesis.”
Solution: i. Mouth - In mouth, large food pieces are crushed with the help of our teeth and mixed with saliva
secreted by the salivary glands, using the tongue. Salivary amylase, the enzyme present in saliva, breaks
down starch to give sugar.
ii. Stomach - The muscular walls of the stomach help in mixing the food thoroughly with the digestive juices
secreted by the gastric glands present in the wall of the stomach. These glands release hydrochloric acid, a
protein digesting enzyme called pepsin, and mucus, which protects the inner lining of the stomach. The
hydrochloric acid creates an acidic medium which facilitates the action of the enzyme pepsin.
iii. Small intestine - The small intestine is the site of the complete digestion of carbohydrates, proteins and
fats. It receives the secretions of the liver and pancreas for this purpose.
Bile juice from liver makes the acidic food coming from stomach alkaline for facilitating the action of
pancreatic enzymes. Bile also emulsifies fats so as to increase the efficiency of enzyme action.
The pancreas secretes pancreatic juice which contains enzymes like trypsin for digesting proteins and lipase
for breaking down emulsified fats.
The walls of the small intestine contain glands which secrete intestinal juice. The enzymes present in it
finally convert the proteins into amino acids, complex carbohydrates into glucose and fats into fatty acids
and glycerol.

OR

(a) The three events that occur during the process of photosynthesis are:
(i) Absorption of light energy by chlorophyll.
(ii) Conversion of light energy to chemical energy and splitting of water molecules into hydrogen and
oxygen.
(iii) Reduction of carbon dioxide to carbohydrates.
Stomata help in exchange of gases (carbon dioxide and oxygen) for the purpose of photosynthesis.

www.vedantu.com 11 / 12
(b)

Experimental set-up to show that light is essential for photosynthesis:


i. Keep a potted plant in a dark room for three days so that all the starch gets used up.
ii. Now cover one half of a leaf of this plant with black paper or metal foil on both sides.
iii. Then keep the plant in sunlight for about six hours.
iv. Pluck the leaf which was half covered and remove the paper or foil.
v. Mark the covered area.
vi. Dip this leaf in boiling water for a few minutes.
vii. Then immerse it in a beaker containing alcohol.
viii. Carefully place this beaker in a water-bath and heat till the alcohol begins to boil.
ix. The leaf gets decolourised.
x. Now, dip the leaf in a dilute solution of iodine for a few minutes.
xi. Take out the leaf and rinse off the iodine solution. Observe the colour of the leaf.
The part containing starch will be turned blue-black by iodine.
You will find that the portion of the leaf exposed to sunlight will turn blue-black whereas the covered half of
the leaf remains colourless. This is because the covered part did not receive sunlight and hence could not
form carbohydrates.
This proves that light is essential for photosynthesis.
Marks: 5

www.vedantu.com 12 / 12
CBSE-XII-2017 EXAMINATI CBSE – X – 2011 EXAMINATION

SCIENCE
SET-1 Paper & Solution
Time: 3 Hrs. Max. Marks: 80

General Instructions:
1. The question paper comprises of two Sections, A and B. You are to attempt both the sections.
2. All questions are compulsory.
There is no overall choice. However, internal choice has been provided in all the three questions of five marks category. Only
one option in such question is to be attempted.
3. All questions of Section A and all questions of Section B are to be attempted separately.
4. Question numbers 1 to 4 in Section A are one-mark questions. These are to be answered in one word or in one sentence.
5. Question numbers 5 to 13 in Section A are two-mark questions. These are to be answered in about 30 words each.
6. Question numbers 14 to 22 in Section A are three-mark questions. These are to be answered in about 50 words each.
7. Question numbers 23 to 25 in Section A are five-mark questions. These are to be answered in about 70 words each.
8. Question numbers 26 to 41 in Section B are multiple choice questions based on practical skills. Each question is a one-mark
question. You are to select one most appropriate response out of the four provided to you.

SECTION A

Question 1. Why is it necessary to conserve our environment?


Solution: Conservation of environment is required for preventing damage to the environment and depletion
of natural resources.
Marks: 1

Question 2. Distinguish between biodegradable and non-biodegradable wastes.


Solution:
Biodegradable wastes Non-biodegradable wastes
Substances which are easily decomposed by Substances which are not decomposed by
microorganisms are called biodegradable wastes. microorganisms are called non-biodegradable wastes.

Marks: 1

Question 3. What will be the colour of scattered sunlight when the size of the scattering particles is
relatively large?
Solution: If the size of the scattering particles is relatively large, then the scattered light will appear white.
Marks: 1

Question 4. Draw the structure of Butanone molecule, CH3COC2H5.


Solution: Structure of butanone, CH3COC2H5

Marks: 1

www.vedantu.com 1 / 14
Question 5. Explain with the help of a diagram, how we are able to observe the sunrise about two minutes
before the Sun gets above the horizon.
Solution: Sunrise takes place when the sun is just above the horizon. But due to refraction of sunlight
caused by the atmosphere, we can see the rising sun about 2 minutes before it is actually above the horizon.
This happens because when the sun is slightly below the horizon, the sun’s light coming from less dense air
to more dense air is refracted downwards as it passes through the atmosphere. Because of this atmospheric
refraction, the sun appears to be raised above the horizon when actually it is slightly below the horizon.

Marks: 2

Question 6. List any four reasons for vegetative propagation being practised in the growth of some type of
plants.
Solution: Reasons for vegetative propagation:
i. It is done for plants which have lost the capacity to produce seeds.
ii. To produce plants which are genetically similar to the parent plant.
iii. It helps in producing those plants which either produce very few seeds or produce such seeds which are
not viable.
iv. It can be used to produce plants which reach maturity and produce fruits and seeds faster.
Marks: 2

Question 7. State the role of


i. Seminal vesicle
ii. Prostate gland in the human body.
Solution: i. Seminal vesicles - secrete alkaline secretions which lower the pH of semen and provide
nourishment.
ii. Prostate Gland - secretions of these glands keep the sperm active and mobile.
Marks: 2

Question 8. List any four disadvantages of using fossil fuels for the production of energy.
Solution: Disadvantages of using fossil fuels for the production of energy:
i. Burning of fossil fuels (e.g. coal and petroleum products) causes air pollution.
ii. The oxides of carbon, nitrogen and sulphur which are released on burning fossil fuels are acidic oxides.
These cause acid rain which adversely affects our water and soil resources.
iii. Green house gases like carbon dioxide released during the combustion of fossil fuels enhances the
process of global warming.

www.vedantu.com 2 / 14
iv. Fossil fuels were formed over millions of years ago and have limited reserves. If we were to continue
consuming these sources at such alarming rates, we would soon run out of energy.
Marks: 2

Question 9. Give two examples for each of the following:


i. Renewable sources or energy
ii. Non-renewable sources of energy
Solution: i. Solar energy and wind energy are the renewable sources of energy.
ii. Fossil fuels and uranium are the non-renewable sources of energy.
Marks: 2

Question 10. How does the metallic character of elements change along a period of the periodic table from
the left to the right and why?
Solution: Metallic character decreases from left to right along a period of the periodic table because on
moving from left to right, size of the atoms decreases and nuclear charge increases. Hence, the tendency to
release electrons decreases. Thus, the electropositive character decreases.
Marks: 2

Question 11. In the modern periodic table, the element calcium (atomic number = 20) is surrounded by
elements with atomic numbers 12, 19, 21 and 38. Which of these elements has physical and chemical
properties resembling those of calcium and why?
Solution: Ca: Electronic configuration is: 2,8,8,2
The physical and chemical properties of elements with atomic number 12 and 38 will resemble those of
calcium.
This is because they all belong to the second group and all of them have two electrons in the valence shell.
Marks: 2

Question 12. State any four characteristics of the image of the objects formed by a plane mirror.
Solution: Four characteristics of images formed by a plane mirror are:
i. The image formed by a plane mirror is always virtual.
ii. The image formed by a plane mirror is always erect.
iii. Size of the image is same as the size of the object and the image is laterally inverted.
iv. The image formed by a plane mirror is at the same distance behind the mirror as object is in front of it.
Marks: 2

Question 13. Draw a diagram to show dispersion of white light by a glass prism. What is the cause of this
dispersion?
Solution:

www.vedantu.com 3 / 14
Different colours of white light bend through different angles with respect to the incident ray, as they pass
through a prism. Thus the rays of each colour emerge along different paths and become distinct. It is the
band of distinct colours that we see in a spectrum.
Marks: 2

Question 14. (a) What is meant by the power of accommodation of an eye?


(b) A person with a myopic eye cannot see objects beyond 1.2 m directly. What should be the type of the
corrective lens used? What would be its power?
Solution: (a) The process by which the ciliary muscles change the focal length of an eye lens to focus
distant or near objects clearly on the retina is called the accommodation of the eye. The ability of the eye to
do this is called the power of accommodation of the eye.
(b) The person is able to see nearby objects clearly, but he is unable to see objects beyond 1.2 m. This
happens because the image of an object beyond 1.2 m is formed in front of the retina and not at the retina, as
shown in the given figure.

To correct this defect of vision, the person must use a concave lens. The concave lens will bring the image
back to the retina as shown in the given figure.

Focal length of the corrective lens used = − (Distance of far point of the myopic eye) = −1.2 m
1 1
Power of the lens    0.83D
focal length 12
Marks: 3

Question 15. What does HIV stand for? Is AIDS an infectious disease? List any four modes of spreading
AIDS.
Solution: HIV stands for Human Immuno Deficiency Virus.
Yes, HIV is an infectious agent which spreads through sexual contact.
Modes by which can HIV spread:
i. Through sexual contact.
ii. From pregnant mothers to the growing foetus.
iii. Through transfusion of infected blood.
iv. By sharing of needles or syringes.
Marks: 3

www.vedantu.com 4 / 14
Question 16. Describe any three ways in which individuals with a particular trait may increase in
population.
Solution: Different ways in which individuals with a particular trait may increase in population are
variation, natural selection and genetic drift.
Variation: Variation is defined as the occurrence of differences among the individuals. No two individuals
are exactly alike. Variations arising during the process of reproduction can be inherited and lead to increased
survival of the individuals.
Natural selection: It results in adaptations in population to fit their environment better. Thus, natural
selection directs evolution in the population of a particular species.
Genetic drift: The change in the frequency of certain genes in a population over generations is called genetic
drift.
Marks: 3

Question 17. State the evidence we have for the origin of life from inanimate matter.
Solution: J.B.S. Haldane suggested that life must have developed from the simple inorganic molecules
which were present on Earth soon after it was formed. He speculated that the conditions on Earth at that time
could have given rise to more complex organic molecules which were necessary for life. The first primitive
organisms would arise from further chemical synthesis. Later on, Stanely L. Miller and Harold C. Urey
conducted experiments to understand the origin of organic molecules. They created an atmosphere similar to
that thought to exist on early Earth (this had molecules like ammonia, methane and hydrogen sulphide, but
no oxygen) over water. This was maintained at a temperature just below 100 °C and sparks were passed
through the mixture of gases to simulate lightning. At the end of a week, 15% of the carbon (from methane)
had been converted to simple compounds of carbon including amino acids which make up protein
molecules. This is considered as evidence for origin of life on the Earth from inanimate matter.
Marks: 3

Question 18. Give an example of body characteristics used to determine how close two species are in terms
of evolution and explain it.
Solution: Homologous organs, analogous organs and vestigial organs help to identify evolutionary
relationships.
Homologous organs are those organs which have similar basic structure but have been modified to perform
different functions. Example - forelimbs of reptiles, frog, lizard, bird and humans are homologous organs.
Such homologous characteristics help to identify an evolutionary relationship between apparently different
species.
Analogous organs are those organs which are different in basic structure but perform the same function.
Example - wings of bird and wings of bat.
Vestigial organs are certain reduced and non-functional organs present in some organisms. Example -
vermiform appendix in human body.
Marks: 3

Question 19. Write chemical equations to show what happens when:


i. Ethanol is heated with concentrated sulphuric acid at 443 K.
ii. Ethanol reacts with ethanoic acid in the presence of an acid acting as a catalyst.
iii. An ester reacts with a base.
Solution: i. CH3CH2OH 
conc.sulphuric acid at 443 K
 CH2 = CH2 + H2O

www.vedantu.com 5 / 14
ii. CH3CH2OH + CH3COOH  
acid
 CH3COOCH2CH3 + H2O
iii. CH3COOCH2CH3 + NaOH 
 CH3COONa + CH3CH2OH
Marks: 3

Question 20. The atomic number of an element is 16. Predict


i. the number of valence electrons in its atom
ii. its valency
iii. its group number
iv. whether it is a metal or a non-metal
v. the nature of oxide formed by it
vi. the formula of its chloride
Solution: Atomic number = 16
Electronic configuration = 2, 8, 6
i. Number of valence electrons in its atom = 6
ii. Valency = 2
iii. Group number =16
iv. It is a non-metal
v. Acidic oxide
vi. XCl2
Marks: 3

Question 21. An object is placed between infinity and the pole of a convex mirror. Draw a ray diagram and
also state the position, the relative size and the nature of the image formed.
Solution:

An object is placed between infinity and the pole of a convex mirror, the image formed is:
i. Behind the mirror at focus (F),
ii. Virtual and erect,
iii. Highly diminished
Marks: 3

Question 22. What is the principle of reversibility of light? Show that the incident ray of light is parallel to
the emergent ray of light when light falls obliquely on a side of a rectangular glass slab.
Solution: The principle of reversibility of light states that light will follow exactly the same path if its
direction of travel is reversed.
When light falls obliquely on a rectangular glass slab, the incident ray is parallel to the emergent ray; as
shown in the figure. Angle of incidence is equal to the angle of emergence.

www.vedantu.com 6 / 14
Marks: 3

Question 23. With the help of suitable diagrams, explain the various steps of budding in Hydra.
OR
What is binary fission in organisms? With the help of suitable diagrams, describe the mode of reproduction
in Amoeba.
Solution: In Hydra, a bud develops as an outgrowth due to repeated cell divisions at one specific site. These
buds develop into tiny individuals and when fully mature, detach from the parent body and become new
independent individuals.

OR
Binary fission is an asexual method of reproduction. Amoeba reproduces by this method. During this
process, nuclear division takes place first, followed by the appearance of a constriction in the cell
membrane, which gradually increases inwards and divides the cytoplasm into two parts. Finally, two
daughter organisms are formed.

www.vedantu.com 7 / 14
Marks: 5

Question 24. (a) State two properties of carbon which lead to a very large number of carbon compounds.
(b) Why does micelle formation take place when soap is added to water? Why are micelles not formed when
soap is added to ethanol?
OR
Explain isomerism. State any four characteristics of isomers. Draw the structures of possible isomers of
butane, C4H10.
Solution: (a) Two properties of carbon which lead to a very large number of carbon compounds are:
i. Tetravalency: Carbon has valency 4 i.e., it is tetravalent. Hence, it is capable of bonding with four other
atoms of other monovalent elements.
ii. Catenation: Carbon has the unique ability to form bonds with other atoms of carbon to form long chains.
(b) A soap molecule has two parts –one hydrophobic part and the other hydrophilic part. When added to
water, the hydrophobic part arranges itself towards the dirt and the hydrophilic end arranges itself towards
the water.
Micelle formation does not take place when soap is added to ethanol because the hydrophobic part of soap
molecules is soluble in ethanol.
OR
Isomers are compounds with same molecular formula but different structures.
Four characteristics of isomers:
i. Isomers have different physical properties.
ii. Isomers may have same or different chemical properties.
iii. All isomers have the same number of atoms.
iv. Isomers have different structural arrangements.
Isomers of butane, C4H10.

Marks: 5

www.vedantu.com 8 / 14
Question 25. (a) What is meant by ‘power of a lens’?
(b) State and define the S.I. unit of power of a lens.
(c) A convex lens of focal length 25 cm and a concave lens of focal length 10 cm are placed in close contact
with each other. Calculate the lens power of this combination.
OR
(a) Draw a ray diagram to show the formation of image of an object placed between infinity and the optical
centre of a concave lens.
(b) A concave lens of focal length 15 cm forms an image 10 cm from the lens. Calculate
i. The distance of the object from the lens.
ii. The magnification for the image formed
iii. The nature of the image formed.
Solution: (a) The degree of convergence or divergence of light rays achieved by a lens is expressed in terms
of its power. The power of a lens is defined as the reciprocal of its focal length. The power P of a lens of
focal length f is given by:
1
P
f (in meters)
(b) The S.I. unit of power of a lens is ‘dioptre’.
1 dioptre is the power of a lens whose focal length is 1 metre.
(c) Focal length of convex lens = +25 cm
100
Power of convex lens, P1 = =4D
25
Focal length of concave lens = −10 cm
100
Power of concave lens, P₂ = = –10 D
10
Hence, power of this combination, P = P1 +P2 = (4 D) + (−10 D) = −6 D
OR
(a) Ray diagram showing the formation of image of an object placed between infinity and optical centre of a
concave lens:

(b) A concave lens always forms a virtual, erect image on the same side of the object.
Focal length of concave lens, f = −15 cm
Image distance, v = −10 cm
i. Let 'u' be the object distance; then using lens formula:
1 1 1
 
f v u
1 1 1
or,  
u v f
Substituting the values,

www.vedantu.com 9 / 14
1  1   1   1 
       
u  10   15   30 
Or, u = −30 cm = −0.3 m
Thus, object distance is 30 cm
v 10 1
ii. Magnification, m     0.33
u 30 3
iii. The positive sign shows that the image is erect and virtual. The image is one-third the size of the object.
Marks: 5

SECTION B

Question 26. The shape of yeast cells is


(a) Only spherical
(b) Only oval.
(c) Irregular
(d) Both oval and spherical.
Solution: (b) Only oval.
Yeast cells are usually oval.
Marks: 1

Question 27. A student added acetic acid to test tubes I, II, III and IV containing the labeled substances and
then brought a burning splinter near the mouth of each test tube.

The splinter would be extinguished when brought near the mouth of test tube.
(a) I
(b) II
(c) III
(d) IV
Solution: (c) III
Sodium bicarbonate reacts with acetic acid to release carbon dioxide gas which is a nonsupporter of
combustion.
Marks: 1

Question 28. Acetic add reacts with solid sodium hydrogen carbonate,
(a) Slowly forming no gas
(b) Vigorously with effervescence
(c) Slowly without effervescence
(d) Vigorously without gas formation

www.vedantu.com 10 / 14
Solution: (b) Vigorously with effervescence
Acetic acid reacts with solid sodium hydrogen carbonate vigorously and effervescence is produced due to
evolution of CO2 gas.
Marks: 1

Question 29. Vapours of acetic acid smell:


(a) Pungent like vinegar
(b) Sweet like rose
(c) Suffocating like sulphur dioxide
(d) Odorless like water
Solution: (a) Pungent like vinegar
Vapours of acetic acid smell pungent like vinegar.
Marks: 1

Question 30. A clean aluminium foil was placed in an aqueous solution of zinc sulphate. When the
aluminium foil was taken out of the zinc sulphate solution after 15 minutes, its surface was found to be
coated with a silvery grey deposit. From the above observation it can be concluded that:
(a) Aluminium is more reactive than zinc
(b) Zinc is more reactive than aluminium
(c) Zinc and aluminium both are equally reactive
(d) Zinc and aluminium both are non-reactive
Solution: (a) Aluminium is more reactive than zinc
Aluminium is more reactive than zinc and is hence able to displace zinc from its solution.
Marks: 1

Question 31. The colour of raisins as used in the experiment, ‘To determine the percentage of water
absorbed by raisins’, was
(a) White
(b) Yellow
(c) Dark brown
(d) Pink
Solution: (c) Dark brown
The raisins are dark brown in colour.
Marks: 1

Question 32. Following are the steps involved in the experiment- 'To determine the percentage of water
absorbed by raisins'. They are not in proper sequence.
I. Soak the raisins in fresh water.
II. Weight dry raisins.
III. Weigh soaked raisins.
IV. Wipe out soaked raisins.
The correct sequence of steps is
(a) I, II, III, IV
(b) II, I, IV, III
(c) II, I, III, IV

www.vedantu.com 11 / 14
(d) I, II, IV, III
Solution: (b) II, I, IV, III
The correct sequence is- II, I, IV, III
Marks: 1

Question 33. During the course of an experiment, to determine the percentage of water absorbed by raisins,
the raisins are weighed
(a) Every half an hour.
(b) Every hour.
(c) Once- only after completing the experiment.
(d) Two times- Before soaking and after soaking for three hours.
Solution: (d) Two times− Before soaking and after soaking for three hours.
Raisins are weighed two times– before soaking and after soaking for three hours.
Marks: 1

Question 34. The given figures illustrate binger fission in Amoeba in improper order.

The correct order is


(a) III, IV, II, I
(b) IV, III, II, I
(c) II, III, IV, I
(d) I, III, IV, II
Solution: (c) II, III, IV, I
The correct order of binary fission is- II, III, IV, I
Marks: 1

Question 35. The steps involved in observing a slide under a microscope are given below. They are not in
proper sequence.
I. Focus the object under high power of the microscope.
II. Place the slide on the stage of the microscope.
III. Arrange the mirror to reflect maximum light to the slide.
IV. Focus the object under low power of the microscope.
The proper sequence of steps is
(a) II, III, IV, I
(b) I, II, III, IV
(c) IV, III, II, I
(d) III, I, II, IV
Solution: (a) II, III, IV, I
The steps to observe a slide under the microscope are- II, III, IV, I
Marks: 1

www.vedantu.com 12 / 14
Question 36. In which diagram the angle of refraction r has been correctly depicted?

(a) I
(b) II
(c) III
(d) IV
Solution: (d) IV
Angle of refraction is measured with respect to the normal at the first point of incidence.
Marks: 1

Question 37. For a ray of light passing through a glass slab, the lateral displacement was correctly measured
as:

(a) AB
(b) PQ
(c) CD
(d) PR
Solution: (c) CD
Lateral displacement is the sideways shift of the emergent ray from the direction of the incident ray.
Marks: 1

Question 38. Iron nails were dipped in an aqueous solution of copper sulphate. After about 30 minutes, it
was observed that the colour of the solution changed from
(a) Colorless to light green.
(b) Blue to light green.
(c) Blue to colourless.
(d) Green to blue.
Solution: (b) Blue to light green
The blue coloured copper sulphate solution changes to light green iron sulphate solution after displacement
of copper by iron from copper sulphate solution.
Marks: 1

www.vedantu.com 13 / 14
Question 39. To find the focal length of a concave mirror, Sita should choose which one of the following
(a) A mirror holder and screen holder
(b) A screen holder and a scale
(c) A mirror holder, a screen holder and a scale
(d) A screen, a mirror, holders for them and a scale
Solution: (d) A screen, a mirror, holders for them and a scale
A screen, a mirror, holders for them and scale are needed to find the focal length of a concave mirror.
Marks: 1

Question 40. By using a convex lens, a student obtained a sharp image of his classroom window grill on a
screen. In which direction should he move the lens to focus a distant tree instead of the grill?
(a) Towards the screen
(b) Away from the screen
(c) Very far away from the screen
(d) Behind the screen
Solution: (a) Towards the screen
The lens should be moved towards the screen because the distant tree can be considered an object at infinity
whose image will be formed at the focus, while earlier the image of nearer grill was formed at a distance
farther than the focal length.
Marks: 1

Question 41. To determine the focal length of a convex lens by obtaining a sharp image of a distant object,
the following steps were suggested which are not in proper sequence.
I. Hold the lens between the object and the screen.
II. Adjust the position of the lens to form a sharp image.
III. Select a suitable distant object.
IV. Measure the distance between the lens and the screen.
The correct sequence of steps to determine the focal length of the lens is
(a) III, I, II, IV
(b) III, I, IV, II
(c) III, IV, II, I
(d) I, II, III, IV
Solution: (a) III, I, II, IV
The proper sequence to determine the focal length of a convex lens is:
III - Select a suitable distant object.
I - Hold the lens between the object and the screen.
II - Adjust the position of the lens to form a sharp image.
IV - Measure the distance between the lens and the screen.
Marks: 1

www.vedantu.com 14 / 14
CBSE-XII-2017 EXAMINATI CBSE – X – 2012 EXAMINATION

SCIENCE
SET-3 Paper & Solution
Time: 3 Hrs. Max. Marks: 80

General Instructions:
1. The question paper comprises two Sections, A and B. You are to attempt both the sections.
2. All questions are compulsory.
3. There is no overall choice. However, internal choice has been provided in all the three questions of five marks category. Only
one option in such question is to be attempted.
4. All questions of Section A and all questions of Section B are to be attempted separately.
5. Question numbers 1 to 4 in Section A are one-mark questions. These are to be answered in one word or in one sentence.
6. Question numbers 5 to 13 in Section A are two-mark questions. These are to be answered in about 30 words each.
7. Question numbers 14 to 22 in Section A are three-mark questions. These are to be answered in about 50 words each.
8. Question numbers 23 to 25 in Section A are five-mark questions. These are to be answered in about 70 words each.
9. Question numbers 26 to 41 in Section B are multiple choice questions based on practical skills. Each question is a one-mark
question. You are to select one most appropriate response out of the four provided to you.

SECTION A

Question 1. Name the functional group present in each of the following organic compounds:
i. CH3COCH3
ii. C2H5COOH
Solution: i. Ketone
ii. Carboxylic acid
Marks: 1

Question 2. Which phenomenon is responsible for making the path of light visible?
Solution: Tyndall effect is the phenomenon responsible for making the path of light visible.
Marks: 1

Question 3. Which class of carbon compounds are responsible for the depletion of ozone layer at the higher
level of the atmosphere?
Solution: Chlorofluorocarbons (CFCs) are responsible for the depletion of the ozone layer.
Marks: 1

Question 4. Select two non-biodegradable substances from the following waste generated in a kitchen:
Spoilt food, paper bags, milk bags, vegetable peels, tin cans, used tea leaves.
Solution: Milk bags and tin cans.
Marks: 1

Question 5. Define the term puberty. List two changes observed in girls at the time of puberty.
Solution: It is the age at which the reproductive system becomes functional in human beings. The changes
observed in girls at the time of puberty are:
i. Breast enlargement.
ii. Fat tissue starts depositing in areas like the breasts, hips, buttocks and thighs.
Marks: 2

www.vedantu.com 1 / 18
Question 6. What is meant by asexual reproduction? List any two of its different forms.
Solution: It is a mode of reproduction in which new individuals are produced from a single parent without
the involvement of fusion of gametes. The two forms of asexual reproduction are - budding and binary
fission.
Marks: 2

Question 7. What are the advantages of water stored in the ground?


Solution: Advantages of water stored in the ground:
i. Stored ground water does not evaporate.
ii. It does not provide breeding grounds for mosquitoes like stagnant water collected in ponds or artificial
lakes.
iii. It provides moisture for vegetation over a wide area.
iv. Ground water stored does not get contaminated by human and animal wastes.
Marks: 2

Question 8. “Burning fossil fuels is a cause of global warming.” Justify this statement.
Solution: Fossil fuels like coal and petroleum are huge reservoirs of carbon and its compounds. On burning
fossil fuels, huge reservoirs of carbon present in fossil fuels get converted to carbon dioxide and go into air.
The amount of carbon dioxide thus increases in the atmosphere which leads to an increased green house
effect leading to excessive heating of the Earth i.e., global warming.
Marks: 2

Question 9. When we place a glass prism in the path of a narrow beam of white light, a spectrum is
obtained. What happens when a second identical prism is placed in an inverted position with respect to the
first prism? Draw a labelled ray diagram to illustrate it.
Solution: When a second identical prism is placed in an inverted position with respect to the first prism,
recombination of the spectrum occurs and it forms white light again.

Marks: 2

Question 10. List four properties of the image formed by a concave mirror when an object is placed
between the focus and pole of the mirror.
Solution: When an object is placed between the focus and the pole of a concave mirror, the image formed
is:
i. Virtual
ii. Enlarged
iii. Behind the mirror
iv. Erect
Marks: 2

www.vedantu.com 2 / 18
Question 11. An element 'M' has atomic number 12.
(a) Write its electronic configuration.
(b) State the group to which 'M' belongs.
(c) Is 'M' a metal or a non-metal?
(d) Write the formula of its oxide.
Solution: (a) The electronic configuration of M is 2, 8, 2
(b) M belongs to the 2nd group
(c) M is a metal
(d) MO
Marks: 2

Question 12. How can the valency of an element be determined if its electronic configuration is known?
What will be the valency of an element of atomic number 9?
Solution: An element's valency is determined by the number of electrons in its outer shell. Hence the
number of valence electrons obtained from the electronic configuration of the element gives the valency i.e.,
the number of electrons lost, gained or shared by the element to attain a noble gas configuration.
The valency of an element of atomic number 9 would be 1 since the number of valence electrons in its outer
shell is 7 so it needs only one electron to attain the noble gas configuration.
Marks: 2

Question 13. A star at times appears bright and at times fainter. What is this effect called? State the reason
for this effect.
Solution: This effect is called twinkling effect. Atmospheric refraction is the reason behind this effect.
Since the stars are very far, they can be taken as point sized objects. As the path of rays coming from the
stars keep varying due to atmospheric refraction, the apparent position of the stars fluctuates and the amount
of light entering our eyes also varies resulting in a twinkling effect.
Marks: 2

Question 14. F, Cl and Br are elements each having seven valence electrons. Which of these:
i. has the largest atomic radius
ii. is most reactive?
Justify your answer stating reason for each.
Solution: i. F, Cl and Br all are in the same group and thus have the same effective nuclear charge. Br has
the largest atomic radius among all because it uses the largest number of electron energy levels since the
valence electrons are placed in larger orbitals i.e. the principal quantum number increases on going down the
group.
ii. Fluorine is the most reactive since it has the greatest tendency to gain electrons because it has a higher
effective nuclear charge and uses fewer energy levels than Br and Cl.
Marks: 3

Question 15. Explain the meaning of sexually transmitted diseases (STD’s). Give two examples of STD’s
each, caused due to
i. bacterial infection
ii. viral infection.

www.vedantu.com 3 / 18
State in brief how the spread of such diseases may be prevented.
Solution: Sexually transmitted diseases (STD’s) are diseases which are usually passed through sexual
contact with an infected partner.
i. Sexually transmitted diseases caused due to bacterial infection: Gonorrhea and Syphilis.
ii. Sexually transmitted diseases caused due to viral infection: AIDS and Herpes.
A key strategy in the prevention of STD’s involves screening, diagnosis and treatment of patients as well as
their sexual partners to interrupt transmission.
Prevention of transmission of STD's:
(a) Having sex with an infected or any unknown person should be avoided.
(b) Sharing of needles, syringes etc. must be prohibited.
(c) Surgical and dental instruments should be sterilised properly before use.
(d) Avoid blood transfusion from an infected person. Blood should be tested before transfusion.
(e) Adequate medical treatment should be provided to the pregnant woman to protect the child from getting
infected.
Marks: 3

Question 16. Distinguish between homologous organs and analogous organs. In which category would you
place wings of a bird and wings of bat? Justify your answer giving a suitable reason.
Solution:
Homologous Organs Analogous Organs
Homologous organs are organs which are dissimilar Analogous organs are organs which are similar in
in shape, size and function but their origin, basic shape and function but their origin, basic plan and
plan and development are similar. development are dissimilar.
Wings of a bird and bat should be placed in the category of analogous organs as they are similar in function
but are different in their structure and development.
Marks: 3

Question 17. Define the term 'evolution'. ‘Evolution cannot be equated with progress’. Justify this
statement.
Solution: Evolution is a gradual change in the characteristics of a population of animals or plants over
successive generations.
Evolution cannot be equated with progress. There is no real 'progress' in the idea of evolution. Evolution is
simply the generation of diversity and the shaping of the diversity by environmental selection. The only
progressive trend in evolution seems to be that more and more complex body designs have emerged over
time. However again, it is not as if the older designs are inefficient! One of the simplest life forms, bacteria
inhabits the most inhospitable habitats like hot springs, deep-sea thermal vents and the ice in Antarctica.
Marks: 3

Question 18. A blue colour flower plant denoted by BB is crossbred with a white colour flower plant
denoted by bb.
(a) State the colour of flower you expect in their F1 generation plants.
(b) What must be the percentage of white flower plants in F2 generation if flowers of F1 plants are self-
pollinated?
(c) State the expected ratio of the genotypes BB and Bb in the F2 progeny.
Solution: (a) Blue

www.vedantu.com 4 / 18
(b) 25%
(c) 1 : 2
Marks: 3

Question 19. Complete the following equations:


(a) CH4 + O2 
(b) C2H5OH 
Hot conc. H2SO4

(c) CH3COOH + NaOH 
Solution: (a) CH4 + O2  CO2 + H2O + Heat and light
(b) C2H5OH 
Hot conc. H2SO4
 CH2 = CH2 + H2O
(c) NaOH + CH3COOH  CH3COONa + H2O
Marks: 3

Question 20. A student cannot see a chart hanging on a wall placed at a distance of 3 m from him. Name the
defect of vision he is suffering from. How can it be corrected? Draw ray diagrams for the:
i. defect of vision
ii. for its correction
Solution: The student is suffering from myopia or short sightedness. It is a defect of vision due to which a
person cannot see the distant objects clearly though he can see nearby objects clearly.
It can be corrected by using a concave lens of suitable focal length.
i. Ray diagram for defect myopia:

ii. Ray diagram for the correction of this defect:

Marks: 3

Question 21. State the type of mirror preferred as


i. Rear view mirrors in vehicles
ii. Shaving mirrors. Justify your answer giving two reasons in each case

www.vedantu.com 5 / 18
Solution: i. Convex mirror is used as rear view mirror because: (1) It has a large field of view. (2) It
produces erect image of the objects behind the vehicle.
ii. Concave mirror is used as shaving mirror, because: (1) It produces enlarged image when object is placed
close to it. (2) It produces an erect image.
Marks: 3

Question 22. The image of a candle flame placed at a distance of 36 cm from a spherical lens is formed on a
screen placed at a distance of 72 cm from the lens. Identify the type of lens and calculate its focal length. If
the height of the flame is 2.5 cm, find the height of the image.
Solution: Given that:
Object distance, u = −36 cm
Image distance, v = 72 cm
As the image is obtained on a screen it is a real image and hence the spherical lens will be a convex lens.
Focal length f = ?
According to lens formula,
1 1 1
 
v u f
Substituting the values, we get:
1 1 1
 
72 36 f
1 1 1
 
f 72 36
72
f
36
f = 24 cm
Therefore the focal length of the lens = 24 cm
It is given that:
Object height, h1 = 2.5 cm
Image height, h2 =?
v h
We know that magnification, m =  2
u h1
v
 h2  h1 
u
72
h 2  2.5 
36
h2 = -5 cm
The image of the flame formed will be inverted and have a height of 5 cm.
Marks: 3

Question 23. List the sign conventions for reflection of light by spherical mirrors. Draw a diagram and
apply these conventions in the determination of focal length of a spherical mirror which forms a three times
magnified real image of an object placed 16 cm in front of it.
OR

www.vedantu.com 6 / 18
State the law of refraction of light which defines the refractive index of a medium with respect to the other.
Express it mathematically. How is refractive index of any medium 'A' with respect to a medium 'B' related to
the speed of propagation of light in two media A and B? State the name of this constant when one medium is
vacuum or air. The refractive indices of glass and water with respect to vacuum are 3/2 and 4/3 respectively.
If the speed of light in glass is 2 × 108 m/s, find the speed of light in
i. vacuum
ii. water
Solution: Sign conventions of spherical mirror:
1. Object is always placed to the left of mirror.
2. All distances are measured from the pole of the mirror.
3. Distances measured in the direction of the incident ray are positive and the distances measured in the
direction opposite to that of the incident ray are negative.
4. Distances measured along the y-axis (upwards) above the principal axis are positive and that measured
along the y-axis (downwards) below the principal axis are negative.

Given that:
Object distance, u = −16 cm
Magnification, m = 3
v h2
We know that magnification for a spherical mirror, m =  
u h1
v
i.e.,  3
u
 v  3u
Using mirror formula:
1 1 1
 
f u v
1 1 1
 
f 16 3  16
1 48

f 4
u = −12 cm
Negative sign of focal length implies that the focal length is being measured against the direction of incident
light and it is a concave mirror.

www.vedantu.com 7 / 18
OR
Snell's law of refraction states that the ratio of sine of angle of incidence in the first medium to the sine of
angle of refraction in the second medium is a constant and is termed as the refractive index of the second
medium with respect to the first medium,
sin i 1
 n2
sin r
where i and r are the angle which the incident and refracted rays respectively make with the normal.
The constant 1n2 is called the refractive index of the second medium (2) relative to the first medium (1).
Relation between refractive index of any medium 'A' with respect to a medium 'B' and the speed of
propagation of light in two media A and B:
B Speed of light in medium B
nA =
Speed of light in medium A
When one medium is vacuum or air, this constant is called absolute refractive index.
Given:
3
Refractive index of glass, ng 
2
4
Refractive index of water, nw 
3
Speed of light in glass, vg = 2 × 108 m/s
Let speed of light in vacuum be c,
c
We know that, ng 
vg
3
c  ng  vg   2 108  3108 m/s
2
Let vw be the speed of light in water.
c
 nw 
vw
c 3 108
vw   = 2.25 108 m/s
nw 4
3
Speed of light in vacuum = 3 × 108 m/s
Speed of light in water = 2.25 × 10s8 m/s
Marks: 5

Question 24. What is the difference between the chemical composition of soaps and detergents?
State in brief the soaps in removing an oily spot from a shirt. Why soaps are not considered suitable for
washing when water is hard?
OR
List in tabular form three physical and two chemical properties on the basis of which ethanol and ethanoic
acid can be differentiated.
Solution: Detergents are generally ammonium or sulphonate salts of long chain carboxylic acids whereas
molecules of soap are sodium or potassium salts of long-chain carboxylic acids.
Action of soap:

www.vedantu.com 8 / 18
1. Soaps are cleansing agents capable of reacting with water and dislodge the unwanted particles from cloth
or skin.
2. The molecules of soap are sodium or potassium salts of long chain carboxylic acids.
3. A soap molecule has a tadpole shaped structure.
4. One end (long non polar end) of a soap molecule is a hydrocarbon chain which is insoluble in water but
soluble in oil.
5. The other end (short polar end) of a soap molecule is a carboxylate ion which is hydrophilic i.e., water
soluble but insoluble in oil.

6. Soap on mixing with water forms a concentrated solution and causes foaming.
7. The long non-polar end of soap gravitates towards and surrounds the dirt and absorbs the dust in it.
8. The short polar end with the carboxylate ion turns the water away from the dirt.
9. A spherical aggregate of soap molecules is formed in the soap solution in water and is called a micelle.
10. The soap molecule thus helps in dissolving the dirt in water and help to wash our clothes clean.

Hard water contains calcium and magnesium salts. When soap is used in hard water it forms an insoluble
substance scum which remains even after washing hence soaps are not considered suitable for washing in
hard water.
OR
Difference in physical properties:
S. No. Ethanol Ethanoic acid
1. It exists only in liquid form. It can exist both in liquid as well as solid form.
2. It belongs to the functional group alcohols. It belongs to the functional group carboxylic
acids.
3. It has a specific smell but not like vinegar. It smells like vinegar.

www.vedantu.com 9 / 18
Difference in chemical properties:
S. No. Ethanol Ethanoic acid
1. Reaction with sodium bicarbonate: Reaction with sodium bicarbonate:
No salt formation occurs and carbon dioxide It will react with sodium bicarbonate to form a
gas is also not evolved. salt and carbon dioxide gas will be released.
2. It does not give litmus test i.e., no change in It turns blue litmus red.
the colour of litmus solution..

Marks: 5

Question 25. Define the terms pollination and fertilisation. Draw a diagram of a pistil showing pollen tube
growth into the ovule and label the following: pollen grain, male gamete, female gamete and ovary.
OR
Describe in brief the role of
i. testis
ii. seminal vesicle
iii. vas deferens
iv. ureter
v. prostate gland in human male reproductive system.
Solution: Pollination- Transfer of pollen grains from the anther to the stigma is called pollination.
Fertilisation - The process of fusion of male and female gametes to form a zygote which eventually develops
into an embryo is called fertilisation.

OR
i. Testis: It is the organ which produces sperms and the male sex hormone, testosterone.
ii. Seminal vesicle: It produces fluid which makes up a significant percentage of semen.

www.vedantu.com 10 / 18
iii. Vas deferens: Vas deferens is a tube transporting spermatozoa from the epididymis to the prostate part of
the urethra.
iv. Ureter: It carries urine from the kidneys to the urinary bladder.
v. Prostate gland in human male reproductive system: It contributes additional fluid to the ejaculate. Prostate
fluids also help to nourish the sperms.
Marks: 5

SECTION B

Question 26. After observing the prepared slides of binary fission in Amoeba and budding in yeast, the
following observations were reported:
a. Single cells of Amoeba and Yeast were undergoing binary fission and budding respectively.
b. Cytokinesis was observed in the Yeast cell.
c. Elongated nucleus was dividing to form two daughter nuclei in Amoeba.
d. A chain of buds were observed due to reproduction in Amoeba.
The correct observation(s) is/are:
(a) d, a and c
(b) c and d
(c) b only
(d) a and c
Solution: (d) a and c
The correct observations are:
i. Single cells of Amoeba and Yeast were undergoing binary fission and budding respectively.
ii. Elongated nucleus was dividing to form two daughter nuclei in Amoeba.
Marks: 1

Question 27. A student after viewing a prepared slide illustrates budding in yeast in the following order
which is not correct:

The correct order should be:


(a) b, d, e, c, a
(b) b, e, d, c, a
(c) b, c, d, e, a
(d) b, d, c, e, a
Solution: (d) b, d, c, e, a
The correct sequence is b, d, c, e, a
Marks: 1

Question 28. A student has to observe a permanent slide of binary fission in Amoeba. Find the correct
sequence of steps given below for focusing the object under a microscope.

www.vedantu.com 11 / 18
a. Place the slide on the stage, look through the eye-piece and adjust the mirror to get proper illumination.
b. Focus the slide sharp using fine adjustment screw.
c. Look through the eye-piece and raise the objective lens using coarse adjustment screw till the object is
focused.
d. Look through the eye-piece and move the slide till the object is visible.
(a) a, c, d, b
(b) d, c, b, a
(c) a, b, d, c
(d) a, d, c, b
Solution: (d) a, d, c, b
The correct sequence is a, d, c, b
Marks: 1

Question 29. After viewing different slides, a student draws the following diagrams. Select the one which
depicts binary fission in Amoeba:

(a) d
(b) b
(c) a
(d) c
Solution: (c) a
Figure (a) shows binary fission in Amoeba.
Marks: 1

Question 30. Dry raisins were soaked in water for 2 hours to determine the percentage of water absorbed by
raisins. Before final weighing of swollen raisins, the extra water left on the surface of soaked raisins was
removed by:
(a) Dry cotton wool
(b) Hot air blower
(c) Gently rubbing with cotton cloth
(d) Filter paper
Solution: (d) Filter paper
Filter paper is used to soak raisins.
Marks: 1

Question 31. While performing the experiment with raisins to determine the percentage of water absorbed
by them, a student made the following measurements:
Mass of water in the beaker = 40 g

www.vedantu.com 12 / 18
Mass of raisins before soaking = 5 g
Mass of raisins after soaking for 2 hours = 8 g
Mass of water left in the beaker after three experiments = 35 g
The percentage of water absorbed by raisins is:
8g  5g
(a) 100
8g
40g  35g
(b) 100
40g
40g  35g
(c) 100
35g
8g  5g
(d) 100
5g
8g  5g
Solution: (d) 100
5g
Marks: 1

Question 32. Which of the following observations is true about dilute solution of acetic acid?
(a) It smells like vinegar and turns red litmus blue
(b) It smells like onion and turns blue litmus blue
(c) It smells like orange and turns red litmus blue
(d) It smells like vinegar and turns blue litmus red
Solution: (d) It smells like vinegar and turns blue litmus red
Acetic acid smells like vinegar and turns blue litmus red due to its acidic nature.
Marks: 1

Question 33. A student takes Na2CO3 powder in a test tube and pours some drops of acetic acid in it.
He observes:
(a) No reaction in the test tube
(b) Colourless gas with pungent smell
(c) Bubbles of a colourless and odourless gas
(d) White fumes with smell of vinegar
Solution: (c) Bubbles of a colourless and odourless gas
Na2CO3 reacts with acetic acid to evolve carbon dioxide gas.
Marks: 1

Question 34. A student adds 4 ml of acetic to a test tube containing 4 ml of distilled water. He then shakes
the test tube and leaves it to settle. After about 10 minutes he observes:
(a) A layer of water over the layer of acetic acid
(b) A layer of acetic acid over the layer of water
(c) A precipitate settling at the bottom of the test tube
(d) A clear colourless solution
Solution: (d) A clear colorless solution
Acetic acid is completely miscible with water in all proportions.
Marks: 1

www.vedantu.com 13 / 18
Question 35. The colours of aqueous solutions of CuSO4 and FeSO4 as observed in the laboratory are:
(a) Pale green and light blue respectively
(b) Light blue and dark green respectively
(c) Dark blue and dark green respectively
(d) Dark blue and pale green respectively
Solution: (D) Dark blue and pale green respectively
Copper sulphate solution is blue coloured and iron sulphate solution is pale green.
Marks: 1

Question 36. A student prepared an aqueous solution of CuSO4 in beaker X and an aqueous solution of
FeSO4 in beaker Y. He then dropped some iron pieces in beaker X and some zinc pieces in beaker Y. After
about 10 hours, he observed that the solution in X and Y respectively appears:
(a) Blue and green
(b) Colourless and pale green
(c) Colourless and light blue
(d) Greenish and colourless
Solution: (d) Greenish and colourless
Fe  CuSO4(aq)  FeSO4(aq)  Cu
(X) Greenish

Zn  FeSO4(aq)  ZnSO4(aq)  Fe
(Y) Colorless

Marks: 1

Question 37. While tracing the path of a ray of light passing through a rectangular glass slab a student
tabulated his observations as given below:

The correct observation is:


(a) I
(b) II
(c) III
(d) IV
Solution: (d) IV
The IV observation is the correct one. The ratio of sin i and sin r given by the fourth choice gives 1.5.
We know that
sin i n2 1.5
   1.5
sin r n1 1
Marks: 1

www.vedantu.com 14 / 18
Question 38. A student traces the path of a ray of white light through a rectangular glass slab and marks the
angles of incidence ( i ), refraction ( r ) and emergence ( e ) as shown.

Which angle or angles have not been marked correctly?


(a) i only
(b) i and r
(c) r and e
(d) i and e
Solution: (d) i and e
i and e are not marked correctly. Each angle is supposed to be marked from the normal.
Marks: 1

Question 39. To determine the focal length of a convex lens by obtaining a sharp image of a distant object
we generally follow the following steps which are not in proper sequence.
a. Hold the lens between the object and the screen
b. Measure the distance between the lens and the screen
c. Select a well lit distant object
d. Place a screen opposite to the object on the lab table
e. Adjust the position of the lens to form a sharp image
The correct sequence of these steps is:
(a) c, a, d, e, b
(b) c, d, a, e, b
(c) c, d, e, a, b
(d) c, a, e, d, b
Solution: (b) c, d, a, e, b
The correct sequence will be c, d, a, e, b
Marks: 1

Question 40. A student obtained a sharp image of the grills of a window on a screen using a concave mirror.
His teacher remarked that for getting better results a well lit distance object (preferably the Sun) should be
focused on the screen. What should be done for this purpose?
(a) Move the screen and the mirror towards the object
(b) Move the screen and the mirror away from the object
(c) Move the screen slightly away from the mirror
(d) Move the mirror slightly towards the screen
Solution: (c) Move the screen slightly away from the mirror

www.vedantu.com 15 / 18
The screen is moved away from the mirror so as to focus the object for a fixed position of the mirror and the
object.
Marks: 1

Question 41. To determine focal length of a concave mirror a student obtains the image of a well lit distant
object on a screen. To determine the focal length of the given concave mirror he needs to measure the
distance between the:
(a) Cannot be determined
(b) Screen and the object
(c) Mirror and the object
(d) Mirror and the screen
Solution: (d) Mirror and the screen
The distance between mirror and the screen will give the focal length of the mirror as the mirror focuses the
light on the screen.
Marks: 1

www.vedantu.com 16 / 18
CBSE-XII-2017 EXAMINATI CBSE – X – 2013 EXAMINATION

SCIENCE
SET-1 Paper & Solution
Time: 3 Hrs. Max. Marks: 90

General Instructions:
1. The question paper comprises two Sections, A and B. You are to attempt both the sections.
2. There is no overall choice. However, internal choice has been provided in all the five questions of five marks category. Only
one option in such question is to be attempted.
3. All questions of Section-A and Section-B are to be attempted separately.
4. Question numbers 1 to 3 in Section A are one-mark questions. These are to be answered in one word or in one sentence.
5. Question numbers 4 to 7 in Section A are two marks questions, to be answered in about 30 words each.
6. Question numbers 8 to 19 in Section A are three marks questions, to be answered in about 50 words each.
7. Question numbers 20 to 24 in Section A are five marks questions, to be answered in about 70 words each.
8. Question numbers 25 to 42 in Section B are multiple choice questions based on practical skills. Each question is a one-mark
question. You are to select one most appropriate response out of the four provided to you.

SECTION A
Question 1. How many horizontal rows are there in the modern periodic table and what are they called?
Solution: There are 7 horizontal rows in the modern periodic table. They are called periods.
Marks: 1

Question 2. List any two factors that could lead to speciation.


Solution: Geographical isolation and genetic drift could lead to speciation.
Marks: 1

Question 3. Mention one negative effect of our affluent life style on the environment.
Solution: Exploitation and overconsumption of natural resources will ultimately result in their scarcity.
Marks: 1

Question 4. Mention two functions of the human testis.


Solution: (a) They produce male germ cells i.e., sperms.
(b) They act as endocrine glands and secrete the hormone testosterone which controls secondary sexual
characters in males.
Marks: 2

Question 5. Every one of us can do something to reduce our consumption of various natural resources. List
four such activities based on the 3-R approach.
Solution: (a) Use of materials such as paper should be preferred as they can be reused and recycled.
(b) Materials like glass and some plastics can be recycled on heating and get easily converted into different
products like toys, containers which can be reused again.
(c) e-wastes such as unused computers, mobiles etc. can be repaired and used again.
(d) Household wastes such as vegetable wastes etc. can be used as manure for plants.
Marks: 2

Question 6. ‘A ray of light incident on a rectangular glass slab immersed in any medium emerges parallel to
itself.’ Draw a labelled ray diagram to justify the statement.

www.vedantu.com 1 / 19
Solution: When a ray of light is incident on a rectangular glass slab immersed in a medium, the emergent
ray comes out parallel to the incident ray. The path of the light ray is as shown:

EF is the incident ray and GH is the emergent ray which is parallel to the incident ray.
Marks: 2

Question 7. We often observe domestic waste decomposing in the bylanes of residential colonies. Suggest
ways to make people realise that the improper disposal of waste is harmful to the environment.
Solution: Some of ways to make people realise that the improper disposal of waste is harmful to the
environment are:
(a) Improper disposal of waste will serve as a breeding ground for mosquitoes and will create favourable
conditions for the spread of various diseases.
(b) Improper disposal of waste will release harmful gases in the environment which makes the environment
unclean and unhygienic for normal living of organisms.
(c) The waste will flow to water bodies along with the rain water and become a threat to aquatic organisms.
Marks: 2

Question 8. Name the oxidising agent used for the conversion of ethanol to ethanoic acid. Distinguish
between ethanol and ethanoic acid on the basis of (i) litmus test, (ii) reaction with sodium carbonate.
Solution: Alkaline potassium permanganate (KMnO4) or acidified potassium dichromate (K2Cr2O7) can be
used as an oxidising agent for conversion of ethanol to ethanoic acid.
i. Litmus test: Ethanoic acid turns blue litmus solution red whereas ethanol being neutral in nature has no
effect on litmus solution.
ii. Reaction with sodium carbonate: Ethanoic acid reacts with sodium carbonate to form sodium ethanoate
and carbon dioxide gas is evolved.
2CH3COOH + Na2CO3  2CH3COONa + H2O + CO2
Ethanol does not react with sodium carbonate.
CH3CH2OH + Na2CO3  No reaction
Marks: 3

www.vedantu.com 2 / 19
Question 9. (a) Differentiate between alkanes and alkenes. Name and draw the structure of one member of
each.
(b) Alkanes generally burn with clean flame. Why?
Solution: (a) Alkanes: Hydrocarbons in which the carbon atoms are joined by single covalent bonds are
called Alkanes. They have general formula CnH2n+2, where n is the number of carbon atoms. Suffix, -ane is
used while naming alkanes.

Alkenes: Hydrocarbons in which the carbon atoms are joined by a double bond are called Alkenes. They
have general formula CnH2n, where n is the number of carbon atoms. Suffix, -ene is used while naming
alkenes.

(b) Alkanes generally burn with clean flame because in them, the percentage of carbon is comparatively low
as compared to other unsaturated hydrocarbons. Hence they get oxidised completely by the oxygen present
in air.
Marks: 3

Question 10. Given below are some elements of the modern periodic table:
4Be, 9Fe, 14Si, 19K, 20Ca
i. Select the element that has one electron in the outermost shell and write its electronic configuration.
ii. Select two elements that belong to the same group. Give reasons for your answer.
iii. Select two elements that belong to the same period. Which one of the two has bigger atomic size?
Solution: i. 19K has one electron in the outermost shell and its electronic configuration is
2, 8, 8, 1.
ii. 4Be and 20Ca belongs to same group i.e., Group 2.

www.vedantu.com 3 / 19
Electronic configuration:
4Be – 2, 2

20Ca – 2, 8, 8, 2
4Be and 20Ca have same number of valence electrons in outermost shell i.e., 2 so they belong to same group.
iii. 9F and 4Be belongs to the same period i.e., period 2.
Electronic configuration:
9F - 2, 7

4Be - 2, 2

4Be has a bigger atomic size than 9F because the atomic radius decreases as we move from left to right in a
period due to increase in nuclear charge which tends to pull the electrons closer to the nucleus and hence
size of atom reduces.
Marks: 3

Question 11. An element X (atomic number 17) reacts with an element Y (atomic number 20) to form a
compound.
(a) Write the position of these elements in the modern periodic table.
(b) Write the formula of the compound formed.
Justify your answer in each case.
Solution: (a) Element X (atomic number 17)
Electronic configuration of X: 2, 8, 7
No. of electrons in outermost shell = 7
So, Group number = 17
No. of shells = 3
So, Period to which the element belongs = 3
Element Y (atomic number 20)
Electronic configuration of Y: 2, 8, 8, 2
No. of electrons in outermost shell = 2
So, Group number = 2
No. of shells = 4
So, Period to which the element belongs = 4
(b) X has 7 valence electrons so, it needs 1 electron to complete its octet and Y has 2 valence electrons so, it
can donate its 2 electrons to acquire the octet configuration. Hence, X will gain 1 electron and Y will lose 2
electrons, so the chemical reaction is:
X2 + Y  YX2
X = Cl (At. No. = 17) and Y = Ca (At. No. = 20)
So, Cl2 + Ca  CaCl2
Marks: 3

Question 12. ‘The sex of a newborn child is a matter of chance and none of the parents may be considered
responsible for it.’ Justify this statement with the help of flow chart showing determination of sex of a
newborn.
Solution: In human beings, females have two X chromosomes and males have one X and one Y
chromosome. Therefore, the females are represented as XX and males as XY. At the time of mating, large
number of sperms are ejaculated from the male reproductive organ (penis), into the female reproductive
organ i.e., vagina. They travel towards the fallopian tubes, where only one sperm meets with the egg.

www.vedantu.com 4 / 19
The process of fusion of the sperm and ovum is called fertilisation. The sperm has either X or Y
chromosome and egg has only X chromosome. So, if a sperm carrying Y chromosome fuses with the egg,
the newly born child will be male and if a sperm carrying X chromosome fuses with the egg, the newly born
child will be female. There is an equal chance of fusion of either X or Y chromosome with the egg so we
can say that the sex of a new born child is a matter of chance and none of the parent is responsible for it.
Sex determination in humans is shown below:

Marks: 3

Question 13. Tabulate two distinguishing features between acquired traits and inherited traits with one
example of each.
Solution:
Acquired traits Inherited traits
i. The acquired traits are the traits which are i. Inherited traits are the characteristics which one
experienced by an individual during his life time. acquires from his/ her ancestors.
ii. These involve changes in non-reproductive ii. These involve changes in the DNA. Hence, they
tissues (or somatic cells), which cannot be passed are transmitted to the progeny.
on to the germ cells or progeny.
iii. Example- cut-tail of mice, learning to dance etc. iii. Example- height, eye colour, skin color etc.
Marks: 3

Question 14. Write two examples each of sexually transmitted diseases caused by (i) virus, (ii) bacteria.
Explain how the transmission of such diseases can be prevented?
Solution: Sexually transmitted diseases by virus:
Genital Herpes is caused by the Herpes Simplex virus and AIDS is caused by HIV.
Sexually transmitted diseases by bacteria:
Gonorrhea is caused by Nisseria gonorrhoeae and Syphilis is caused by Treponema pallidum.
Prevention of transmission of STD's:
i. Having sex with infected or any unknown person should be avoided.
ii. Sharing of needles, syringes etc. must be prohibited.
iii. The surgical and dental instruments should be sterilised properly before use.
iv. Avoid blood transfusions from infected person. Blood should be tested before transfusion.
v. Adequate medical treatment should be provided to the pregnant woman to protect the child from getting
infected.
Marks: 3

www.vedantu.com 5 / 19
Question 15. (a) Explain the process of regeneration in Planaria.
(b) How is regeneration different from reproduction?
Solution: (a) Regeneration in Planaria:
In Planaria, any part of the body which gets cut is capable of regeneration or developing into a complete
organism. Regeneration is carried out by specialized cells which proliferate and make large numbers of
cells. From these mass of cells, different cells undergo changes to become various cell types and tissues.
These changes take place in an organised sequence referred to as development.
(b)
Regeneration Reproduction
1. It is a type of asexual mode of reproduction in 1. It includes both sexual and asexual mode of
which only single parent is involved. reproduction in which either single parent or both
the parents are involved respectively.
2. This mode of reproduction is dependent on a cut 2. This mode of reproduction is independent of cut
in an organism. in an organism.
Example - Planaria. Example - Sexual reproduction in humans.
Marks: 3

Question 16. An object of height 5 cm is placed perpendicular to the principal axis of a concave lens of
focal length 10 cm. Use lens formula to determine the position, size and nature of the image, if the distance
of the object from the lens is 20 cm.
Solution: Given that:
The height of the object ho = 5 cm
Focal length of the lens = −10 cm
Object distance, u = −20 cm
Using lens formula,
1 1 1
 
v u f
1 1 1
 
v 20 10
1 1 1
 
v 10 20
20
v  6.66 cm
3
Negative sign implies that the image is formed on the same side as the object.
v 20 / 3 1
We know that magnification, m =  
u 20 3
Since magnification is positive, so image is virtual and erect.
Let hi be the height of the image.
v h
We also know that: m =  i
u ho
1 hi

3 5
5
hi   1.66 cm
3

www.vedantu.com 6 / 19
As hi < ho
The image formed is smaller than the object.
Marks: 3

Question 17. Mention the types of mirrors used as (i) rear view mirrors, (ii) shaving mirrors. List two
reasons to justify your answers in each case.
Solution: i. Rear view mirrors: Convex mirrors
Reason: (a) They produce virtual and erect images of the objects. (b) They have a wider field of view as they
are curved outwards.
ii. Shaving mirrors: Concave mirrors
Reason: (a) Image formed is magnified so that the details of the object can be seen clearly. (b) Image formed
is virtual and erect when the object is placed close to the mirror.
Marks: 3

Question 18. State the difference in colours of the Sun observed during sunrise/ sunset and noon. Give
explanation for each.
Solution: During sunrise and sunset, Sun is red in colour while at noon, the Sun appears white.
At the time of sunrise and sunset, the Sun is near the horizon. The rays from the Sun have to travel a much
larger part of the atmosphere to reach an observer on earth. So, most of the blue light gets scattered away.
The red colour which has the largest wavelength is scattered the least and enters into our eyes. Hence, the
Sun appears red at the time of sunrise and sunset.
At noon, the sun is nearly overhead. The sunlight has to pass through much smaller portion of the Earth's
atmosphere. The scattering is much less and the Sun looks white.
Marks: 3

Question 19. (a) What is an ecosystem? List its two main components.
(b) We do not clean ponds or lakes, but an aquarium needs to be cleaned regularly. Explain.
Solution: Ecosystem is a self-sustaining system where the biotic and abiotic organisms of various
Communities live with each other.
(a) The two components of the ecosystem are- Biotic and Abiotic. Biotic system consists of all the living
organisms of particular area like humans, animals etc. and the nonliving component consists of air, minerals,
soil, water and sunlight.
(b) Ponds are an example of a natural ecosystem whereas an aquarium is an example of an artificial
ecosystem. Ponds do not need to be cleaned but aquarium needs to be cleaned because an aquarium does not
contain soil and decomposing bacteria which helps in degrading complex organic substances into simple
inorganic substances. But ponds or lakes have this ability of self-purification, and therefore these do not
need to be cleaned.
Marks: 3

Question 20. (a) Write the function of placenta in females.


(b) List four ways of preventing pregnancy. State two advantages of using such preventive methods.
Solution: (a) Placenta is a disc like tissue which develops between the uterus wall and embryo.
Role of placenta:
i. Exchange of water between mother and the foetus.
ii. Exchange of nutrients.

www.vedantu.com 7 / 19
iii. Exchange of respiratory gases.
iv. Excretion of nitrogenous wastes from foetus. Nitrogenous waste crosses the placenta and is removed by
mother's kidney.
v. Antibodies also cross the placenta and provide immunity to the baby.
(b) Ways of preventing pregnancy:
i. Natural methods: In this method, sexual act is avoided from the 10 th day to the 17th day of the menstrual
cycle, since during this period ovulation is expected.
Therefore, the chances of fertilisation are very high.
ii. Barrier methods: In this method, the fertilisation of ovum and sperm is prevented with the help of
physical devices such as condoms and diaphragm.
iii. Oral contraceptives: In this method, tablets or drugs are taken orally. These contain small doses of
hormones which prevent the release of eggs and prevent fertilisation.
Marks: 5

Question 21. (a) Identify A, B and C in the given diagram and write their functions.
(b) Mention the role of gamete and zygote in sexually reproducing organisms.

Solution: (a) A - Stigma.


Function - The stigma is a sticky surface where the pollen grains land and germinate.
B - Pollen tube.
Function - It carries the pollen grains to the egg cell for fertilisation.
C - Egg cell.
Function - It fuses with the male gamete and leads to the formation of the zygote.
(b) Role of gametes - Gametes play an important role in sexually reproducing organisms as they carry the
entire genetic information of the organism. These gametes upon fusion result in the formation of the zygote,
which develops into a new individual. Any deformation in the gametes will lead to deformity in the newly
formed offspring.
Role of zygote - Zygote is the diploid cell formed by the fusion of male and female gametes during
fertilisation in sexual reproduction. Zygote is the first stage in the development process of an organism and it
contains all the genetic information of both the parents essential for the growth of the new organism.
Marks: 5

Question 22. (a) A person cannot read newspaper placed nearer than 50 cm from his eyes. Name the defect
of vision he is suffering from. Draw a ray diagram to illustrate this defect. List its two possible causes. Draw
a ray diagram to show how this defect may be corrected using a lens of appropriate focal length.

www.vedantu.com 8 / 19
(b) We see advertisements for eye donation on television or in newspaper. Write the importance of such
advertisements.
Solution: (a) The person is suffering from Hypermetropia i.e., far-sightedness. It is a defect of vision due to
which a person cannot see nearby objects clearly though he can see distant objects clearly. In this defect the
image of the nearby object is formed behind the retina of eye.

Hypermetropia is caused due to:


i. decrease in length of the eyeball
ii. increase in focal length of the eye lens
This defect can be corrected using a convex lens of appropriate focal length so that the image gets formed on
the retina.

(b) Eye donations:


Our eyes can live even after our death, so by donating our eyes, we can give vision to a blind person and
give them an opportunity to see the world. By giving such advertisements in newspapers, we can make more
people aware for this noble cause and raise the number of eye donations for blind people.
Marks: 5

Question 23. State Snell's law of refraction of light. Write an expression to relate refractive index of a
medium with speed to light in vacuum.
The refractive index of a medium 'a' with respect to medium 'b' is 2/3 and the refractive index of medium 'b'
with respect to medium 'c' is 4/3. Find the refractive index of medium 'c' with respect to medium 'a'.
Solution: Snells' law of refraction: The ratio of the sine of angle of incidence to the sine of angle of
refraction is constant for a pair of media.
Mathematically, it can be given as follows:
When light travels from medium a to medium b, such that ‘i’ is the angle of incidence and ‘r’ is the angle of
refraction, then
sin i a
 nb
sin r
where, anb is the relative refractive index of medium 'b' with respect to medium 'a'.
Relation between refractive index (n) of a medium and the speed of light in vacuum is:
Speed of light in vacuum (c)
n=
Speed of light in medium (c)
Given that:

www.vedantu.com 9 / 19
na 2
Refractive index of a with respect to b, bna   …………. (i)
nb 3
nb 4
Refractive index of b with respect to c, cnb   …………. (ii)
nc 3
nc nc nb
 Refractive index of c with respect to a, anc   
na nb na
3 3
nc  
a
-------- Using eq. (i) and (ii)
4 2
9
a
nc 
8
Marks: 5

Question 24. (a) Define the term 'isomers'


(b) Draw two possible isomers of the compound with molecular formula C3H6O and write their names.
(c) Give the electron dot structures of the above two compounds.
Solution: (a) Compounds having same molecular formula but different structural formula are known as
isomers.
(b) Two possible isomers of the compound with molecular formula C 3H6O:

(c) Electron dot structure:

Marks: 5

SECTION B
Question 25. A student obtained a sharp image of a burning candle, placed at the farther end of a laboratory
table, on a screen using a concave mirror. For getting a better value of focal length of the mirror, what
should the student do?
(a) He should move the mirror away from the screen.
(b) He should move the mirror slightly towards the screen.
(c) He should move the mirror as well as the screen towards the newly selected object.
(d) He should move only the screen towards the newly selected object.
Solution: (b) He should move the mirror slightly towards the screen.
When an object is at infinity, image is formed at the focus and when an object is beyond C then image is
formed between C and F. As the object is farther, image will be closer to F. So, the student should move the
mirror slightly towards the screen.

www.vedantu.com 10 / 19
Marks: 1

Question 26. A student focused the image of a distant object using a device 'X' on a white screen 'S' as
shown in the figure. If the distance of the screen from the device is 40 cm, select the correct statement about
the device.

(a) The device X is a convex lens of focal length 20 cm.


(b) The device X is a concave mirror of focal length 40 cm.
(c) The device X is a convex mirror of radius of curvature 40 cm.
(d) The device X is a convex lens of focal length 40 cm.
Solution: (d) The device X is a convex lens of focal length 40 cm.
The parallel rays from the distant object fall on the convex lens and converge at its second principal focus
(i.e., where the screen is placed). Then the distance between the screen and the convex lens gives the
approximate focal length of the lens i.e., 40 cm.
Marks: 1

Question 27. Study the following ray diagrams:


I.

II.

III.

www.vedantu.com 11 / 19
IV.

The diagrams showing the correct path of the ray after passing through the lens are:
(a) II and III only
(b) I and II only
(c) I, II and III
(d) I, II and IV
Solution: (c) Ray diagrams I, II and III are correct.
The light ray passing through the optical centre of the lens does not deviate. The light ray parallel to the
principal axis passes through the second focus of the lens. The light ray passing through the first focus
becomes parallel to the principal axis after passing through the lens.
Marks: 1

Question 28. Out of the five incident rays shown in the figure find the three rays which are obeying the laws
of refraction and may be used for locating the position of the image formed by a convex lens:

(a) 1, 2 and 3
(b) 2, 3 and 4
(c) 3, 4 and 5
(d) 1, 2 and 4
Solution: (b) Rays 2, 3 and 4 obey the laws of refraction.
Ray (2) is parallel to the principal axis and passes through the second focus of the lens.
Ray (3) passes through the optical centre and does not deviate.
Ray (4) passes through the first focus of the lens and goes parallel to the principal axis.
Marks: 1

Question 29. Select from the following the best set-up for tracing the path of a ray of light through a
rectangular glass slab:

www.vedantu.com 12 / 19
I.

II.

III.

IV.

(a) I
(b) II
(c) III
(d) IV
Solution: (a) I
The best set up is given in figure I. The incoming light should not fall perpendicularly as the light will
emerge straight and refraction cannot be traced. The light rays should not be very close or far from the
normal as the emergent rays are difficult to trace.
Marks: 1

Question 30. After tracing the path of rays of light through a glass slab for three different angles of
incidence, a student measured the corresponding values of angle of refraction ‘r’ and angle of emergence ‘e’
and recorded them in the table given below:

www.vedantu.com 13 / 19
The correct observations are:
(a) I and II
(b) II and III
(c) I and III
(d) I, II and III
Solution: (d) I, II and III
For light rays passing through a glass slab, the following conditions should hold true:
1. i  e and r  i
2. Also, with the increase in incident angle, the refracted angle also increases.
Both of these conditions are satisfied in all the three observations.
Marks: 1

Question 31. In an experiment to trace the path of a ray of light through a glass prism for different values of
angle of incidence a student would find that the emergent ray:
(a) is parallel to the incident ray
(b) perpendicular to the incident ray
(c) is parallel to the refracted ray
(d) bends at an angle to the direction of the incident ray
Solution: (d) bends at an angle to the direction of incident ray
As the light gets refracted twice at different angles the emergent ray bends at an angle to the direction of
incident ray.
Marks: 1

Question 32. While performing an experiment to trace the path of a ray of light passing through a glass
prism, four students marked the incident ray and the emergent ray in their diagrams in the manner shown
below. Which one is correct?
I.

II.

www.vedantu.com 14 / 19
III.

IV.

(a) I
(b) II
(c) III
(d) IV
Solution: (c) III
When light goes form a rarer medium to a denser medium; it bends towards the normal and vice-versa. This
condition is fulfilled in figure (III).
Marks: 1

Question 33. Study the different conclusions draw by students of a class on the basis of observations of
preserved/available specimens of plants and animals.
I. Potato and sweet potato are analogous organs in plants and animals.
II. Wings of insects and wings of birds are homologous organs in animals.
III. Wings of insects and wings of bats are analogous organs in animals.
IV. Thorns of citrus and tendrils of cucurbita are analogous organs in plants
The correct conclusions are:
(a) I and II
(b) II and IV
(c) III and IV
(d) I and III
Solution: (a) I and II
Analogous organs are those which do not share a common ancestry but perform common functions. Hence,
wings of insects and wings of bats are analogous organs in animals and potato and sweet potato are
analogous organs in plants.
Marks: 1

Question 34. You have potato, carrot, radish, sweet potato, tomato and ginger bought from the market in
your jute bag. Identify two vegetables to represent the correct homologous structures.
(a) Potato and sweet potato
(b) Carrot and tomato
(c) Potato and tomato
(d) Carrot and radish
Solution: (d) Carrot and Radish

www.vedantu.com 15 / 19
Homologous structures are similar in origin but perform different functions. Carrot and radish are
underground roots. So, they are considered homologous structures.
Marks: 1

Question 35. In the figure, the parts marked A, B and C are sequentially:

(a) Plumule, Cotyledon and Radicle


(b) Radicle, Cotyledon and Plumule
(c) Radicle, Plumule and Cotyledon
(d) Plumule, Radicle and Cotyledon
Solution: (d) Plumule, Radicle and Cotyledon
In the figure, the part marked A is Plumule, B is Radicle and C is Cotyledon.
Marks: 1

Question 36. Select the correct statements for the process of budding in yeast:
I. A bud arises from a particular region on a parent body.
II. A parent cell divides into two daughter cells; here the parental identity is lost.
III. Before detaching from the parent body a bud may form another bud.
IV. A bud when detached from the parent body grows into a new individual.
(a) II, III and IV
(b) I, II and III
(c) III, IV and I
(d) None of the above
Solution: (d) None of the above
Yeast reproduces asexually by the process of budding.
Budding is a type of asexual reproduction in which a new organism is formed from a bud of an existing
organism. A small bud is formed at a specific position on the parent cell. The nucleus of parent cell splits
and a part of it enters inside the newly formed bud. The bud develops into a new cell or daughter organism.
The new organism remains attached to the parent organism till it matures. After attaining maturity it
separates from the parent body.
Marks: 1

Question 37. A student after observing a slide showing different stages of binary fission in Amoeba draws
the following diagrams. However these diagrams are not in proper sequence.

www.vedantu.com 16 / 19
The correct sequence is:
(a) I, V, IV, III, II
(b) I, V, III, IV, II
(c) I, III, IV, V, II
(d) None of these
Solution: (d) None of these
The correct procedure in reproduction in Amoeba is,
(a) Chromosome is replicated.
(b) Chromosome splits into daughter strands.
(c) Cytokinesis splits the Amoeba into two daughter cells.
(d) Two Amoebae with identical DNA.
Marks: 1

Question 38. Read the following statements:


I. When a red litmus paper is dipped into the reaction mixture of a saponification reaction, it turns blue and
the reaction is exothermic.
II. When a blue litmus paper is dipped into the reaction mixture of a saponification reaction, its colour does
not change and the reaction is exothermic.
III. When a red litmus paper is dipped into the reaction mixture of a saponification reaction, its colour does
not change and the reaction is endothermic.
IV. When a blue litmus paper is dipped into the reaction mixture of a saponification reaction, its colour does
not change and the reaction is endothermic.
Which of the above statements are correct?
(a) I and II
(b) II and III
(c) III and IV
(d) I and IV
Solution: (a) I and II
Soap solution is basic in nature because it is formed from a strong base NaOH. Hence it turns red litmus
paper blue and does not change the color of blue litmus paper. The saponification reaction is exothermic in
nature. The correct statements are I and II.
Marks: 1

Question 39. A student prepared 20% sodium hydroxide solution in a beaker containing water. The
observations noted by him are given below.
I. Sodium hydroxide is in the form of pellets.
II. It dissolves in water readily.
III. The beaker appears cold when touched from outside.
IV. Red litmus paper turns blue when dipped into the solution.
The correct observations are:
(a) I, II, and III
(b) II, III and IV
(c) III, IV and I
(d) I, II and IV
Solution: (d) I, II and IV

www.vedantu.com 17 / 19
Sodium hydroxide is in the form of small white pellets and is soluble in water. Being basic in nature, it turns
red litmus paper blue. The correct observation is I, II and IV.
Marks: 1

Question 40. Hard water required for an experiment is not available in a school laboratory. However,
following salts are available in the laboratory. Select the salts which may be dissolved in water to make it
hard for the experiment.
1. Calcium Sulphate
2. Sodium Sulphate
3. Calcium Chloride
4. Potassium Sulphate
5. Sodium Hydrogen Carbonate
6. Magnesium Chloride
(a) 1, 2 and 4
(b) 1, 3 and 6
(c) 3, 5 and 6
(d) 2, 4 and 5
Solution: (b) 1, 3 and 6
Hard water contains Ca2+ and Mg2+ ions. Thus the salts which can be added to water to make it hard are
calcium sulphate, calcium chloride and magnesium chloride i.e., the salts 1, 3 and 6.
Marks: 1

Question 41. In an experiment to study the properties of acetic acid, a student takes about 2 ml of acetic acid
in a dry test tube. He adds about 2 ml of water to it and shakes the test tube well. He is likely to observe that:
(a) The acetic acid dissolves readily in water.
(b) The solution becomes light orange.
(c) Water floats over the surface of acetic acid.
(d) Acetic acid floats over the surface of water.
Solution: (a) The acetic acid dissolves readily in water.
The acetic acid dissolves readily in water to form a clear solution.
Marks: 1

Question 42. A student takes 2 ml acetic acid in a dry test tube and adds a pinch of sodium hydrogen
carbonate to it. He makes the following observations:
I. A colourless and odourless gas evolves with a brisk effervescence.
II. The gas turns lime water milky when passed through it.
III. The gas burns with an explosion when a burning splinter is brought near it.
IV. The gas extinguishes the burning splinter which is brought near it.
The correct observations are:
(a) I, II and III
(b) II, III and IV
(c) III, IV and I
(d) IV, I and II
Solution: (d) IV, I and II

www.vedantu.com 18 / 19
When acetic acid is taken in a dry test tube and sodium hydrogen carbonate is added to it, a colourless and
odourless gas evolves with a brisk effervescence i.e., CO2.
CH3COOH NaHCO3  CH3COONa  H2O  CO2
acetic acid Sodium hydrogen Sodium acetate carbon dioxide
carbonate

When CO2 is passed through lime water, it turns lime water milky because of formation of calcium
carbonate ppt.
Ca(OH)2(aq)  CO2(g)  CaCO3(s)  H2O
lime water (white ppt)

CO2 being a non-supporter of combustion extinguishes the burning splinter when it is brought near it. So,
the correct observations are IV, I and II.
Marks: 1

www.vedantu.com 19 / 19
CBSE-XII-2017 EXAMINATI CBSE – X – 2014 EXAMINATION

SCIENCE
SET-3 Paper & Solution
Time: 3 Hrs. Max. Marks: 90

General Instructions:
1. The question paper comprises two Sections, A and B. You are to attempt both the sections.
2. All questions are compulsory.
3. All questions of Section A and all questions of Section B are to be attempted separately.
4. Question numbers 1 to 3 in Section A are one-mark questions. These are to be answered in one word or in one sentence.
5. Question numbers 4 to 7 in Section A are two-mark questions. These are to be answered in about 30 words each.
6. Question numbers 8 to 19 in Section A are three-mark questions. These are to be answered in about 50 words each.
7. Question numbers 20 to 24 in Section A are five-mark questions. These are to be answered in about 70 words each.
8. Question numbers 25 to 42 in Section B are multiple choice questions based on practical skills. Each question is a one-mark
question. You are to select one most appropriate response out of the four provided to you.

SECTION A
Question 1. The atomic numbers of three elements A, B and C are 12, 18 and 20 respectively. State, giving
reason, which two elements will show similar properties.
Solution: The elements are,
A-(Atomic number 12) = Magnesium
B-(Atomic number 18) = Argon
C-(Atomic number 20) = Calcium
Element Calcium and magnesium will show similar properties as they belong to same group (Group II) of
the periodic table. They have same number of valence electrons and they both are metals. While argon is a
noble gas.
Marks: 1

Question 2. No two individuals are absolutely alike in a population. Why?


Solution: No two individuals are absolutely alike in a population due to variations taking place during the
process of DNA copying.
Marks: 1

Question 3. Write one negative effect of affluent lifestyle of few persons on the environment.
Solution: Affluent lifestyle of few persons leads to exploitation and overconsumption of natural resources
which ultimately results in their scarcity.
Marks: 1

Question 4. Draw labelled diagrams to illustrate budding in Hydra.


Solution: Budding in Hydra:

Marks: 2

www.vedantu.com 1 / 18
Question 5. “A concave mirror of focal length 15 cm can form a magnified, erect as well as inverted image
of an object placed in front of it.” Justify this statement stating the position of the object with respect to the
pole of the mirror in both the cases for obtaining the images.
Solution: For magnified and erect image the object is placed between pole P and focus F.
For magnified and inverted image the object is placed either at focus or anywhere between F and C.
Marks: 2

Question 6. State with reason any two possible consequences of elimination of decomposers from the Earth.
Solution: Consequences of elimination of decomposers:
i. If all the decomposers of earth are eliminated then the dead bodies of plants and animals would not be
decomposed into simpler non-polluting substances so whole of the earth in no time would have heaps of
dead bodies of plants and animals. They would start giving foul smell and life would become impossible on
earth.
ii. Elimination of decomposers would cause imbalance of nutrients as they break complex organic material
into simpler substances in different nutrient cycles.
Marks: 2

Question 7. What is water harvesting? How can this technique help in the conservation of water?
Solution: Water harvesting refers to all the processes and steps that are taken for scientific conservation of
fresh water.
Water harvesting helps in the conservation of water as:
(i). It increases the ground water resources.
(ii). It reduces storm water discharge, urban floods and overloading of sewage treatment plants.
Marks: 2

Question 8. Study the following table in which positions of six elements A, B, C, D, E and F are shown as
they are in the modern periodic table:

On the basis of the above table, answer the following questions:


(i) Name the element which forms only covalent compounds.
(ii) Name the element which is a metal with valency three.
(iii) Name the element which is a non-metal with valency three.
(iv) Out of D and E, which is bigger in size and why?
(v) Write the common name for the family to which the elements C and F belong.
Solution:

www.vedantu.com 2 / 18
(i) Element E - Silicon forms only covalent compounds.
(ii) Aluminium is a metal with the valency 3.
(iii) Nitrogen is a non-metal with the valency 3.
(iv) Out of D (Aluminium) and E (Silicon), aluminium has a larger size than silicon. This is because atomic
size decreases across the period.
(v) Common name for the family to which the elements C (Neon) and F (Argon) belong is 'Noble gas' or
'Inert gas'.
Marks: 3

Question 9. The elements Be, Mg and Ca each having two electrons in their outermost shells are in periods
2, 3, and 4 respectively of the modern periodic table. Answer the following questions, giving justification in
each case:
(i) Write the group to which these elements belong.
(ii) Name the least reactive element.
(iii) Name the element having largest atomic radius.
Solution: (i) Elements Be, Mg and Ca belong to Group II.
(ii) Beryllium (Be) is the least reactive element. This is because, as we move down the group, number of
shells increases and the effective nuclear charge decreases. Thus, the tendency to lose electrons increases.
(iii) Calcium has the largest atomic radius. Since, number of shells increases down the group atomic radius
also increases.
Marks: 3

Question 10. A carboxylic acid (molecular formula C2H402) reacts with an alcohol in the presence of an acid
catalyst to form a compound 'X'. The alcohol on oxidation with alkaline KMnO4 followed by acidification
gives the same carboxylic acid C2H402.
Write the name and structure of
(i) carboxylic acid, (ii) alcohol and (iii) the compound 'X'.
Solution: Ethanoic acid reacts with ethanol in the presence of concentrated sulphuric acid as a catalyst to
produce the ester, ethyl ethanoate. The reaction is slow and reversible.

(i) CH3-COOH:- Ethanoic acid


(ii) CH3 - CH2 - OH:- Ethyl alcohol or Ethanol

(iii) Compound :- Ethyl ethanoate.


Marks: 3

Question 11. Define the term ‘structural’ isomerism'. Explain why propane cannot exhibit this property.
Draw the structures of possible isomers of butane, C4H10.
Solution: Structural isomerism: Molecules having same molecular formula but different structure are said to
be structural isomers.
Propane has the chemical formula C3H8 and it is represented as CH3-CH2-CH3. In alkanes, isomerism arises
when a particular compound can be represented in the form of both straight chain and branched chain. Also,
the branching cannot be done from the first or the last carbon atom of the structure.

www.vedantu.com 3 / 18
The structural formulae of propane show that they do not have sufficient number of carbon atoms to exist in
the form of branched isomer. Hence, they do not exhibit structural isomerism.
Isomers of Butane: There are two isomers. N-butane and iso-butane

Marks: 3

Question 12. A student wants to project the image of a candle flame on a screen 90 cm in front of a mirror
by keeping the flame at a distance of 15 cm from its pole.
(a) Suggest the type of mirror he should use.
(b) Determine the linear magnification in this case.
(c) Find the distance between the object and its image.
(d) Draw ray diagram to show the image formation in this case.
Solution: (a) Concave mirror
(b) Linear magnification of a concave mirror is given by:
v
m
u
(90)

(  15)
= –6
(c) The distance between the object and image = 90 – 15 = 75 cm.
(d) Ray diagram:

Marks: 3

Question 13. Draw a ray diagram to show the path of the refracted ray in each of the following cases:
A ray of light incident on a concave lens is
(i) passing through its optical centre.
(ii) parallel to its principal axis.

www.vedantu.com 4 / 18
(iii) directed towards its principal focus.
Solution: (i) A ray of light passing through the optical centre of the concave lens will emerge without any
deviation.

(ii) A ray of light parallel to the principal axis, after refraction from a concave lens, appears to diverge from
the principal focus on the same side of the lens.

(iii) A ray of light directed towards the principal focus of a concave lens, becomes parallel to its principal
axis after refraction through the lens.

Marks: 3

Question 14. A narrow beam PQ of white light is passing through a glass prism ABC as shown in the
diagram.

Trace it on your answer sheet and show the path of the emergent beam as observed on the screen DE.
(i) Write the name and cause of the phenomenon observed.
(ii) Where else in nature is this phenomenon observed?
(iii) Based on this observation, state the conclusion which can be drawn about the constituents of white light.

www.vedantu.com 5 / 18
Solution:

(i) Dispersion of light


Cause: The dispersion of white light occurs because colors of white light travel at different speeds through
glass prism. Different colours undergo different deviations on passing through prism.
(ii) Rainbow is the example of dispersion of light observed in nature.
It is caused due to dispersion of sunlight by water droplets in the atmosphere. It always forms in the
direction opposite to the sun.
(iii) White light is a mixture of seven colours. The sequence of colours given by the prism is Violet, Indigo,
Blue, Green, Yellow, Orange and Red. VIBGYOR is the acronym for this sequence. The red light bends the
least and violet light bends the most.
Marks: 3

Question 15. "Energy flow in a food chain is unidirectional" Justify this statement. Explain how the
pesticides enter a food chain and subsequently get into our body.
Solution: The flow of energy in the ecosystem is said to be unidirectional because the energy lost as heat
from the living organisms of a food chain cannot be reused by plants in photosynthesis.
Pesticides are non-biodegradable wastes which pass along the food chain from crops to man or other animals
and birds and harm them.
Marks: 3

Question 16. Write one difference between asexual and sexual mode of reproduction. Which species is
likely to have better chances of survival - the one reproducing asexually or the one reproducing sexually?
Justify your answer.
Solution: The difference between asexual and sexual reproduction is that only one parent is needed in
asexual reproduction whereas two parents are needed in sexual reproduction.
The organisms reproducing sexually have better chances of survival because it promotes diversity of
characters in an offspring due to combinations of genes which can lead to variation whereas in asexual
reproduction evolutionary change is not possible as only one parent is involved therefore no variation takes
place.
Marks: 3

Question 17. What is the effect of DNA copying, which is not perfectly accurate, on the reproduction
process? How does the amount of DNA remain constant though each new generation is a combination of
DNA copies of two individuals?
Solution: The DNA copying which is not perfectly accurate in the reproduction process results in variations
in populations for the survival of species.

www.vedantu.com 6 / 18
The amount of DNA remains constant because the gametes are special type of cells called reproductive cells
which contain only half the amount of DNA as compared to the normal body cells of an organism.
Marks: 3

Question 18. List three main factors responsible for the speciation and briefly describe each one of them.
Solution: The process by which new species develop from the existing species is known as speciation. The
important factors which could lead to speciation are:
i. Geographical isolation of a population caused by various type of barriers such as mountain ranges, rivers
and seas.
ii. Genetic drift caused by drastic changes in the frequencies of particular genes by chance alone.
iii. Variations caused in individuals due to natural selection.
Marks: 3

Question 19. “A trait may be inherited, but may not be expressed.” Justify this statement with the help of a
suitable example.
Solution: Some traits are determined by the combined effect of more than one pair of genes. These are
referred to as polygenic or continuous, traits.
An example of this is human stature. The combined size of all of the body parts from head to foot
determines the height of an individual. There is an additive effect. The sizes of all of these body parts are, in
turn, determined by numerous genes. Human skin, hair, and eye color are also polygenic traits because they
are influenced by more than one allele at different loci. The result is the perception of continuous gradation
in the expression of these traits.
Marks: 3

Question 20. List two reasons for carbon forming a large number of compounds. Name the type of bonding
found in most of its compounds. Why does carbon form compounds mainly by this kind of bonding?
Give reason why the carbon compounds
(i) generally have low melting and boiling points.
(ii) do not conduct electricity in molten state.
Solution: The two characteristic properties of the carbon element which leads to the formation of a very
large number of organic compounds are: Catenation and Tetravalency.
Catenation: Carbon has the unique ability to form bonds with other atoms of carbon, giving rise to large
molecules. This property is called catenation.
Tetravalency: Carbon has a valency of four. So, it is capable of bonding with four other atoms of carbon or
atoms of some other mono-valent element.
1. Covalent bonding is found in most of the carbon compounds.
2. Carbon form compounds mainly by covalent bonding because-
The bonds that carbon forms with most other elements are very strong making these compounds
exceptionally stable. One reason for the formation of strong bonds by carbon is its small size. This enables
the nucleus to hold on to the shared pairs of electrons strongly. The bonds formed by elements having larger
atoms are much weaker.
(i) Carbon compounds generally have low melting points and boiling points because-
The force of attraction between the molecules of carbon compounds is not very strong. These weak
intermolecular forces make them very easy to pull apart from each other. Since they are easy to separate,
carbon compounds have low melting and boiling points.

www.vedantu.com 7 / 18
(ii) Carbon compounds do not conduct electricity in molten state because bonding in carbon compounds
does not involve any formation of ions.
Marks: 5

Question 21. (a) List the parts of the human eye that control the amount of light entering into it. Explain
how they perform this function.
(b) Write the function of retina in human eye.
(c) Do you know that the corneal-impairment can be cured by replacing the defective cornea with the cornea
of the donated eye? How and why should we organise groups to motivate the community members to donate
their eyes after death?
Solution: (a) The iris and pupil are the two parts of human eye that controls the amount of light entering the
eyes. The iris automatically adjusts the size of the pupil according to the intensity of light received by the
eye.
If the amount of light received by the eye is large (as during the daytime), then the iris contracts the pupil
(makes the pupil small) and reduces the amount of light entering the eye. On the other hand, if the amount of
light received by the eye is small (as in a dark room or during night), the iris expands the pupil (makes the
pupil large) so that more light may enter the eyes. Thus, the iris regulates (or controls) the amount of light
entering the eye by changing the size of the pupil.
(b) The retina acts as a screen on which the image is formed in the eye. The retina has a large number of
light-sensitive cells. When the image falls on the retina then these light-sensitive cells get activated and
generate electrical signals.
(c) Yes.
There is a need for awareness among various communities about eye donation after death as it will help a
person suffering from an eye ailment to get the treatment.
The following steps can be taken to organize groups to motivate them:
(i) The person who is taking the initiative can communicate with people and after realizing people who are
like minded should gather them and form an awareness group.
(ii) The group should then approach various communities and organizations in their area and promote their
campaign.
(iii) They can also approach various hospitals and clinics to promote the campaign.
Marks: 5

Question 22. (a) Explain the following terms related to spherical lenses:
(i) optical centre
(ii) centres of curvature
(iii) principal axis
(iv) aperture
(v) principal focus
(vi) focal length
(b) A converging lens has focal length of 12 cm. Calculate at what distance the object should be placed from
the lens so that it forms an image at 48 cm on the other side of the lens.
Solution: (a)
(i) Optical centre:
The central point of the lens is known as optical centre. It is represented as O. The optical centre of a lens
has a property that a ray of light passing through it does not suffer any deviation and goes straight.

www.vedantu.com 8 / 18
(ii) Centre of Curvature:
The centre of sphere of part of which a lens is formed is called the centre of curvature of the lens. Since
concave and convex lenses are formed by the combination of two parts of spheres, therefore they have two
centres of curvature. One centre of curvature is usually denoted by C1 and second is denoted by C2.
(iii) Principal Axis:
The principal axis of a lens is a line passing through the optical centre of the lens and perpendicular to both
the faces of the lens.
(iv) Aperture:
The diameter of sphere of part of which a lens is formed is called the aperture.
(v) Principal Focus:
The convex lens converge the rays incident on it after refraction, to a point on the principal axis. This point
is known as principal focus of the convex lens.
The rays incident on concave lens appear to diverge from a point on the principal axis. This point is known
as the principal focus of concave lens.
(vi) Focal Length:
The focal length of a lens is the distance between optical centre and principal focus of the lens.
(b) Given,
Image distance: v = +48 cm (It is on the other side of the lens)
Focal length: f = +12 cm (It is a converging lens or convex lens)
Object distance: u =? (To be calculated)
Now, putting these values in the lens formula:
1 1 1
 
f v u
1 1 1
  
12 48 u
1 1 1
  
u 48 12
1 1 3
 
u 48
1 1
 
u 24
 u = –24 cm
Therefore, the object should be placed at a distance of 24 cm from the convex lens. The minus sign with the
object distance shows that the object is on its left side.
Marks: 5

Question 23. (a) Draw a sectional view of human female reproductive system and label the part where
(i) eggs develop.
(ii) fertilisation takes place.
(iii) fertilised egg gets implanted.
(b) Describe, in brief, the changes the uterus undergoes
(i) to receive the zygote.
(ii) if zygote is not formed.
Solution: (a) (i) Ovary
(ii) Fallopian tubes

www.vedantu.com 9 / 18
(iii) Uterus

(b) (i) If the uterus receives the zygote, the female becomes pregnant. The embryonic development of the
zygote starts immediately. The embryo moves down into the uterus forming a thick and soft lining of blood
vessels around itself. This process is called implantation. After implantation, a special tissue develops
between the uterine wall and the embryo called placenta, where the exchange of nutrients, oxygen and waste
products takes place.
(ii) If the egg released by the ovary is not fertilized and the zygote is not formed, then the thick lining of the
uterus breaks down and comes out through the vagina in the form of blood and mucous. This is called
menstruation.
Marks: 5

Question 24. (a) Name the parts labelled as A, B, C and D in the diagram given below:

(b) What is pollination? State its significance.


(c) How does fertilisation occur in flowers? Name the parts of the flower that develop into (i) seed, and (ii)
fruit after fertilisation.
Solution: (a) A - Pollen grain
B - Pollen tube
C - Ovary
D - Female germ cell
(b) Pollination is the process of transfer of pollen grains from anther to stigma of the flower.
Significance of pollination: Pollination is a significant event because it precedes fertilization. It brings the
two types of gametes closer for the process of fertilization.

www.vedantu.com 10 / 18
Also, cross pollination introduces variations in the plants due to mixing of different genes which increases
adaptability towards environment or surroundings.
(c) Fertilization in plants: In plants, pollination is followed by fertilization. Once the pollen grains are
deposited on the stigma, they form tubes called pollen tubes. The pollen tubes grow through the ovules and
reach the ovary where ovules are located. The pollen tube normally enters the ovule through a small opening
called micropyle. Inside the ovule, the pollen tube releases two male gametes into the embryo sac.

The embryo sac of the ovule contains the egg. One male gamete fuses with the egg. This fusion of male and
female gamete is called syngamy and its product is called zygote. The other male gamete fuse with two
polar bodies and this process is called triple fusion, where three nuclei are involved in the fusion process,
one male gamete and two polar nuclei. Thus, inside each embryo sac, two fusions, syngamy and triple
fusion, takes place. This mechanism of two fusions occurring in an embryo sac is called double-fertilization.
After fertilization, ovules develop into the seeds and ovary develops into the fruit.
Marks: 5

SECTION B
Question 25. In the following diagram showing the structure of embryo of a dicot seed, what are the parts
marked I, II and III sequentially?

(A) Plumule, Cotyledon, Radicle


(B) Plumule, Radicle, Cotyledon
(C) Cotyledon, Plumule, Radicle
(D) Radicle, Plumule, Cotyledon
Solution: (A)
In the figure, the part marked I is Plumule, II is Cotyledon and III is Radicle.
Marks: 1

Question 26. Study the following statements:


I. Wings of birds and wings of bats are homologous organs.
II. Wings of birds and wings of insects are modified forelimbs.
III. Wings of birds and wings of insects are analogous organs.
IV. Wings of birds and forelimbs of horse are homologous organs.

www.vedantu.com 11 / 18
The correct statements are
(A) I and II
(B) II and Ill
(C) III and IV
(D) I and IV
Solution: (C)
III and IV are the correct statements.
Marks: 1

Question 27. Which of the following pairs of two vegetables represent the correct homologous structures?
(A) Sweet potato and potato
(B) Sweet potato and tomato
(C) Carrot and potato
(D) Radish and carrot
Solution: (D)
Radish and carrot are the modifications of roots. Hence, they are the homologous structures.
Marks: 1

Question 28. When you study a slide showing different stages of budding in yeast, you observe the
following stages:
I. The bud may get separated from the parent body and develop into a new individual.
II. The body of the bud develops and gives rise to another baby bud.
III. A bud comes out in any direction from the body of the parent cell.
IV. Thus they may form a colony.
The proper sequence of the above stages is
(A) II, I, III, IV
(B) II, III, I, IV
(C) III, II, l, IV
(D) III, I, II, IV
Solution: (C)
This is the correct sequence of budding in yeast.
Marks: 1

Question 29. A student has to focus his compound microscope to observe a prepared slide showing different
stages of binary fission in Amoeba. The steps he is likely to follow are listed below in a haphazard manner:
I. Adjust the diaphragm and the mirror of the microscope so that sufficient light may enter to illuminate the
slide.
II. Fix the slide on the stage carefully.
III. Adjust the microscope to high power and focus.
IV. Adjust the microscope to low power and focus.
The correct sequence of the above steps to observe the slide under the microscope is
(A) I, II, IV, III
(B) II, I, IV, III
(C) II, IV, I, III
(D) I, IV, II, III

www.vedantu.com 12 / 18
Solution: (B)
This is the correct sequence.
Marks: 1

Question 30. A student takes about 2 mL ethanoic acid in a dry test tube and adds a pinch of sodium
hydrogen carbonate to it. He reports the following observations:
I. Immediately a colourless and odourless gas evolves with a brisk effervescence.
II. The gas turns lime water milky when passed through it.
III. The gas burns with an explosion when a burning splinter is brought near it.
IV. The gas extinguishes the burning splinter that is brought near it.
The correct observations are
(A) I, II and III
(B) II, III and IV
(C) III, IV and I
(D) I, II and IV
Solution: (D)
Statements I, II and IV are true. The carbon dioxide gas gets liberated. It is colourless, odourless and turns
lime water milky. The gas extinguishes the burning splinter when brought near it. This is because; carbon
dioxide does not burn or support combustion.
Marks: 1

Question 31. In an experiment to study the properties of ethanoic acid, a student takes about 3 mL of
ethanoic acid in a dry test tube. He adds an equal amount of distilled water to it and shakes the test tube well.
After some time he is likely to observe that
(A) a colloid is formed in the test tube.
(B) the ethanoic acid dissolves readily in water.
(C) the solution becomes light orange.
(D) water floats over the surface of ethanoic acid.
Solution: (B)
Ethanoic acid is readily soluble in water.
Marks: 1

Question 32. We need 20% aqueous solution of sodium hydroxide for the study of saponification reaction.
When we open the lid of the bottle containing solid sodium hydroxide we observe it in which form?
(A) Colourless transparent beads
(B) Small white beads
(C) White pellets/flakes
(D) Fine white powder
Solution: (C)
Sodium hydroxide is present in the form of white flakes or pellets.
Marks: 1

Question 33. While studying saponification reaction, a student measures the temperature of the reaction
mixture and also finds its nature using blue/red litmus paper. On the basis of his observations the correct
conclusion would be

www.vedantu.com 13 / 18
(A) the reaction is exothermic and the reaction mixture is acidic.
(B) the reaction is endothermic and the reaction mixture is acidic.
(C) the reaction is endothermic and the reaction mixture is basic.
(D) the reaction is exothermic and the reaction mixture is basic.
Solution: (C)
The reaction is endothermic and the reaction mixture is basic in nature.
Saponification is defined as the hydrolysis of an ester under basic conditions leading to the formation of
sodium salt of fatty acids. It is an endothermic reaction meaning it absorbs surrounding heat.
Marks: 1

Question 34. In a locality, hard water, required for an experiment, is not available. However, the following
salts are available in the school laboratory:
1. Sodium sulphate
2. Calcium sulphate
3. Magnesium chloride
4. Sodium chloride
5. Calcium chloride
6. Potassium sulphate
Which of the above salts may be dissolved in water to obtain hard water for the experiment?
(A) 2, 3 and 5
(B) 1, 2 and 5
(C) 1, 2, 4 and 6
(D) 3 and 5 only
Solution: (A)
Hard water is an impure form of water having high mineral contents like, Calcium, Magnesium, sulphides,
chlorides, bicarbonates etc.
Marks: 1

Question 35. A student focused the Sun rays using an optical device 'X' on a screen S as shown.

From this it may be concluded that the device 'X' is a (select the correct option)
(A) Convex lens off focal length 10 cm.
(B) Convex lens of radius of curvature 20 cm.
(C) Convex lens of focal length 20 cm.
(D) Concave mirror of focal length 20 cm.
Solution: (C)

www.vedantu.com 14 / 18
Optical device ‘X’ is a convex lens as sun rays are focused on the screen.
Marks: 1

Question 36. A student has obtained an image of a well-illuminated distant object on a screen to determine
the focal length, F1of the given spherical mirror. The teacher then gave him another mirror of focal length,
F2 and asked him to obtain a focused image of the same object on the same screen. The student found that in
order to focus the same object using the second mirror, he has to move the mirror away from the screen.
From this observation, it may be concluded that both the spherical mirrors given to the student were (select
the correct option)
(A) Concave and F1 < F2
(B) Concave and F1 > F2
(C) Convex and F1 < F2
(D) Convex and F1 > F2
Solution: (A)
Since the image is focused, the spherical mirror is a concave mirror.
For second mirror the distance is increased to focus the image on the screen. Hence, focal length is more
than that of first mirror.
Marks: 1

Question 37. In the following diagram, the path of a ray of light passing through a glass prism is shown:

In this diagram the angle of incidence, the angle of emergence and the angle of deviation respectively are
(select the correct option):
(A) X, R and T
(B) Y, Q and T
(C) X, Q and P
(D) Y, Q and P
Solution: (D)
Marks: 1

Question 38. Study the following diagrams in which the path of a ray of light passing through a glass prism
as traced by four students P, Q, R and S is shown:

www.vedantu.com 15 / 18
The student who has traced the path correctly is
(A) P
(B) Q
(C) R
(D) S
Solution: (B)
Marks: 1

Question 39. A student is using a convex lens of focal length 18 cm to study the image formation by it for
the various positions of the object. He observes that when he places the object at 27 cm, the location of the
image is at 54 cm on the other side of the lens. Identify from the following diagram the three rays that are
obeying the laws of refraction and may be used to draw the corresponding ray diagram.

(A) 1, 2 and 4
(B) 1, 3 and 5
(C) 2, 4 and 5
(D) 2, 3 and 4
Solution: (D)
Ray 2, 3 and 4 are obeying the laws of refraction.
Ray 2 is parallel to the principal axis and passes through the principal focus after refraction.
Ray 3 passes from the optical centre of the lens and emerges without any deviation.
Ray 4 is passing through the principal focus and after refraction from a convex lens emerges parallel to the
principal axis.
Ray 1 and 5 cannot pass through the focus after refraction as they are not parallel to the principal axis.
Marks: 1

Question 40. A student is using a convex lens of focal length 10 cm to study the image formation by a
convex lens for the various positions of the object. In one of his observations, he may observe that when the
object is placed at a distance of 20 cm from the lens, its image is formed at (select the correct option)

www.vedantu.com 16 / 18
(A) 20 cm on the other side of the lens and is of the same size, real and erect.
(B) 40 cm on the other side of the lens and is magnified, real and inverted.
(C) 20 cm on the other side of the lens and is of the same size, real and inverted.
(D) 20 cm on the other side of the lens and is of the same size, virtual and erect.
Solution: (C)
Focal length f = 10 cm
The object is placed at 2F (2 × 10 = 20 cm). Hence the image is also formed at 2F.

Marks: 1

Question 41. A student traces the path of a ray of light passing through a rectangular glass slab and marks
the angle of incidence i, angle of refraction r and angle of emergence e, as shown.

The correctly marked angle(s) is/are


(A) i only
(B) e only
(C) r only
(D) i and e
Solution: (C)

www.vedantu.com 17 / 18
Other angles are not taken with respect to normal.
Marks: 1

Question 42. After tracing the path of a ray of light passing through a rectangular glass slab for four
different values of the angle of incidence, a student reported his observations in tabular form as given below:

The best observation is


(A) I
(B) II
(C) III
(D) IV
Solution: (A)
For glass slab, i = e
Refractive index of glass = 1.5
sin 30
 1.5
sin19
30  29
Marks: 1

www.vedantu.com 18 / 18
CBSE-XII-2017 EXAMINATION CBSE-X-2015 EXAMINATION

SCIENCE
SET-3 Paper & Solution
Time: 3 Hrs. Max. Marks: 90

General Instructions:
1. The question paper comprises two Sections, A and B. You are to attempt both the sections.
2. All questions are compulsory.
3. All questions of Section A and all questions of Section B are to be attempted separately.
4. Question numbers 1 to 3 in Section A are one-mark questions. These are to be answered in one word or in one
sentence.
5. Question numbers 4 to 7 in Section A are two-mark questions. These are to be answered in about 30 words
each.
6. Question numbers 8 to 19 in Section A are three-mark questions. These are to be answered in about 50 words
each.
7. Question numbers 20 to 24 in Section A are five-mark questions. These are to be answered in about 70 words
each.
8. Question numbers 25 to 42 in Section B are multiple choice questions based on practical skills. Each question
is a one-mark question. You are to select one most appropriate response out of the four provided to you.

SECTION A
1. Write the number of covalent bonds in the molecule of butane, C4H10.
Answer:
There are thirteen covalent bonds—ten C-H and three C-C bonds—present in a molecule of butane.

2. Name two simple organisms having the ability of regeneration.


Answer:
Hydra and Planaria have the ability of regeneration.

3. Which of the following are always at the second tropic level of food chains?
Answer:
Primary consumers (green plants) are always at the second trophic level.

4. Draw a ray diagram to show the path of the reflected ray corresponding to an incident ray of light parallel
to the principal axis of a convex mirror and show the angle of incidence and angle of reflection on it.
Answer:
A light ray is incident on a convex mirror parallel to the principal axis. The ray diagram is shown below.

www.vedantu.com 1 / 15
CBSE-X-2015 EXAMINATION

In the above diagram, ‘i’ is the angle of incidence and ‘r’ is the angle of reflection.

5. Why is sustainable management of natural resources necessary? Out of the two—reuse and recycle—
which, in your opinion, is better to practice? Give reason.
Answer:
Sustainable management of natural resources is necessary to preserve the natural resources for the future
generations and also to control environmental pollution.
Reusing is better than recycling because recycling requires a large amount of energy and money, but reusing
creates lesser air and water pollution.

6. What is meant by biodiversity? List two advantages of conserving forests and wild life.
Answer:
The existence of many different kinds of plants and animals in an environment is called biodiversity.
Two advantages of conserving forests and wild life:
i. They add to the natural beauty of the environment.
ii. They provide valuable things which are required for our survival.

7. Write the name and general formula of a chain of hydrocarbons in which an addition reaction with
hydrogen is possible. State the essential condition for an addition reaction. Stating this condition, write a
chemical equation giving the name of the reactant and the product of the reaction.
Answer:
The addition of hydrogen is possible in alkenes and alkynes. This is because of the presence of double and
triple bonds, respectively.
The general formula of alkenes is C n H2n and that of alkynes is Cn H2n-2 Conditions for addition reactions
are
• Presence of an unsaturated compound, i.e. an unsaturated hydrocarbon.
• Presence of a species to be added to an unsaturated compound.
• Presence of a catalyst such as finely divided palladium or nickel.

CH 2  CH 2  H 2 
Ni
 CH 3  CH 3
ETHENE ETHANE

CH 3  C  CH  2 H 2 
Ni
 CH 2  CH  CH 3
PROPYNE PROPENE

www.vedantu.com 2 / 15
CBSE-X-2015 EXAMINATION

8. List two tests for experimentally distinguishing between an alcohol and a carboxylic acid and describe
how these tests are performed.
Answer:
Carboxylic acid can be distinguished from an alcohol by performing the following tests:
i. Test with 3 NaHCO3 solution in water.
On adding carboxylic acid to baking soda, carbon dioxide is liberated with brisk effervescence.
On adding a solution of baking soda to alcohol, no brisk effervescence occurs.
ii. Test with blue litmus solution.
Carboxylic acid turns blue litmus red. There is no change in colour when a blue litmus solution is added to
alcohol.
Answer:
Given below are some elements of the modern periodic table. Atomic number of the element is given in
parentheses.
A (4), B (9), C (14), D(19), E(20)
(a) Select the element that has one electron in the outermost shell. Also, write the electronic configuration of
this element.
(b) Which two elements amongst these belong to the same group? Give reasons for your answer.
(c) Which two elements amongst these belong to the same period? Which one of the two has bigger atomic
radius?
Answer:
(a) Element D (19) has one electron in its outermost shell.
Its electronic configuration is 2, 8, 8, 1.
(b) Elements A (4) and E (20) have two electrons in their outermost shells.
Electronic configuration of A: 2, 2
Electronic configuration of E: 2, 8, 8, 2
Since they both have a valency of two, they belong to group 2 of the periodic table.
(c) Elements A (4) and B (9) belong to the second period, and elements D (19) and E (20) belong to the
fourth period of the periodic table.
Since the effective nuclear charge which pulls the outermost electron closer to the nucleus increases from
left to right in a period, the atomic radii of the elements decreases.
A (4) has a bigger atomic radius than B (9) and D (19) has a bigger atomic radius than E (20).

10. Taking the example of an element of atomic number 16, explain how the electronic configuration of the
atom of an element relates to its position in the modern periodic table and how valency of an element is
calculated on the basis of its atomic number.
Answer:
Atomic number of the element = 16
Electronic configuration = 2, 8, 6
The period number is equal to the number of shells which starts filling up in it.
The atom of an element has three shells. So, the period number is 3.
The atom of an element has six valence electrons in the outermost shell.
So, the group number of the element will be 16 (6 + 10).
The valency of an element is determined by the number of valence electrons present in the outermost shell.
The atom of an element has six valence electrons in the outermost shell, so the valency of the element is 2.

11. Name the parts A, B and C shown in the diagram and write their functions

www.vedantu.com 3 / 15
CBSE-X-2015 EXAMINATION

Answer:
A - Stigma
Function: Pollen lands and germination starts
B - Pollen tube
Function: It carries the pollen to the egg cell for fertilisation.
C - Egg cell
Function: It fuses with the male gamete to form a zygote.

12. List any four methods of contraception used by humans. State in brief two advantages of adopting such
preventive methods.
Answer:
Four methods of contraception used by humans—
Intrauterine devices, oral contraceptive methods, surgical methods and natural methods (coitus interrupts).
Two advantages of adopting such preventive methods:
i. Helps in preventing unwanted pregnancies
ii. Reduces the chance of getting STDs such as AIDS

13. What are chromosomes? Explain how in sexually reproducing organisms the number of chromosomes in
the progeny is maintained.
Answer:
Chromosomes are thread-like structures found in the nucleus at the time of cell division. They are made of
proteins and DNA.
In sexually reproducing organisms, the gametes undergo meiosis, and hence, each gamete contains only half
a set of chromosomes. When two gametes fuse, the zygote formed contains the full set of chromosomes.
Hence, the formation of gametes by meiosis helps to maintain the number of chromosomes in the progeny.

14. Explain the following:


(a) Speciation
(b) Natural Selection
Answer:

www.vedantu.com 4 / 15
CBSE-X-2015 EXAMINATION

(a) Speciation: The process by which new species develop from the existing species is known as speciation.
The factors which could lead to speciation are:
i. Geographical isolation of population caused by various types of barriers such as mountain ranges, rivers
and seas. This leads to reproductive isolation because of which there is no flow of genes between separated
groups of population.
ii. Genetic drift caused by drastic changes in the frequencies of particular genes by chance alone.
iii. Variations caused in individuals because of natural selection.
(b) Natural Selection: Natural selection is the process of evolution of a species whereby characteristics
which help individual organisms to survive and reproduce are passed on to their offspring, and those
characteristics which do not help are not passed on. Charles Darwin proposed the theory of natural selection.
According to him, nature selects the fittest.
There are always changes in the progeny when an animal reproduces by sexual reproduction.
Example: If one of the progeny of deer is tall and the other is short, then the tall one with long legs will
survive. Because the progeny with short height cannot reach the leaves of tall trees and cannot get food, they
will starve and hence die. Thus, it proves the theory of natural selection.

15. Explain with an example for each, how the following provides evidences in favour of evolution in
organisms:
(a) Homologous organs
(b) Analogous organs
(c) Fossils
Ans.
(a) Homologous organs: Organs which have the same basic structure but different functions are called
homologous organs.
Example: The forelimbs of a man, lizard, frog, bird and bat have the same basic design of bones, but they
perform different functions. The forelimbs of a man are used for grasping, the forelimbs of a lizard are used
for running, the forelimbs of a frog are used to prop up the front ends of the body when at rest and the
forelimbs of a bird and bat are modified for flying. Hence, all these organisms use their forelimbs for
performing different functions, but the forelimbs have originated from the same structural pattern.
(b) Analogous organs: Organs which have different basic structure but similar appearance and perform
similar functions are called analogous organs. Example: The wings of an insect and a bird have different
structures, but they perform the same function of flying. Because the wings of insects and birds have
different structures but perform similar functions, they are analogous organs.
(c) Fossils: The remains of dead animals or plants which lived in the remote past are known as fossils. The
fossils provide evidence for evolution. For example, a fossil bird called Archaeopteryx looks like a bird, but
it has many other features which are found in reptiles. It has feathered wings like those of birds but teeth and
tail like those of reptiles. Therefore, Archaeopteryx is a connecting link between the reptiles and birds and
hence suggests that birds have evolved from reptiles.

16. With the help of scattering of light, explain the reason for the difference in colours of the Sun as it
appears during sunset/sunrise and noon.
Answer:
At the time of sunrise and sunset, when the Sun is near the horizon, sunlight travels a greater distance
through the atmosphere to reach us. During this time, most of the shorter wavelengths present in it are
scattered away from our line of sight by the molecules of air and other fine particles in the atmosphere. So,

www.vedantu.com 5 / 15
CBSE-X-2015 EXAMINATION

light reaching us directly from the rising or setting Sun consists mainly of the longer wavelength red colour
because of which the Sun appears red.
Thus, at sunrise and sunset, the Sun and the surrounding sky appear red. At noon, the Sun is overhead. So,
the sunlight has to travel a relatively shorter distance. Hence, there is only slight scattering of shorter
wavelengths of blue and violet colour. Therefore, the Sun appears mostly white.

17.An object of height 5 cm is placed perpendicular to the principal axis of a concave lens of focal length 10
cm. If the distance of the object from the optical centre of the lens is 20 cm, determine the position, nature
and size of the image formed using the lens formula.
Answer:
Given: Height of the object = h = 5 cm
Focal length of the concave lens = f = −10 cm
Object distance = u = −20 cm
Using the lens formula, we get
1 1 1
 
f v u
1 1 1
  
10 v 20
1 1 1
  
10 v 20
1 1 1
  
10 20 v
2  1 1
 
20 v
3 1
 
20 v
 v  6.67cm

Hence, the image is formed 6.67 cm in front of the lens on the same side as the object.
Because v is negative, we can say that the image is virtual.
From the magnification formula for the lens, we get
h' v
m 
h u
vh
h' 
u
6.67(5)
 h' 
20
 h '  1.67
Hence, the size of the image is h′ = 1.67 cm.
Because the height of the image is positive and smaller than the height of the object, the image is erect and
diminished. So, we can conclude that the image is virtual, erect and diminished.

18. Differentiate between biodegradable and non-biodegradable substances with the help of one example
each. List two changes in habit that people must adopt to dispose non-biodegradable waste, for saving the
environment.

www.vedantu.com 6 / 15
CBSE-X-2015 EXAMINATION

Answer:
Biodegradable wastes Non-biodegradable wastes
(a) Waste materials which can be broken (a) Waste materials which cannot be
down to non-poisonous substances in broken down into non-poisonous or
nature in due course of time by the action harmless substances in nature are called
non-biodegradable wastes non-biodegradable wastes
(b) Examples: Cattle dung, wool, paper, (b) Examples: Plastics, polythene bags,
compost metal articles, glass objects
The changes which people must adopt to dispose non-biodegradable wastes for saving the environment are
environment are
(a) Household waste, chemical waste and hospital waste should be disposed of by dumping them in the low-
lying areas of the ground called a landfill.
(b) Broken plastic articles such as buckets, bowls, cups, plates etc. should be sent to plastic processing
factories.

19.Write the importance of ciliary muscles in the human eye. Name the defect of vision that arises due to
gradual weakening of the ciliary muscles. What types of lenses are required by the person suffering from
this defect to see the objects clearly?
Akshay, sitting in the last row in his class, could not see clearly the words write on the blackboard. When
the teacher noticed it, he announced if any student sitting in the front row could volunteer to exchange his
seat with Akshay. Salman immediately agreed to exchange his seat with Akshay. He could now see the
words written on the blackboard clearly. The teacher thought it fit to send the message to Akshay’s parents
advising them to get his eyesight checked. In the context of the above event, answer the following questions:
(a) Which defect of vision is Akshay suffering from? Which type of lens is used to correct this defect?
(b) State the values displayed by the teacher and Salman.
(c) In your opinion, in what way can Akshay express his gratitude towards the teacher and Salman?
Answer:
The curvature of the eye lens can be adjusted by the ciliary muscles. This changes the focal length of the
lens. The defect which arises because of the gradual weakening of the ciliary muscles is known as
presbyopia. A bifocal lens can be used to correct presbyopia. Answers to the context questions:
(a) Akshay is not able to see from a far distance, so he is suffering from myopia or nearsightedness. A
concave lens should be used to correct this defect.
(b) The teacher displayed presence of mind and pro-activeness, and she is of a considerate nature. Salman
displayed the virtue of friendship and is caring in nature.
(c) Akshay should thank the teacher and Salman in front of the entire class.

20. What is meant by power of a lens? Define its SI unit. [5] You have two lenses A and B of focal lengths
+10 cm and –10 cm, respectively. State the nature and power of each lens. Which of the two lenses will
form a virtual and magnified image of an object placed 8 cm from the lens? Draw a ray diagram to justify
your answer. [5]
Answer:
The power of a lens is defined as the reciprocal of its focal length. It is represented by the letter P. The
power P of a lens of focal length f is given as
1
p
f
The SI unit of power is dioptre (D).
Given: Focal length of lens A, FA = +10 cm = +0.1 m

www.vedantu.com 7 / 15
CBSE-X-2015 EXAMINATION

Focal length of lens B, FB = −10 cm = −0.1 m


To calculate the power of lens A:
The power of lens A,
1
p
fA
1
 p
0.1
 p  10 D
The positive sign indicates that it is a converging or convex lens.
To calculate the power of lens B:
The power of lens B,

1
p
fB
1
 p
0.1
 p  10 D
The negative sign indicates that it is a diverging or concave lens.
In a convex lens, when the object is placed between the pole and focus, the image formed is always virtual
and magnified.
On the other hand, a concave lens produces virtual, erect but diminished image. Here the object is placed 8
cm from the lens which is at a distance less than the focal length, i.e. less than 10 cm. Thus, the 8 cm
position of the object placed in front of the convex lens will produce a virtual and magnified image. The
diagram for the same is as shown below:

21. One half of a convex lens of focal length 10 cm is covered with a black paper.
Can such a lens produce an image of a complete object placed at a distance of 30 cm from the lens?
Draw a ray diagram to justify your answer. A 4 cm tall object is placed perpendicular to its principal axis of
a convex lens of focal length 20 cm. The distance of the object from the lens is 15 cm. Find the nature,
position and the size of the image.
Answer:
A convex lens can produce the complete image of the object even though half of the lens is covered. This is
because light coming from the object can be refracted from the other half of the lens. However, the intensity
of light will be reduced.

www.vedantu.com 8 / 15
CBSE-X-2015 EXAMINATION

Given: Height of the object = h = 4 cm


Focal length of the convex lens = f = 20 cm
Object distance = u = −15 cm
Using the lens formula, we get
1 1 1
 
f v u
1 1 1
  
20 v 15
1 1 1
  
20 v 15
1 1 1
  
20 15 v
1 1
 
60 v
 v  60
Hence, the image is formed 60 cm in front of the lens on the same side as the object.
Because v is negative, we can say that the image is virtual. From the magnification formula for the lens, we
get

h' v
m 
h u
vh
h' 
u
60(4)
 h' 
15
 h '  16cm
Hence, the size of the image is h′ = 16 cm.
Because the height of the image is positive and greater than the height of the object, the image is erect and
magnified. So, we can conclude that the image is virtual, erect and magnified.

22.How do Mendel’s experiments show that the


(a) traits may be dominant or recessive
(b) traits are inherited independently
Answer:
(a) Mendel crossed pure bred tall pea plants with pure bred dwarf pea plants and found that only tall pea
plants were produced in the first generation and there were no dwarf pea plants. He concluded that the first
generation showed the traits of only one of the parent plants—tallness. The trait of the other parent plant—
dwarfness— did not show up in the progeny of the first generation.

www.vedantu.com 9 / 15
CBSE-X-2015 EXAMINATION

He then crossed the tall pea plants obtained in the first generation (F1 generation) and found that both tall
plants and dwarf plants were obtained in the second generation (F2 generation) in the ratio of 3:1.
Mendel noted that the dwarf trait of the parent pea plant which disappeared in the first generation progeny
reappeared in the second generation. In this way, Mendel’s experiments with tall and dwarf pea plants
showed that the traits may be dominant and recessive.

(b) When Mendel crossed pure-bred tall pea plants with pure-bred dwarf pea plants, he found that only tall
pea plants were produced in the F1 generation. When he further crossed the tall pea plants of the F1
generation, he found that the tall plants and dwarf plants were obtained in the ratio 3:1 in the F2 generation.
Mendel noted that all the pea plants produced in the F2 generation were either tall or dwarf. There were no
plants with intermediate height (or medium height) in between the tall and dwarf plants.
In this way, Mendel’s experiment showed that the traits (like tallness and dwarfness) are inherited
independently. This is because if the traits of tallness and dwarfness had blended (or mixed up), then
medium-sized pea plants would have been produced.

23.
(a) Name the human male reproductive organ that produces sperms and also secretes a hormone. Write the
functions of the secreted hormone.
(b) Name the parts of the human female reproductive system where
i. fertilisation takes place
ii. Implantation of the fertilised egg occurs Explain how the embryo gets nourishment inside the mother’s
body.
Answer:
(a) Testes produce sperms and secrete a hormone called testosterone. The function of testosterone is to
control the development of male sex organs and male features such as a deeper voice, moustache, beard and
more body hair as compared to females.
(b) i. Fertilisation takes place in the oviduct or fallopian tubes.
ii. Implantation of the fertilised egg occurs in the uterus. After implantation, a disc-like special tissue called
placenta develops between the uterus wall and the embryo. The placenta helps in the exchange of nutrients,
oxygen and waste products between the embryo and the mother. Thus, it provides nourishment to the
growing embryo

www.vedantu.com 10 / 15
CBSE-X-2015 EXAMINATION

24. Both soap and detergent are some type of salts. What is the difference between them? Describe in brief
the cleansing action of soap. Why do soaps not form lather in hard water? List two problems that arise due
to the use of detergents instead of soaps.
Answer:
Difference between soap and detergent: The molecules of soap are sodium or potassium salts of long-chain
carboxylic acids. Detergents are generally ammonium or sulphonate salts of long chain carboxylic acids.
Cleansing action of soap can be described as follows:
• A soap molecule has a tadpole-shaped structure.
• At one end (long non-polar end) of the soap molecule is a hydrocarbon chain which is insoluble in water
but soluble in oil.
• At the other end (short polar end) of the soap molecule, there is a carboxylate ion which is hydrophilic, i.e.
water soluble but insoluble in oil.

• Soap on mixing with water forms a concentrated solution and causes foaming.
• The long non-polar end of soap gravitates towards and surrounds the dirt and absorbs the dust in it.
• The short polar end with the carboxylate ion repels the water away from the dirt.
• A spherical aggregate of soap molecules is formed in the soap solution in water and is called a micelle.
•Thus, the soap molecule dissolves the dirt and our clothes get clean. Soaps do not form lather in hard water
because Hard water contains calcium and magnesium salts. Soap molecules react with calcium and
magnesium salts to form an insoluble precipitate called scum. Two problems arise because of the use of
detergents instead of soap:

i. Soaps are biodegradable, while detergents are non-biodegradable; hence, detergents accumulate in the
environment and cause problems.
ii. Certain phosphate additives are added to detergents. These phosphate additives act as nutrients for algae
which form a thick green scum over the river water and upset the animal life in the river.

SECTION B

www.vedantu.com 11 / 15
CBSE-X-2015 EXAMINATION

25. A student traces the path of a ray of light through a rectangular glass slab for the different values of
angle of incidence. He observes all possible precautions at each step of the experiment. At the end of the
experiment, on analyzing the measurements, which of the following conclusions is he likely to draw?
(A) i   e   r
(B) i   e   r
(C) i   e   r
(D) i   e   r
Answer:
(A)i   e   r
In refraction through a rectangular slab, the angle of incidence is equal to the angle of emergence. Also, the
angle of refraction should be smaller than the angle of incidence.

26.A student traces the path of a ray of light through a triangular glass prism for different values of angle of
incidence. On analyzing the ray diagrams, which one of the following conclusions is he likely to draw?
(A) The emergent ray is parallel to the incident ray.
(B) The emergent ray bends at an angle to the direction of the incident ray.
(C) The emergent ray and the refracted ray are at right angles to each other.
(D) The emergent ray is perpendicular to the incident ray.
Answer:
(B) The emergent ray bends at an angle to the direction of the incident ray. In refraction of light through a
glass prism, there is deviation or change in the path of light passing through the prism.

27.A student obtains a sharp image of the distant window (W) of the school laboratory on the screen (S)
using the given concave mirror (M) to determine its focal length. Which of the following distances should he
measure to get the focal length of the mirror?

(A) MW
(B) MS
(C) SW
(D) MW MS
Answer:
(B) MS The focal length of a concave mirror is the distance between its pole and principal focus. That is, the
distance of the image formed (screen) from the concave mirror will be equal to the focal length of the
concave mirror.

28.A student used a device (X) to obtain/focus the image of a well illuminated distant building on a screen
(S) as shown below in the diagram. Select the correct statement about the device (X).

www.vedantu.com 12 / 15
CBSE-X-2015 EXAMINATION

(A) This device is a concave lens of focal length 8 cm.


(B) This device is a convex mirror of focal length 8 cm.
(C) This device is a convex lens of focal length 4 cm.
(D) This device is a convex lens of focal length 8 cm.
Answer:
(D) This device is a convex lens of focal length 8 cm. The incident rays after passing through the lens
converge at the focus. So, the device 'X' is a converging or a convex lens. The distance between the lens and
the screen gives the focal length of the lens.

29. Given below is the list of vegetables available in the market. Select from these the two vegetables having
homologous structures: Potato, sweet potato, ginger, radish, tomato, carrot, okra (Lady’s finger)

(A) Potato and sweet potato


(B) Radish and carrot
(C) Okra and sweet potato
(D) Potato and tomato
Answer:
(B) Radish and carrot Radish and carrot are homologous structures as these are modifications of the root.
Tomato and okra are fruits. Potato is a modification of the stem.

30. A student was asked to observe and identify the various parts of an embryo of a red kidney bean seed.
He identified the parts and listed them as under: [1]
I. Tegmen
II. Testa
III. Cotyledon
IV. Radicle
V. Plumule
The correctly identified parts among these are
(A)I, II and III
(B)II, III and IV
(C)III, IV and V
(D)I, III, IV and V
Answer:
(C) III, IV and V An embryo has two large cotyledons and one embryo axis or tigellum. The upper end of
the embryo axis is the plumule, and the lower end of the embryo axis which projects beyond the cotyledons

www.vedantu.com 13 / 15
CBSE-X-2015 EXAMINATION

is the radical. The testa is the thick outer seed coat, and the tegmen is the inner transparent seed coat of
seeds.

31. While preparing soap a small quantity of common salt is generally added to the reaction mixture of
vegetable oil and sodium hydroxide. Which one of the following may be the purpose of adding common
salt?
(A)To reduce the basic nature of the soap
(B)To make the soap neutral
(C) To enhance the cleansing power of the soap
(D)To favour the precipitation of the soap
Answer:
(D) To favour the precipitation of the soap During saponification, the soap formed remains in a suspended
form in the mixture. It is precipitated as a solid from the suspension by adding common salt to the
suspension. This process is called salting out of soap.

32.A student takes about 4 ml of distilled water in four test tubes marked P, Q, R and S. He then dissolves in
each test tube an equal amount of one salt in one test tube, namely sodium sulphate in P, potassium sulphate
in Q, calcium sulphate in R and magnesium sulphate in S. After that he adds an equal amount of soap
solution in each test tube. On shaking each of these test tubes well, he observes a good amount of lather
(foam) in the test tube marked.
(A)P and Q
(B)Q and R
(C) P, Q and S
(D)P, R and S
Answer:
(A) P and Q Lather (foam) is formed by the reaction of soap solution with sodium sulphate and potassium
sulphate in the test tubes P and Q, respectively. They are dissolved in water to give a neutral solution.
Sulphates, chlorides and bicarbonates of calcium and magnesium make the water hard. Thus, the lather is
not formed in the test tubes R and S.

33. What do we observe on pouring acetic acid on red and blue litmus papers? [1]
(A)Red litmus remains red and blue litmus turns red.
(B)Red litmus turns blue and blue litmus remains blue.
(C) Red litmus turns blue and blue litmus turns red.
(D)Red litmus becomes colourless and blue litmus remains blue.
Answer:
(A) Red litmus remains red and blue litmus turns red. Acids turn blue litmus paper red.
They have no effect on red litmus paper.

34. Students were asked to observe the permanent slides showing different stages of budding in yeast under
high power of a microscope.
(a) Which adjustment screw (coarse/fine) were you asked to move to focus the slides?
(b) Draw three diagrams in correct sequence showing budding in yeast.
Answer:
(A) A fine screw is used to focus the slides of budding in yeast under high power of a microscope.
(B) Sequence showing budding in yeast:

www.vedantu.com 14 / 15
CBSE-X-2015 EXAMINATION

35. When you add sodium hydrogen carbonate to acetic acid in a test tube, a gas liberates immediately with
brisk effervescence. Name this gas. Describe the method of testing this gas.
Answer:
Carbon dioxide gas gets liberated. When a pinch of sodium hydrogen carbonate is added to acetic acid in a
test tube, a brisk effervescence is produced because of the liberation of carbon dioxide gas. When this gas is
passed through the lime water, it turns lime water milky.
This shows that the gas liberated is carbon dioxide gas. The chemical reaction can be represented as
CH3COOH (aq) + NaHCO3(s) → CH3COONa (aq) + H2O(l) + CO2(g) ↑

36. A 4 cm tall object is placed on the principal axis of a convex lens. The distance of he object from the
optical centre of the lens is 12 cm and its sharp image is formed at a distance of 24 cm from it on a screen on
the other side of the lens. If the object is now moved a little away from the lens, in which way (towards the
lens or away from the lens) will he have to move the screen to get a sharp image of the object on it again?
How will the magnification of the image be affected?
Answer:
Given that Object distance, u = −12 cm
Image distance, v = 24 cm

1 1 1
 
f v u
1 1 1
  
f 24 12
1 1 1
  
f 24 12
1 1 2
  
f 24
1 3
 
f 24
 f  8cm
 The focal length of the lens is 8 cm.
Now if the object is moved away from the lens, the screen has to be moved towards the lens. This is because
when we move the object away from the lens, the object distance is increased. Hence, by the lens formula,
the image distance decreases.
Magnification is given as
v
m
u
Because the image distance (v) decreases, the value of magnification also decreases.

www.vedantu.com 15 / 15
CBSE-XII-2017 EXAMINATI CBSE – X – 2016 EXAMINATION

SCIENCE
SET-3 Paper & Solution
Time: 3 Hrs. Max. Marks: 90

General Instructions:
1. The question paper comprises two Sections, A and B. You are to attempt both the sections.
2. All questions are compulsory.
3. There is no choice in any of the questions.
4. All questions of Section A and all questions of Section B are to be attempted separately.
5. Question numbers 1 to 3 in Section A are one-mark questions. These are to be answered in one word or in one sentence.
6. Question numbers 4 to 6 in Section A are two-mark questions. These are to be answered in about 30 words each.
7. Question numbers 7 to 18 in Section A are three-mark questions. These are to be answered in about 50 words each.
8. Question numbers 19 to 24 in Section A are five-mark questions. These are to be answered in about 70 words each.
9. Question numbers 25 to 33 in Section B are multiple choice questions based on practical skills. Each question is a one-mark
question. You are to select one most appropriate response out of the four provided to you.
10. Question numbers 34 to 36 in Section B are two-marks questions based on practical skills. These are to be answered in brief.

SECTION A

Question 1. Write the name and structure of an aldehyde with four carbon atoms in its molecule.
Solution: 1-butanal

Marks: 1

Question 2. List two functions ovary of human female reproductive system.


Solution: Two functions of the ovaries of the human female reproductive system are
1. To produce ova, which are female gametes
2. To secrete the female hormones oestrogen and progesterone.
Marks: 1

Question 3. In a food chain of frog, grass, insect and snake, assign trophic level to frog.
Solution: Grass   Insect   Frog   Snake
Frog is a secondary consumer.
Marks: 1

Question 4. The refractive indices of glass and water with respect to air are 3/2 and 4/3 respectively. If
speed of light in glass is 2 × 108 m/s, find the speed of light in water.
Solution: Given: ng = 3/2 and nw = 4/3
Speed of light in air
Refractive index of glass, ng =
Speed of light in glass
3 Speed of light in air

2 2 108
3
 Speed of light in air =  2 108  3108 m/s
2

www.vedantu.com 1 / 16
 Thus, the speed of light in air is 3 × 108 m/s.
Marks: 2

Question 5. List four stakeholders which may be helpful in the conservation of forests.
Solution: Four stakeholders which may help in the conservation of forests are
1. The Forest Department of the Government which owns the forest land and controls the resources from
forests
2. People who live in and around the forest and are dependent on forest produce to lead their lives
3. Industrialists who use various forest products for their factories
4. Forest and wildlife activists who want to see forests in their original form.
Marks: 2

Question 6. The construction of large dams leads to social and environmental problems. List two problems
of each category.
Solution: Social problems arise because the construction of dams causes the displacement of a large number
of tribals and peasants who are then rendered homeless. They are neither given sufficient compensation or
rehabilitation nor do they get any benefits from these projects.
Construction of dams leads to several environmental problems such as deforestation and loss of biodiversity
because large areas of forest land get submerged in water leading to an ecological imbalance.
Marks: 2

Question 7. The position of eight elements in the Modern Periodic Table is given below where atomic
numbers of elements are given in the parenthesis.
Period No.
2 Li(3) Be(4)
3 Na(11) Mg(12)
4 K(19) Ca(20)
5 Rb(37) Sr(38)
(i) Write the electronic configuration of Ca.
(ii) Predict the number of valence electrons in Rb.
(iii) What is the number of shells in Sr?
(iv) Predict whether K is a metal or a non-metal.
(v) Which one of these elements has the largest atom in size?
(vi) Arrange Be, Ca, Mg and Rb in the increasing order of the size of their respective atoms.
Solution: (i) Electronic configuration of Ca (20): 2, 8, 8, 2
(ii) Rb belongs to Group 1, and all Group 1 elements have one valence electron.
(iii) Sr belongs to Period 5, and thus, it has five shells.
(iv) K is a metal with electronic configuration 2, 8, 8, 1. Thus, it will donate its one electron to acquire the
noble gas configuration.
(v) The atomic size increases down the group and decreases across a period. Rb is the element which has the
largest atomic size.
(vi) Be < Mg < Ca < Rb
Marks: 3

www.vedantu.com 2 / 16
Question 8. Write three different chemical reactions showing the conversion of ethanoic acid to sodium
ethanoate. Write balanced chemical equation in each case. Write the name of the reactants and the products
other ethanoic acid and sodium ethanoate in each case.
Solution: Three different chemical reactions showing the conversion of ethanoic acid to sodium ethanoate:
2CH3COOH + Na2CO3   2CH3COONa + H2O + CO2
CH3COOH + NaHCO3 
 CH3COONa + H2O + CO2
CH3COOH + NaOH 
 CH3COONa + H2O + CO2
Marks: 3

Question 9. An element `X’ belong to 3rd period and group 13 of the Modern Periodic Table.
(a) Determine the valence electrons and the valency of `X’.
(b) Molecular formula of the compound formed when `X’ reacts with an element `Y’ (atomic number = 8).
(c) Write the name and formula of the compound formed when `X’ combines with chlorine.
Solution: Period of X = 3
Group of X = 13
Atomic number of X = 13
Electronic configuration: 2, 8, 3
(a) Number of valence electrons = 3 and valency = 3
(b) Atomic number of Y = 8
Electronic configuration = 2, 6
Valency of Y = 2
Molecular formula of the compound formed when ‘X’ reacts with an element ‘Y’ is X2Y3.
(c) Atomic number of Cl = 17
Electronic configuration = 2, 8, 7
Valency of Y = 1
Molecular formula of the compound formed when ‘X’ reacts with an element ‘Y’ is XCl3.
Marks: 3

Question 10. An element `X’ has mass number 35 and number of neutrons 18. Write atomic number and
electronic configuration of `X’. Also write group number, period number and valency of `X’.
Solution: Mass number of X = 35
Number of neutrons = 18
Atomic number = Mass number – Number of neutrons
= 35 – 18 = 17
Electronic configuration of X = 2, 8, 7
Group of X = 17
Period of X = 3
Valency of X = 1.
Marks: 3

Question 11. (a) List two reasons for the appearance of variations among the progeny formed by sexual
reproduction.

www.vedantu.com 3 / 16
(b)

(i) Name the part marked `A’ in the diagram.


(ii) How dose `A’ reachese part `B’?
(iii) State the importance of the part `C’.
(iv) What happens to the part marked `D’ after fertilisation is over?
Solution: (a) Two reasons for the appearance of variations among the progeny formed by sexual
reproduction are
1. Sexual reproduction results in new combinations of genes which are brought together during the
formation of gametes.
2. Gene combinations are different in gametes.
(b)
(i) Part ‘A’ labelled is pollen grain.
(ii) Part ‘B’ is stigma. The pollen grain reaches the stigma during pollination.
(iii) Part ‘C’ is the pollen tube. The pollen tube carries the gametes to the embryo sac for fertilisation.
(iv) Part ‘D’ is the egg cell. After fertilisation with the male gametes, the egg cell forms the zygote.
Marks: 3

Question 12. Define reproduction. How does it help in providing stability to the population of species?
Solution: Reproduction is the ability of living organisms to produce living beings similar to them.
Reproduction maintains the number of chromosomes specific to a species in each generation. Multicellular
organisms have specialised cells in their gonads, which have only half the number of chromosomes and half
the amount of DNA as compared to non-reproductive body cells. So, when these germ cells from 2 different
individuals combine during sexual reproduction to form a new individual, it results in the re-establishment
of the number of chromosomes and the DNA content in the new generation. Thus, it provides stability to the
population of a species.
Marks: 3

Question 13. Explain the term “Regeneration” as used in relation to reproduction of organisms. Describe
briefly how regeneration is carried out in multicellular organisms like Hydra.
Solution: Regeneration is the ability of organisms to generate lost or damaged parts of the body.
When a hydra is bisected anywhere in the upper 7th or 8th part of the body column, the upper half will
regenerate a foot at its basal end and the lower half will regenerate a head at its apical end; each half

www.vedantu.com 4 / 16
generates the organ which it is missing. The regeneration is precise, and the head and foot are always formed
specifically at the apical and basal ends, respectively.
Marks: 3

Question 14. “Two areas of study namely `evolution’ and `classification' are interlinked”. Justify this
statement.
Solution: All living things are identified and categorised on the basis of their body design in form and
function. After a certain body design comes into existence, it will shape the effects of all other subsequent
design changes simply because it already exists. So, characteristics which came into existence earlier are
likely to be more basic than characteristics which have come into existence later. This means that the
classification of life forms will be closely related to their evolution. On connecting this idea of evolution to
classification, it is seen that some groups of organisms with ancient body designs have not changed very
much. However, other groups of organisms have acquired their particular body designs relatively recently.
Because there is a possibility that complexity in design will increase over evolutionary time, it may not be
wrong to say that older organisms are simpler, while younger organisms are more complex.
Marks: 3

Question 15. How do Mendel’s experiment show that traits are inherited independently?
Solution: Mendel carried out dihybrid crosses by crossing two pea plants differing in contrasting traits of
two characters. For example, he crossed a pea plant having yellow colour and round seed characters with
another pea plant bearing green colour and wrinkled seed characters. In the F2 generation, he obtained pea
plants with two parental and two recombinant phenotypes as yellow round and green wrinkled (parental) and
yellow wrinkled and green round (recombinant). This indicated that traits separated from their original
parental combinations and got inherited independently.

Marks: 3

www.vedantu.com 5 / 16
Question 16. The activities of man had adverse effects on all forms of living organisms in the biosphere.
Unlimited exploitation of nature by man disturbed the delicate ecological balance between the living and
non-living components of the biosphere. The unfavourable conditions created by man himself threatened the
survival not only of himself but also of the entire living organisms on the mother earth. One of your
classmates is an active member of `Eco club’ of your school which is creating environmental awareness
amongst the school students, spreading the same in the society and also working hard for preventing
environmental degradation of the surroundings.
(a) Why is it necessary to conserve our environment?
(b) State the importance of green and blue dust-bins in the safe disposal of the household waste.
(c) List two values exhibited by your classmate who is an active member of Eco-club of your school.
Solution: (a) It is necessary to conserve our environment because
(i) It helps in protecting the ozone layer.
(ii) It helps in maintaining animal and human food chains.
(iii) It provides us with many useful products such as medicines and wood.
(b) Disposal of household waste is carried out in green and blue bins, respectively. It will be very useful in
the separate disposal of biodegradable and non-biodegradable wastes. This will also ensure the application
of 3Rs—reduce, reuse and recycle.
(c) Two values exhibited are
(i) Creating environmental awareness among students and society.
(ii) Working hard on prevention of environmental degradation of surroundings.
Marks: 3

Question 17. The image formed by a spherical mirror is real, inverted and is of magnification –2. If the
image is at a distance of 30 cm from the mirror, where is the object placed? Find the focal length of the
mirror. List two characteristics of the image formed if the object is moved 10 cm towards the mirror.
Solution: Given: Magnification, m = –2
Distance of the image, v = –30 cm
v
Magnification, m = 
u
v (30)
u    
m (2)
 u = –15 cm
Substituting these values in the mirror formula
1 1 1
 
f v u
1 1
 
(30) (15)
1 1

f 10
 f = –10 cm
When the object is moved 10 cm towards the mirror the new position of the object is
u' = – (15 – 10) = –5 cm
Substituting the new value in the mirror formula
1 1 1
 
f v u'

www.vedantu.com 6 / 16
1 1 1 1 1
   
v' f u' 10 (5)
1 1

v' 10
 v' = 10 cm
Thus, the image is located 10 cm behind the mirror.
v' 10
And magnification, m' = –  
u' (  5)
m' = 2
Since magnification is positive the image is erect and virtual.
Thus, the image is erect, virtual and magnified in nature.
Marks: 3

Question 18. Describe an activity to show that colours of white light splitted by a glass prism can be
recombined to get white light by another identical glass prism. Also draw ray diagram to show the
recombination of the spectrum of white light.
Solution: The seven colours of a spectrum can be recombined to give back white light as
1. Two identical glass prisms are placed such that their refracting surfaces are in opposite direction (placed
inverted). When a beam of light is allowed to fall on the surface of one prism, a patch of ordinary white light
is obtained on a screen placed behind the second prism.
2. The first prism disperses the white light into seven coloured rays. The second prism receives all the seven
coloured rays from the first prism and recombines them into original white light. This is because the
refraction produced by the second prism is equal and opposite to that produced by the first prism. Hence, the
light coming out of the second prism will be white.

Marks: 3

Question 19. It is desired to obtain an erect image of an object, using concave mirror of focal length of 12
cm.
(i) What should be the range of distance of an object placed in front of the mirror?
(ii) Will the image be smaller or larger than the object? Draw ray diagram to show the formation of image in
this case.
(iii) Where will the image of this object be, if it is placed 24 cm in front of the mirror? Draw ray diagram for
this situation also justify your answer.
Show the positions of pole, principal focus and the centre of curvature in the above ray diagrams.
Solution: (i) To obtain an erect image, the object should be placed within the focus, i.e., between the pole
and the focus. Here, the focal length of the mirror is 12 cm.
Hence, the object should be placed at a distance less than 12 cm.

www.vedantu.com 7 / 16
(ii) The image will be larger than the object (enlarged).

(iii) Since f = 12 cm  Centre of curvature = 2f = 24 cm


For an object placed at a distance 24 cm, i.e., at the centre of curvature of a concave mirror, the image
formed will be real, inverted and of the same size as that of the object.

Marks: 5

Question 20. Define evolution. How does it occur? Describe how fossils provide us evidences in support of
evolution.
Solution: Evolution is the formation of more complex organisms from pre-existing simpler organisms over
a certain period. Accumulation of variation in genetic material forms the basis of evolutionary processes.
Fossils provide a unique view into the history of life by showing the forms and features of life in the past.
Fossils tell us how species have changed across long periods of the Earth’s history.
Importance of fossils in the evolutionary process:
1. Some invertebrates living on the sea bed died and were buried in the sand.
2. More sand was accumulated and formed sandstone under pressure.
3. After millions of years, dinosaurs living in the area died and their bodies were buried in the mud.
4. The mud got compressed into the rock, just above the rock containing earlier invertebrate fossils.
5. Again millions of years later, the bodies of horse-like creatures dying in the area were fossilised in the
rocks above the earlier rocks.
6. Much later, because of erosion and water flow, some rocks wore out and exposed the horse-like fossils.
Marks: 5

Question 21. What is placenta? Describe its structure. State its functions in case of a pregnant human
female.
Solution: The placenta is an organ attached to the lining of the womb during pregnancy. The placenta is
composed of both maternal tissue and tissue derived from the embryo. It contains blood spaces on the
mother’s side and villi on the embryo’s side.
Functions of the placenta:
1. It provides food and oxygen to the foetus.

www.vedantu.com 8 / 16
2. The foetus gives away waste products and carbon dioxide to the mother’s blood for excretion.

Marks: 5

Question 22. A carbon compound `P’ on heating with excess conc. H2SO4 forms another carbon compound
`Q’ which on addition of hydrogen in the presence of nickel catalyst forms a saturated carbon compound
`R’. One molecule of `R’ on combustion forms two molecules of carbon dioxide and three molecules of
water. Identify P, Q and R and write chemical equations for the reactions involved.
Solution: P 
Excess H2SO4
 Q 
H2 /Nickel catalyst
 R 
O2
 2CO2 + 3H2O
P 
Excess H2SO4
Q
CH3CH2OH 
Excess H2SO4
 CH2 = CH2
Q 
H2 /Nickel catalyst
R
CH2 = CH2 
H2 /Nickel catalyst
 CH3 – CH3
R 
O2
 2CO2 + 3H2O
CH3 – CH3 
O2
 2CO2 + 3H2O
CH3CH2OH 
Excess H2SO4
 CH2 = CH2 
H2 /Nickel catalyst
 CH3 – CH3 
O2
 2CO2 + 3H2O
P Q R
Ethanol Ethene Ethane
Marks: 5

Question 23. What is atmospheric refraction? Use this phenomenon to explain the following natural events:
(a) Twinkling of stars
(b) Advanced sun-rise and delayed sun-set.
Draw diagrams to illustrate your answers.
Solution: (a) Stars emit light on their own; when this light travels through the Earth's atmosphere which has
variable optical density, the continuously changing atmosphere refracts the light from the stars in different
amounts from one moment to the next. The light seems to be bright and dim as it keeps changing because of
continuous refraction through the different layers of the atmosphere of the Earth. Hence, we say light
twinkles at night.

www.vedantu.com 9 / 16
(b) The sunrise is advanced because of the atmospheric refraction of sunlight. An observer on the Earth sees
the Sun two minutes before the Sun reaches the horizon. A ray of sunlight entering the Earth's atmosphere
follows a curved path because of atmospheric refraction before reaching the Earth. This happens because of
a gradual variation in the refractive index of the atmosphere. For an observer on the Earth, the apparent
position of the Sun is slightly higher than the actual position. Hence, the Sun is seen before it reaches the
horizon.

The increased atmospheric refraction of sunlight occurs also at sunset. In this case, the observer on the Earth
continues to see the setting Sun for two minutes after the Sun has dipped below the horizon, thus delaying
the sunset. The advanced sunrise and delayed sunset increase the duration of the day by four minutes.
Marks: 5

Question 24. (a) Define focal length of a divergent lens.


(b) A divergent lens of focal length 30 cm forms the image of an object of size 6 cm on the same side as the
object at a distance of 15 cm from its optical center. Use lens formula to determine the distance of the object
from the lens and the size of the image formed.

www.vedantu.com 10 / 16
(c) Draw a ray diagram to show the formation of image in the above situation.
Solution: (a) The focal length of a diverging lens is half the value of its radius of curvature. Conventionally,
the sign of the focal length of the diverging lens is taken as negative.
(b) Given:
f = −20 cm (It is a diverging lens.)
v = −15 cm (Image is formed on the same side of the lens.)
Using the lens formula,
1 1 1
 
f v u
1 1 1
  
u v f
1 1 1
  
( 15) (  30) 30
 u  30 cm
Given: Height of the object, h = 6 cm
Height of the image, h' = ?
v h'
Magnification, m = 
u h
v (15)
 h'  h  6 
u (30)
 h'  3 cm
(c)

Marks: 5

SECTION B

Question 25. A student while observing an embryo of a pea seed in the laboratory listed various parts of the
embryo as given below:
Testa, Tegmen, Radicle, Plumule, Micropyle, Cotyledon.
On examining the list the teacher remarked that only three parts are correct.
Select three correct parts from the above list:
(a) Testa, Radicle, Cotyleddon
(b) Tegmen, Radicle, Micropyle
(c) Cotyledon, Plumule, Testa

www.vedantu.com 11 / 16
(d) Radicle, Cotyledon, Plumule
Solution: (d) Radicle, Cotyledon, Plumule
A dicot embryo consists of radicle, plumule and a pair of cotyledons. Testa, tegmen and micropyle are the
parts of the seed coat.

Marks: 1

Question 26. If you are asked to select a group of two vegetables, out of the following, having homologous
structures which one would you select?
(a) Carrot and radish
(b) Potato and sweet potato
(c) Potato and tomato
(d) Lady finger and potato
Solution: (a) Carrot and radish
Homologous structures are fundamentally same in structure and origin but are modified to perform different
functions in different organisms. They indicate common ancestry. From the given plants, carrot and radish
are homologous structures because they both are underground roots. While potato is a stem, sweet potato is
a root, tomato is a fruit and lady finger is a vegetable.
Marks: 1

Question 27. In the following ray diagram the correctly marked angle are:

(a) i and e
(b) A and D
(c) i , e and D
(d) r , A and D
Solution: (d) r , A and D
The angle between the normal and the incident ray is the angle of incidence.
The angle between the normal and the emergent ray is the angle of emergence.

www.vedantu.com 12 / 16
The correctly marked angles are shown in the diagram below:

Marks: 1

Question 28. In your laboratory you trace the path of light rays through a glass slab for different values of
angle of incidence ( i ) and in each case measure the values of the corresponding angle of refraction ( r )
and angle of emergence ( e ). On the basis of your observations your correct conclusion is:
(a) i is more than r , but nearly equal to e
(b) i is less then r , but nearly equal to e
(c) i is more than e , but nearly equal to r
(d) i is less than e , but nearly equal to r
Solution: (a) i is more than r , but nearly equal to e

On entering a glass slab, the incident light gets refracted. According to Snell’s law, we get
sin i

sin r
For glass  > 1
 sin r < sin i
or r < i

www.vedantu.com 13 / 16
In refraction of light through a glass slab, the emergent ray is parallel to the incident ray. Thus, i = e .
Marks: 1

Question 29. To determine the approximate value of the focal length of a given concave mirror, you focus
the image of a distant object formed by the mirror on a screen. The image obtained on the serene, as
compared to the object is always:
(a) Laterally inverted and diminished
(b) Inverted and diminished
(c) Erect and diminished
(d) Erect and highly diminished
Solution: (b) inverted and diminished
Images obtained on the screen are always diminished and inverted in nature.
Marks: 1

Question 30. Suppose you have focused on a screen the image of candle flame placed at the farthest end of
the laboratory table using a convex lens. If your teacher suggests you to focus the parallel rays of the sun,
reaching your laboratory table, on the same screen, what you are expected to do is to move the:
(a) lens slightly towards the screen
(b) lens slightly away from the screen
(c) lens slightly towards the sun
(d) lens and screen both towards the sun
Solution: (a) lens slightly towards the screen
To focus the parallel rays of the Sun, the lens should be slightly moved towards the screen.
Marks: 1

Question 31. For preparing soap in the laboratory we require an oil and a base. Which of the following
combinations of an oil and a base would be best suited for the preparation of soap?
(a) Castor oil and calcium hydroxide
(b) Turpentine oil and sodium hydroxide
(c) Castor oil and sodium hydroxide
(d) Mustard oil and calcium hydroxide
Solution: (a) P and Q
Dissolving chloride salts of calcium or magnesium in distilled water will make the water hard and lather
formation will take place with difficulty.
Marks: 1

Question 32. A student puts a drop of reaction mixture of a saponification reaction first a blue litmus paper
and then on a red litmus paper. He may observe that:
(a) There is no change in the blue litmus paper and the red litmus paper turns white.
(b) There is no change in the red litmus paper and the blue litmus paper turns red.
(c) There is no change in the blue litmus paper and the red litmus paper turns blue.
(d) No change in colour is observed in both the litmus papers.
Solution: (c)

www.vedantu.com 14 / 16
The molecules of soap are sodium or potassium salts of long chain carboxylic acids. So, when a student puts
a drop of reaction mixture of a saponification reaction first on a blue litmus paper and then on a red litmus
paper, he will observe that there is no change in the blue litmus paper and the red litmus paper turns blue.
Marks: 1

Question 33. In the neighbourhood of your school, hard water required for an experiment is not available.
Select from the following group of salts available in your school, a group each member of which, if
dissolved in distilled water, will make it hard:
(a) Sodium chloride, calcium chloride
(b) Potassium chloride, sodium chloride
(c) Sodium chloride, magnesium chloride
(d) Calcium chloride, magnesium chloride
Solution: (d) Calcium chloride, magnesium chloride
Dissolving chloride salts of calcium or magnesium in distilled water will make the water hard, and lather
formation will take place with difficulty.
Marks: 1

Question 34. A student is observing a permanent slide showing sequentially the different stages of asexual
reproduction taking place in yeast. Name this process and draw diagrams, of what he observes, in a proper
sequence.
Solution: Yeast reproduces asexually by the process of budding. Different stages of budding as observed by
the student are depicted below:

Marks: 2

Question 35. An object of height 2.5 cm is placed at a distance of 15 cm from the optical centre `O’ of a
convex lens of focal length 10 cm. Draw a ray diagram to find the position and size of the image formed.
Mark optical `O’, principal focus F and height of the image on the diagram.
Solution: Ray diagram:

Marks: 2

www.vedantu.com 15 / 16
Question 36. A student adds a spoon full of powdered sodium hydrogen carbonate to a flask containing
ethanoic acid. List two main observations, he must note in his note book, about the reaction that takes place.
Also write chemical equation foe the reaction.
Solution: Two main observations about the reaction:
(i) Brisk effervescence of carbon dioxide which turns lime water milky.
(ii) It is a neutralisation reaction and heat is released.
CH3COOH + NaHCO3   CH3COONa + H2O + CO2
Marks: 2

www.vedantu.com 16 / 16
CBSE-XII-2017 EXAMINATI CBSE - X - 2017 EXAMINATION

SCIENCE
SET-3 Paper & Solution
Time: 3 Hrs. Max. Marks: 90

General Instructions:
1. The question paper comprises two Sections, A and B. You are to attempt both the sections.
2. All questions are compulsory.
3. There is no choice in any of the questions.
4. All questions of Section A and all questions of Section B are to be attempted separately.
5. Question numbers 1 to 3 in Section A are one-mark questions. These are to be answered in one word or in one sentence.
6. Question numbers 4 to 6 in Section A are two-mark questions. These are to be answered in about 30 words each.
7. Question numbers 7 to 18 in Section A are three-mark questions. These are to be answered in about 50 words each.
8. Question numbers 19 to 24 in Section A are five-mark questions. These are to be answered in about 70 words each.
9. Question numbers 25 to 33 in Section B are multiple choice questions based on practical skills. Each question is a one-mark
question. You are to select one most appropriate response out of the four provided to you.
10. Question numbers 34 to 36 in Section B are two-marks questions based on practical skills. These are to be answered in brief.

SECTION A

Question 1. Write the molecular formula of the 2nd and 3rd member of the homologous series where the first
member is ethyne.
Solution: The molecular formula of the 2nd and 3rd members of a homologous series where the first member
is ethyne (C2H2) is formed by adding -CH2-:
2nd member of alkyne series = propyne (C3H4) CH3 - CH2 - C  CH
3rd member of alkyne series = butyne (C4H6) CH3 - CH2 - C  CH
Marks: 1

Question 2. Why is variation important for a species?


Solution: Variation increases the chances of survival of a species in a constantly changing environment.
Marks: 1

Question 3. In the following food chain, 20,000 J of energy was available to the plants. How much energy
would be available to man in this chain?
Plants  Sheep  Man
Solution: According to the 10% law, 2 J of energy will be available for the man in this chain.
Marks: 1

Question 4. An object is placed at a distance of 15 cm from a concave lens of focal length 30 cm. List four
characteristic (nature, position, etc.) of the image formed by the lens.
Solution: Given,
u = -15 cm (It is to the left of the lens)
f = -30 cm (It is a concave lens)
1 1 1
Using the lens formula  
f v u
1 1 1 1 1
   
v f u (30) (15)

www.vedantu.com 1 / 14
1 3 1
  
v 30 10
 v = -10 cm
The negative sign of the image distance shows that the image is formed on the left side of the concave
mirror. Thus, the image formed by a mirror is virtual, erect and on the same side as the object.
Marks: 2

Question 5. You being an environmentalist are interested in contributing towards the conservation of nature
resources. List four activities that you can do on your own.
Solution: Four activities which can be done as an environmentalist to conserve natural resources are
1) Using public transport for commuting instead of using a personal vehicle.
2) Avoid using clothes, accessories or articles made of animal skin.
3) Using energy-efficient electrical appliances to save electricity.
4) Ensuring no leakage of water taps and pipes at home.
Marks: 2

Question 6. Why are coal and petroleum categorized as natural resources? Given a reason as to why they
should be used judiciously.
Solution: Coal and petroleum have been formed by natural processes. They have been formed by the
degeneration of dead plant and animal biomass buried deep in the earth several million years ago.
It has taken millions of years for the formation of these fossil fuels, and the present rate of consumption of
these fossil fuels far exceeds the rate at which they are formed.
If exhausted, these resources will not be available for use in the near future, and hence, they should be used
judiciously.
Marks: 2

Question 7. Distinguish between esterification and saponification reactions with the help of the chemical
equations for each. State one use of each (i) esters, and (ii) saponification process.
Solution:
Esterification Saponification
1. Carboxylic acid reacts with alcohols in the 1. On treating an ester with a base such as NaOH, it
presence of a little conc. sulphuric acid to form is converted back to alcohol and sodium salt of
esters. carboxylic acid.
2. Example: Ethanoic acid reacts with ethanol in the 2. Example: Ethyl ethanoate on reaction with
presence of a little conc. sulphuric acid to form sodium hydroxide gives ethanol and sodium
esters. ethanoate.

Use of esters:
Esters are used in synthetic flavours, perfumes, cosmetics, lacquers, paints and varnishes.
Use of saponification reaction:
It is used in the preparation of soaps on a commercial basis.
Marks: 3

www.vedantu.com 2 / 14
Question 8. Write the structural formula of ethanol. What happens when it is heated with excess of conc.
H2SO4 at 443 K? Write the chemical equation for the reaction stating the role of conc. H 2SO4 in this
reaction.
Solution: Structural formula of ethanol:

On adding conc. sulphuric acid to ethanol and heating the mixture up to 443 K (443 K - 273 = 170 °C) gives
ethene.

The role of conc. H2SO4 in the above reaction is that it is used as a dehydrating agent and causes
dehydration of ethanol.
Marks: 3

Question 9. What is periodicity in properties of elements with reference to the Modern Periodic Table? Why
do all the elements of the same group have similar properties? How does the tendency of elements to gain
electrons change as we move from left to right in a period? State the reason of this change?
Solution: Properties which reappear at regular intervals or in which there is gradual variation at regular
intervals are called periodic properties, and the phenomenon is known as the periodicity of elements.
Elements in the same group or column have the same number of electrons in their outermost shell. Hence,
elements of the same group have similar properties.
On moving across a period from left to right, the tendency to gain electrons increases. This is due to an
increase in the nuclear pull and a decrease in atomic size.
Marks: 3

Question 10. Write the electronic configuration two elements X and Y whose atomic numbers are 20 and 17
respectively. Write the Molecular formula of the compound formed when element. X reacts with element Y.
Draw electron-dot structure of the product and also state the nature of the bond formed between both the
elements.
Solution: Atomic number of X = 20, electronic configuration = 2, 8, 8, 2
Atomic number of Y = 17, electronic configuration = 2, 8, 7
Molecular formula of the compound = XY2
Electron-dot structure of the compound:

An ionic bond is formed between the two elements.


Marks: 3

Question 11. How did Mendel explain that it is possible that a trait is inherited but not expressed in an
organism?
Solution: Mendel explained that it is possible that a trait is inherited but not expressed
in an organism with the help of a monohybrid cross.

www.vedantu.com 3 / 14
1. He crossed pure-bred tall plants (TT) with pure-bred dwarf plants (tt).
2. The progeny he received in the first filial generation was tall. The dwarfness did not show up in the F1
generation.
3. He then crossed the tall pea plants of the F1 generation and found that the dwarf plants were obtained in
the second generation. He obtained three tall plants and one dwarf plant.
Marks: 3

Question 12. What is an organic evolution? It cannot be equated with progress. Explain with the help of a
suitable example.
Solution: Organic evolution can be defined as the slow, progressive, natural and sequential development in
primitive organisms to form more complex organisms or a new species.
Marks: 3

Question 13. List the two types of reproduction. Which one of the two is responsible for bringing in more
variations in its progeny and how?
Solution: Two types of reproduction:
1. Sexual reproduction
2. Asexual reproduction
Sexual reproduction is responsible for bringing in more variations in its progeny.
It takes place by the combination of male and female gametes.
Gametes are formed from one cell which involves copying of DNA and the cellular apparatus. DNA
copying is not absolutely accurate, and errors result in new variations. With every DNA copied, a new
variation is introduced, and this DNA copy may already have several variations accumulated from the
previous generations.
Marks: 3

Question 14. What is vegetative propagation? State two advantages and two disadvantages of this method.
Solution: Vegetative propagation is a type of reproduction in which several plants are capable of producing
naturally through their roots, stems and leaves.

www.vedantu.com 4 / 14
Advantages of vegetative propagation:
Plants not capable of producing sexually are produced by this method.
It is a fast and certain method to obtain plants with desired features.
Disadvantages of vegetative propagation:
There is no possibility for variation.
The new plant grows in the same area as the parent plant which leads to competition for resources.
Marks: 3

Question 15. List three techniques that have been developed to prevent pregnancy. Which one of these
techniques is not meant for males? How does the use of these techniques have a direct impact on the health
and prosperity of a family?
Solution: Techniques to prevent pregnancy:
Use of intra-uterine devices such as Lippes loop and Copper T
Use of condoms
Surgical methods (e.g., tubectomy)
Use of intra-uterine devices is not meant for males.
The use of these techniques will keep the mother in good health. With a small family size, parents will be
able to provide quality resources to the child such as food, clothes and education. This will improve the
overall mental and physical well-being of the family.
Marks: 3

Question 16. “A lens can form a magnified erect image as well as magnified inverted image of an object
placed in front of it”. State the nature of this lens and draw ray diagrams to justify the above statement. Mark
the positions of O, F and 2F in the diagram.
Solution: Convex lens can form a magnified erect image as well as a magnified inverted image of an object
placed in front of it.

www.vedantu.com 5 / 14
Marks: 3

Question 17. What is “dispersion of white light”? Draw a labelled diagram to illustrate the recombination of
the spectrum of white light. Why it is essential that the two prisms used for the purpose should be identical
and placed in an inverted position with respect to each other?
Solution: The phenomenon of splitting of white light into its constituent seven colours on passing through a
glass prism is called dispersion of light.

It is essential to place the two identical prisms in an inverted position with respect to each other because the
refraction produced by the second prism is equal and opposite to that produced by the first prism.
Marks: 3

Question 18. (a) Water is an elixir of life a very important natural resource. Your science teacher wants you
to prepare a plan for a formative assessment activity, “How to save water, the vital natural resource”. Write
any two ways that you will suggest to bring awareness in your neighborhood, on how to save water’.
(b) Name and explain any one way by which the underground water table does not go down further.
Solution: Two ways by which awareness on how to save water can be created in the neighborhood:
1. By bringing to notice the current situation of drought in rural areas and its dreadful effects on humans and
animals
2. Making people realise the importance of water in life and the shortage of water and its consequences in
the near future
Khadin is one way of recharging groundwater.
A khadin consists of a 100-300-m long embankment called bund made of earth. The bund is built across the
lower edge of the sloping farmland.
Rainwater from the catchment area flows down the slope and collects in front of the bund forming a
reservoir.
Pathways through the bund allow excess water to flow through and collect in shallow wells dug behind the
bund.

www.vedantu.com 6 / 14
The water which collects in the reservoir and wells seeps into the land and recharges the groundwater.
Marks: 3

Question 19. With the help of one example for each, distinguish between the acquired traits and the
inherited traits. Why are the traits/experiences acquired during the entire lifetime of an individual not
inherited in the next generation? Explain the reason of this fact with an example.
Solution:
Acquired Trait Inherited Trait
A trait or characteristic which develops in response A characteristic feature inherited from the previous
to the environment and cannot be inherited. generation.
Example: A person learns to swim. Example: A girl has brown eyes just like her
mother.
Only those traits are inherited which are developed because of changes in genes.
An acquired trait or experience is developed as a response to the environment; it is not inherited. These are
not developed due to the changes in genes.
Example: Human beings experiencing weight loss due to starvation. There will be reduction in weight as a
response to starvation. This will result in the reduction in the number of body cells or overall body-mass
ratio of the individual. It will not have any effect on the genetic constitution of the individual. Because there
is no change in the gene of the individual, it is not an acquired trait.
Marks: 5

Question 20. (a) Write the functions of each of the following parts in a human female reproductive system:
(i) Ovary
(ii) Uterus
(iii) Fallopian tube
(b) Write the structure and functions of placenta in a human female.
Solution: (i) Ovary: It produces female gametes. One ovum is released by one ovary every month. It also
secretes hormones oestrogen and progesterone.
(ii) Uterus: It protects and nourishes the developing embryo.
(iii) Fallopian tube: It passes down the ovum towards the uterus released by the ovary.
Structure of the placenta in human female:
1. The placenta is a disc which is embedded in the uterine wall.
2. It contains villi on the embryo side. The mother’s end of the placenta has blood spaces which surround the
villi.
Functions of the placenta in human female:
1. Nutrients and oxygen are received by the foetus from the mother’s blood.
2. The foetus gives away waste products and carbon dioxide to the mother’s blood for excretion.
Marks: 5

Question 21. Why certain compounds are called hydrocarbons? Write the general formula for homologous
series of alkanes, alkenes and alkynes and also draw the structure of the first member of each series. Write
the name of the reaction that converts alkenes into alkanes and also write a chemical equation to show the
necessary conditions for the reaction to occur.
Solution: Certain compounds contain only carbon and hydrogen. So, these organic compounds are called
hydrocarbons.

www.vedantu.com 7 / 14
General formula for the homologous series of alkanes = CnH2n+2
First member of the alkane family is methane.

General formula for the homologous series of alkenes = CnH2n


First member of the alkene family is ethene.

General formula for the homologous series of alkynes = CnH2n-2


First member of the alkyne family is ethyne.

Catalytic hydrogenation is the reaction used to convert alkenes to alkanes.

Marks: 5

Question 22. (a) A student suffering from myopia is not able to see distinctly the object placed beyond 5 m.
List two possible reasons due to which this defect of vision may have arisen. With the help of ray diagrams
explain.
(i) Why the student is unable to see distinctly the objects placed beyond 5 cm from his eyes.
(ii) The type of the corrective lens used to restore proper vision and how this defect is corrected by the use
of this lens.
(b) If, in this case, the numerical value of the focal length of the corrective lens is 5 m, find the power of the
lens as per the new Cartesian sign convention.
Solution: (a) This defect may arise due to excessive curvature of the eye lens or elongation of the eyeball.
(i) A person with this defect has the far point nearer than infinity. Such a person may see clearly up to a
distance of a few meters.

In a myopic eye, the image of a distant object is formed in front of the retina and not at the retina itself.

www.vedantu.com 8 / 14
(ii) This defect can be corrected by using a concave lens of suitable power. A concave lens of suitable power
will bring the image back onto the retina and thus the defect is corrected.

(b) Given: Focal length f = -5 m ( it is a concave lens)


1 1
Power, P    0.2D
f (in m) 5
The negative sign indicates that it is a diverging lens or concave lens.
Marks: 5

Question 23. Analyse the following observation table showing variation of image distance (v) with object
distance (u) in case of a convex lens and answer the questions that follow without doing any calculations:

(a) What is the focal length of the convex lens? Give reason to justify your answer.
(b) Write the serial number of the observation which is not correct. On what basis have you arrived at this
conclusion?
(c) Select an appropriate scale and draw a ray diagram for the observation at S. No. 2. Also find the
approximate value of magnification.
Solution: (a) When the object distance and the image distance are the same, it means that the object is
placed at 2f or the image is formed at 2f. From the table, it is clear that 2f = 40 cm. Therefore, the focal
length of the convex lens is 20 cm.
(b) Serial number 6 is incorrect. Given that the object is placed at 15 cm which is between the focal length
and the lens. Thus, the image should be formed on the same side as the object. The data given in the
observation serial number 6 does not satisfy the condition.
(c)

www.vedantu.com 9 / 14
v
Magnification, m 
u
Let us consider the third observation where
u = -40 cm and v = 40 cm
v 40
m  
u 40
 m = -1
Marks: 5

Question 24. (a) If the image formed by a mirror for all position of the object placed in front of it is always
diminished, erect and virtual, state the type of the mirror and also draw a ray diagram to justify your answer.
Write one use such mirrors are put to and why.
(b) Define the radius of curvature of spherical mirrors. Find the nature and focal length of a spherical mirror
whose radius of curvature is +24 cm.
Solution: (a) A convex mirror always forms a diminished, erect and virtual image of the object placed in
front of it.

Use of a convex mirror:


i. Convex mirrors are commonly used as rear view mirrors in vehicles.
ii. They are preferred because they always give an erect image, although diminished. Also, they have a
wider field of view as they are curved outwards. Thus, convex mirrors enable the driver to view a much
larger area than would be possible with a plane mirror.

www.vedantu.com 10 / 14
(b) The radius of curvature of a spherical mirror is the radius of the sphere of which the reflecting surface of
the spherical mirror is a part and represented by R.
Radius of curvature R = 24 cm
Radius of curvature = 2 × focal length
i.e., R = 2 f
24 = 2 × f
24
f  12
2
f = 12 cm
Marks: 5

SECTION B
Question 25. Study the following ray diagram:

In this diagram, the angle of incidence, the angle of emergence and the angle of deviation respectively have
been represented by
(A) y, p, z
(B) x, q, z
(C) p, y, z
(D) P, z, y
Solution: (A) y, p, z
The angle between the incident ray and the normal is known as the angle of incidence, and the angle
between the emergent ray and the normal is known as the angle of emergence. The emergent ray is bent at
an angle with the direction of the incident ray. This angle is called the angle of deviation.
Marks: 1

Question 26. A student very cautiously traces the path of a ray through a glass slab for different values of
the angle of incidence (  i). He then measures the corresponding values of the angle of refraction (  r) and
the angle of emergence (  e) for every value of the angle of incidence. On analysing these measurements of
angles, his conclusion would be
(A)  i >  r >  e
(B)  i =  e >  r
(C)  i <  r <  e
(D)  i =  e <  r
Solution: (B)  i =  e >  r
Because the emergent ray is parallel to the incident ray, the angle of incidence is equal to the angle of
emergence. The refracted ray travels from a rarer medium to a denser medium (considering the first
refraction); it bends towards the normal. Thus, the angle of incidence is greater than the angle of refraction.
If we consider the second refraction, then light travels from a denser medium to a rarer medium, due to

www.vedantu.com 11 / 14
which it bends away from the normal after refraction. So, in this case, the angle of refraction is again less
than the angle of emergence.
Marks: 1

Question 27. Study the given ray diagrams and select the correct statement from the following:

(A) Device X is a concave mirror and device Y is a convex lens, whose focal lengths are 20 cm and 25 cm
respectively.
(B) Device X is a convex lens and device Y is a concave mirror, whose focal lengths are 10 cm and 25 cm
respectively
(C) Device X is a concave lens and device Y is a convex mirror, whose focal lengths are 20 cm and 25 cm
respectively.
(D) Device X is a convex lens and device Y is a concave mirror, whose focal lengths are 20 cm and 25 cm
respectively.
Solution: (D) Device X is a convex lens and device Y is a concave mirror, whose focal lengths are 20 cm
and 25 cm respectively.
Device X is a convex lens and device Y is a concave mirror whose focal lengths are 20 cm and 25 cm,
respectively. A parallel ray of light incident on a concave mirror gets reflected, and the image is seen on a
screen placed before it. A parallel ray of light incident on a convex lens converges to a point.
Marks: 1

Question 28. A student obtains a blurred image of a distant object on a screen using a convex lens. To
obtain a distinct image on the screen he should move the lens.
(A) away from the screen
(B) towards the screen
(C) to a position very far away from the screen
(D) either towards or away from the screen depending upon the position of the object.
Solution: (B) Inverted and diminished
When the object is at infinity, the distance of the image from the lens will be equal to the focal length of the
lens.
Marks: 1

Question 29. While studying the saponification reaction, what do you observe when you mix an equal
amount of colorless vegetable oil and 20% aqueous solution of NaOH in a beaker?
(A) The color of the mixture has become dark brown
(B) A brisk effervescence is taking place in the beaker
(C) The outer surface of the beaker has become hot
(D) The outer surface of the beaker has become cold
Solution: (C) The outer surface of the beaker has become hot.

www.vedantu.com 12 / 14
When 20% NaOH solution was added to the beaker containing vegetable oil, it was observed that the
beaker’s surface was warm when touched.
A whitish suspension was formed by heating the mixture of vegetable oil and 20% NaOH solution.
Marks: 1

Question 30. When you add a few drops of acetic acid to a test-tube containing sodium bicarbonate powder,
which one of the following is your observation?
(A) No reaction takes place
(B) A colorless gas with pungent smell is released with brisk effervescence
(C) A brown colored gas is released with brisk effervescence
(D) Formation of bubbles of a colorless and odorless gas
Solution: (D) Formation of bubbles of a colorless and odorless gas.
NaHCO3 + HC2H3O2  NaC2H3O2 + H2O + CO2
There is double displacement in which acetic acid reacts with sodium bicarbonate to form sodium acetate
and carbonic acid.
NaHCO3 + HC2H3O2  NaC2H3O2 + H2CO3
Carbonic acid is unstable and undergoes a decomposition reaction to produce carbon dioxide gas.
H2CO3  H2O + CO2
Carbon dioxide escapes from the solution as bubbles.
Marks: 1

Question 31. A student requires hard water for an experiment in his laboratory which is not available in the
neighbouring area. In the laboratory there are some salts, which when dissolved in distilled water can
convert it into hard water. Select from the following groups of salts, a group, each salt of which when
dissolved in distilled water will make it hard.
(A) Sodium chloride, Potassium chloride
(B) Sodium sulphate, Potassium sulphate
(C) Sodium sulphate, Calcium sulphate
(D) Calcium sulphate, Calcium chloride
Solution: (D) Calcium sulphate, calcium chloride
Hard water can be prepared by dissolving sulphates, chlorides or bicarbonate salts of Ca2+ or Mg2+ ions.
Marks: 1

Question 32. To perform an experiment to identify the different parts of an embryo of a dicot seed, first of
all you require a dicot seed. Select dicot seeds from the following group:
Wheat, Gram, Maize, Pea, Barley, Ground-nut
(A) Wheat, Gram and Pea
(B) Gram, Pea and Ground-nut
(C) Maize, Pea and Barley
(D) Gram, Maize and Ground-nut
Solution: (B) Gram, Groundnut, Pea
Dicot seeds have two cotyledons.
Marks: 1

Question 33. The following vegetables are kept in a basket:

www.vedantu.com 13 / 14
Potato, Tomato, Radish, Brinjal, Carrot, Bottle-gourd
Which two of these vegetables correctly represent the homologous structures?
(A) Carrot and Tomato
(B) Potato and Brinjal
(C) Radish and Carrot
(D) Radish and Bottle-gourd
Solution: (C) Radish and Carrot
The structures which are same in structure and origin but are modified to perform different functions are
called homologous structures.
Although radish and carrot store food and are used as food, the nutrients which each provide are different.
Marks: 1

Question 34. Draw in sequence (showing the four stages), the process of binary fission in Amoeba.
Solution: Binary fission in amoeba:

Marks: 2

Question 35. A student focuses the image of a candle flame, placed at about 2 m from a convex lens of focal
length 10 cm, on a screen. After that he moves gradually the flame towards the lens and each time focuses
its image on the screen.
(A) In which direction does he move the lens to focus the flame on the screen?
(B) What happens to the size of the image of the flame formed on the screen?
(C) What difference is seen in the intensity (brightness) of the image of the flame on the screen?
(D) What is seen on the screen when the flame is very close (at about 5 cm) to the lens?
Solution: (a) As the candle is moved towards the lens, the image distance increases. Thus, the student
moves the lens away from the screen to focus the image.
(b) The size of the image increases when the object is moved towards the lens.
(c) Intensity decreases.
(d) When the candle is moved very close to the lens, no image is formed on the screen. A virtual image is
formed behind the candle on the same side of the screen.
Marks: 2

Question 36. Mention the essential material (chemicals) to prepare soap in the laboratory. Describe in brief
the test of determining the nature (acidic/alkaline) of the reaction mixture of saponification reaction.
Solution: Chemicals required: Vegetable oil, common salt and 20% sodium hydroxide solution.
When a red litmus paper is dipped in the reaction mixture, the paper changes its colour to blue. Hence, the
reaction mixture of the saponification reaction is basic in nature.
Marks: 2

www.vedantu.com 14 / 14
X - CBSE BOARD - 2018
Date: 16.03.2018 Science - Question Paper Solutions

SECTION - A
1. Write the energy conversion that takes place in a hydropower plant.
Ans. Potential energy of water stored in a dam is converted into kinetic energy of the falling water. The
water falls on the turbine, so kinetic energy of the flowing water is converted into the kinetic energy of
the armature of the generator connected to the turbine. Then kinetic energy is converted into the
electrical energy known as hydro-electricity.

2. A Mendelian experiment consisted of breeding pea plants bearing violet .flowers with pea plants
bearing white flowers. What will be the result in F1 pregeny ?
Ans. In mendelian experiment, breeding of pea plants bearing violet flowers with pea plant bearing white
flower leds to production of all violet coloured flowers (F1 progeny plants). The plants bearing violet
coloured of the flower is dominant over write coloured flower in pea plant.

3. (a) Name one gustatory receptor and one alfactory receptor present in human beings.
(b) Write a and b in the given flow chart of neuron through which information travels as an electrical
impulse.

Dendrite 
 a 
 b 
 End point of Newron

Ans. (a) Gustatory receptors – Tongue


Olfactory receptors – Nose

(b) Dendrite →Cyton →Axon →End point of Newron

www.vedantu.com 1
4. If the image formed by a spherical mirror for all positions of the object placed in front of it is always
erect and diminished, what type of mirror is it ? Draw a labelled ray diagram to support your answer.
Ans. To get erect and diminished image mirror used is convex mirror.

5. A compound ‘X’ on heating with excess conc. sulphuric acid at 443 K gives an unsaturated compound
‘Y’. ‘X’ also reacts with sodium metal to evolve a colourless gas ‘Z’. Identify ‘X’, ‘Y’ and ‘Z’. Write
the equation of the chemical reaction of formation of ‘Y’ and also Write the role of sulphuric acid in
the reaction.
Ans. Compound ‘X’ on heating with excess conc. sulphuric acid at 443 gives unsaturated compound.
443 K
CH 3 – CH 2 – OH  H 2 SO4  conc.   CH 2  CH 2  H 2O
 Alcohol 
CH 3 – CH 2 – OH  Na 
 CH 3 – CH 2 – ONa  H 2

Concentrated H 2 SO4 act as a dehydrating agent

X  C2 H 5OH

Y  C2 H 4

Z  H2 

6. State the laws of refraction of light. Explain the term ‘absolute refractive index of a medium’ and write
an expression to relate it with the speed of light in vacuum.
Ans. Laws of Refraction of light :
Refraction of light follows the following two laws :
First Law : The incident ray, the normal to the transparent surface at the point of incidence and the
refracted ray, all lie in one and the same plane.

Second Law : The ratio of sine of the incidence angle  i  to the sine of the refracted angle of the
medium is called refractive index. It is denoted by n.

sin i
i.e., n
sin r

Refractive index of second medium with respect to the first medium is denoted by 2 n1 .

2 2

www.vedantu.com 2
sin i
Thus, eq.(i) can be written as 2 n1 
sin r
This law is called Snell’s law as it was stated by Prof. Willenbrord Snell (Dutch mathematicial and
astronomer).
Absolute Refractive index :
Absolute refractive index of a medium is defined as the ratio of the speed of light in vacuum or air to
the speed of light in the medium. It is denoted by n.

speed of light in air c


Then, n  
speed of light in medium v
It has no unit.

OR
What is meant by power of a lens ? Write its SI unit. A student uses a lens of focal length 40 cm and
another of –20 cm. Write the nature and power of each lens.

1
Ans. P
f
Unit of Power is Dioptre (D)
Focal length = 40 cm
Focal length = –20 cm

1 100
Convex lens P    2.5 D
f 40

1 100
Concave lens P    5 D
f 20

7. Write one main difference between asexual and sexual mode of reproduction. Which species is likely
to have comparatively better chances of survival - the one reproducing asexually or the one reproducing
sexually ? Give reason to justify your answer.
Ans. Asexual reproduction involves single individual which produces new generation whereas sexual
reproduction involves two individuals one is male parent & other is female parent to produce new
individuals. Sexually reproducing species is likely to have comparitively better chance of survival as it
involves two different individuals.
Sexual mode of reproduction incorporates such a process of combining DNA from two different
gametes of two different parents i.e., male and female gametes of male and female parents respectively.
Thus sexual reproduction involves variation in the new individuals which helps in survival of the species.

www.vedantu.com 3
8. Show how would you join three resistors, each of resistance 9  so that the equivalent resistance of
the combination is (i) 13 (ii) 6  ?
9
9
9 9 9  9
Ans. (i) 9  9  2  9   4.5   9   13.5 
9
two 9  resistors are connected in parallel and one in series.

(ii) 2 resistors connected in series   9  9    18 

18  and 9  are connected in series. 9 9

18  9
 6 9
18  9

OR
(a) Write Joule’s law of heating.
(b) Two lamps, one rated 100 W; 220 V, and the other 60 W; 220 V, are connected in parallel to
electric mains supply. Find the current drawn by two bulbs from the line, if the supply voltage is
220 V.
Ans. (a) Joule’s law of heating H = I2Rt
When electric current flows through resistance element, the flowing charges suffer resistance, the
work done to overcome resistance is converted to heat energy.
(b) P1  100W ,V1  220V

P2  60W ,V2  220V

P  VI

P1 100 10
I1     0.45 Amp
V1 220 22

P2 60 3
I2     0.27 Amp
V2 220 11

9. Name the hormones secreted by the following endocrine glands and specify one function of each :
(a) Thyroid (b) Pituitary (c) Pancreas
Ans. Following are the hormones & functions secreted by given glands.
(a) Thyroid : Thyroid gland secretes thyroxin hormone.
Function : Thyroxin regulates carbohydrate, protein & fat metabolism in the body to provide
best balance for the growth.

www.vedantu.com 4
(b) Pituitary gland : Pitutary gland secretes growth hormone.
Function : Growth hormone regulates growth & development of the body.
(c) Pancreas : Pancreas secretes insulin hormone.
Function : Insulin helps in regulating blood sugar level.

10. 2 mL of sodium hydroxide solution is added to a few pieces of granulated zinc metal taken in a test
tube. When the contents are warmed, a gas evolves which is bubbled through a soap solution before
testing. Write the equation of the chemical reaction involved and the test to detect the gas. Name the
gas which will be evolved when the same metal reacts with dilute solution of a strong acid.

Ans. (i) Zn  2 NaOH 


 Na2 ZnO2  H 2 
(ii) The liberated H2 gas can be detected by putting a burning matchstick at the opening of test tube,
it will burn with a pop sound with blue flume.

(iii) Zn  2 HCl 
 ZnCl2  H 2 

OR
The pH of a salt used to make tasty and crispy pakoras is 14. Identify the salt and write a chemical
equation for its formation. List its two uses.
Ans. (i) The salt is NaHCO3.

(ii) Na2 CO3  H 2O  CO2 


 2 NaHCO3
(iii) (a) In medicine as antacid.
(b) Used in cakes to make it more fluffy.

11. (a) Why are most carbon compounds poor conductors of electricity ?
(b) Write the name and structure of a saturated compound in which the carbon atoms are arranged in
a ring. Give the number of single bonds present in this compound.
Ans. (a) An carbon is a non-metal so most of its compound are poor conductors of electricity.
H H
H H
H H Name - Cyclohexane
H Single bond - 18
(b) H
H H
H H

www.vedantu.com 5
12. Decomposition reactions require energy either in the form of heat or light or electricity for breaking
down the reactants. Write one equation each for decomposition reactions where energy is supplied in
the form of heat, light and electricity.

Ans. (a) 2 Pb  NO3  2 s    2 PbO  4 NO2  O2

(b) 2 AgCl s   Sunlight 


 2 Ag s   Cl2 g 

electricity
(c) 2 NaCl l    2 Na l   Cl2 g 

13. What is a dam ? Why do we seek to build large dams ? While building large dams, which three main
problems should particularly be addressed to maintain peace among local people ? Mention them.
Ans. A dam is a barrier constructed to hold back water and raise its level forming a reservoir used to
generator electricity or as a water supply.
Large dams are built in order to generate electricity from water supply.
The dams can be constructed only in a limited number of place preferably in hilly terrains. large areas
of agricultural land and human habitation are to be sacrificed as they get submerged. large eco-
systems are destroyed when submerged under the water in dams. The vegetation large amounts of
methane which is also a green-house gas. It created the problem of satisfactory rehabilitation of
displaced people.

14. Students in a school listened to the news read in the morning assembly that the mountain of garbage
in Delhi, suddenly exploded and various vehicles got buried under it. Several people were also injured
and there was traffic jam all around. In the brain storming session the teacher also discussed this issue
and asked the students to find out a solution to the problem of garbage. Finally they arrived at two
main points - one is self management of the garbage we produce and the second is to generate lee
garbage at individual level.
(a) Suggest two measures to manage the garbage we produce.
(b) As an individual, what can we do to generate the least garbage ? Give two points.
(c) List two values the teacher instilled in his students in this episode.
Ans. (a) (i) Garbage should be kept in proper place indicated by municipality.
(ii) We can put wet garbage and dry garbage in separate container so that they can be used for
recycling.
(b) (i) We should be careful in kitchen not to cook more food than necessary.
(ii) We can also give green vegetable and food waste to nearby pet animals like cow etc.

(c) (i) Teacher is environment corrsious.


(ii) Teacher wants to make his students responsible citizens.

www.vedantu.com 6
15. (a) List the factors on which the resistance of a conductor in the shape of a wire depends.
(b) Why are metals good conductors of electricity whereas glass is a bad conductor of electricity?
Give reason.
(c) Why are alloys commonly used in electrical heating devices ? Give reason.
Ans. (a) Resistance of conductor depends on following factor:
(i) Resistance of conductor is directly proportional to length (l) of the conductor.
R
(ii) Resistance of conductor is inversely proportional to area of cross section of conductor.
1
R
A
(iii) Resistance also depends on a material of conductor (  )

 R
A
(iv) Resistance and resistivity also depends on temperature.
(b) Metals have more free electrons than glass to carry currents. That’s why glass is bad conductor
and metals are good conductors.
(c) Alloys are used rather than pure metals in electrical heating devices, since they have low electrical
conductivity and also low melting point.

16. (a) State Fleming’s left hand rule.


(b) Write the principle of working of an electric motor.
(c) Explain the function of the following parts of an electric motor.
(i) Armature (ii) Brushes (iii) Split ring
Ans. (a) Hold the forefinger the centre finger and the thumb of your left hand at right angles to one another
If the forefinger points in the direction of magnetic field, and centrefinger points in the direction of
current, the thumb gives direction of motion conductor.
(b) Electric motor is based on the principal that a current carrying conductor placed perpendicular to
the magnetic field experience a force.
(c) (i) Armature : It contains of a single loop of inculated copper wire in the form o a rectangle.
rectangle.
(ii) Brushes : Two carbon brushes B1 and B2 press against the commutator. These brushes act
as the contacts between the commutator and the terminals of the battery.
(iii) Split-ring : It consists of two halves (R1 and R2) of a metallic ring. The two ends of the
armature coil are connected to these two halved of the ring. Commutator reverses the direction
of current in the armature coil.

www.vedantu.com 7
17. (a) Write the function of following parts in human female reproductive system :
(i) Ovary (ii) Oviduct (iii) Uterus
(b) Describe in brief the structure and function of placenta.
Ans. (a) Functions of following parts of human reproductive system.
(i) Ovary – production of egg every month.
(ii) Oviduct or fallopian tube.
Function – carries egg from the ovary to the womb.
(iii) Uterus function :
* Implantation of zygote in the lining of the uterus.
* Nourishes the growing embryo as its lining is thickened due to rich supply of blood.
(b) Structure & function of placenta :
The placenta is a disc – like structure which is embedded in the uterine wall.
Placenta contain villi on the embryo side of the tissue.
Function of placenta :
1. It helps in nourishment of the embryo.
2. It also helps in exchange of gases by providing O2 & removal of CO2 from the foetus.
3. A placenta is also involved in removal of metabolic wastes generated by embryo through the
placenta.

18. (a) Write the steps involved in the extraction of pure metals in the middle of the activity series from
their carbonate ores.
(b) How is copper extracted from its sulphide ore ? Explain the various steps supported by chemical
equations. Draw labelled diagram for the electrolytic refining of copper.
Ans. (a) Let us consider the case of Zn metal extraction from its carbonate ore.
Steps involved are
(i) The ore ZnCO3 is first concentrated by gravity separation method.
(ii) The ore is calcinated absence of it gets converted to oxide

ZnCO3   ZnO  CO2 
(iii) The oxide is reduced by coke

ZnO  c 
 Zn  CO
(iv) The impure Zn thus obtained can be purified by electrolysis.
(b) (i) Copper from its sulphide ore can be extracted simply by heating in air.
The steps involved are

1. 2Cu2 S  3O2 g    2Cu2O  2SO 2


2. 2Cu2O  Cu2 S   6Cu s   SO2 g 

www.vedantu.com 8
(ii)

19. (a) Mention any two components of blood.


(b) Trace the movement of oxygenated blood in the body.
(c) Write the function of valves present in between atria and ventricles.
(d) Write one structural difference between the composition of artery and veins.
Ans. (a) Two components of blood are (i) Blood plasma (ii) Blood cells
(b) Movement of oxygenated blood in the body as follows
(i) Pulmonary veins  Left atrium  Left ventricle  Systemicaorta  All part of the blood
(c) The valves in the heart are to prevent the backflow of blood when the atria or ventricles contract.
(d) Ateries are thick walled whereas veins are thin walled.

OR
(a) Define excretion.
(b) Name the basic filtration unit present in the kidney.
(c) Draw excretory system in human beings and label the following organs of excretory system which
perform following functions :
(i) form urine
(ii) is a long tube which collects urine from kidney
(iii) store urine until it is passed out.
Ans. (a) The biological process involved in the removal of these harmful metabolic wastes from the body
is called excretion.
(b) The nephron is the filtration units present in the kidney.

www.vedantu.com 9
(c)

20. (a) The modern periodic table has been evolved through the early attempts of Dobereiner, Newland
and Mendeleev. List one advantage and one limitation of all the three attempts.
(b) Name the scientist who first of all showed that atomic number of an element is a more fundamental
property than its atomic mass.
(c) State Modern periodic law.
Ans. (a) Dobereiner triad :
Advantage : The three elements of a triad were found to process similar properties.
Limitation : Some elements which are dissimilar were being grouped into a triad.
Newland’s octave :
Advantage : If the elements known at that time were arranged in the increasing order of their
atomic weights, the properties of every eighth element were similar to those of the first one.
Limitation : This classification did not include elements beyond atomic weight 40 (calcium)
Mendeleev :
Advantage : The physical and chemical properties of elements are periodic function of their
atomic weights.
Limitation : Position of rare earths was not clear. They were placed in group III A.
(b) Mendeleev
(c) The modern periodic law can be stated as physical and chemical properties of the elements
are periodic functions of their atomic numbers.

21. (a) A student is unable to see clearly the words written on the black board placed at a distance of
approximately 3 m from him. Name the defect of vision the boy is suffering from. State the
possible causes of this defect and explain the method of correcting it.
(b) Why do stars twinkle ? Explain.

www.vedantu.com 10
Ans. (a) Myopia is that defect of human eye by virtue of which it can see clearly the objects lying at short
distance from it. But the far off objects cannot be seen clearly by the myopic eye.
Causes of Myopia : The two possible causes of this defect are :
Increase in the length of the eye ball, as if distance of retina from the eye lens has increased.
Decrease in focal length of the eye lens when the eye is fully relaxed. This is as if the ciliary
muscles holding the eye lens do not relax fully and have some tension.
Correction : The image of a distant object (i.e., at infinity) is formed in front of the retina of eye
suffering from myopia as shown in figure. (a)

As the image of the object lying at infinity is not formed on the retina of the eye, so such object
can not be seen clearly by the myopic eye. The far point of such an eye is near to the eye as
shown in figure (b).

This defect can be corrected by using a concave lens of suitable focal length. So, a man suffering
from this defect wears spectacles having concave lens of suitable focal length. The concave lens
diverges the rays of light entering the eye from inf inity. Hence this lens makes the rays of light
appear come from the far point  O '  of the defective eye as shown in figure (c).

Let x = distance of far point of myopic eye, f = focal length of concave lens to be used.
As the object to be seen is at infinity and its image is to be formed at the far point, therefore,
u   and v = –x. Distance of far point O ' from eye lens is taken same as the distance of far
point O ' from concave lens.

1 1 1
From,  
f v u

or f   x
Hence, focal length of concave lens used for correcting the myopic eye is equal to distance of far
point of the myopic eye.

www.vedantu.com 11
(b) Twinkling of Stars
Light emitted by stars passes through the atmosphere of the earth reaching our eyes. The
atmosphere of the easth is not uniform but consists of many layers of different densities. The
layers close to the surface of the earth are optically denser. As we go higher and higher, the
density of layers and refractive index decreases progressively. As the light from a star enters the
upper most layer of the atmosphere, it bends towards the normal as it enters the next layer.

This process continues till the light enters our eyes. So due to refraction of light, the apparent
position of the star is different from the actual position of the star. Moreover, the different layers
of the atmosphere are mobile and the temperature and the density of layers of atmosphere changes
continuously. Hence, the apparent position of the star changes continuously. The change int he
apparent position of the star continuously leads to the twinkling of a star.

OR

(a) Write the function of each of the following parts of human eye :
(i) Cornea (ii) Iris (iii) Crystalline lens (iv) Ciliary muscles
(b) Why does the sun appear reddish early in the morning ? Will this phenomenon be observed by an
astronaut on the Moon ? Give reason to justify your answer.
Ans. (a) (i) Cornea : Its function is to act as a windoe to the world, i.e., to allow the light to enter the eye
ball.
(ii) Iris : Its function is to control the amount of light entering in the eye.
(iii) Crystalline lens : Its function is to focus the images of the objects at different distances,
clearly on the retina.
(iv) Ciliary muscles : Its function is to alter the focal length of the crystalline lens, so that the
image of the objects at various distances if clearly focussed on the retina.
(b) Colour of the sun at sunrise and sunset
At the time of sunrise and sunset, the position of the sun is very far away from us. The sunlight
travels longer distance through the atmosphere of the earth before reaching our eyes. Scattering
of blue light is more than the scattering of red light. As a result of this, more red light reaches our
eyes than any other colour. Therefore at sunset and sunrise sun appears red.

www.vedantu.com 12
Earth

This phenomenon will not be observed by an stronour on moon, since there is no atmosphere so
no scattering of light takes place, thus the sun appears dark

SECTION - B
22. Name the process by which an amoeba reproduces. Draw the various stages of its reproduction in
a proper sequence.
Ans. Amoeba reproduces by binary fission. Amoeba splits into two daughter cells in any plane.

OR
A student is viewing under a microscope a permanent slide showing various stages of asexual
reproduction by budding in yeast. Draw diagrams of what he observes. (in proper sequence)
Ans.

www.vedantu.com 13
23. An object of height 4.0 em is placed at a distance of 30 cm from the optical centre ‘O’ of a convex
lens of focal length 20 cm. Draw a ray diagram to find the position and size of the image formed.
Mark optical centre ‘O’ and principal focus ‘F’ on the diagram. Also find the approximate ratio of
size of the image to the size of the object.
Ans. Given data :

u  30 cm ; f  20 cm ; ho  4 cm ; v  ? ; hi  ?

1 1 1
 
v u f

1 1 1
  
v  30  20

1 1 1
  
v 20 30
v  60 cm

v hi
magnification ratio   m
u ho

60 cm h
  i
30 cm 4cm

 hi  8 cm

hi 8
ratio of size of image to size of object   2
h0 4

24. A student added few pieces of aluminium metal to two test tubes A and B containing aqueous solutions
of iron sulphate and copper sulphate. In the second part of her experiment, she added iron metal to
another test tubes C and D containing aqueous solutions of aluminium sulphate and copper sulphate.
In which test tube or test tubes will she observe colour change ? On the basis of this experiment, state
which one is the most reactive metal and why.

Ans. Test tube A : 2Al  3FeSO4  Al2  SO4 3  3Fe

Test tube B : 2 Al  3CuSO4  Al2  SO4 3  3Cu

Test tube C : 2 Fe  Al2  SO4 3  FeSO4  2 Al

Test tube D : Fe  CuSO4  FeSO4  Cu


The colour charge in test tube (green) that is from blue to light green.

www.vedantu.com 14
Aluminium is more reactive than iron because aluminium reacts with oxygen in air to form a larger
of oxide which protects the aluminium from further oxidation. Therefore, Aluminium does not
carrode as much as iron.

25. What is observed when a solution of sodium sulphate is added to a solution of barium chloride taken
in a test tube ? Write equation for the chemical reaction involved and name the type of reaction in this
case.
Ans. When sodium sulphate is added to Barium chloride it gives white ppt of barium sulphate which is
insoluble in water. The reaction also creates sodium chloride, which remains dissolved in water and
so cannot be seen.
It is double displacement type of reaction.
Na2 SO4  BaCl2 
 2 NaCl  BaSO4

26. The values of current (I) flowing through a given resistor of resistance (R), for the corresponding
values of potential difference (V) across the resistor are as given below:

V (volts) 0.5 1.0 1.5 2.0 2.5 3.0 4.0 5.0


I (amperes) 0.1 0.2 0.3 0.4 0.5 0.6 0.8 1.0

Plot a graph between current (I) and potential difference (V) and determine the resistance (R) of the
resistor.

1  0.5 0.5
Ans. Resistance  R  =Slope of line    5
0.2  0.1 0.1
V

5
4
3
2
1
I
0.1 0.2 0.3 0.4 0.5 0.6 0.7 0.8 0.9 0.10

27. List the steps of preparation of temporary mount of a leaf peel to observe stomata.
Ans. The following are the steps of preparation of temperory amount of a leaf peel to observe stomata.
(i) Remove a healthy leaf from the plotted plant.
(ii) Remove a part of the peel from the lower surface of the leaf by folding the leaf over and gently
pulling the peel a part using forceps.

www.vedantu.com 15
(iii) Put a few drops of saffranin stain in a watch glass.
(iv) After 2-3 minutes take out the peel and place it on a clean glass slide.
(v) Put a drop of glycerin over the peel and place a clean coverslip gently over it.
(vi) Remove the excess stain and glycerin with the help of blotting paper.
(vii) Observe the slide under the low-power and high-power magnifications of the compound micro-
scope.

www.vedantu.com 16
CBSE Class 10
Science
Previous Year Question Paper 2020
Series: JBB/1 Set– 1
Code no. 31/1/1

• Please check that this paper contains 15 printed pages.


• Code number given on the right-hand side of the question paper should be
written on the title page of the answer-book by the candidate.
• Please check that this question paper contains 30 questions.
• Please write down the Serial Number of the question in the answer-
book before attempting it.
• 15 minutes of time has been allotted to read this question paper. The
question paper will be distributed at 10.15 a.m. From 10.15 a.m. to 10.30
a.m., the students will read the question paper only and will not write any
answer on the answer-book during this period.

SCIENCE

Time Allowed: 3 hours Maximum Marks: 80


General Instructions:
Read the following instructions very carefully and strictly follow them:
(i) Question paper comprises three sections - A, B and C.
There are 30 questions in the question paper. All questions are compulsory.
(ii) Section A - question no. 1 to 14 - all questions or part thereof are of one mark
each. These questions comprise multiple choice questions (MCQ), very short
answer (VSA), and Assertion-Reason type questions. Answers to these questions
should be given in one word or one sentence.

Class X Science www.vedantu.com 1


(iii) Section B - question no. 15 to 24 are short answer type questions, carrying 3
marks each, Answers to these questions should not exceed 50 to 60 words.
(iv) Section C - question no. 25 to 30 are long answer type questions, carrying 5
marks each. Answer to these questions should not exceed
80 to 90 words.
(v) Answer should be brief and to the point. Also, the abovementioned word limit
be adhered to as far as possible.
(vi) There is no overall choice in the question paper. However, an internal choice
has been provided in some questions in each Section.
Only one of the choices in such questions have to be attempted.
(vii) In addition to this, separate instructions are given with each section and
question, wherever necessary.

SECTION - A
1. Name acyclic unsaturated carbon compound. 1 Mark
Ans: Acyclic unsaturated carbon compounds are those which contain double or
triple bonds between carbon atoms and are acyclic in nature. For example:
Butene.

2. The change in magnetic field lines in a coil is the cause of induced electric
current in it. Name the underlying phenomenon. 1 Mark
Ans: According to the phenomenon of electromagnetic induction, the change in
magnetic field lines in a coil is the cause of induced electric current in it.

Answer question numbers 3(a) to 3(d) and 4(a) to 4(d) on the basis of your
understanding of the following paragraphs and the related studied concepts.
3. The growing size of the human population is a cause of concern for all
people. The rate of birth and death in a given population will determine its
size. Reproduction is the process by which organisms increase their
population. The process of sexual maturation for reproduction is gradual

Class X Science www.vedantu.com 2


and takes place while general body growth is still going on. Some degree of
sexual maturation does not necessarily mean that the mind or body is ready
for sexual acts or for having and bringing up children. Various contraceptive
devices are being used by human beings to control the size of the population.
(a) List two common signs of sexual maturation in boys and girls. 1 Mark
Ans: Two common signs of sexual maturation in boys and girls are:
1. Pubic, underarm and facial hair development.
2. Common signs of acne development.
(b) What is the result of reckless female foeticide? 1 Mark
Ans: The reckless female foeticide will ultimately result in men: women ratio.
Since due to foeticide the number of females decreases with respect to the number
of males.
(c) Which contraceptive method changes the hormonal balance of the body?
1 Mark
Ans: The oral contraceptive method changes the hormonal balance of the body
so this woman does not ovulate by taking such pills.
(d) Write two factors that determine the size of a population. 1 Mark
Ans: Two factors which determine the size of population are:
1. Birth rate
2. Death rate

4. Human body is made up of five important components, of which water is


the main component. Food as well as potable water are essential for every
human being. The food is obtained from plants through agriculture.
Pesticides are being used extensively for a high yield in the fields. These
pesticides are absorbed by the plants from the soil along with water and
minerals and from the water bodies these pesticides are taken up by the
aquatic animals and plants. As these chemicals are not biodegradable, they
get accumulated progressively at each trophic level. The maximum
concentration of these chemicals gets accumulated in our bodies and greatly
affects the health of our mind and body.

Class X Science www.vedantu.com 3


(a) Why is the maximum concentration of pesticides found in human beings?
1 Mark
Ans: Pesticides are extensively used for good production of crops and these crops
will be consumed by us. Hence these pesticides transfer to the human body.
Therefore the maximum concentration of pesticides found in human beings.
(b) Give one method which could be applied to reduce our intake of
pesticides through food to some extent. 1 Mark
Ans: Before eating food we must ensure to wash it properly so that if there is a
layer of pesticides on the fruit then it gets washed.
(c) Various steps in a food chain represent: 1 Mark
(a) Food web (b) Trophic level
(c) Ecosystem (d) Biomagnification
Ans: The various steps which are represented in the food chain are known as
trophic level Each trophic level represents a class of organism.
Hence the correct option is (b).
(d) With regard to various food chains operating in an ecosystem, man is a:
(a) Consumer (b) Producer
(c) Producer and consumer (d) Producer and decomposer 1 Mark
Ans: According to various food chains operating in the ecosystem, a man is a
consumer which does not produce anything of its own. It only consumes.
Hence the correct option is (a).

5. Calcium oxide reacts vigorously with water to produce slaked lime.

CaO(s ) + H 2O(l) → Ca ( OH )2(aq.)

This reaction can be classified as:


(A) Combination reaction (B) Exothermic reaction
(C) Endothermic reaction (D) Oxidation reaction
Which of the following is a correct option?

Class X Science www.vedantu.com 4


(a) (A) and (C) (b) (C) and (D)
(c) (A), (C) and (D) (d) (A) and (B) 1 Mark
Ans: The above reaction is an exothermic in nature which releases large amounts
of heat and it is an example of a combination reaction.
Hence the correct option is (d).
Or
When hydrogen sulphide gas is passed through a blue solution of copper
sulphate, a black precipitate of copper sulphide is obtained and the sulphuric
acid so formed remains in the solution. The reaction is an example of a:
1 Mark
(a) Combination reaction (b) Displacement reaction
(c) Decomposition reaction (d) Double displacement reaction
Ans: The reaction can be represented as:
H 2S( g ) +CuSO 4(aq.) → CuS(s) +H 2SO 4(aq.)

Thus it is a double displacement reaction.


Hence the correct option is (d).

6. In a double displacement reaction such as the reaction between sodium


sulphate solution and barium chloride solution: 1 Mark
(A) exchange of atoms takes place
(B) exchange of ions takes place
(C) a precipitate is produced
(D) an insoluble salt is produced
The correct option is:
(a) (B) and (D) (b) (A) and (C)
(c) only (B) (d) (B), (C) and (D)
Ans: The reaction can be represented as:

Class X Science www.vedantu.com 5


BaCl2 +NaSO 4 → NaCl+BaSO 4 ( ↓ )

Sodium chloride is not an insoluble salt therefore the correct options are A and
C.
Hence the correct option is (b).

7. Baking soda is a mixture of: 1 Mark


(a) Sodium carbonate and acetic acid
(b) Sodium carbonate and tartaric acid
(c) Sodium hydrogen carbonate and tartaric acid
(d) Sodium hydrogen carbonate and acetic acid
Ans: Baking soda is sodium bicarbonate which is NaHCO3 which is a mixture
of sodium hydrogen carbonate and tartaric acid.
Hence the correct option is (c).

8. The chemical formula for plaster of Paris is: 1 Mark


(a) CaSO 4 .2H 2O (b) CaSO 4 .H 2O

1
(c) CaSO 4 . H 2O (d) 2CaSO 4 .H 2O
2
Ans: The correct formula for plaster of Paris is CaSO 4 .2H 2O .

Hence the correct option is (a).

9. The laws of reflection hold true for: 1 Mark


(a) plane mirrors only (b) concave mirrors only
(c) convex mirrors only (d) all reflecting surfaces
Ans: The laws of reflection hold true for all reflecting surfaces.
Hence the correct option is (d).

Class X Science www.vedantu.com 6


Or
When an object is kept within the focus of a concave mirror, an enlarged
image is formed behind the mirror. This image is: 1 Mark
(a) real (b) inverted
(c) virtual and inverted (d) virtual and erect
Ans: The nature of the image will be virtual and erect.
Hence the correct option is (d).

10. At the time of short circuit, the electric current in the circuit: 1 Mark
(a) vary continuously (b) does not change
(c) reduces substantially (d) increases heavily
Ans: At time of short circuit the current increases heavily which will burn our
devices too.
Hence the correct option is (d).
Or
Two bulbs of 100 W and 40 W are connected in series. The current through
the 100 W bulb is 1A. The current through the 40W bulb will be: 1 Mark
(a) 0.4A (b) 0.6A
(c) 0.8A (d) 1A
Ans: The current will always be equal in series connection. Therefore current will
be 1A.
Hence the correct option is (d).

12. Incomplete combustion of coal and petroleum: 1 Mark


(A) increases air pollution.
(B) increases the efficiency of machines.
(C) reduces global warming.

Class X Science www.vedantu.com 7


(D) produce poisonous gases.
The correct option is:
(a) (A) and (B) (b) (A) and (D)
(c) (B) and (C) (d) (C)and (D)
Ans: Incomplete combustion of petrol will increase the air pollution and produce
the poisonous gases consequently.
Hence the correct option is (b).
For question numbers 13 and 14, two statements are given – one labelled
Assertion (A) and the other labelled Reason (R). Select the correct answer to
these questions from the codes (a), (b), (c) and (d) as given below:
(a)Both A and R are true and R is correct explanation of the
Assertion.
(b) Both A and R are true, but R is not the correct explanation of the
Assertion.
(c) A is true but R is false.
(d) A is false but R is true.

13. Assertion (A): Esterification is a process in which a sweet-smelling


substance is produced.
Reason (R): When esters react with sodium hydroxide an alcohol and sodium
salt of carboxylic acid are obtained. 1 Mark
Ans: Esterification is the process by which ester is formed and ester is a sweet
smelling substance. When this ester reacts with sodium hydroxide an alcohol and
sodium salt of carboxylic acid are obtained. Thus Assertion is true but reason is
not correct explanation because ester smell because of volatile in nature.
Hence the correct option is (b).

14. Assertion (A): In the process of nuclear fission, the amount of nuclear
energy generated by the fission of an atom of uranium is so tremendous that

Class X Science www.vedantu.com 8


it produces 10 million times the energy produced by the combustion of an
atom of carbon from coal.
Reason (R): The nucleus of a heavy atom such as uranium, when bombarded
with low energy neutrons, splits apart into lighter nuclei. The mass
difference between the original nucleus and the product nuclei gets
converted to tremendous energy. 1 Mark
Ans: Nuclear fission produces a large amount of energy and this energy is
produced as the result of bombardment of uranium with low energy neutrons.
Thus both are true and reason is the correct explanation of assertion.
Hence the correct option is (a).

SECTION-B
15. 1g of copper powder was taken in a China dish and heated. What change
takes place on heating? When hydrogen gas is passed over this heated
substance, a visible change is seen in it. Give the chemical equations
reactions, the name and the color of the products formed in each case.
3 Marks
Ans: When 1g Copper powder is taken in a china dish and then it is heated, a
black colored substance is formed which is known as copper oxide. The above
reaction can be represented by using the equation as:
Heat
2Cu+O 2  → 2CuO(black)

This is because the copper is being oxidized to copper oxide which is black in
color. Now when hydrogen gas is passed over this heated substance which is
copper oxide then the copper metal is obtained. The reaction can be represented
as:
Heat
CuO(black) +H 2  → Cu (s) + H 2O(l)

Thus we get copper back on heating the copper oxide with hydrogen gas and also
water is formed as a by-product of reaction.

16. List the important products of the Chlor-alkali process. Write one
important use of each. 3 Marks

Class X Science www.vedantu.com 9


Ans: The chlor-alkali process is used in the electrolysis of sodium chloride
( NaCl ) . The important products which are formed during the electrolysis of
sodium chloride are:
2NaCl+2H 2O → 2NaOH + Cl2 + H 2

The important products are sodium hydroxide, chlorine and hydrogen gas. When
the electric current is passed through the aqueous solution of sodium chloride
then hydrogen gas is formed at cathode and chlorine gas is formed at anode. Their
uses are listed below:
(a) Sodium Hydroxide: It is used in manufacturing of soaps and detergents.
(b) Chlorine gas: It is used in pesticides and manufacturing of PVC pipes.
(c) Hydrogen gas: It is used in fuels and reduction reactions also.

How is washing soda prepared from sodium carbonate? Give its chemical
equation. State the type of this salt. Name the type of hardness of water which
can be removed by it? 3 Marks
Ans: Washing soda can be prepared from sodium carbonate by adding ten
molecules of water of crystallization. It is then known as sodium carbonate
decahydrate or we can simply call it a washing soda in common name. The
reaction of adding water of crystallization can be shown as:
Na 2CO3 +10H 2O → Na 2CO3 .10H 2O

Washing soda is a sodium salt and we know that sodium salt is basic in nature.
Therefore sodium carbonate is basic in nature and it is basic salt.
Basically there are two types of hardness in water which are temporary hardness
and permanent hardness. Washing soda is used to remove both types of hardness.
When we add washing soda in water it adds a large amount of carbonate ions in
water which will react with calcium and magnesium ions to form insoluble
precipitates and thus cleansing action is completed.

17. A 3 ml ethanol is taken in a test tube and warmed gently in a water bath.
A 5% a solution of alkaline potassium permanganate is added first drop by
drop to this solution, then in excess.

Class X Science www.vedantu.com 10


(i) How is 5% solution of KMnO 4 prepared?

(ii) State the role of alkaline potassium permanganate in this reaction. What
happens when you add it in excess?
(iii) Write the chemical equation of this reaction. 3 Marks
Ans:
(i) 5% solution of KMnO 4 is the solution in which there is 5 g of KMnO 4 is
dissolved in 100 ml solution. Therefore it is ratio of weight of KMnO 4 and the
volume of solution. It can be prepared in the following steps as:
(a)Take 100 ml of distilled water in a beaker.
(b) Now add 5 g of KMnO 4 into beaker gently and thus it from the 5% solution
of KMnO 4 .

(ii) When the alkaline potassium permanganate is added to ethanol then the
ethanol gets oxidized. This is because alkaline potassium permanganate is an
oxidizing agent which will oxidize the alcohol into carboxylic acid. The reaction
between them can be shown as:
C2 H5OH
KMnO 4  → CH 3COOH

Here in the above reaction alkaline potassium permanganate acts as an oxidizing


agent. When we add excess alkaline potassium permanganate then the purple
color will persist.
(iii) The equation for the chemical reaction will be:
KMnO 4 + C2 H 5OH → CH 3COOH

18. A squirrel is in a scary situation. Its body has to prepare for either
fighting or running away. State the immediate changes that take place in its
body so that the squirrel is able to either fight or run? 3 Marks
Ans: There will be some immediate changes that take place in its body for the
squirrel to be ready to fight or run away. These changes are listed below as:
(a) There will be release of adrenaline hormone which will cause an increase in
the pumping rate of the heart and also the breathing rate of squirrels increases.

Class X Science www.vedantu.com 11


(b) There will be a change in blood pressure and the dilation of the pupil which
helps while running.
(c) The hypothalamus of the squirrel activates the sympathetic nervous system
and thus releases the adrenaline from adrenal gland.
(d) At that time the digestive activities and reproductive activities stop for a
moment.
All these changes made squirrels to be active while running or fighting like
situations.
Or
Why is chemical communication better than electrical impulses as a means
of communication between cells in a multicellular organism? 3 Marks
Ans: Chemical communications is better than electrical impulses as means of
communication between cells in a multicellular organism because of the
following reasons:
(a) Chemical communication involves the communication which takes place
through hormones. Thus it does not require any specialized tissue like nervous
tissue which is used for electrical impulses.
(b) Since electric communication is limited to only certain regions. These regions
are connected by nerves while the chemical connection is not specific to some
regions, it is all over the body.
(c) In chemical communication signals are sent persistently and steadily while in
nervous communication it can be done only at certain intervals of time.

19. Define the term pollination. Differentiate between self pollination and
cross pollination. What is the significance of pollination? 3 Marks
Ans: Pollination can be defined as the act of transferring of pollen grains from
the anther of male flower to stigma of female flower. After the pollination the
seeds can be produced which will create offspring.

Self-Pollination Cross Pollination


When the pollen grains transfer from When the pollen grains transfer from
anther of the same flower to the anther of flower to the stigma of
stigma of the same flower then this different flowers then this kind of

Class X Science www.vedantu.com 12


kind of pollination is called self- pollination is called cross
pollination. pollination.
Here pollinating agents are not Here pollinating agents are necessary
necessary. for the transfer of pollen grains.

Significance of pollination:
The transfer of pollen grains is significant as it helps in fertilization by bringing
male gamete to female egg and thus it helps in the production of seed and which
will ultimately produce new offspring.

20. What are homologous structures? Give an example. Is it necessary that


homologous structures always have a common ancestor? Justify your
answer. 3 Marks
Ans: Homologous structures are those structures which have common basic
structure but they perform different operations or functions. Thus we can say that
homologous structures are similar to each other but they are modified to perform
different functions.
For example: forelimbs of reptiles, amphibians and mammals have the same
structure but they can perform different functions.
The Arms of human beings, wings of birds and the front legs of horses are
homologous structures.
Yes, homologous structures have common ancestors but they are modified to
carry out different operations or activities. The basic structure of the arms of the
humans and wings of the bird are similar in structure but they are modified and
hence perform different operations in different individuals. Arms of human
beings cannot be used for flying while that of birds does. Thus we can say that
they have common ancestors but perform different functions.

21. Why is the Tyndall effect shown by colloidal particles? State four
instances of observing the Tyndall effect. 3 Marks
Ans: The scattering of light in a colloidal dispersion medium by colloidal particle
is known as the Tyndall effect. When the same light is passed through pure
medium then no such scattering of light takes place. Thus it is a property of
colloidal particles to scatter the light when it passes through it. Hence we can say

Class X Science www.vedantu.com 13


that the Tyndall effect is used to determine whether a mixture is a true solution
or a colloidal solution. The instances where we can see this Tyndall effect are
listed below:
(a) The Tyndall effect can be shown when headlights of a car turns on on a foggy
day. The light from the headlight gets scattered by the colloidal particles of the
fog and thus we can see colloidal particles too.
(b) When sunlight passes through the canopy of the forest then we can see the
scattering of light at the top of the tree which is the canopy region.
(c) The Tyndall effect can be visible when light enters a dark room through a hole
and thus we can easily see the light scattered by the particles in the way of the
light.
(d)When we direct the beam of light into the glass full of milk then we can also
see this effect too.
Or
Differentiate between a glass slab and a glass prism. What happens when a
narrow beam of (i) a monochromatic light, and (ii) white light passes through
(a) glass slab and (b) glass prism? 3 Marks
Ans: The difference between glass slab and glass prism is listed as:
Glass Slab Glass Prism
It is a transparent substance in which It is also a transparent substance in
reflecting surfaces are parallel to each which reflecting surfaces are at an
other. angle.
It is rectangular in shape. It has two triangular sides inclined to
each other and a rectangular base.
The direction of incident ray and The direction of incident ray and
emergent ray are parallel to each other. emergent ray are not parallel to each
other.

(i) When the narrow beam of monochromatic light get passed through:
(a) Glass slab: It will deviate from the actual path but we know that the direction
of the incident ray and the emergent ray are parallel to each other.
(b) Glass prism: There is splitting of white into seven colors and the direction of
incident ray and emergent ray are not parallel to each other.

Class X Science www.vedantu.com 14


(ii) When the narrow beam of white light get passed through:
(a) Glass slab: When it strikes the first surface then splitting of white light takes
place and when it strikes on the second surface then it comes out as single white
light.
(b) When it falls on a prism, white light gets dispersed into seven different colors
on the screen placed in front of the prism.

22. Draw a labelled diagram to show (i) reddish appearance of the sun at the
sunrise or the sunset and (ii) white appearance of the sun at noon when it is
overhead. 3 Marks
Ans: When the sun is near the horizon then it appears to be reddish in color
because as the light passes through the atmosphere enters at slant angle and due
to which a large amount of scattering of light takes place. The blue light of the
sky gets scarier than the red light of the sun thus we can see the reddish color of
the sun more precisely. The same phenomena does not happen at noon.

Class X Science www.vedantu.com 15


23. A V-I graph for a nichrome wire is given below. What do you infer from
this graph? Draw a labelled circuit diagram to obtain such a graph.
3 Marks

Ans: From the above we can see that there is a linear relation between the
potential difference(V) and current (A), therefore it gives a straight line between
them. We can depict that when we increase the potential difference across the
nichrome wire then the current also increases in a constant ratio respectively.
Thus we can say that the resistance of nichrome wire is constant. Since there is a
linear relation between the current and potential difference we can say that the
nichrome wire follows the ohm’ law.
According to ohm’s law there is a linear relation between the applied potential
difference and the current passing through wire. It can be represented as:
V∝I
⇒ V=IR
Hence we can draw the circuit diagram of the above graph as:

Class X Science www.vedantu.com 16


Thus the resistance of nichrome wire is R and current passing through it is I. It is
an ohmic device which follows ohm’s law.

24. 3 Marks
(a) Write the mathematical expression for Joule’s law of heating:
Ans: (a) According to Joule’s of heating when a current, I passes through
conductor of resistance r, for time t, then the amount of heat produced in the
conductor will be equal to product of square of current , the resistance and time.
This can be represented as:

H=i 2 rt
The S.I unit of energy is Joules.
(b) Compute the heat generated while transferring 96000 coulomb of charge
in two hours through a potential difference 40 V.
Ans: Here the amount of charge, Q is 9600 C and time for this flow of charge is
2 hr which is equal to 7200 s . Hence we can find the electric current as:
Q
i=
t
9600 40
i= = A
7200 3
Since we know that V=ir the above equation can be reduced as:
H=Vit
It is given that V=40 V , therefore the amount of heat can be calculated as:
40
H=40× ×7200
3
H=3840000 J
H=3.84×106 J

SECTION-C

Class X Science www.vedantu.com 17


25. Carbon cannot reduce the oxides of sodium, magnesium and aluminium
to their respective metals. Why? Where are these metals placed in the
reactivity series? How are these metals obtained from their ores? Take an
example to explain the process of extraction along with chemical equations.
5 Marks
Ans: Carbon cannot reduce the oxides of sodium, magnesium and aluminium to
their respective metals because sodium, magnesium and aluminium have higher
affinity towards oxygen then that of carbon. This is because all these are highly
reactive metals and thus it requires a lot of energy to reduce their oxides.
According to the reactivity series of metals which helps in finding the more
reactive elements, it is found that sodium, magnesium and aluminium are more
reactive than carbon and thus placed at higher levels than carbon in the reactivity
series of elements. Therefore being less reactive, carbon is not used for extraction
of sodium, magnesium and aluminium from their ores.
Such metals which are highly reactive than carbon elements are extracted by the
help of electrolytic reduction process. It is also known as electrolysis of the
molten solution of the ore. Thus we can say that sodium can be obtained with the
help of electrolysis of its salt. In electrolysis a sufficient amount of current is
passed through the aqueous solution and thus it breaks into its respective ions and
thus metals get collected at cathode. The reaction can be represented as:

At cathode: Na + + e- → Na

1
At anode: Cl- → Cl2 +e-
2
Therefore we get the pure metal at cathode. This method is basically used for
elements which cannot be reduced by carbon.

26. The position of certain elements in the Modern Periodic Table are shown
below.
1 2 3 to 12 13 14 15 16 17 18
Group
Period
1 G H
2 A I B C
3 D E F

Class X Science www.vedantu.com 18


Using the above table answer the following questions giving reasons in each
case:
(i) Which element will form only covalent compounds?
(ii) Which element is a non-metal with valency 2?
(iii) Which element is a metal with valency 2?
(iv) Out of H, C and F which has the largest atomic size?
(v) To which family docs H, C and F belong? 5 Marks
Ans: (i) The element which forms covalent compounds must have valence
electrons greater than three and less than eight. Thus we can say that elements E
and B will have a tendency to form covalent compounds.
(ii) Non-metals are present at the right hand side of the periodic table. They have
a tendency to accept electrons. Thus we can say that element E will be non-metal
and it can accept two electrons to form inert in nature.
(iii) Metals are present on the left hand side of the periodic table and they have a
tendency to lose electrons. Thus we can say that G, A and D are metals. But D
element will have a valency of 2.
(iv) As we go down the group the size of the element increases in the same group.
Hence we can say that out of H, C and F , F has the largest atomic size. The size
of atoms increases as the number of shells increases.
OR
Define atomic size. Give its unit of measurement. In the modern periodic
table what trend is observed in the atomic radius in a group and a period
and why is it so? 5 Marks
Ans: Atomic size: The distance between the centre of the nucleus of an atom and
its outermost shell is known as the atomic size of an atom. The atomic size varies
as the number of shells inside the atom increases. A atom having more number
of shells will have more atomic size than an atom having less number of shells.
The atomic size is the distance between the nucleus of atom to outermost shell,
hence it is measure in Angstrom  A  .
o

 
o
1A = 10−10 m

Class X Science www.vedantu.com 19


Trend observed in atomic radius in a group and period in modern periodic table:
Along the periods: When we move along the periods from left to right in a period
the atomic radius of elements gradually decreases. The decrease in atomic radius
is due to an increase in effective nuclear energy and more attraction between the
electron and the nucleus of the atom. Since the shell does not increase and number
of electrons increases which increases the attraction between the electrons and
nucleus of the atom. This attraction will ultimately shrink the size of atoms.
Therefore we can say that along the periods while moving from left to right the
atomic radius decreases generally.
Down the groups: When we move down the group from top to bottom the atomic
radius increases gradually. This is because the number of shells increases at each
down element. Thus the number of shells increases at every element while
moving down the group which ultimately increases the size of the element.
Therefore we can say that the atomic radius increases while moving down the
group.

27. 5 Marks
(a) Why is there a difference in the rate of breathing between aquatic
organisms and terrestrial organisms? Explain.
Ans: One complete breath includes single inhalation and exhalation of air. Thus
rate of breathing is the number of times a complete breath takes place in a minute.
Rate of breathing is different in aquatic organisms and terrestrial organisms. This
is because aquatic organisms like fishes obtain their oxygen from water which is
present in dissolved state through their gills. Since we know that the amount of
dissolved oxygen in water is less as compared to oxygen present in undissolved
form in air. Thus they have high breathing rates to meet the breathing
requirements. They breathe faster than humans to meet the oxygen demand of the
body. Thus it is also seen that swimmers breathe at a faster rate when they swim
across the river. Therefore there is a difference of breathing rates in both aquatic
organisms and terrestrial organisms.
(b) Draw a diagram of the human respiratory system and label - pharynx,
trachea, lungs, diaphragm and alveolar sac on it.
Ans: The human respiratory system can be shown as:

Class X Science www.vedantu.com 20


Or
5 Marks
(a) Name the organs that form the excretory system in human beings.
Ans: Human excretory system consists of following organs:
1.Two kidneys : Kidneys are located at one on each side of the spine at the level
of the liver. Kidneys are the important organ of the human body and it is present
in pairs. Kidney helps in purification of blood.
2.A urinary bladder: It is a sac-like structure and it stores urine until the process
of expelling the urine. It receives urine from the ureters which are connected to
each kidney separately.
3. Urethra: It is a tube -like structure which arises from the urinary bladder. It is
shorter in females and longer in males. In males, it is a common path for sperms
and urine both.
(b) Describe in brief how urine is produced in the human body.
Ans: The human body produces the waste chemical compounds like urea and uric
acid with the help of urine. These waste are filtered with the help of the kidney.

Class X Science www.vedantu.com 21


Thus we can say that the main role of the kidney is to filter our blood and produce
the waste excreted out in the form of urine.
Basically the kidney contains a basic structure called nephrons which helps in the
filtration process. It has two parts:
1. Bowman’s Capsule: It is a cup shaped structure which consists of a coiled tube
of blood capillary. This helps in the filtration process.
2. Renal Tubule: Now the filtrate obtained from Bowman’s capsule goes into
renal tubule. But some of the substances like amino acids and salts get absorbed
in their initial state. The rest of water which contains only uric acid and urea gets
passed through the tubule.
Now it moves through a duct and moves towards the urinary bladder where it gets
stored and excreted out from the body in the form of urine.

28. 5 Marks
(a) What is the law of dominance of traits? Explain with an example.
Ans: The law of dominance of traits states that in a heterozygous or we can say
in a hybrid condition the allele having the characters expressed over the other
allele is the dominant allele.

Class X Science www.vedantu.com 22


Here in the parent generation one of them is homozygous dominant and the other
one is homozygous recessive allele. The first one gamete has the dominant pea
shape structure while the other one gamete has the recessive structure. After he
fertilization takes place, the first generation F1 would have heterozygous
conditions among the both alleles the ‘R’ character expresses over the ‘r’ allele.
Thus we can say that here ‘R’ is the dominant allele.
(b) Why are the traits acquired during the lifetime of an individual not
inherited? Explain.
Ans: The traits which are inherited during the lifetime cannot be inherited to
successive generations as the changes do not reflect in the DNA of the germ cells.
The traits acquired during the lifetime brings the change in non-reproductive
tissues which cannot be passed to germ cells. Thus we can say these acquired
traits cannot be passed to successive generations.
For example: a swimmer good at swimming cannot pass this acquired trait to
successive generations as it brings changes only in non-reproductive cells which
do not take part in reproduction.

29. Draw a ray diagram in each of the following cases to show the formation
of image, when the object is placed:
(i) between the optical centre and principal focus of a convex lens.
State the signs and values of magnifications in the above-mentioned cases (i)
and (ii). 5 Marks
Ans: (i) Let us suppose O be the object and I be the image then the image can be
formed as:

Class X Science www.vedantu.com 23


The image formed is enlarged and erect. Therefore its magnification will be
positive and since it is enlarged therefore magnification will be greater than one.

(ii) anywhere in front of a concave lens.


Ans: Let us suppose O be the object and I be the image then the image can be
formed as:
The image formed is diminished and erect. Therefore its magnification will be
positive and since it is diminished therefore magnification will be less than one.

(iii) at 2F of a convex lens.


Ans: Let us suppose O be the object and I be the image then the image can be
formed as:
The image will be formed at 2F on the other side of the lens and its magnification
will be the same as that of the object.

Class X Science www.vedantu.com 24


Or
An object 4.0 cm in size, is placed 25.0 cm in front of a concave mirror of
focal length 15.0 cm. 5 Marks
(i) At what distance from the mirror should a screen be placed in order to
obtain a sharp image?
Ans: Here, height of object, h i =4 cm

Object distance u=25 cm


Focal length f=15 cm
We know that:
1 1 1
⇒ = +
f v u
V is the image distance , object distance is taken as negative and focal length of
concave mirror is always negative. Therefore,
1 1 1
⇒ = +
-15 v -25
1 1 1
⇒ = +
v -15 25
1 -5+3
⇒ =
v 75
1 -2
⇒ =
v 75

Class X Science www.vedantu.com 25


v=-37.5 cm
(ii) Find the size of the image.
-v h i
Ans: Magnification, m= =
u ho

- ( -37.5 ) h i
⇒ m= =
-25 4
4×37.5
⇒ hi =
-25
⇒ h i =-6 cm

Therefore height of image will be −6 cm . The image will be real and inverted.
(iii) Draw a ray diagram to show the formation of the image in this case.
Ans:

30. 5 Marks
(a) What is an electromagnet? List any two uses.
Ans: An electromagnet is a type of magnet in which the magnetic field is
produced by passing the electric current. Thus whenever the electric current
passes through the electromagnet then it will behave like a magnet. It is also
known as a temporary magnet. Its magnetism is not permanent in nature.
Uses of electromagnet:
1.It is used in lifting heavy weight objects.

Class X Science www.vedantu.com 26


2.It is used in transformers.
(b) Draw a labelled diagram to show how an electromagnet is made.
Ans:

(c) State the purpose of the soft iron core used in making an electromagnet.
Ans: The soft iron core is used in electromagnets because the soft iron core has
high magnetic properties. Because of its magnetic capability it is used in
electromagnetism. It can provide a strong magnet when it is used as a core
material in the electromagnets.
(d) List two ways of increasing the strength of an electromagnet if the
material of the electromagnet is fixed.
Ans: Ways of increasing the strength of an electromagnet if the material of the
electromagnet is fixed are listed below as:
1. By increasing the number of turns in the coil of electromagnet we can increase
its magnetic strength.
2. By increasing the amount of current we can also increase its strength. But the
amount of current should be increased in the limit of the coil as more current will
burn out the coil. Thus the heating effect of the coil must be remembered while
increasing current.

Class X Science www.vedantu.com 27


Class X Science
CBSE-2019
Time: 3Hr
Maximum Marks: 80

General Instructions:
I. The question paper comprises five sections. A. B. C. D and E. You have to attempt all the
sections.
II. All questions are compulsory.
III. Internal choice is given in sections B, C, D and E.
IV. Question numbers 1 and 2 in Section A are one mark questions. They are to be
answered in one word or in one sentence.
V. Question numbers 3 to 5 in Section B are two-marks questions. These are to be
answered in about 30 words each.
VI. Question numbers 6 to 15 in Section C are three-marks questions. These are to be
answered in about 50 words each.
VII. Question numbers 16 to 21 in Section D are five-marks questions. These are to be
answered in about 70 words each.
VIII. Question numbers 22 to 27 in Section E are based on practical skills. Each question is a
two marks question. These are to be answered in brief.

Section A

1. If you could use any source of energy for heating your food which one would you prefer?
State one reason for your choice.

ANSWER: If I could use any source of energy for heating food then I would
prefer solar energy because:
1)It is available in abundance
2)Energy from Sun is available to every individual
3)Solar energy also preserves the natural nutrients of food by cooking it slowly at low
temperature.
4)It is economical.

2. Write the function of voltmeter in an electric circuit.

ANSWER: The function of the voltmeter in a circuit is to measure the voltage drop across any
appliance. The voltmeter is always connected parallel in the circuit.

3. What happens to the image distance in the normal human eye when we decrease the
distance of an object, say 10 m to 1 m ? Justify your answer.
ANSWER: If the object distance is greater than 25 cm the image is always formed on the retina
as the focal length of the human eye lens gets adjusted depending on the object distance. This
special property of human eye is called as power of accommodation. Hence, image distance will
remain same if object distance changes from 10 m to 1 m.

4. List two different functions performed by pancreas in our body.

ANSWER: The pancreas contains both the endocrine and exocrine portions. The exocrine region
secretes sodium bicarbonate and many digestive enzymes. The pancreatic amylase causes
breakdown of starch, pancreatic lipase causes breakdown of lipids, and trypsin causes digestion
of proteins The endocrine regions secrete hormones i.e. insulin and glucagon. These hormones
regulate the level of glucose in the blood. Insulin decreases while glucagon increases the blood
glucose level.

5. How it can be proved that the basic structure of the Modern Periodic Table is based on the
electronic configuration of atoms of different elements?
OR
The electronic configuration of an element is 2, 8, 4. State its:
(a) group and period in the Modern Periodic Table.
(b) name and write its one physical property.

ANSWER: Modern periodic law states that the physical and chemical properties of an element
in the periodic table are the periodic function of the atomic number of that element. Electronic
configuration of the elements plays an important role in the placement of elements in the
modern periodic table. The valence shell electron of an element decides its position in a
particular group or period. For example, if the electronic configuration of an element is 2, 1, it
means that the element belongs to group I and 2nd period (valence electron is present in the
2nd shell). Similarly, if the electronic configuration of an element is 2, 8, 1, it means that the
element belongs to group I and 3rd period (valence electron is present in the 3rd shell).
Thus, the modern periodic table is based on the electronic configuration of the different
elements.
OR
(a) The electronic configuration of the given element is 2,8,4. Since, the number of valence
electrons for the given element is 4. Hence, the element belongs to group 14 (10+4) and the 3rd
period.
(b) The atomic number of the given element is 14. Hence, the given element is Silicon. Silicon is
a metalloid.

6. How can we help in reducing the problem of waste disposal? Suggest any three methods.
OR

Define an ecosystem. Draw a block diagram to show the flow of energy in an ecosystem.
ANSWER: The three methods we can utilise for reducing the problem of waste disposal are:
Segregation of waste into biodegradable and non-biodegradable
Following the principle of 3R- reduce, reuse and recycle Converting biodegradable waste into
useful commodities like energy from biogas, using the compost as fertiliser OR
Ecosystem refers to the living and non-living components in an area and the interactions
between them. Energy flows across the trophic levels as shown in the diagram below, following
the ten percent law. Only ten percent of the energy available to a trophic level is passed on to
the next trophic level. The remaining is dissipated away.

Assuming 10000 J is the energy available to the producers, then 1000 J will be available to the
primary consumers, 100 J will be available to secondary consumers and 10 J will be available to
tertiary consumers.

7. List three advantages each of:


(i) exploiting resources with short term aims, and
(ii) using a long term perspective in managing our natural resources.

ANSWER: (i)Advantages of exploiting resources with short term aims:


(i) The current basic needs of the population can easily be fulfilled.
(ii) Fast industrialisation.
(iii) Variety of useful products can be made to make life easy.

(ii)Advantages of long term perspective in managing our natural resources:


(i) Sustainable management of resources
(ii) The resources can be used efficiently by the present as well as future generations.
(iii) The pollution and the degradation of the environment can be reduced.

8. What is a rainbow? Draw a labelled diagram to show the formation of a rainbow.


ANSWER: The rainbow is a natural phenomenon in which white sunlight splits into beautiful
colours by water droplets, which remain suspended in air after the rain. Formation of a
rainbow:

9. Nervous and hormonal systems together perform the function of control and coordination
in human beings. Justify this statement with the help of an example.
ANSWER: In human beings, control and coordination is brought about by both nervous system
and endocrine system. Nervous system works by generation and transmission of electrical
impulse while the endocrine system works by secreting chemical messengers called hormones.
These systems complement the action of each other to control and coordinate different
functions in our body. For example, when an emergency stimulus is detected by the nervous
system, the stimulus is received and analysed by CNS that send message to effectors to provide
proper response. At the same time, the sympathetic nervous system activates adrenal gland to
release adrenaline that prepares body by increasing heart rate, blood pressure, respiration and
dilates pupil etc. Thus, both these systems interact and work together to produce an effective
response.

10. Trace the sequence of events which occur when a bright light is focused on your eyes.

ANSWER: When bright light is focussed on our eyes, then it goes to the brain, the brain reverts
back the message by motor neuron which contracts the pupil. The sequence of events which
occur is:

Receptor               
→ Sensory neuron → Brain → Motor Neuron → Eye → Eye muscle contracts
11. What is photosynthesis? Explain its mechanism.

ANSWER: Photosynthesis is a process in which the energy of light is utilised to produce simple
organic compounds in autotrophs. In the process of photosynthesis the energy of sunlight is
captured by photosensitive pigment like chlorophyll in green plants. The energy harnessed by
light is utilised for photolysis of water molecules within the chloroplasts of green plants. As a
result nascent oxygen is released as molecular oxygen gas. Hydrogen released by the reduction
of water is further utilised by the plants in light independent reaction where carbon dioxide is
reduced to simple carbohydrates or sugars in a series of metabolic reactions within the plant
chloroplasts.
The detailed equation of photosynthesis can be represented as:human

6CO2 + 12 H 2 0 → C6 H12 06 + 602 + 6 H 2 0

12. Name the plant Mendel used for his experiment. What type of progeny was obtained by
Mendel in F1 and F2 generations when he crossed the tall and short plants? Write the ratio
he obtained in F2 generation plants.

OR

List two differences between acquired traits and inherited traits by giving an example of
each.

ANSWER: Mendel used pea plant (Pisum sativum) for his experiments. When Mendel crossed
tall and short plants, the progeny obtained in Fl generation were tall. When the Fl plants were
selfed, the F2 generation showed three tall and one dwarf plant. The genotypic ratio of F2
generation is 1:2:1 (TT : Tt: Tt: tt) and the phenotypic ratio is 3:1 (Tall: Dwarf).
OR

Differences between Acquired Traits and Inherited Traits

Acquired Traits Inherited Traits


i. These are somatic variations. i. These are genetic variations.
ii. Acquired traits develop due to the ii. Inherited traits develop due to a reshuffling of
effects of environmental factors, use and genetic material and mutations.
disguise of organs and special
(conscious) efforts.
iii. These traits develop throughout the iii. These traits are transferred (inherited) by the
lifetime of an individual. parents to their offspring.
iv. Example-Learning of dance, music, etc. iv. Example-Attached or free earlobe and curly hair.
and muscular body of a wrestler.

13. 2 g of silver chloride is taken in a china dish and the china dish is placed in sunlight for
sometime. What will be your observation in this case? Write the chemical reaction involved
in the form of a balanced chemical equation. Identify the type of chemical reaction.
OR

Identify the type of reactions taking place in each of the following cases and write the
balanced chemical equation for the reactions.
(a) Zinc reacts with silver nitrate to produce zinc nitrate and silver.
(b) Potassium iodide reacts with lead nitrate to produce potassium nitrate and lead iodide.

ANSWER: When 2 g of silver chloride, AgCI, is kept in sunlight then AgCI breaks down into Ag
and C12. The color of the silver chloride turns to grey. The following change can be represented
by the chemical reactions as:
sunlight 2AgCl(s) 2Ag(s) + Cl2(g) . This type of reaction is an example of a photochemical
decomposition reaction.

(a) The given reaction is a displacement reaction in which more reactive zinc will displace less
reactive silver from silver nitra Answer. Zn(s) + 2AgNO3(aq) -+Zn(NO3)2(aq) + 2Ag(s)
(b) The given reaction is a double displacement reaction. 2KI(aq) + Pb(NO3)2(aq) 2KNO3(aq) +
Pb12(s)

14. Based on the group valency of elements write the molecular formula of the following
compounds giving justification for each:
Oxide of first group elements.
Halide of the elements of group thirteen, and Compound formed when an element A of group
2 combines with an element B of group seventeen.

ANSWER: Oxides of the first group elements have the common formula of M20.
Example- Na20, K20. This is because, the first group elements have a common valency of 1, and
the valency of Oxygen is 2 so, to satisfy the combining capacity of Oxygen two 1st group metals
are required.

Halides of group 13 elements have a common formula of MX3 , where M-metal and X- halogen
element. Example- AICI3, BF3 . This is because the valency of group 13 elements is 3 and that of
halogens is 1 so, to satisfy the combining capacity of aluminum or other group 13th elements
three of halogens are required in the molecular formula.

The general formula for those kinds of compounds would be AB2


Example- MgCl2, CaCl2. This is because the valency of group 2 elements is 2 and that of group
17th elements if 1 so to satisfy the combining capacity of group 2 elements two of group 17
elements are required in the molecular formula.
15. Explain the following:

(a) Sodium chloride is an ionic compound which does not conduct electricity in solid state
where as it does conduct electricity in molten state as well as in aqueous solution.

(b) Reactivity of aluminium decrease if it is dipped in nitric acid.

(c) Metals like calcium and magnesium are never found in their free state in nature.

ANSWER: (a) NaCI conducts electricity in the molten state and in aqueous Answers because
ions are free to move whereas in solid state ions are not free to move.

(b) When aluminium is dipped in nitric acid a layer of aluminium oxide is formed on the metal.
This happens because nitric acid is a strong oxidizing agent. The layer of aluminium oxide
prevents further reaction of aluminium. Due to this, the reactivity of aluminium decreases.

(c) Because Calcium and Magnesium are highly reactive.

16. (a) With the help of a suitable circuit diagram prove that the reciprocal of the equivalent
resistance of a group of resistances joined in parallel is equal to the sum of the reciprocals of
the individual resistances.
(b) In an electric circuit, two resistors of 12 0 each are joined in parallel to a 6 V battery. Find
the current drawn from the battery.

OR

An electric lamp of resistance 20 0 and a conductor of resistance 4 0 are connected to a 6 V


battery as shown in the circuit. Calculate:

(a) the total resistance of the circuit,


(b) the current through the circuit,
(c) the potential difference across the
(i) electric lamp and (ii) conductor, and
(d) power of the lamp.
ANSWER: (a)

Let there are n resistances, each of value RI, R2 Rn, respectively, are connected in parallel to a
battery of voltage V. if the equivalent resistance of the circuit is Req, then current drawn from
V
i=
Req
the battery is
The total current /then divides into Ii, iz, i3 in, respectively in the given resistors. As all the
resistances are connected in parallel, hence the voltage across each resistor is Vvolt. Now we
can write,
i = i1 + i2 + i3 + ..... + ieq

V V V V V
= + + .... +   ..... (1)
Req R1 R2 R3 Rn

From eq. 1,
1 1 1 1 1
= + + .... +
Req R1 R2 R3 Rn
Hence, reciprocal of the equivalent resistance is equal to the sum of reciprocal of each resistor
joined in parallel (b) Let net resistance of the given parallel combination be Root, Then
1 1 1
= +
Rnet 12 12
1 2 1
= =
Rnet 12 6

⇒ Rnet = 6 Ω
V 6  V
= i = = 1 A
Hence, current, Rnet 6 Ω

OR

Resistance of electric lamp = 20 Ω


Resistance of conductor= 4 Ω
Voltage battery= 6V
Apply Ohms law
V   
= IR
6 V= I × 24 Ω
6  V
=I =  0.25  A
24 Ω
Hence, current in the circuit is 0.25A

(c)

Potential difference across the loop


Vlamp = IR
Vlamp 0.25 A × 20 Ω = 5 V
=

5 V
∴Vlamp =

Potential difference across the conductor


VConductor = IR
= 0.25 A × 4 Ω = 1 V
VConductor
1 V
∴VConductor =
( 0.25)
2 2
I= R 20 1.25 W
×=
Power of lamp

17. (a) Draw magnetic field lines produced around a current carrying straight conductor
passing through cardboard. Name, state and apply the rule to mark the direction of these
field lines.
(b) How will the strength of the magnetic field change when the point where magnetic field is
to be determined is moved away from the straight wire carrying constant current? Justify
your answer.
ANSWER:

(a) To mark the direction of magnetic field lines, we will use The Right hand thumb rule. The
right-hand thumb rule is used to find the direction of the magnetic field lines, according to this
rule if we place our right-hand thumb along the direction of the current flowing in a current
carrying wire, the direction in which the fingers wrap the wire represents the direction of the
magnetic field. As we can see in the given figure, where the current is going downward, the
direction of magnetic field lines is clockwise, according to the rule.

(b) Using a compass needle we can determine the magnetic field. When we move away from
the compass needle from the straight wire, the deflection of the needle decreases which
implies the strength of the magnetic field decreases, as the strength of magnetic field produced
by a straight wire at any point is inversely proportional to the distance of the point from the
wire.
18. An object is placed at a distance of 60 cm from a concave lens of focal length 30 cm.
(i) Use lens formula to find the distance of the image from the lens.
(ii) List four characteristics of the image (nature, position, size, erect/inverted) formed by the
lens in this case.
(iii) Draw ray diagram to justify your answer of part (ii).

ANSWER: We have, (i) Object distance, u= —60 cm Focal length of the concave lens, f= —30 cm
Using lens formula,
1 1 1
− =
v u f
1 1 1
− =
v ( −60 ) ( −30 )
1 −1 1
= −
v 30 60
1 −3
=
v 60
v = −20 cm
The image will be formed at a distance of 20 cm in front of the lens.
(ii) Nature of the image is virtual. The position of the image is between F1 and optical center 0.
Size of the image is diminished. The image is Erect.

(iii)
19. Define pollination. Explain the different types of pollination. List two agents of
pollination? How does suitable pollination lead to fertilization?

OR

Identify the given diagram. Name the parts 1 to 5.

What is contraception? List three advantage of adopting contraceptive measures.

ANSWER: Transfer of pollen grain from the anther of a flower to stigma is termed as
pollination.
Based on the transfer of pollen grains between plants, pollination can be of two types:
1) Self Pollination: When pollen of a plant is transferred to the stigma of a flower on the same
parent plant then it is termed as self-pollination.
2) Cross Pollination: When pollen of a plant is transferred to the stigma of a flower of a plant
different from the one from which pollen is obtained then it is termed as cross-pollination.
Some of the most common agents of pollination that helps in carrying the pollen from the
anther to the stigma of a flower are insects and wind.
As the pollen of the right type is deposited on to the stigma of the flower of the same species,
quite often it leads to germination of pollen grain as a result of a chemical cross-talk between
the pollen and the carpel. Germination leads to the growth and extension of the pollen tube
through the style of the flower to its ovary. The pollen tube carries the male gametes all the
way to the ovule inside the ovary, leading to fertilisation of male gamete with the female
gamete inside the ovule.

OR

(a) The labelled parts of the female reproductive system are as follows:

1 - Oviduct or Fallopian Tube 2 - Ovary 3 - Uterus 4 - Cervix 5 - Vagina


(b) Contraception includes methods or ways to prevent fertilisation and pregnancy in a fertile
female as a result of successful copulation between a fertile male and female.
Some of the major advantages of adopting various contraceptive methods include:
(i) Prevention of unwanted pregnancies
(ii) Help in family planning and population control
(iii) Some contraceptive devices like condoms and female diaphragm prevent spread of
STDs like AIDS and Hepatitis B.

20. Write the chemical formula and name of the compound which is the active ingredient of
all alcoholic drinks. List its two uses.
Write chemical equation and name of the product formed when this compound reacts with-
(i) Sodium metal
(ii) hot concentrated sulphuric acid

OR

What is methane? Draw its electron dot structure. Name the type of bonds formed in this
compound. Why are such compounds:
(i) poor conductors of electricity? and
(ii) have low melting and boiling points? What happens when this compound burns in
oxygen?

ANSWER: Name of the compound which is the active ingredient of all alcoholic drinks is
Ethanol, and its chemical formula is CH3CH2OH. Uses- (i) Ethanol is used in medical
wipes/swabs and antibacterial hand sanitizers. (ii) Ethanol is also used in body lotions as
preservatives and stabilizer of its ingredients.
Chemical reactions of ethanol-
CH 3CH 2OH + Na → CH 3CH 2O − Na + + H 2
CH 3CH 2OH + H 2 SO4 → C2 H 4 + H 2 0
OR

Methane is a colourless, odourless and highly flammable gas which is the main component of
natural gas. It is also called as marsh gas as it is produced when vegetation decomposes
naturally in any swampy or marshlands.

The type of bonds present in methane are all covalent bonds between four hydrogen atoms
and the single carbon atom at the center of the molecule.
(i) Methane is a poor conductor of electricity because in methane all bonds are covalent bonds
and therefore no free electrons are present in the molecule that can help in the conduction of
electricity.
(ii) Covalent compounds have low intermolecular forces of attraction between the molecules
and thus show low melting and boiling points. Since, methane is also a covalent compound thus
methane has very low melting and low boiling point. When methane is burned in the presence
of Oxygen it forms Carbon dioxide gas and water as a product of the reaction.
CH 4 + O2 → CO2 + H 2O
Methane Water Carbon dioxide Water

21. Write the main difference between an acid and a base. With the help of suitable examples
explain the term neutralization and the formation of –
(i) acidic,
(ii) basic and
(iii) neutral salts

ANSWER:

Acid Base
1. Turns blue litmus red 1. Turns red litmus blue
2. Sour in taste 2. Bitter in taste
3. Release H+ ion in water3. Release OH- ion in water
4. pH is less than 7 4. pH is more than 7
5. Example: HCI 5. Example: NaOH

Neutralization is a type of chemical reaction in which an acid and base react to form salt and
water with evolution of heat. i) When neutralisation of a strong acid with a weak base takes
place, it leads to the formation of acidic salt. Example

HCl       
+ NH 4OH → NH 4Cl + H 2 O
NaOH       
+ CH 3COOH → CH 3COO − Na + + H 2 O
NaOH       
+ HCl → NaCl + H 2 O
22. In the experimental set up to show that "CO2 is given out during respiration", name the
substance taken in the small test tube kept in the conical flask. State its function and the
consequence of its use.

ANSWER: In the above mentioned experimental setup, lime water is taken in a small test tube
which is kept in the conical flask. Lime water is used to detect the presence of carbon dioxide.
When carbon dioxide passes through lime water, it turns milky due to the formation of calcium
carbonate. Hence, it is used in the above experiment to check whether CO2 is released during
respiration.

23. A student is observing the temporary mount of a leaf peel under a microscope. Draw
labelled diagram of the structure of stomata as seen under the microscope

OR

Draw a labelled diagram in proper sequence to show budding in hydra.

ANSWER: Structure of stomata

OR

Budding of hydra

24. List four precautions which a student should observe while determining the focal length
of a given convex lens by obtaining image of a distant object on a screen.
ANSWER: Following are the precautions while making the image by the help of convex lens: 1.)
Fix the convex lens vertically in the lens holder.
2.) The base of the lens and white screen should be in a line with the measuring scale.
3.) Record the position of the lens and screen only when a well-defined sharp image is formed.
4.) There should not be any obstacle in the path of the convex lens.

25. While studying the dependence of potential difference ( V) across a resistor on the
current (I) passing through it, in order to determine the resistance of the resistor, a student
took 5 readings for different values of current and plotted a graph between V and t. He got a
straight line graph passing through the origin. What does the straight line signify? Write the
method of determining the resistance of the resister using this graph.

OR

What would you suggest to a student if while performing an experiment he finds that the
pointer/needle of the ammeter and voltmeter do not coincide with the zero marks on the
scales when the circuit is open? No extra ammeter/voltmeter is available in the laboratory.
ANSWER:

Straight line graph between potential difference ( V) and current(I) shows that resistor is a
linear element and follows Ohm's Law.
Calculation of Resistance: At any point on the graph, resistance is given as,
V (value of potential difference at that point)
R= o
I o (value of current at that point)
1
In other words, the Slope of the graph at any point gives the value R
Hence,
Io 1
slope
= =
Vo R
1 V (Value of Potential difference at that point)

= R = o
slope I o (Value of Current at that point)
OR
This is called the zero error of the scale of ammeter or voltmeter. If there is a zero error, then
this error is subtracted from the value that depicts when the circuit is closed, otherwise, the
accurate current or potential difference will not be recorded.

26. In three test tubes A, B, and C, three different liquids namely, distilled water,
underground water and distilled water in which a pinch of calcium sulphate is dissolved,
respectively are taken. Equal amount of soap Answer is added to each test tube and the
contents are shaken. In which test tube will the length of the foam (lather) be longest? Justify
your answer.

ANSWER:

Test tube A contains distilled water which is considered as soft water, free from ions like Mg2÷,
Ca2+, etc. So, in this case, the length of foam will be the longest(maximum). Test tube B
contains underground water which contains ions like Mg2+, Ca2÷, etc. which react with soaps
to form salts of fatty acids called scum, which are insoluble in water. So, in this case, the length
of foam will be the less in comparison to test tube A. Test tube C contains distilled water with
CaSO4, which contains Ca2÷ ions which react with soaps to form salts of fatty acids called scum,
which are insoluble in water. So, in this case also, the length of foam will be the less in
comparison to test tube A.

27. Blue litmus is added to two test tubes A and B containing dilute HCl and NaOH. Answer
respectively. In which test tube a colour change will be observed? State the colour change
and give its reason.

OR

What is observed when 2 mL of dilute hydrochloric acid is added to 1 g of sodium carbonate


taken in a clean and dry test tube? Write chemical equation for the reaction involved.
ANSWER: Test tube A contains dil. HCl and test tube B contains dil. NaOH. On adding blue
litmus Answer to both the test tubes, the colour of the test tube A will change from blue to red.
This is because HCl is an acid and acids turn blue litmus to red.

OR

On adding dilute hydrochloric acid to sodium carbonate taken in a clean and dry test tube, a
brisk effervescence will be observed due to the evolution of carbon dioxide gas.

2 HCl     
+ Na2CO3 → 2 NaCl     
+ H 2O + CO2

You might also like